Kvpy 11 Years Solved Papers 2019 2009 Stream SA by Arihant

You might also like

Download as pdf or txt
Download as pdf or txt
You are on page 1of 257

https://iit-jeeacademy.blogspot.

com
https://iit-jeeacademy.blogspot.com

Kishore Vaigyanik Protsahan Yojana

11 Years’
SOLVED PAPERS
2019 - 2009

Stream SA
https://iit-jeeacademy.blogspot.com
https://iit-jeeacademy.blogspot.com

Included
5 Practice
Sets

Kishore Vaigyanik Protsahan Yojana

11 Years’
SOLVED PAPERS
2019 - 2009

Stream SA

Authors
Lakshman Prasad (Mathematics)
Deepak Paliwal, Mansi Garg (Physics)
Neha Minglani Sachdeva (Chemistry)
Sanubia Saleem (Biology)

ARIHANT PRAKASHAN (Series), MEERUT


https://iit-jeeacademy.blogspot.com

ARIHANT PRAKASHAN (Series), MEERUT


All Rights Reserved

© Publishers
No part of this publication may be re-produced, stored in a retrieval system or distributed in
any form or by any means, electronic, mechanical, photocopying, recording, scanning, web
or otherwise without the written permission of the publisher. Arihant has obtained all the
information in this book from the sources believed to be reliable and true. However, Arihant
or its editors or authors or illustrators don’t take any responsibility for the absolute accuracy
of any information published and the damages or loss suffered there upon.
All disputes subject to Meerut (UP) jurisdiction only.

Administrative & Production Offices


Regd. Office
‘Ramchhaya’ 4577/15, Agarwal Road, Darya Ganj, New Delhi -110002
Tele: 011- 47630600, 43518550; Fax: 011- 23280316

Head Office
Kalindi, TP Nagar, Meerut (UP) - 250002
Tel: 0121-7156203, 7156204

Sales & Support Offices


Agra, Ahmedabad, Bengaluru, Bareilly, Chennai, Delhi, Guwahati,
Hyderabad, Jaipur, Jhansi, Kolkata, Lucknow, Nagpur & Pune.

ISBN : 978-93-24195-37-1
Price : ` 195.00
PO No : TXT-XX-XXXXXXX-X-XX
Published by Arihant Publications (India) Ltd.
For further information about the books published by Arihant
log on to www.arihantbooks.com or email to info@arihantbooks.com
Follow us on
https://iit-jeeacademy.blogspot.com

Kishore Vaigyanik Protsahan Yojana

ABOUT THE EXAM


KVPY i.e. Kishore Vaigyanik Protsahan Yojana is a National Level Fellowship (scholarship) Program in
Basic Science (Physics, Chemistry, Mathematics & Biology) upto Pre-Phd Level, run by Department of
Science & Technology, Government of India and Conducted by IISC (Indian Institute of Science)
Bangalore, Karnataka Annually.
It Was Started in 1999 to Encourage Basic Sciences Students to take up Research Career in Sciences.
The Objective of the Exam is to Encourage Talented Students for Research Career in Sciences.

ELIGIBILITY CRITERIA
KVPY scholarships are given only to Indian Nationals to study in India. There are three
streams in KVPY; SA, SB & SX. Eligibility criteria for different streams is discussed below;
Ÿ For SA Class 11 Students who passed class 10 with minimum 75% (65% for SC/ST/PWD)
marks in Mathematics & Science.
The fellowship of students selected in SA will be activated only if they pursue
undergraduate courses in Basic Sciences (B.Sc./B.S./B.Stat./B.Math/Integrated M.Sc. or
M.S.) and have secured a minimum of 60% (50% for SC/ST/PWD) marks in science
subjects in class 12th.

Ÿ For SX Class 12 Students aspiring to pursue undergraduate program (B.Sc. etc ) with
basic sciences (Physics, Chemistry, Mathematics & Biology) who passed class 10 with
minimum 75% (65% for SC/ST/PWD) marks in Mathematics & Science.
The fellowship of students selected in SX will be activated only if they pursue
undergraduate courses in Basic Sciences (B.Sc./B.S./B.Stat./B.Math/Integrated M.Sc. or
M.S.) and have secured a minimum of 60% (50% for SC/ST/PWD) marks in science
subjects in class 12th.

Ÿ For SB B.Sc. Ist year Students who passed class 12 with 60% marks in Maths & Sciences
(PCMB) & class 10 with minimum 75% marks in Mathematics & Science.
In order to activate fellowship, in the first year of undergraduate course they should
secure minimum 60% (50% for SC/ST/PWD) marks.

Those students who are intending or pursuing undergraduate program under distance
education scheme or correspondence course of any university are not eligible.
https://iit-jeeacademy.blogspot.com

SYLLABUS OF KVPY
There is no prescribed syllabus for KVPY aptitude test, it aims to assess the understanding &
analytical ability of the students than his/her factual knowledge. However questions are
framed from syllabus upto class 10/12/Ist Year of Undergraduate Courses in basic sciences, as
applicable. There are two Questions Papers in KVPY; one for stream SA & Other for SB/SX
(Question Paper is same for SB & SX).

QUESTION PAPERS PATTERN


There are two Questions Papers in KVPY; one for stream SA & Other for SX/SB
(Question Paper is same for SB & SX).
Ÿ Question Paper for SA Stream caries 80 Questions for 100 marks. There are Two Parts in
the Question Paper; Part I has 15 Questions of 1 mark each for Mathematics, Physics,
Chemistry & Biology while Part II has 5 Questions of 2 marks each for Mathematics,
Physics, Chemistry & Biology.
Ÿ Question Paper for SB/SX Stream caries 120 Questions for 160 marks. There are Two
Parts in the Question Paper; Part I has 20 Questions of 1 mark each for Mathematics,
Physics, Chemistry & Biology while Part II has 10 Questions of 2 marks each for
Mathematics, Physics, Chemistry & Biology.

MODE OF EXAM
KVPY is conducted in Online Mode in English & Hindi Medium.

TIME OF EXAM
Ÿ Normally the notification or advertisement for KVPY appear in National Newspapers on
May 11 (Technology Day) and Second Sunday of July every year.
Ÿ Generally the exam is conducted in the month of November.

SELECTION PROCESS
After scrutiny of application forms on the basis of eligibility criteria for various streams all
eligible students are called for Aptitude Test conducted in English & Hindi Medium at
different centers across the country. On the basis of performance in aptitude test shortlisted
students are called for an interview, which is the final stage of selection procedure.
https://iit-jeeacademy.blogspot.com

FELLOWSHIPS
The selected students are eligible to receive KVPY fellowship after class 12th/Ist Year of
Undergraduate course only if they pursue Undergraduate Courses in Basic Science, upto Pre-
PhD or 5 Years whichever is earlier.
Details of fellowships are listed below;

Monthly Annual
Basic Science Fellowship Contingency Grant
SA/SX/SB during Ist to IIIrd year
of B.Sc./B.S./BB.Stat./B.Math/ Rs. 5000 Rs. 20000
Integrated M.Sc or M.S.
SA/SX/SB during M.Sc. / IVth to Vth
years of Integrated M.Sc /M.S./ Rs. 7000 Rs. 28000
M.Math/ M.Stat.

CONTINUATION / RENEWAL OF FELLOWSHIP


Ÿ The fellow should continue to study basic science and should also maintain a minimum
level of academic performance as Ist division or 60% (50% for SC/ST/PWD) marks in
aggregate. Also the fellow has to pass all the subjects prescribed for that particular year.
Ÿ In each year marks are to be certified by the Dean or Head of the Institution.
Ÿ The fellowship will be discontinued if above marks are not obtained. However if fellow
passed all the subjects & obtain marks more than 60% (50% for SC/ST/PWD) in
subsequent year, the fellowship can be renewed only for that year onwards.
Ÿ If KVPY fellow opts out of the basic science at any stage then monthly fellowship and
contingency grant will be forfeited from him.

KVPY Timeline 2020


IMPORTANT DATES
Opening of Application Portal : 2nd Week of July 2020
Last Date of Submission of Online Application : 1st Week of September 2020
KYPY Aptitude Test : 1st Week of November 2020
APPLICATION FEE
For General Category : Rs. 1000/-
For SC/ST/ Persons with Disabilities : Rs. 500/-

For more details visit:www.kvpy.iisc.ernet.in


https://iit-jeeacademy.blogspot.com

CONTENTS

KVPY SA QUESTION PAPERS (2019-2009)

QUESTION PAPER QUESTION PAPER QUESTION PAPER


2019 2018 2017 (19 Nov)
Pg. No. 1-16 Pg. No. 1-15 Pg. No. 16-31

QUESTION PAPER QUESTION PAPER QUESTION PAPER


2017 (05 Nov) 2016 2015
Pg. No. 32-47 Pg. No. 48-62 Pg. No. 63-76

QUESTION PAPER QUESTION PAPER QUESTION PAPER


2014 2013 2012
Pg. No. 77-92 Pg. No. 93-107 Pg. No. 108-122

QUESTION PAPER QUESTION PAPER QUESTION PAPER


2011 2010 2009
Pg. No. 123-137 Pg. No. 138-150 Pg. No. 151-164

KVPY PRACTICE SETS (1-5) 167-232


https://iit-jeeacademy.blogspot.com

KVPY Question Paper 2019 Stream : SA 1

KVPY
KISHORE VAIGYANIK PROTSAHAN YOJANA

QUESTION PAPER 2019


Stream : SA
MM 100

Instructions
There are 80 questions in this paper.
This question paper contains two parts; Part I and Part II. There are four sections; Mathematics, Physics, Chemistry
and Biology in each part.
Out of the four options given with each question, only one is correct.

PART-I (1 Mark Questions)


MATHEMATICS Then, which of the following statements are true?
1. Let ABC be an equilateral triangle with side length I. g has exactly two distinct real roots.
a. Let R and r denote the radii of the circumcircle and II. g can have more than two distinct real roots.
the incircle of triangle ABC respectively. Then, as a III. There exists a real number α such that g(x) ≥ α
R for all real x.
function of a, the ratio
r (a) Only I (b) Both I and III
(a) strictly increases (b) strictly decreases (c) Only II (d) Both II and III
(c) remains constant
4. Let a n , n ≥ 1, be an arithmetic progression with first
(d) strictly increases for a < 1and strictly decrease for a > 1
term 2 and common difference 4. Let M n be the
2. Let b be an non-zero real number. Suppose the 10
1
quadratic equation 2x + bx + = 0 has two distinct
2 average of the first n terms. Then the sum ∑ M n is
b n =1

real roots. Then (a) 110 (b) 335


1 5 1 5 (c) 770 (d) 1100
(a) b + > (b) b + <
b 2 b 2 5. In a triangle ABC, ∠BAC = 90°; AD is the altitude
1
(c) b2 − 3b > − 2 (d) b2 + 2 < 4 from A on to BC. Draw DE perpendicular to AC and
b
DF perpendicular to AB. Suppose AB = 15 and
3. Let p(x) = x + ax + b have two distinct real roots,
2
BC = 25. Then the length of EF is
where a , b are real number. Define g(x) = p(x3 ) for all (a) 12 (b) 10
real number x. (c) 5 3 (d) 5 5
https://iit-jeeacademy.blogspot.com

2 KVPY Question Paper 2019 Stream : SA

6. The sides a , b, c of a triangle satisfy the relations (b) given any positive real number α, we can choose C and
Area (C )
c2 = 2ab and a 2 + c2 = 3b2. Then the measure of T as above such that ratio is less than α
Area (T )
∠BAC, in degrees, is Area (C )
(a) 30 (b) 45 (c) give any C and T as above, the ratio is
Area (T )
(c) 60 (d) 90
independent of C and T
7. Let N be the least positive integer such that (d) there exist real numbers a and b such that for any
whenever a non-zero digit c is written after the last circle C and triangle T as above, we must have
Area (C )
digit of N, the resulting number is divisible by c. The a< <b
sum of the digits of N is Area (T )
(a) 9 (b) 18 15. The number of three digit numbers abc such that the
(c) 27 (d) 36 arithmetic mean of b and c and the square of their
8. Let x1, x2,…, x11 be 11 distinct positive integers. If we geometric mean are equal is
replace the largest of these integers by the median of (a) 9 (b) 18
the other 10 integers, then (c) 36 (d) 54
(a) the median remains the same
(b) the mean increases PHYSICS
(c) the median decreases
(d) the mean remains the same 16. Various optical processes are involved in the
9. The number of cubic polynomials P (x) satisfying formation of a rainbow. Which of the following
provides the correct order in time in which these
P(1) = 2, P(2) = 4, P(3) = 6, P(4) = 8 is
processes occur?
(a) 0
(a) Refraction, total internal reflection, refraction.
(b) 1
(b) Total internal reflection, refraction, total internal
(c) more than one but finitely many reflection.
(d) infinitely many (c) Total internal reflection, refraction, refraction.
10. A two-digit number ab is called almost prime if one (d) Refraction, total internal reflection, total internal
obtains a two-digit prime number by changing at reflection.
most one of its digits a and b. (For example, 18 is an 17. A specially designed Vernier calliper has the main
almost prime number because 13 is a prime number). scale least count of 1 mm. On the Vernier scale, there
Then the number of almost prime two-digit numbers are 10 equal divisions and they match with 11 main
is scale divisions. Then, the least count of the Vernier
(a) 56 (b) 75 (c) 87 (d) 90 calliper is
11. Let P be an interior point of a convex quadrilateral (a) 0.1 mm (b) 0.909 mm
ABCD and K , L , M , N be the mid-points of AB, BC, (c) 1.1 mm (d) 0.09 mm
CD, DA respectively. If Area (PKAN ) = 25, Area 18. A steel ball is dropped in a viscous liquid. The
(PLBK ) = 36, and Area (PMDN ) = 41 then Area distance of the steel ball from the top of the liquid is
(PLCM ) is shown below. The terminal velocity of the ball is
(a) 20 (b) 29 (c) 52 (d) 54 closest to
12. The number of non-negative integer solutions of the
equations 6x + 4 y + z = 200 and x + y + z = 100 is 0.4
(a) 3 (b) 5 (c) 7 (d) Infinite
13. Let N 1 = 255 + 1 and N 2 = 165. 0.3
Distance (m)

Then
(a) N1 and N 2 are coprime 0.2
(b) the HCF (Highest Common Factor) of N1 and N 2 is 55
(c) the HCF of N1 and N 2 is 11
(d) the HCF of N1 and N 2 is 33 0.1

14. Let l > 0 be a real number, C denote a circle with


circumference l and T denote a triangle with 0
0 0.5 1 1.5 2
perimeter l. Then
Time (s)
(a) given any positive real number α, we can choose C and
Area (C ) (a) 0.26 m/s (b) 0.33 m/s
T as above such that ratio is greater than α (c) 0.45 m/s (d) 0.21 m/s
Area (T )
https://iit-jeeacademy.blogspot.com

KVPY Question Paper 2019 Stream : SA 3

19. A student in a town in India, where the price per unit hemisphere at a height h from the horizontal surface,
(1 unit = 1 kWh) of electricity is ` 5.00, purchases a then the speed of the particle is
1 kVA UPS (uninterrupted power supply) battery. A (a) (2g (R − h ))
day before the exam, 10 friends arrive to the (b) (2g (R + h ))
student’s home with their laptops and all connect (c) 2gR
their laptops to the UPS. Assume that each laptop (d) 2gh
has a constant power requirement of
24. The nuclear radius is given by R = r0 A1/ 3 , where r0 is
90 W. Consider the following statements.
constant and A is the atomic mass number. Then, the
I. All the 10 laptops can be powered by the UPS, if
nuclear mass density of U238 is
connected directly.
(a) twice that of Sn119
II. All the 10 laptops can be powered, if connected (b) thrice that of Sn119
using an extension box with a 3 A fuse. (c) same as that of Sn119
III. If all the 10 friends use the laptop for 5 h, then (d) half that of Sn119
the cost of the consumed electricity is about
25. The electrostatic energy of a nucleus of charge Ze is
` 22.50.
kZ 2e2
Select the correct option with the true statements. equal to , where k is a constant and R is the
R
(a) I only (b) I and II only nuclear radius. The nucleus divides into two
(c) I and III only (d) II and III only Ze
daughter nuclei of charges and equal radii. The
20. Frosted glass is widely used for translucent windows. 2
The region, where a transparent adhesive tape is change in electrostatic energy in the process when
stuck over the frosted glass becomes transparent. they are far apart is
The most reasonable explanation for this is 0.375kZ 2e2 0125
. kZ 2e2
(a) (b)
(a) diffusion of adhesive glue into glass R R
(b) chemical reaction at adhesive tape-glass interface kZ 2e2 0.5kZ 2e2
(c) (d)
(c) refractive index of adhesive glue is close to that of glass R R
(d) adhesive tape is more transparent than glass
26. Two masses M1 and M 2 carry positive charges Q1 and
21. Consider two equivalent, triangular hollow prisms A Q2, respectively. They are dropped to the floor in a
and B made of thin glass plates and arranged with laboratory set up from the same height, where there
negligible spacing as shown in the figure. A beam of is a constant electric field vertically upwards. M1 hits
white light is incident on prism A from the left. Given the floor before M 2. Then,
that, the refractive index of water is inversely related (a) Q1 > Q2 (b) Q1 < Q2
to temperature, the beam to the right of prism B (c) M1Q1 > M2Q2 (d) M1Q2 > M2Q1
would not appear white, if
27. Which one of the following schematic graphs best
represents the variation of pV (in Joules) versus T (in
Kelvin) of one mole of an ideal gas? (The dotted line
ht A B represents pV = T )
te lig
Whi

(a) both prisms are filled with hot water (70°C)


(b) both prisms are filled with cold water (7°C) (a) pV (J) (b) pV (J)
(c) both prisms are empty
(d) prism A is filled with hot water (70°C) and prism B
with cold water (7°C)
T (K) T (K)
22. A ball is moving uniformly in a circular path of
radius 1 m with a time period of 1.5 s. If the ball is
suddenly stopped at t = 83
. s, the magnitude of the
displacement of the ball with respect to its position at
t = 0 s is closest to
(a) 1 m (b) 33 m (c) pV (J) (d) pV (J)
(c) 3 m (d) 2 m
23. A particle slides from the top of a smooth
hemispherical surface of radius R which is fixed on a T (K) T (K)
horizontal surface. If it separates from the
https://iit-jeeacademy.blogspot.com

4 KVPY Question Paper 2019 Stream : SA

. × 1012 L of water annually. Its


28. Mumbai needs 14 34. IUPAC name of the following compound
effective surface area is 600 km 2 and it receives an O
average rainfall of 2.4 m annually. If 10% of this rain
water is conserved, it will meet approximately
(a) 1% of Mumbai’s water needs
HO
(b) 10% of Mumbai’s water needs is
(c) 50% of Mumbai’s water needs (a) 1-hydroxycyclohex-4-en-3-one
(d) 100% of Mumbai’s water needs (b) 1-hydroxycyclohex-3-en-5-one
29. A mass M moving with a certain speed V collides (c) 3-hydroxycyclohex-5-en-1-one
elastically with another stationary mass m. After the (d) 5-hydroxycyclohex-2-en-1-one
collision, the masses M and m move with speeds V ′ 35. In water-gas shift reaction, hydrogen gas is produced
and v, respectively. All motion is in one dimension. from the reaction of steam with
Then,
(a) methane (b) coke
(a) V = V ′ + v (b) V ′ = V + v
(c) carbon monoxide (d) carbon dioxide
(V + v)
(c) V ′ = (d) v = V + V ′ 36. Treatment with lime can remove hardness of water
2
caused by
30. Four ray 1, 2, 3 and 4 are incident normally on the
(a) CaCl 2 (b) CaSO4
face PQ of an isosceles prism PQR with apex angle
(c) Ca(HCO3 )2 (d) CaCO3
∠Q = 120°. The refractive indices of the material of
the prism for the above rays 1, 2, 3 and 4 are 1.85, 37. The most polarisable ion among the following is
1.95, 2.05 and 2.15 respectively and the surrounding (a) F− (b) I− (c) Na + (d) Cl −
medium is air. Then, the rays emerging from the face 38. For a multi-electron atom, the highest energy level
QR are among the following is
(a) 4 only (b) 1 and 2 only 1
(a) n = 5, l = 0, m = 0, s = +
(c) 3 and 4 only (d) 1, 2, 3 and 4 2
1
(b) n = 4, l = 2, m = 0, s = +
2
CHEMISTRY (c) n = 4, l = 1, m = 0, s = +
1
2
31. The hybridisations of N, C and O shown in the 1
following compound (d) n = 5, l = 1, m = 0, s = +
2
R
N C O
39. The oxide, which is neither acidic nor basic is
(a) As2O3 (b) Sb4 O10 (c) N2O (d) Na 2O
respectively, are
2 2 2 2 2
40. The element whose salts cannot be detected by
(a) sp , sp, sp (b) sp , sp , sp flame test is
(c) sp 2, sp, sp (d) sp, sp, sp 2
(a) Mg (b) Na (c) Cu (d) Sr
32. The following compounds 41. The plot of concentration of a reactant vs time for a
chemical reaction is shown below.
are
Concentration

(a) geometrical isomers (b) positional isomers


(c) optical isomers (d) functional group isomers
33. The major product of the following reaction
Br
Br 1. Excess alc. KOH
2. NaNH2 O Time
Ph
3. H3O
+ The order of this reaction with respect to the reactant
is is
H (a) 0 (b) 1 (c) 2
H (d) not possible to determine from this plot

(a) Ph H (b) Ph 42. During the free expansion of an ideal gas in an


Br isolated chamber,
Br Br (a) internal energy remains constant
H H (b) internal energy decreases
(c) Ph (d) Ph (c) work done on the system is negative
Br H (d) temperature increases
https://iit-jeeacademy.blogspot.com

KVPY Question Paper 2019 Stream : SA 5

43. The number of moles of water present in a spherical 51. The mode of speciation mediated by geographical
water droplet of radius 1.0 cm is, isolation is referred as
. g cm −3 ]
[Given : Density of water in the droplet = 10 (a) adaptive radiation
π 2π (b) allopatric speciation
(a) (b) (c) parapatric speciation
18 27
2π (d) sympatric speciation
(c) 24π (d)
9 52. Which one of the following metabolic conversion
44. Among the following, the correct statement about requires oxygen?
cathode ray discharge tube is (a) Glucose to pyruvate
(a) the electrical discharge can only be observed at high (b) Glucose to CO 2 and ethanol
pressure and at low voltage. (c) Glucose to lactate
(b) in the absence of external electrical or magnetic field, (d) Glucose to CO 2 and H2 O
cathode rays travel in straight lines. 53. Where are the proximal and distal convoluted
(c) the characteristics of cathode rays depend upon the tubules located within the human body?
material of electrodes. (a) Adrenal cortex
(d) the characteristics of cathode rays depend upon the (b) Adrenal medulla
gas present in the cathode ray tube.
(c) Renal cortex
45. For a spontaneous process, (d) Renal medulla
(a) enthalpy change of the system must be negative
54. In a diploid organism, when the locus X is
(b) entropy change of the system must be positive
inactivated, transcription of the locus Y is triggered.
(c) entropy change of the surrounding must be positive Based on this observation, which one of the following
(d) entropy change of the system plus surrounding must statements is CORRECT?
be positive
(a) X is dominant over Y
(b) X is epistatic to Y
BIOLOGY (c) Y is dominant over X
(d) Y is epistatic to X
46. Which one of the following is a CORRECT statement
about primates’ evolution?
55. Which one of the following sequences represents the
CORRECT taxonomical hierarchy?
(a) Chimpanzees and gorillas evolved from macaques
(a) Species, genus, family, order
(b) Humans and chimpanzees evolved from gorillas
(b) Order, genus, family, species
(c) Human, chimpanzees and gorillas evolve from a
common ancestor (c) Species, order, genus, family
(d) Humans and gorillas evolved from chimpanzees (d) Species, genus, order, family

47. The crypts of Lieberkuhn are found in which one of 56. Which one of the following organs is NOT a site for
the following parts of the human digestive tract? the production of white blood cells?
(a) Oesophagus (a) Bone marrow (b) Kidney
(b) Small intestine (c) Liver (d) Spleen
(c) Stomach 57. Which one of the following anatomical structures is
(d) Rectum involved in guttation?
48. Removal of the pancreas impairs the breakdown of (a) Cuticle (b) Hydathodes
(a) lipids and carbohydrates only (c) Lenticels (d) Stomata
(b) lipids and proteins only 58. Which one of the following parts of the eye is affected
(c) lipids, proteins and carbohydrates in cataract?
(d) proteins and carbohydrates only (a) Cornea (b) Conjunctiva
49. Microscopic examination of a blood smear reveals an (c) Retina (d) Lens
abnormal increase in the number of granular cells 59. Which one of the following organisms is a bryophyte?
with multiple nuclear lobes. Which one of the (a) Liverwort (b) Volvox
following cell types has increased in number? (c) Chlamydomonas (d) Fern
(a) Lymphocytes (b) Monocytes
60. During oogenesis in mammals, the second meiotic
(c) Neutrophils (d) Thrombocytes
division occurs
50. Which one of the following genetic phenomena is (a) before fertilisation
represented by the blood group AB? (b) after implantation
(a) Codominance (b) Dominance (c) before ovulation
(c) Overdominance (d) Semidominance (d) after fertilisation
https://iit-jeeacademy.blogspot.com

6 KVPY Question Paper 2019 Stream : SA

PART-II (2 Marks Questions)


MATHEMATICS 67. In an hour-glass approximately 100 grains of sand
fall per second (starting from rest); and it takes 2 s
61. Let a , b, c, d be distinct real numbers such that a , b for each sand particle to reach the bottom of the
are roots of x2 − 5cx − 6d = 0, and c, d are roots of hour-glass. If the average mass of each sand particle
x2 − 5ax − 6b = 0. Then b + d is is 0.2 g, then the average force exerted by the falling
(a) 180 (b) 162 (c) 144 (d) 126 sand on the bottom of the hour-glass is close to
(a) 0.4 N (b) 0.8 N
62. Let S = {1, 2, 3, … , 100}. Suppose b and c are chosen at
(c) 1.2 N (d) 1.6 N
random from the set S. The probability that
4x2 + bx + c has equal roots is 68. A student uses the resistance of a known resistor
(1 Ω ) to calibrate a voltmeter and an ammeter using
(a) 0.001 (b) 0.004 (c) 0.007 (d) 0.01 the circuits shown below. The student measures the
63. Let N be the set of positive integers. For all n ∈ N , let ratio of the voltage to current to be 1 × 103 Ω in
fn = (n + 1)1/ 3 − n1/ 3 and circuit (a) and 0999
. Ω in circuit (b). From these
  measurements, the resistance (in Ω) of the voltmeter
1
A = n ∈ N : fn + 1 < < fn  and ammeter are found to be close to
 3(n + 1)2/3

v A
Then,
(a) A = N 1=W 1=W

v
(b) A is a finite set
(c) the complement of A in N is nonempty, but finite
(d) A and its complement in N are both infinite (a) (b)
64. A prime number p is called special if there exist 2
(a) 10 and 10 −2
(b) 10 and 10−3
3

primes p1, p2, p3 , p4 such that p = p1 + p2 = p3 − p4. (c) 10−2 and 102 (d) 10−2 and 103
The number of special primes is 69. A hot air balloon with a payload rises in the air.
(a) 0 Assume that the balloon is spherical in shape with
(b) 1 diameter of 11.7 m and the mass of the balloon and
(c) more than one but finite the payload (without the hot air inside) is 210 kg.
(d) infinite Temperature and pressure of outside air are 27 °C
65. Let ABC be a triangle in which AB = BC. Let X be a and 1 atm = 105 N/m 2, respectively. Molar mass of
point on AB such that AX : XB = AB : AX. If dry air is 30 g. The temperature of the hot air inside
AC = AX, then the measure of ∠ABC equals . JK −1mol−1]
is close to [The gas constant, R = 831
(a) 18° (b) 36° (c) 54° (d) 72° (a) 27 °C (b) 52 °C
(c) 105 °C (d) 171 °C

PHYSICS 70. A healthy adult of height 1.7 m has an average blood


pressure (BP) of 100 mm of Hg. The heart is typically
66. A water-proof laser pointer of length 10 cm placed in at a height of 1.3 m from the foot. Take, the density
a water tank rotates about a horizontal axis passing of blood to be 103 kg/m3 and note that 100 mm of Hg
through its centre of mass in a vertical plane as is equivalent to 13.3 kPa (kilo pascals). The ratio of
shown in the figure. The time period of rotation is BP in the foot region to that in the head region is
60 s. Assuming the water to be still and no reflections close to
from the surface of the tank, the duration for which (a) one (b) two
the light beam escapes the tank in one time period is (c) three (d) four
close to (Take, refractive index of water = 133
. )

10cm CHEMISTRY
Laser

40cm 71. PbO2 is obtained from


30cm 30cm
(a) the reaction of PbO with HCl
(b) thermal decomposition of Pb(NO3 )2 at 200°C
(c) the reaction of Pb3 O4 with HNO3
(a) 8.13 s (b) 14.05 s
(d) the reaction of Pb with air at room temperature
(c) 16.67 s (d) 23.86 s
https://iit-jeeacademy.blogspot.com

KVPY Question Paper 2019 Stream : SA 7

72. For one mole of a van der Waals’ gas, the 77. Papaya is a dioecious species with XY sexual
 pV  genotype for male and XX for female. What will be
compressibility factor Z =   at a fixed volume
 RT  the genotype of the embryos and endosperm nuclei
will certainly decrease, if after double fertilisation?
(a) 50% ovules would have XXX endosperm and YY
[Given : ‘‘a’’ and ‘‘b’’ are standard parameters for
embryo, while the other 50% would have XXY
van der Waals’ gas] endosperm and XX embryo
(a) ‘‘b’’ increases and ‘‘a’’ decreases at constant (b) 100% ovules would have XXX endosperm and XY
temperature embryo
(b) ‘‘b’’ decreases and ‘‘a’’ increases at constant (c) 100% ovules would have XXY endosperm and XX
temperature embryo
(c) temperature increases at constant ‘‘a’’ and ‘‘b’’ values (d) 50% ovules would have XXX endosperm and XX
(d) ‘‘b’’ increases at constant ‘‘a’’ and temperature embryo, while the other 50% would have XXY
73. The correct statements among the following. endosperm and XY embryo
i. E 2s (H) > E 2s (Li) < E 2s (Na) > E 2s (K). 78. Solid and dotted lines represent the activities of
ii. The maximum number of electrons in the shell pepsin and salivary amylase enzymes of the digestive
with principal quantum number n is equal to tract, respectively. Which one of the following graphs
best represents their activity vs pH?
2n 2 .
iii. Extra stability of half-filled subshell is due to

Activity

Activity
smaller exchange energy.
(a) (b)
iv. Only two electrons, irrespective of their spin,
may exist in the same orbital are.
(a) i and ii (b) ii and iii 1 5 10 1 5 10
pH pH
(c) iii and iv (d) i and iv
Activity

Activity
74. An organic compound contains 46.78% of a halogen
(c) (d)
X. When 2.00 g of this compound is heated with
fuming HNO3 in the presence of AgNO3 , 2.21 g AgX
was formed. The halogen X is 1 5 10 1 5 10
[Given : atomic weight of Ag = 108, F = 19, Cl = 355
., pH pH
Br = 80, I = 127]
(a) F (b) Cl (c) Br (d) I 79. If the gene pool of the locus X in the human genome
75. An organic compound X with molecular formula is 4, then what would be the highest possible number
C6H10, when treated with HBr, forms a gem- of genotypes in a large population?
dibromide. The compound X upon warming with (a) 6 (b) 8
HgSO4 and dil. H2SO4, produces a ketone, which (c) 10 (d) 16
gives a positive iodoform test. The compound X is 80. Match the plant hormones in Column I with their
primary function in Column II.
(a) (b)
Column I Column II
(c) C (d)
P. Abscisic acid i. Promotes disease
resistance
Q. Ethylene ii. Maintains seed dormancy
BIOLOGY R. Cytokinin iii. Promotes seed germination
76. A cell weighing 1 mg grows to double its initial mass S. Gibberellin iv. Promotes fruit ripening
before dividing into two daughter cells of equal mass. v. Inhibits leaf senescence
Assuming no death, at the end of 100 divisions what
will be the ratio of the mass of the entire population Choose the correct combination
of these cells to that of the mass of the earth? Assume (a) P–iii, Q–iv, R–i, S–ii
that mass of the earth is 1024 kg and 210 is (b) P–ii, Q–iv, R–v, S–iii
approximately equal to 1000. (c) P–v, Q–iii, R–ii, S–i
(a) 10−28 (b) 10−3 (d) P–iv, Q–ii, R–iii, S–v
(c) 1 (d) 103
https://iit-jeeacademy.blogspot.com

8 KVPY Question Paper 2019 Stream : SA

Answers
PART-I
1 (c) 2 (c) 3 (b) 4 (a) 5 (a) 6 (b) 7 (a) 8 (c) 9 (a) 10 (d)
11 (c) 12 (c) 13 (d) 14 (a) 15 (b) 16 (a) 17 (a) 18 (b) 19 (c) 20 (c)
21 (d) 22 (d) 23 (a) 24 (c) 25 (a) 26 (d) 27 (a) 28 (b) 29 (d) 30 (c)
31 (a) 32 (d) 33 (a) 34 (d) 35 (d) 36 (c) 37 (b) 38 (b,d) 39 (c) 40 (a)
41 (a) 42 (a) 43 (b) 44 (b) 45 (d) 46 (c) 47 (b) 48 (c) 49 (c) 50 (a)
51 (b) 52 (d) 53 (c) 54 (d) 55 (a) 56 (b) 57 (b) 58 (d) 59 (a) 60 (d)

PART-II
61 (c) 62 (a) 63 (a) 64 (b) 65 (b) 66 (c) 67 (a) 68 (b) 69 (c) 70 (c)
71 (c) 72 (b) 73 (a) 74 (c) 75 (d) 76 (c) 77 (d) 78 (a) 79 (c) 80 (b)

Solutions
1. (c) For an equilateral triangle ABC 8 b3 − 8 4. (a) The sum of first n ,n ≥ 1terms of
⇒ b2 − > 0⇒ >0
having side length a. If R and r are radii b b arithmetic progression with first term 2
of the circumcircle and the incircle of (b − 2)(b2 + 2b + 4) and common difference 4, is
triangle ABC respectively, then ⇒ >0
n
b Sn = [4 + (n − 1)4] = 2n 2
R = sec 30° = 
a a 2  a
 = ⇒ b ∈ (−∞ , 0) ∪ (2, ∞ ) …(i) 2
2 2  3 3 So, the average of the first n terms
For option (c),
S
A b2 − 3b > − 2 Mn = n = 2n
n
⇒ b2 − 3b + 2 > 0 10 10
⇒ (b − 2)(b − 1) > 0 Now, ∑ Mn = 2 ∑ n
n =1 n =1
b ∈ (−∞ , 1) ∪ (2, ∞ )
10 × 11
mean if b ∈ (−∞ , 0) ∪ (2, ∞ ) = 2 ×   = 110
R  2 
r then b2 − 3b > − 2
30° 60° 3. (b) Let the given quadratic polynomial 5. (a) It is given that in triangle ABC,
B
a/ M a/
C p (x) = x2 + ax + b has two distinct real ∠BAC = 90°, AD is the altitude from A on
2 2 to BC.
roots α and β, then
p (x) = x2 + ax + b = (x − α )(x − β ) B

a a 1 a and since g (x) = p (x3 ) = (x3 − α )(x3 − β ) D


and r = tan 30° = × = F
2 2 3 2 3 let α = α31 and β = β31
a then g (x) = (x3 − α31 )(x3 − β31 )
R = (x − α1 )(x − β1 )(x2 + α1 x + α12 )
∴ = 3 = 2, which is independent of a
r a
(x2 + β1 x + β12 ) A
2 3 E C
Q the discriminants of quadratic
and it is constant. equations Since, AB = 15 and BC = 25
2. (c) Given quadratic equation x2 + α1 x + α12 and x2 + β1 x + β12 are ∴ AC = BC 2 − AB 2 = 625 − 225
1
2x + bx + = 0, has two distinct real
2
negative.
b = 400 = 20
∴ g (x) has exactly two distinct real roots 1
roots, so Now, since area of ∆ABC = (BC )(AD )
and since g (x) = x6 + ax3 + b is an even 2
D>0 degree polynomial, so there exists a real 1
⇒  1
b − 4(2)  > 0
2 number ‘α’ such that g (x) ≥ α for all real x. = (AB )(AC )
 b 2
https://iit-jeeacademy.blogspot.com

KVPY Question Paper 2019 Stream : SA 9

1 1 Now,
⇒ (BC )(AD ) = × 15 × 20 12. (c) Given equations
2 2 P (1) = a + b + c + d = 2 …(i) 6x + 4 y + z = 200, …(i)
⇒ 25 × AD = 300 P (2) = 8a + 4b + 2c + d = 4 …(ii) and x + y + z = 100 …(ii)
⇒ AD = 12 P (3) = 27a + 9b + 3c + d = 6 …(iii) By Eqs. (i) and (ii), we get
Q AEDF is a rectangle, then P (4) = 64a + 16b + 4c + d = 8 …(iv) 5x + 3 y = 100
EF = AD = 12
From Eqs. (i) and (ii), we get For non-negative integer solutions, when
6. (b) It is given that the sides of 7a + 3b + c = 2 …(v) x = 2, then y = 30
triangle, a , b and c satisfy the following
relations
From Eqs. (ii) and (iii), we get x = 5, then y = 25
c2 = 2ab …(i) 19a + 5b + c = 2 …(vi) x = 8, then y = 20
and a + c = 3b
2 2 2
…(ii) From Eqs. (iii) and (iv), we get x = 11, then y = 15
From Eqs. (i) and (ii), we get 37a + 7b + c = 2 …(vii) x = 14, then y = 10
A
Now, from Eqs. (v) and (vi), we get x = 17, then y = 5
12a + 2b = 0 …(viii) and x = 20, then y = 0
and from Eqs. (vi) and (vii), we get In every case z = 100 − (x + y) > 0
Ö2a
18a + 2b = 0 …(ix)
So, total number of non-negative integral
a From Eqs. (viii) and (ix), we get solutions are 7.
a = 0 and b = 0,
13. (d) It is given that, N 2 = 165
c = 2 and d = 0.
= 3 × 5 × 11and N1 = 255 + 1
C B So, P (x) = 2x
a As we know that, if n is odd integer then
∴no cubic polynomial is possible.
a 2 + 2ab = 3b2 xn + yn is divisible by x + y.
⇒ a + 2ab + b2 = 4b2
2 10. (d) Since in the group of first 10 two
So, N1 = 255 + 155 is divisible by 2 + 1 = 3
digit number 10-19, has atleast 1 prime
⇒ (a + b)2 = (2b)2 = (b + b)2 and N1 = 255 + 155
number similarly in other groups of 10
⇒ a = b, so c = 2a two digits numbers = (25 )11 + (15 )11 = (32)11 + (1)11
∴ ∠A = ∠B = 45° 20-29, 30-39, 40-49, 50-59, 60-69, 70-79, is divisible by 32 + 1 = 33
7. (a) As N be the least positive integer 80-89 and 90-99 have almost 1 prime ∴the HCF of N1 and N 2 is 33.
and when a non-zero digit C is written numbers. 14. (a) It is given that circumference of
after the last digit of N, the resulting So, the number of almost prime two-digit circle C is l and the perimeter of triangle
number is divisible by C. number is 90. T is l.
So, 10N + C is divisible by C 11. (c) Let a convex quadrilateral ABCD Now, let the radius of circle C is r, so
∴10N must be divisible by C. and K , L, M, N be the mid-point of AB, l
2πr = l ⇒ r =
Now, the least integer (N ) which is BC, CD, DA respectively. 2π
divisible by digit ‘C’ i.e. (1 to 9) must be A l2
K ∴area of circle C is A1 = πr 2 =
L.C.M of {1, 3, 4, 6, 7, 9}. B 4π
x x
= L.C.M of {4, 7, 9} w Now, as we know that area of triangle
y
= 252 = N L N will be maximum for given perimeter if it
y w is an equilateral triangle, let the length
and sum of digits of number ‘N’ is P
2+ 5+ 2= 9 of side of equilateral triangle is ‘a’, then
z z l
8. (c) Let the given 11 distinct positive 3a = l ⇒ a =
3
integers are in increasing order
C M D and area of equilateral triangle is
x1 , x2 , x3 , x4 , x5 , x6 , x7 , x8 , x9 , x10 , x11 , so x11
is largest of these integers and the Now, as area ∆AKP = area ∆BKP = x (let) A2 =
3 2
a
median is x6 . Similarly 4
Now, median of first 10 numbers is ∆BLP = ∆CLP = y 3  l2  l2
So, A2 =  =
x6 + x6 ∆CPM = ∆DPM = z 4  9  12 3
= m (Let).
2 and ∆DNP = ∆ANP = w l2
Now, we have to replace largest number It is given that Area (PKAN ) = x + w = 25 A1
Q = 4π = 3 3 > 1
x11 by m and then increasing order will be π
area (PLBK ) = x + y = 36 A2 l2
x1 , x2 , x3 , x4 , x5 , m, x6 , x7 , x8 , x9 , x10
and area (PMDN ) = z + w = 41 12 3
x + x6
Q m < x6 as x5 < 5 < x6 So area (PLCM ) = y + z Since, as we took an equilateral triangle,
2
= ( x + y ) + ( z + w) − ( x + w) which has maximum area. But we can
So, median decreases. take a triangle T such that the ratio
= area (PLBK ) + area (PMDN ) − area (C)
9. (a) Let the equation of a cubic is greater than any positive real
area (PKAN )
polynomial area (T )
= 36 + 41 − 25 = 77 − 25 = 52
P (x) = ax3 + bx2 + cx + d number α.
https://iit-jeeacademy.blogspot.com

10 KVPY Question Paper 2019 Stream : SA

te
15. (b) It is given that, the number of D(m) Whi
three digit number abc, such that
A B
b+ c R
= bc …(i) x2=0.4 R
2
V
te
the above relation is true if b = c = 0 x1=0.3 Whi V
And if neither b nor c is zero, then
1 1 If bending of light caused by B is less
+ = 2, and b, c ∈{1, 2, 3, 4, 5, 6, 7, 8, 9}
b c than or more than that of A, then out
Then b = c = 1 (s) going beam of light is not white.
1.6 1.9
and a ∈{1, 2, 3, 4, 5, 6, 7, 8, 9} t1 t2 So, when both prisms are filled with
water at different temperatures, their
So, total number of such three digit From data of graph,
refractive indices are different and the
number are 2 × 9 = 18 Terminal velocity,
dispersion produced by A and B are not
16. (a) Formation of rainbow is shown x − x1 0.4 − 0.3
v= 2 = equal and opposite. Hence, with condition
below. t2 − t1 1.9 − 1.6 in (d) beam to right of prism B will be
0.1 coloured.
White light
Refraction = = 0.33 m/s
0.3 22. (d) Time period of rotation of ball
Red
Vio 19. (c) Power requirement for 1 laptop, = 1.5 s
let Total
internal
P1 = 90 W So, in time interval of 7.5 s (= 1.5 × 5) s
reflection So, power requirement for 10 laptops, ball completes 5 revolutions.
P = 10 × P1 = 10 × 90 Also, ball covers one-fourth of circular
et
Refraction = 900 W = 0.9 kW 1.5
ol path in time = 0.375 s.
Vi
d

4
Re

In 5h, electrical energy used by all


42° laptops, So, in remaining 0.8 s (= 8.3 − 7.5s) ball is
40°
E = P × t = 0.9 × 5 very near to other end of diameter as
Observer shown in the figure.
= 4.5 kWh
So, processes involved in formation of
rainbow in correct order are: refraction, Cost of electrical energy used is Position of ball at
total internal reflection, refraction. Cost = E × Unit cost t=0 and at t=7.5 s
Hence, the correct order is given in = 4.5 × 5
option (a). = ` 22.50 nt
me 3s
So, statement III is correct. l a ce t=8.
17. (a) Here, 10 divisions of vernier scall p t
= 11main scale divisions Dis ball a
For laptop charger, input voltage is Position of
11 220 V. of ball at
So, 1 vernier scale division = main
10 So, current when all 10 laptops are t=7.5+0.375
scale divisions connected through an extension, =7.875s
Now, we use formula for least count, P 900 Position
I= = ≈ 4.1A of ball at
Least count = 1main scale division − 1 V 220
t=8.3s
vernier scale division. As, line current exceeds current rating of Clearly, displacement of ball is nearly
⇒ LC = 1MSD − 1VSD fuse, therefore 3A fuse cannot be used. equals to diameter (= 2 m) of circular path.
=  1 −  MSD
11 So, statement II is incorrect.
23. (a) Condition given in question is
 10  20. (c) Frosted glass has a rough layer shown below.
1 which causes irregular refraction and
=− MSD
10 makes glass translucent. Particle
1 When a transparent tape which has separates here
= − × 1mm
321

10 refractive index close to that of glass is R–h


v
= −01
. mm pasted over the rough surface of glass,
the tape glue fills the roughness of glass. R h
So, magnitude of least count is 0.1 mm.
This makes glass surface more smooth
18. (b) Velocity = Slope of distance and so refraction is more regular. This
− Time graph Let v = speed of particle when it
makes region of tape transparent.
Last portion of given graph is a straight separates from hemisphere.
21. (d) Prism B is inverted relative to As there is no friction, loss of potential
line which indicates that velocity is
prism A. So, dispersion of light caused by
constant, i.e. terminal velocity is reached. energy appears in form of kinetic energy
prism A and B is in opposite direction.
of particle.
https://iit-jeeacademy.blogspot.com

KVPY Question Paper 2019 Stream : SA 11

1 2h
∴ mg (R − h ) = mv2 26. (d) Time of fall = 28. (b) Surface area over which rain is
2 anet received, A = 600 km 2
⇒ v = 2 g (R − h ) = 600 × (103 )2 m 2
qE
24. (c) Given, nuclear radius is = 6 × 108 m 2
1
+ E Average rainfall, h = 2.4 m
R = r0 A 3
mg Volume of water received by rain, V
Here, atomic mass number of nucleus = A = A × h = 6 × 108 × 2.4 m3
∴Nuclear density d is given by Water conserved = 10% of volume
Mass number received by rain
d=
Volume Net acceleration of charged masses is 10
= 6 × 108 × × 2.4m3 = 1.44 × 108 m3
A A qE 100
⇒ d= = anet = g −
4 3 1
= 1.4 × 108 × 103 L = 1.4 × 1011 L
πR 4 m
3 π (r0 A 3 )3
3 As, M1 hits the floor before M2. Percentage of total water consumption
A 3 2h 2h received by rain is
⇒ d= = ⇒ >
4 3
πr0 ⋅ A 4 π r03 a1 a2 1.4 × 1011 × 100
= = 10%
3 1 1 1.4 × 1012
As r0 = a constant, so nuclear density is a ⇒ >
a1 a2 29. (d) Collision is elastic, so both linear
constant quantity. momentum and kinetic energy are
⇒ a2 > a1
∴ Nuclear mass density of U238 is same conserved.
as that of Sn119 . When reciprocal is taken in equality sign
is reversed, then. We have following situation,
25. (a) Electrostatic energy of a nucleus

1 2 3
QE QE V
of charge Ze is g − 1 > g− 2 Before M m=0
2 2 M1 M2 collision
kZ e
U1 = ...(i) Q1 E Q2E
R ⇒ − >−

1 2 3
M1 M2 After V¢ v
When this nucleus is divided into two M m
Q1 E Q2E collision
equal nuclei of radius r, then as density ⇒ <
of nuclear matter is a constant, we have M1 M2 According to figure,
initial density = final density Here, multiplication with − 1reverse sign MV = MV ′ + mv ...(i) (linear momentum
M of inequality. conservation)
M
= 2 So,
Q1 Q2
<
1 1
MV 2 = MV ′2 + mv2
1
...(ii)
4 3 4 3
πR πr M1 M2 2 2 2
3 3
or M2Q1 < M1Q2 (kinetic energy conservation)
R3 R
⇒ r3 = or r = 1 ...(ii) ⇒ M1Q2 > M2Q1 From Eqs. (i) and (ii), we get
2
23 M (V − V ′ ) = mv ...(iii)
27. (a) From gas equation,
and M (V 2 − V ′2 ) = mv2 ...(iv)
Now, final electrostatic energy is given by pV = nRT
2 Dividing Eq. (iv) by Eq. (iii), we have
2k   e2
Z Here, n = 1mole
kZ ′ 2e2  2 M (V 2 − V ′2 ) mv2
U2 = 2 × ⇒U 2 = So, pV = RT ...(i) =
r   M (V − V ′ ) mv
Substituting the value of R in Eq. (i), we
 R
 1  get or V + V ′ = v
 23  pV = 8.3 T 30. (c) Total internal, reflection occurs
1
1 kZ 2e2 Clearly, slope of pV versus T line is 8.3, when n ≥ .
[from Eq. (ii)] = 2 ⋅ sin ic
R which is greater than one. Hence,
23 following graph is correct. 4
1
⇒ U 2 = 2 ⋅ U1 pV (J)
3
RT 2 Q
=
23 pV
1
[from Eq. (i)] = 0.63U1 R T 120°
e= V= 3
op p
So, change in electrostatic energy in this Sl 4
process is
∆U = U1 − U 2 (QU1 > U 2 ) 30°
1
= U1 − 0.63U1 = (1 − 0.63) U1 = 0.375U1 e =
Slop P R
kZ 2e2 2
= 0.375 [From Eq. (i)] T (K) 1
R
https://iit-jeeacademy.blogspot.com

12 KVPY Question Paper 2019 Stream : SA

In given situation, angle of incidence of 4π


35. (d) Water-gas shift reaction is ∴Mass = volume × density = g.
each of ray is 30° over face PR. FeO ⋅ Cr2 O3 (Catalyst) 3
CO + H2O  
→
So, i = 30° (Atomic weight of water = 18 )
CO2 + H2
1 1 4π 2π
⇒ = =2 In this reaction, hydrogen gas is ∴ n= =
sin i sin 30° 3 × 18 27
produced from the reaction of steam with
Hence, for total internal reflection at carbon dioxide. 44. (b) Cathode ray is observed only at
surface PR, n ≥ 2 . As refractive index for 36. (c) Temporary hardness (caused by low pressure and high voltage, which
3 and 4 is more than 2, only rays 1 and 2, bicarbonates of calcium or magnesium) travel in straight line in the absence of
pass from face PR while rays 3 and 4 pass can be removed by using lime, Ca(OH)2. electrical and magnetic fields.
through face QR (as shown in diagram). Characteristics of cathode rays are
Ca(HCO3 )2 + Ca(OH)2 → 2CaCO3
31. (a) Hybridisation is determined from independent of the material of electrode
+ 2H2O
the steric number (number of atoms or the gas present in the tube.
37. (b) Among anions with same charge,
bonded to the central atom + the number 45. (d) For a spontaneous process in an
the one having greatest size has
of lone pairs). Number of hybrid orbitals isolated system, the change in entropy is
maximum polarisability. Thus, I− ion
must be equal to the steric number. positive, i.e, ∆S > O.
having most polarisability.
From the Lewis structure. However, if a system is not isolated, the
38. (b, d) Among the orbitals of a entropy change of both the system and
R
multi-electron atom, the one with
:

N== C==O: surroundings are to be taken into account


greatest value of n + l has the greatest
:

(i) Steric number of N-atom = 3 because system and surroundings


energy.
(2 bonded atoms + 1 lone pair), together constitute the isolated system
∴Hybridisation = sp 2 (3 hybrid Between two orbitals with same value of thus, the total entropy change (∆Stotal ) is
orbitals). n + l e.g. options (b) and (d), the one with sum of the change in entropy of the
greater value of n has greater energy. system (∆Ssystem ) and the change in
(ii) Steric number of C-atom = 2
(2 bonded atoms), 39. (c) N2O is a neutral oxide, which is entropy of the surroundings
∴Hybridisation = sp (2 hybrid neither acidic nor basic. (∆Ssurroundings ),
orbitals). 40. (a) Of all the s-block elements, Mg i.e., ∆Stotal = ∆Ssystem + ∆SSurroundings for a
(iii) Steric number of O-atom = 3 and Be salts do not impart colour to spontenceus process, ∆Stotal must be
(1 bonded atom + 2 lone pair) flame. positive, i.e., ∆Stotal is also termed as
∴Hybridisation = sp 2 (3 hybrid ∆Suniverse.
41. (a)
orbitals). 46. (c) The correct statement for
For a reaction
32. (d) One isomer is an alkyne and the primates’ evolution is that human,
X → Y , chimpanzees and gorillas share a
other one is an alkadiene. Since, they
− d[X ]
have two different functional groups, rate = ; [X ] = concentration of X. common ancestor. From fossil records,
they are functional group isomers. dt primatologists came to know that human,
If reaction is nth order, chimpanzee and gorilla are evolved from
33. (a)
rate ∝ [X ]n a common ancient ancestor about 10
Br million years ago. Recent studies on
1.Excess alcoholic KOH Br d[X ]
Br
–HBr Ph From the graph, the slope is gorilla genome confirmed that gorilla
Ph dt
diverged from the common ancestor
2.NaNH2 + 3.H3O
+ constant.
Br Ph—CºC Na Ph—CºCH about 6 million years ago.
Ph –HBr ∴Rate is constant at any concentration.
–2NH3 or PhºH 47. (b) The crypts of Lieberkuhn are
∴ n=0
found in small intestine. Crypts are
42. (a) In a free expansion, external invagination of the epithelium around
34. (d) O pressure ( pex ) = 0 the villi and lined largely with younger
1 ∴ W = − pex ⋅ ∆V = 0 epithelial cell which are involved in
6 2 secretion of mucus.
and the system is isolated.
3
HO 5 Heat does not enter or leave, q = 0. 48. (c) Removal of pancreas impairs the
4 breakdown of lipids, proteins and
Principal functional group is ketone. ∆U = q + W = 0, whereU = internal
energy. carbohydrates because pancreas produces
∴C1 is carbonyl carbon atom. insulin and other important enzyme like
Locants for hydroxyl groups and double 43. (b) Number of moles, trypsin, chymotrypsin, amylase and
mass (m)
bonds are 5 and 2, which are preferred n= lipase which helps in breakdown of
over 3 and 5, since the lower number at molar mass (M ) macromolecules.
first difference (2 compared to 3) is Given, radius = 1.0 cm, 49. (c) Microscopic examination of blood
preferred. 4π
∴volume = cm3 smear reveals an abnormal increase in
Hence, the IUPAC name of given 3 neutrophils. Neutrophils have a
compound is 5-hydroxycyclohex-2-en- Given, density = 1.0 g cm − 3 , multilobed nucleus and granulated
1-one. cytoplasm.
https://iit-jeeacademy.blogspot.com

KVPY Question Paper 2019 Stream : SA 13

Their number increases in blood in deals with identification, nomenclature 60. (d) The second meiotic division
response of bacterial infection, acute and classification. Carlous Linnaeus occurs after fertilisation. Oogenesis is the
inflammation and Eclampsia. invented binomial nomenclature and formation of female gametes (egg).
Neutrophils are produced by developed a classification system known Oogenesis begins in female before birth.
hematopoiesis in the bone marrow and as taxonomic hierarchy. The various During early fetal development, germ cell
are active phagocytic cells. units of classification is kingdom, differentiate into oogonia. After several
Lymphocytes are white blood cells phylum, class, order, family, genus and mitotic divisions, oogonia begins meiosis
which occurs in blood, lymph and species. and known as primary oocytes. It
lymphoid organs. 56. (b) Kidneys are not associated with remains arrested after diplotene of
Monocytes are mononuclear phagocytic the production of white blood cells. prophase-I of meiosis-I until the female
cells. Kidneys regulate blood volume and becomes sexually mature. After puberty,
composition, release erythropoietin and primary oocyte completes meiosis-I and
Platelets are known as thrombocytes
excrete waste in the urine. produces secondary oocytes and it arrests
and helps in blood clotting.
Bone marrow is involved in at metaphase-II and it completes meiosis
50. (a) Blood group AB represents -II only after fertilisation.
codominance. In codominance a hematopoiesis. It is the site of
B-lymphocytes synthesis and maturation. 61. (c) It is given that the quadratic
heterozygous individual expresses both
Liver produces monocytes (a type of equation
alleles simultaneously with blending. No
single allele is dominant over the other. white blood cells). x2 − 5cx − 6d = 0 has roots a and b, then
Expression of both A and B alleles at In spleen, B and T-lymphocytes are a + b = 5c …(i)
same time results in AB type blood. present. 50% of spleen cells are and ab = − 6d …(ii)
51. (b) Allopatric speciation is a genetic B-lymphocytes and 30-40% are and, the quadratic equation
T-lymphocytes.
divergence permitted by geographical x2 − 5ax − 6b = 0 has roots c and d, then
isolation. It is a speciation that occurs 57. (b) Hydathodes are involved in c + d = 5a …(iii)
when population of the same species guttation. Hydathodes are specialised
and cd = − 6b …(iv)
becomes isolated due to geographic pore located along the leaf margins and
barriers such as mountain ranges and tip which secrets water droplets. The Now, from Eqs. (i) and (iii), we have
water bodies. The population is exudation of water droplets from the tip (a + b) − (c + d ) = 5c − 5a
reproductively isolated and each of the or margin of the leaves is called ⇒ (a − c) + (b − d ) = − 5(a − c)
population accumulates different guttation. Hydathodes mediated ⇒ (b − d ) = 6(c − a ) …(v)
mutation and become diverge. guttation occurs under high humidity
∴a and c are the roots of equations.
52. (d) Conversion of glucose to CO2 and and in the absence of transpiration.
x2 − 5cx − 6d = 0 and x2 − 5ax − 6b = 0,
H2O requires oxygen. In aerobic Cuticle is an extracellular layer which
covers the epidermis of plants which respectively.
respiration glucose reacts with oxygen
forming ATP, carbon dioxide and water provides protection against dessication ∴ a 2 − 5ac − 6d = 0
are released as byproducts. and external environmental stress. and c2 − 5ac − 6b = 0
C6 H12O6 + 6O2 → 6CO2 + 6H2O + ATP Lenticels and stomata both regulates ⇒ (a 2 − c2 ) − 6(d − b) = 0
gaseous exchange between internal plant 6(d − b)
53. (c) Proxima and distal convoluted ⇒ a+ c= = 36 …(vi)
tissues and atmosphere and also regulates a−c
tubules are located in renal cortex.
water movement through transpiration.
Convoluted means the tubules one tightly From Eqs. (i) and (iii), we have
coiled. Proximal convoluted tubules are 58. (d) Cataract affect the lens in eye. It
(a + b) + (c + d ) = 5(a + c)
associated with the reabsorption of occurs due to the clouding of lens and
⇒ b + d = 4(a + c) = 4(36) [from Eq. (vi)]
filtered water, Na + , K+ . glucose, amino prevent light and image from reaching to
acid, Cl − , HCO3− , Ca 2 + , Mg 2 + and retina. Cataract makes a person vision ⇒ b + d = 144
secretion of H+ , NH+4 , urea whereas distal blurry and less colourful. 62. (a) The quadratic equation
convoluted tubules are associated with 59. (a) Liverwort These are 4x2 + bx + c = 0
reabsorption of water, Na + , Cl − and Ca 2 + . non-vascular plants and one of the three has equal roots if b2 − 16c = 0
54. (d) When one gene masks or ancient lines of bryophytes (liverworts,
⇒ b2 = 24 c
modifies the expression of another gene hornworts and mosses).
Now,c should be chosen from the set
at distinct locus is known as epistasis. Volvox It is a spherical multicellular
Gene that masks other or expresses itself green algae and used as a genetic model S = {1, 2, 3, … , 100}, such that it is a
is epistatic gene and gene that is masked of morphogenesis. perfect square number, so
is hypostatic gene. Here, X is inactivated Chlamydomonas It is a genus of c = 1, 4, 9, 16, 25, 36, 49, 64, 81, 100
by Y and triggers its own expression that unicellular green algae found in soil, ∴number of ordered pair (b, c) will be 10.
means Y is epistatic to loci X because it freshwater and oceans. 10
So, required probability =
masks the expression of X. Fern These are vascular plants that 100 × 100
55. (a) The correct taxonomic hierarchy possess true roots, leaves and stem and 1
= = 0.001
is species, genus, family, order. are reproduced by spores. Ferns and 1000
Taxonomy is the branch of biology that lycophytes are pteridophytes.
https://iit-jeeacademy.blogspot.com

14 KVPY Question Paper 2019 Stream : SA

63. (a) It is given that for n ∈ N ∴XB = x2 ⋅ AB 67. (a) Force = Rate of change of
fn = (n + 1)1/3 − n1/3 Q AB = AX + XB = x ⋅ AB + x2 ⋅ AB momentum
(n + 1) − n ⇒ x2 + x − 1 = 0
=
(n + 1)23
/
+ (n + 1)23
/
n 23
/
+ n 23
/ −1 ± 1 + 4 ± 5 − 1
⇒ x= =
1 2 2
=
(n + 1)23
/
+ (n + 1)23
/
n 23
/
+ n 23
/ 5−1
Q x > 0, so x =
2
Q∀ n ∈ N
AB 2 + BC 2 − AC 2
3n 23
/
< (n + 1)23
/
+ (n + 1)23
/
n 23
/
+ n 23
/
Now, cosθ =
2(AB )(BC )
< 3(n + 1)23
/

1 2(AB 2 ) − (AX 2 )
⇒ ⇒ cosθ = [Q AB = BC ]
3(n + 1) 23
/
2(AB 2 ) Velocity with which a sand particle
1 1 strikes the bottom of hour-glass,
< < 2(AB ) − (x ⋅ AB )
2 2 2
= v = u + gt
(n + 1)23
/
+ (n + 1)23
/
n 23
/
+ n 23
/
3n 23
/
2(AB 2 )
1 1 ⇒ v = 0 + 10 × 2 = 20 ms − 1
⇒ < fn < 2− x 2
3(n + 1)23
/
3n 23/ = Change in momentum of particle
2 = pf − pi = 0 − mv
Similarly, 2
= − 0.2 × 10− 3 × 20
 5 − 1
1
< fn + 1 <
1 2− 
 2  = − 4 × 10− 3 kg-ms − 1
3(n + 2)23
/
3(n + 1)23
/
=
2 Momentum imparted to base by the
1
∴ fn + 1 < < fn + 1 , ∀ n ∈ N particle = 4 × 10− 3 kg-ms − 1
3(n + 1) 23
/ 8 − (5 + 1 − 2 5 )
=
8 Total change of momentum imparted per
So, set A = N . second by all 100 particles
2 5+ 2
64. (b) It is given that for prime = = 4 × 10− 3 kg-ms−1 × 100 s− 1
8
numbers p1 , p2 , p3 , p4 the special prime = 0.4 kg-ms−1
5+1
number = = cos 36° So, force on bottom = 0.4 N
p = p1 + p2 = p3 − p4 4
So, θ = ∠ABC = 36°
68. (b) When a voltmeter put in series, it
Case I still reads potential drop and when an
If all p1 , p2 , p3 , p4 are odd, then ( p1 + p2 ) 66. (c) When angle of incidence of laser ammeter is connected in parallel, it still
and ( p3 − p4 ) are even, which is not on surface of water is less than critical shows current through it.
possible. incidence, it goes out otherwise reflected
Case a
back into the tank.
Case II B
If one of p1 and p2 is even, say p2 is 2 and Laser comes out from V
p4 must be 2. a path of 2 ic rotation
So, p = p1 + 2 = p3 − 2
1W A
the above equation is satisfied only if ic
ic ic
p = 5, p1 = 3 and p3 = 7 I I2
I1
So, the number of special prime p is 1.
65. (b) It is given that in ∆ABC, A
AB = BC Let I = current through cell, then
For water, ic = sin − 1   = sin − 1 
A 1 1  potential drop read by voltmeter is

 n  1.33  V = I ⋅ RV (this is reading of voltmeter)
⇒ ic = sin − 1 (0.75) Where, RV is the resistance of voltmeter
X ⇒ ic ≈ 50° In loop AB,
If ω = angular speed and t = time to travel V AB = I1 × 1 = I 2 × RA and I = I1 + I 2
q an arc of 2 ic , then using ωt = 2 ic . Where, RA is the resistance of ammeter
B C 2i
We have, t = c We substitute for I1 from above equation
AX AB 1 ω to get
and = = (say) 50
XB AX x 2× × π ⇒ I = I 2RA + I 2 = I 2 (RA + 1)
⇒ AX = x ⋅ AB = 180 = 16.67 s I
 2π  ⇒ I2 =
and XB = xAX   (RA + 1)
 60 
(this is reading of ammeter)
https://iit-jeeacademy.blogspot.com

KVPY Question Paper 2019 Stream : SA 15

Now given, where ρo = density of outside air van der Waals’ gas, the actual pressure
voltmeter reading IRV and ρi = density of inside air. and volume are:
= 1 × 103 =
ammeter reading  I  ⇒ V (ρo − ρi ) = 210 an 2
  p = p0 − 2 , V = V 0 + nb
 RA + 1 210 × 3  4 3  V
⇒ ρo − ρi = QV = πr 
So, RV (RA + 1) = 1000 ...(i) 4 πr3  3  ∴As “a” increases, p decreases, so
pV
RT
Case b PM PM 210 × 3 1 1
⇒ − = 3
⇒ − decreases, when temperature is constant.
RTo RTi To Ti
4 × π × 
B 11.7  pV
 and, as “b” decreases, V decreases, so
A  2  RT
680 × 8 × 8.31 decreases, when temperature is constant.
=
1W
4 × π × (11.7)3 × 105 × 30 × 10− 3 Note option (c) : When temperature
V
T − To 1 increases, pV also increases and
⇒ i =
I
I1 I2 ToTi 1387 therefore Z would not necessarily
decrease.
⇒ ToTi = 1387 (Ti − To )
A 73. (a) (i) Energy of the 2sorbital of
⇒ 300 Ti = 1387 Ti − 300 × 1387
Let I = current through cell, then different elements decreases as nuclear
ammeter reading in this case is I. (as, To = 27°C = 300 K) charge (equal to atomic number) of atom
300 × 1387
So, Ti = ≈ 383 K increases.
1087 (ii) There are n 2 orbitals in a shell with
Also, in loop AB,
∴ Ti = 383 − 273 = 110° C principal quantum number n.
V AB = I1 × 1 = I 2 × RV
So, temperature of hot air is near to 105° C. ∴total number of electrons = 2n 2.
As, I = I1 + I 2 = I 2RV + I 2
70. (c) (iii) Extra stability of half-filled orbitals
= I 2 (RV + 1)
is due to greater exchange energy.
I
So, I 2 = Head
(iv) For two electrons will be in the same
(RV + 1)
orbital, their spin quantum numbers
0.4m
Hence, voltmeter reading is V = I 2RV must be different.
IRV 1.7m
= (this is reading of voltmeter) It is not irrespective of their spin.
(RV + 1)
Heart 74. (c) Mass of AgX = 2.21g
Now given, voltmeter reading ÷ ammeter
reading = 0.999 Ω. Mass of X = 46.78% of 2.00 g
46.78 × 2.00
 IRV  1.3m =
 (R + 1)  100
So, 0.999 =  V 
Foot ≅ 0.94 g
I
∴Mass of Ag in AgX must be
RV Pressure at head level = pheart − ρgh
⇒ 0.999 = 2.21 − 0.94 g
RV + 1 = 13.3 − 103 × 10 × 0.4 = 1.27 g
So, RV = 999Ω ...(ii) = 9.3 kPa 1.27
∴Number of moles of Ag = ,
≈ 103 Ω Pressure at foot level = pheart + ρgh 108
Substituting RV in Eq (i), we get = 13.3 + 103 × 10 × 1.3 which is also equal to number of moles
1 = 26.3 kPa of X.
RA =
999 26.3 mass
So, ratio = ≈ 2.9 or 3 ∴Atomic mass of X =
or RA = 10− 3 Ω 9.3 'n '
0.94 × 108
69. (c) Hot air balloon will rise in the 71. (c) (a) PbO + HCl → PbCl 2 + H2O = ≅ 80
atmosphere when upthrust of buoyant 1.27
(not correct option)
force is greater than weight of balloon ∴The halogen must be bromine (Br).
(b) 2Pb(NO3 )2 → 2PbO + 4NO2 + O2
and its payload. 75. (d)
(not correct option)
Upthrust = Weight of atmospheric air Br Br
(c) Pb 3 O4 + 4HNO3 → 2Pb(NO3 )2
displaced by balloon + PbO2 + 2H 2 O (correct option) + 2HBr
So, upthrust ≥ weight of balloon and its (d) Pb + air (contains O2 , H2O and CO2)
payload room
→ (Hydrobromination, Markownikoff’s
⇒ (Volume of air displaced × density of temperature product, which is a gem-dibromide).
atmospheric air × Acceleration due to Protective layer of varying composition, O
gravity) ≥ (Volume of air of inside balloon mainly PbCO3 is formed only on the + H+
× density of air inside balloon × surface. (not correct option)
+ H2 O
+ Hg2+
acceleration due to gravity) + (Weight of X
payload of balloon) 72. (b) If p0 and V 0 are used as notation (dil. H2SO4
+ HgSO4)
⇒V ⋅ ρo ⋅ g ≥ V ⋅ ρi ⋅ g + 210 × g of ideal pressure and ideal volume of a
https://iit-jeeacademy.blogspot.com

16 KVPY Question Paper 2019 Stream : SA

Acid catalysed hydration of alkyne gives 1024 Graph (c) and (d) represents constant
= =1
ketone. In the case of terminal alkyne, 1024 activity of both enzyme. At low pH
the product is a methyl ketone, which So, option (c) is correct activity of pepsin increases and become
gives haloform test. stable as pH is increasing and activity of
O
77. (d) In papaya, sexual genotype for amylase increases at above pH 5 and
O
male is XY and for female is XX. In become stable at high pH.
I2 + NaOH double fertilisation, the X nuclei fuses
ONa
with egg and polar nuclei then resulting 79. (c) Gene pool of locus X = 4
+ CHI3 + other n
genotype of embryo and endosperm is XX Possible genotype = (n + 1)
products
and XXX. When Y nuclei fuses with the 2
(NaI+H2O)
egg cell and polar nuclei then resulting n = Total number of gene for ‘X’ loucs
Yellow product genotype for embryo and endosperm is n=4
(positive haloform test) XY and XXY. n
= (n + 1)
76. (c) Mass of one cell = 1 mg = 10− 6 kg So, 50% XXX and XXY is genotype of 2
Division in the cell is calculated as 2n endosperm and 50% XX and XY is 4
genotype of embryo. = (4 + 1) = 2(5) = 10
So, after 100 divisions, 2
Number of cells = 2n = 2100 78. (a) Graph (a) represents the activity So, highest possible genotype in a
of pepsin at low pH and salivary amylase population is 10.
Total mass of cells = Total no. of cells ×
activity at high pH. Enzymes have a
Mass of one cell
particular pH where they have the proper
80. (b) The correct combination of plant
= 2100 × D 10− 6 kg conformation to have maximum catalytic
hormones with their function is as follows
= 210 = 103 activity. Pepsin have maximum catalytic (P) Abscisic acid—Maintains seed
= (103 )10 × 10− 6 kg activity at a very low pH (2.0) and no dormancy
= 1030 × 10− 6 kg longer functional once moved to alkaline (Q) Ethylene—Promotes fruit ripening
condition and optimum pH for salivary (R) Cytokinin—Inhibits leaf senescence
= 10+ 24 kg
amylase ranges from 6 to 7 and it is most (S) Gibberellin—Promotes seed
Mass of earth is 1024 kg active at pH 6.8.
Total mass of cells germination
Ratio = Graph (b) represents minimum activity of
Mass of earth pepsin and salivary amylase.
https://iit-jeeacademy.blogspot.com

KVPY Question Paper 2018 Stream : SA 1

KVPY
KISHORE VAIGYANIK PROTSAHAN YOJANA

QUESTION PAPER 2018


Stream : SA
MM 100

Instructions
There are 80 questions in this paper.
This question paper contains two parts; Part I and Part II. There are four sections; Mathematics, Physics, Chemistry
and Biology in each part.
Out of the four options given with each question, only one is correct.

PART-I (1 Mark Questions)


MATHEMATICS Then
(a) both I and II are true (b) both I and II are false
1. The number of pairs (a , b) of positive real numbers
(c) I is true and II is false (d) I is false and II is true
satisfying a 4 + b4 < 1 and a 2 + b2 > 1 is
5. The number of polynomials p(x) with integer
(a) 0 (b) 1
coefficients such that curve y = p(x) passes through
(c) 2 (d) More than 2
(2, 2) and (4, 5) is
2. The number of real roots of the polynomial equation (a) 0 (b) 1
x4 − x2 + 2x − 1 = 0 is (c) more than 1 but finite (d) infinite
(a) 0 (b) 2 6. The median of all 4-digit numbers that are divisible
(c) 3 (d) 4 by 7 is
3. Suppose the sum of the first m terms of an arithmetic (a) 5797 (b) 5498.5 (c) 5499.5 (d) 5490
progression is n and the sum of its first n terms is m, 7. A solid hemisphere is attached to the top of a
where m ≠ n. Then, the sum of the first (m + n ) terms cylinder, having the same radius as that of the
of the arithmetic progression is cylinder. If the height of the cylinder were doubled
(a) 1 − mn (b) mn − 5 (keeping both radii fixed), the volume of the entire
(c) − (m + n ) (d) m + n system would have increased by 50%. By what
4. Consider the following two statements percentage would the volume have increased if the
radii of the hemisphere and the cylinder were
I. Any pair of consistent liner equations in two
doubled (keeping the height fixed)?
variables must have a unique solution.
(a) 300% (b) 400%
II. There do not exist two consecutive integers, the (c) 500% (d) 600%
sum of whose squares is 365.
https://iit-jeeacademy.blogspot.com

2 KVPY Question Paper 2018 Stream : SA

8. Consider a ∆PQR in which the relation


PHYSICS
QR2 + PR2 = 5 PQ 2 holds. Let G be the point of
intersection of medians PM and QN. Then, ∠QGM is 16. A block of wood is floating on water at 0°C with
always volume V0 above water. When the temperature of
water increases from 0 to 10°C, the change in the
(a) less than 45° (b) obtuse
volume of the block that is above water is best
(c) a right angle (d) acute and larger than 45°
described schematically by the graph.
9. Let a , b, c be the side-lengths of a triangle and l, m, n
l+m+ n (a) (b)
be the lengths of its medians. Put K = . V0 V0
a+ b+ c
Then, as a , b, c vary, K can assume every value in the
interval 10°C 10°C
(a)  ,  (b)  ,
1 2 1 4 0°C 0°C

 4 3 2 5 (c) (d)
(c)  , 1 (d)  ,
3 4 5
 V0 V0
4  5 4
10. Let x0 , y0 be fixed real numbers such that x02 + y02 > 1.
10°C 10°C
If x, y are arbitrary real numbers such that 0°C 0°C
x2 + y2 ≤ 1, then the minimum value of
17. A very large block of ice of the size of a volleyball
(x − x0 )2 + ( y − y0 )2 is
court and of uniform thickness of 8 m is floating on
(a) ( x02 + y02 − 1)2 (b) x02 + y02 − 1 water. A person standing near its edge wishes to
(c) (|x0| + |y0| − 1)2 (d) (|x0| + |y0|)2 − 1 fetch a bucketful of water using a rope. The smallest
length of rope required for this is about
11. Let PQR be a triangle is which PQ = 3. From the (a) 3.6 m (b) 1.8 m (c) 0.9 m (d) 0.4 m
vertex R, draw the altitude RS to meet PQ at S.
Assume that RS = 3 and PS = QR. Then, PR equals
18. A box filled with water has a small hole on its side
near the bottom. It is dropped from the top of a
(a) 5 (b) 6
tower. As it falls, a camera attached on the side of
(c) 7 (d) 8
the box records the shape of the water stream coming
12. A 100 mark examination was administered to a class out of the hole. The resulting video will show
of 50 students. Despite only integer marks being (a) the water coming down forming a parabolic stream
given, the average score of the class was 47.5. Then, (b) the water going up forming a parabolic stream
the maximum number of students who could get (c) the water coming out in a straight line
marks more than the class average is (d) no water coming out
(a) 25 (b) 35
(c) 45 (d) 49
19. An earthen pitcher used in summer cools water in it
essentially by evaporation of water from its porous
13. Let S be the sum of the digits of the number 152 × 518 surface. If a pitcher carries 4 kg of water and the rate
in base 10. Then, of evaporation is 20 g per hour, temperature of water
(a) S < 6 (b) 6 ≤ S < 140 in it decreases by ∆T in two hours. The value of ∆T is
(c) 140 ≤ S < 148 (d) S ≥ 148 close to (ratio of latent of evaporation to specific heat
of water is 540°C)
14. Let PQR be an acute-angled triangle in which
(a) 2.7°C (b) 4.2°C (c) 5.4°C (d) 10.8°C
PQ < QR. From the vertex Q draw the altitude QQ1,
the angle bisector QQ2 and the median QQ3 , with 20. Two plane mirrors are kept on a horizontal table
Q1, Q2, Q3 lying on PR. Then, making an angle θ with each other as shown
(a) PQ1 < PQ2 < PQ3 schematically in the figure. The angle θ is such that
any ray of light reflected after striking both the
(b) PQ2 < PQ1 < PQ3
mirrors returns parallel to its incident path. For this
(c) PQ1 < PQ3 < PQ2
to happen, the value of θ should be
(d)PQ3 < PQ1 < PQ2
15. All the vertices of a rectangle are of the form (a , b)
with a , b integers satisfying the equation
(a − 8)2 − (b − 7)2 = 5 . Then, the perimeter of the θ

rectangle is
(a) 20 (b) 22
(c) 24 (d) 26 (a) 30° (b) 45° (c) 60° (d) 90°
KVPY Question Paper 2018 Stream : SA 3

21. A certain liquid has a melting point of −50°C and a 27. Select the correct statement about rainbow.
boiling point of 150°C. A thermometer is designed (a) We can see a rainbow in the western sky in the late
with this liquid and its melting and boiling points are afternoon
designated at 0°L and 100°L. The melting and boiling (b) The double rainbow has red on the inside and violet on
points of water on this scale are the outside
(a) 25°L and 75°L, respectively (c) A rainbow has an arc shape, since the earth is round
(b) 0°L and 100°L, respectively (d) A rainbow on the moon is violet on the inside and red
on the outside
(c) 20°L and 70°L, respectively
(d) 30°L and 80°L, respectively 28. Remote sensing satellites move in an orbit that is at
an average height of about 500 km from the surface
22. One can define an alpha-volt (α-V ) to be the energy
of the earth. The camera onboard one such satellite
acquired by an α-particle when it is accelerated by a
has a screen of area A on which the images captured
potential of 1 V. For this problem, you may take a
by it are formed. If the focal length of the camera lens
proton to be 2000 times heavier than an electron.
is 50 cm, then the terrestrial area that can be
Then,
observed from the satellite is close to
(a) 1 α- V = 1eV/4000 (b) 1 α-V = 2 eV
(a) 2 × 10 3 A (b) 106 A (c) 1012 A (d) 4 × 1012 A
(c) 1 α-V = 8000 eV (d) 1 α-V = 1eV
23. In a particle accelerator, a current of 500 µA is 29. Letters A, B, C and D are written on a cardboard as
shown in the figure below.
carried by a proton beam in which each proton has a
speed of 3 × 107 m/s. The cross-sectional area of the A
beam is 1.50 mm2. The charge density in this beam
(in C/m3 ) is close to D B
(a) 10−8 (b) 10−7 (c) 10−6 (d) 10−5 C
24. Which of the following is not true about the total The cardboard is kept at a suitable distance behind a
lunar eclipse? transparent empty glass of cylindrical shape. If the
(a) A lunar eclipse can occur on a new moon and full moon glass is now filled with water, one sees an inverted
day image of the pattern on the cardboard when looking
(b) The lunar eclipse would occur roughly every month, if through the glass. Ignoring magnification effects, the
the orbits of earth and moon were perfectly coplanar image would appear as
(c) The moon appears red during the eclipse because the
blue light is absorbed in earth’s atmosphere and red is (a) A (b) A
transmitted
(d) A lunar eclipse can occur only on a full moon day B D D B

25. Many exoplanets have been discovered by the transit C C


method, where in one monitors, a dip in the intensity
(c) C (d) A
of the parent star as the exoplanet moves in front of
it. The exoplanet has a radius R and the parent star B D B D
has radius 100 R. If I 0 is the intensity observed on A
C
earth due to the parent star, then as the exoplanet
transits 30. If a ball is thrown at a velocity of 45 m/s in vertical
(a) the minimum observed intensity of the parent star is upward direction, then what would be the velocity
0.9 I 0 profile as function of height? (Assume, g = 10 m/s 2)
(b) the minimum observed intensity of the parent star is
0.99 I 0 (a) (b)
(c) the minimum observed intensity of the parent star is 45 45
0.999 I 0
(d) the minimum observed intensity of the parent star is v(m/s) v(m/s)
0.9999 I 0 0 0
0 Height 101 0 Height 101
26. A steady current I is set up in a wire whose (m) (m)
cross-sectional area decreases in the direction of the
(c) (d)
flow of the current. Then, as we examine the
narrowing region, 45 45
(a) the current density decreases in value v(m/s) v(m/s)
(b) the magnitude of the electric field increases
0 0
(c) the current density remains constant 0 Height 101 0 Height 101
(d) the average speed of the moving charges remains (m) (m)
constant
https://iit-jeeacademy.blogspot.com

4 KVPY Question Paper 2018 Stream : SA

37. The correct statement about the following compounds


CHEMISTRY
31. The number of water molecules in 250 mL of water is
closest to Br Br
[Given, density of water is 1.0 g mL−1; Avogadro’s X Y
number = 6023
. × 1023 ]
is
(a) 83.6 × 1023 (b) 13.9 × 1023
(d) 33.6 × 1023 (a) Both are chiral
(c) 1.5 × 1023
(b) Both are achiral
32. Among the following, the correct statement is (a) (c) X is chiral and Y is achiral
pH decreases when solid ammonium chloride is added
(d) X is achiral and Y is chiral
to a dilute aqueous solution of NH3
(b) pH decreases when solid sodium acetate is added to a 38. The most acidic proton and the strongest nucleophilic
dilute aqueous solution of acetic acid nitrogen in the following compound
(c) pH decreases when solid NaCl is added to a dilute
O
aqueous solution of NaOH
CH3
(d) pH decreases when solid sodium oxalate is added to a b c
dilute aqueous solution of oxalic acid N N
a H H
33. The solubility of BaSO4 in pure water (in g L −1) is N
H
closest to
10
. × 10− at 25°C. Molecular
[Given; Ksp for BaSO4 is 10 respectively, are
−1 (a) N a − H; Nb (b) Nb − H; N c
weight of BaSO4 is 233 g mol ] (c) N a − H; N c (d) N c − H; N a
. × 10−5
(a) 10
39. The chlorine atom of the following compound
. × 10−3
(b) 10
(c) 2. 3 × 10−5
c
Cl
(d) 2. 3 × 10−3 b
Cl d Cl
34. Among the following, the incorrect statement is
(a) no two electrons in an atom can have the same set of Cl
a
four quantum numbers O
(b) the maximum number of electrons in the shell with
principal quantum number, n is equal to n 2 + 2 that reacts most readily with AgNO3 to give a
(c) electrons in an orbital must have opposite spin precipitate is
(d) in the ground state, atomic orbitals are filled in the (a) Cl a (b) Clb (c) Cl c (d) Cl d
order of their increasing energies 40. Among the following sets, the most stable ionic
35. A container of volume 2.24 L can with stand a species are
maximum pressure of 2 atm at 298 K before + – + +

exploding. The maximum amount of nitrogen (in g) and


(a) and (b)
that can be safely put in this container at this
+
temperature is closest to –
– –

(a) 2.8 (b) 5.6 (d) and


(c) and
(c) 1.4 (d) 4.2
36. The compound shown below
41. The correct order of energy of 2s-orbitals in H, Li, Na
O NO2
and K, is
(a) K < Na < Li < H (b) Na < Li < K < H
(c) Na < K < H < Li (d) H < Na < Li < K
42. The hybridisation of xenon atom in XeF4 is
can be readily prepared by Friedel-Craft’s reaction (a) sp3 (b) dsp 2
between (c) sp3 d 2 (d) d 2sp3
(a) benzene and 2-nitrobenzoyl chloride 43. The formal oxidation numbers of Cr and Cl in the
(b) benzyl chloride and nitrobenzene ions Cr2O72− and ClO3− , respectively are
(c) nitrobenzene and benzoyl chloride
(d) benzene and 2-nitrobenzyl chloride (a) + 6 and +7 (b) +7 and +5
(c) +6 and +5 (d) +8 and +7
https://iit-jeeacademy.blogspot.com

KVPY Question Paper 2018 Stream : SA 5

44. A filter paper soaked in salt X turns brown when 53. Which one of the following proteins does not play a
exposed to HNO3 vapor. The salt X is role in skeletal muscle contraction?
(a) KCl (b) KBr (c) KI (d) K2SO4 (a) Actin (b) Myosin
45. The role of haemoglobin is to (c) Troponin (d) Microtubule
(a) store oxygen in muscles 54. Which one of the following reactions is catalysed by
(b) transport oxygen to different parts of the body high-energy ultraviolet radiation in the stratosphere?
(c) convert CO to CO2 (a) O2 + O → O3 (b) O2 → O + O
(d) convert CO2 into carbonic acid (c) O3 + O3 → 3O2 (d) O + O → O2
55. Which one of the following statements is true about
BIOLOGY trypsinogen?
(a) It is activated by enterokinase
46. Which one of the following molecules is a secondary (b) It is activated by renin
metabolite?
(c) It is activated by pepsin
(a) Ethanol (b) Lactate
(c) Penicillin (d) Citric acid (d) It does not need activation

47. Lecithin is a 56. Which one of the following organisms respires


(a) carbohydrate (b) phospholipid through the skin?
(c) nucleoside (d) protein (a) Blue whale (b) Salamander
(c) Platypus (d) Peacock
48. The water potential (ψ p ) of pure water at standard
temperature and atmospheric pressure is 57. Which one of the following human cells lacks a
(a) 0 (b) 0.5 (c) 1.0 (d) 2.0 nucleus?
(a) Neutrophil (b) Neuron
49. Action potential in neurons is generated by a rapid
(c) Mature erythrocyte (d) Keratinocyte
influx of
(a) chloride ions (b) potassium ions 58. The first enzyme that the food encounters in human
(c) calcium ions (d) sodium ions digestive system is
(a) pepsin (b) trypsin
50. Erythropoietin is produced by
(a) heart (b) kidney (c) chymotrypsin (d) amylase
(c) bone marrow (d) adrenal gland 59. Glycoproteins are formed in which one of the
51. Tendrils are modifications of following organelles?
(a) stem or leaf (b) stem only (a) Peroxisome (b) Lysosome
(c) leaf only (d) aerial roots only (c) Golgi apparatus (d) Mitochondria
52. Which one of the following combinations of 60. An example of nastic movement (external
biomolecules is present in the ribosomes? stimulus-dependent movement) in plants is
(a) RNA, DNA and protein (a) folding up of the leaves of Mimosa pudica
(b) RNA, lipids and DNA (b) climbing of tendrils
(c) RNA and protein (c) growth of roots from seeds
(d) RNA and DNA (d) growth of pollen tube towards the ovule

PART-II (2 Marks Questions)


MATHEMATICS (a) 9 (b) 12 (c) 15 (d) 18
61. What is the sum of all natural numbers n such that 63. The number of solid cones with integer radius and
the product of the digits of n (in base 10) is equal to integer height each having its volume numerically
n 2 − 10n − 36? equal to its total surface area is
(a) 12 (b) 13 (c) 124 (d) 2612 (a) 0 (b) 1 (c) 2 (d) infinite

62. Let m (respectively, n) be the number of 5-digit 64. Let ABCD be a square. An arc of a circle with A as
integers obtained by using the digits 1, 2, 3, 4, 5 with centre and AB as radius is drawn inside the square
repetitions (respectively, without repetitions) such joining the points B and D. Points P on AB, S on
that the sum of any two adjacent digits is odd. Then AD, Q and R on arc BD are taken such that PQRS is
m a square.
is equal to
n
https://iit-jeeacademy.blogspot.com

6 KVPY Question Paper 2018 Stream : SA

Further suppose that PQ and RS are parallel to AC. (a) T (b) T


area PQRS
Then, is
area ABCD
1 1 1 2 E E
(a) (b) (c) (d)
8 5 4 5
65. Suppose ABCD is a trapezium whose sides and (c) T (d) T
height are integers and AB is parallel to CD. If the
area of ABCD is 12 and the sides are distinct, then
| AB − CD|
E E
(a) is 2
(b) is 4
(c) is 8 CHEMISTRY
(d) cannot be determined from the data
71. Among the following, the species with identical bond
order are
PHYSICS (a) CO and O2−
2 (b) O−2 and CO
(c) O2−
2 and B2 (d) CO and N+2
66. A coffee maker makes coffee by passing steam
72. The quantity of heat (in J) required to raise the
through a mixture of coffee powder, milk and water.
temperature of 1.0 kg of ethanol from 293.45 K to the
If the steam is mixed at the rate of 50 g per minute in boiling point and then change the liquid to vapor at
a mug containing 500 g of mixture, then it takes that temperature is closest to
about t0 seconds to make coffee at 70° C when the
[Given, boiling point of ethanol 351.45 K. Specific
initial temperature of the mixture is 25°C. The value heat capacity of liquid ethanol 2.44 J g −1K −1. Latent
of t0 is close to (ratio of latent heat of evaporation to heat of vaporisation of ethanol 855 J g −1]
specific heat of water is 540°C and specific heat of the
. × 102
(a) 142 (b) 9.97 × 102
mixture can be taken to be the same as that of water)
. × 105
(c) 142 (d) 9.97 × 105
(a) 30 (b) 45 (c) 60 (d) 90
73. A solution of 20.2 g of 1,2-dibromopropane in MeOH
67. A person in front of a mountain is beating a drum at upon heating with excess Zn produces 3.58 g of an
the rate of 40 per minute and hears no distinct echo. unsaturated compound X. The yield (%) of X is closest
If the person moves 90 m closer to the mountain, he to [Atomic weight of Br is 80.]
has to beat the drum at 60 per minute to not hear (a) 18 (b) 85 (c) 89 (d) 30
any distinct echo. The speed of sound is
(a) 320 ms−1 (b) 340 ms−1 (c) 360 ms−1 (d) 380 ms−1 74. The lower stability of ethyl anion compared to methyl
anion and the higher stability of ethyl radical
68. A glass beaker is filled with water up to 5 cm. It is compared to methyl radical, respectively, are due to
kept on top of a 2 cm thick glass slab. When a coin at
(a) + I-effect of the methyl group in ethyl anion
the bottom of the glass slab is viewed at the normal
σ → p-orbital conjugation in ethyl radical
incidence from above the beaker, its apparent depth
from the water surface is d cm. Value of d is close to (b) − I-effect of the methyl group in ethyl anion and
(the refractive indices of water and glass are 1.33 and σ → σ * conjugation in ethyl radical
1.5, respectively) (c) + I effect of the methyl group in both cases
(a) 2.5 cm (b) 5.1 cm (c) 3.7 cm (d) 6.0 cm (d) + I- effect of the methyl group in ethyl anion and
σ → σ * conjugation in ethyl radical
69. A proton of mass m and charge e is projected from a
very large distance towards an α-particle with 75. The F-Br-F bond angles in BrF5 and the Cl-P-Cl bond
velocity v. Initially α-particle is at rest, but it is free angles in PCl5 , respectively, are
to move. If gravity is neglected, then the minimum (a) identical in BrF5 but non-identical in PCl5
separation along the straight line of their motion will (b) identical in BrF5 and identical in PCl5
be
(c) non-identical in BrF5 but identical in PCl5
(a) e2/4 π ε0 mv2 (b) 5e2/4 π ε0 mv2 (d) non-identical in BrF5 and non-identical in PCl5
(c) 2e2/4 π ε0 mv2 (d) 4e2/4 π ε0 mv2
70. A potential is given by V (x) = k(x + a )2 / 2 for x < 0 and BIOLOGY
V (x) = k(x − a )2 / 2 for x > 0. The schematic variation of 76. If the genotypes determining the blood groups of a
oscillation period T for a particle performing periodic couple are IA IO and IA IB , then the probability of their
motion in this potential as a function of its energy E first child having type O blood is
is (a) 0 (b) 0.25 (c) 0.50 (d) 0.75
https://iit-jeeacademy.blogspot.com

KVPY Question Paper 2018 Stream : SA 7

77. A cross was carried out between two individuals Column I Column II Column III
heterozygous for two pairs of genes was carried out. P. Hypermetropia (i) Near-sightedness a. Convex lens
Assuming segregation and independent assortment,
the number of different genotypes and phenotypes Q. Myopia (ii) Far-sightedness b. Concave
lens
obtained respectively would be
(a) 4 and 9 (b) 6 and 3 (a) P–ii–b (b) Q–i–b
(c) 9 and 4 (d) 11 and 4 (c) P–i–a (d) Q–i–a
78. If the H+ concentration of an aqueous solution is 80. Which one of the following properties causes the
0.001 M, then the pOH of the solution would be plant tendrils to coil around a bamboo stick?
(a) 0.001 (b) 0.999 (c) 3 (d) 11 (a) Tendril has spines
79. Consider the following vision defects listed in (b) The base of the tendril grows faster than the tip
Columns I and II and the corrective measures in (c) Part of the tendril in contact with the bamboo stick
Column III. Choose the correct combination. grows at a slower rate than the part away from it.
(d) The tip of the tendril grows faster than the base

Answers
PART-I
1 (d) 2 (b) 3 (c) 4 (b) 5 (a) 6 (b) 7 (c) 8 (c) 9 (c) 10 (a)
11 (c) 12 (d) 13 (b) 14 (a) 15 (a) 16 (a) 17 (c) 18 (d) 19 (c) 20 (d)
21 (a) 22 (b) 23 (d) 24 (a) 25 (d) 26 (b) 27 (*) 28 (c) 29 (d) 30 (a)
31 (a) 32 (a) 33 (d) 34 (b) 35 (d) 36 (a) 37 (c) 38 (b) 39 (a) 40 (d)
41 (a) 42 (c) 43 (c) 44 (c) 45 (b) 46 (c) 47 (b) 48 (a) 49 (d) 50 (b)
51 (a) 52 (c) 53 (d) 54 (b) 55 (a) 56 (b) 57 (c) 58 (d) 59 (c) 60 (a)

PART-II
61 (b) 62 (c) 63 (b) 64 (d) 65 (b) 66 (b) 67 (c) 68 (b) 69 (b) 70 (b)
71 (c) 72 (d) 73 (b) 74 (a) 75 (d) 76 (a) 77 (c) 78 (d) 79 (b) 80 (c)

* No options are correct.

Solutions
1. (d) We have, 2. (b) Given,
a 4 + b4 < 1 and a 2 + b2 > 1 x4 − x2 + 2x − 1 = 0 = − (m − n )
The graph of x2 + y2 = 1 and x4 + y4 = 1 ⇒ x4 − (x − 1)2 = 0 ⇒ 2a + (m + n − 1) d = − 2 [m ≠ n ]
are ⇒ (x − x + 1) (x2 + x − 1) = 0
2
m+ n
Y ∴ Sm + n = (2a + (m + n − 1) d )
⇒ x2 − x + 1 = 0 2
or x2 + x − 1 = 0 m+ n
x4+y4=1 = (− 2) = − (m + n )
⇒ x2 − x + 1 = 0 has no real roots. 2
⇒ x2 + x − 1 = 0 has two real roots 4. (b) (I) Any pair of consistent linear
X′ X 3. (c) Given, Sm = n and Sn = m equation in two variables must have a
unique solution. This statement is false.
m
Sm = [2a + (m − 1)d ] = n ...(i) Consistent equation may have unique or
2
x2+y2=1 n infinite solution.
Sn = (2a + (n − 1) d ) = m ...(ii)
2 (II) There do not exists two consecutive
Y′ integers the sum of whose square is 365.
On subtracting Eq. (ii) from Eq. (i), we get
Clearly from graph. d This statement is also false
(m − n ) a + (m − n ) (m + n − 1) 132 + 142 = 365.
There are many positive real number 2
(a , b) satisfying a 4 + b4 < 1 and a 2 + b2 > 1.
https://iit-jeeacademy.blogspot.com

8 KVPY Question Paper 2018 Stream : SA

2 3 ∴ OG 2 + GM 2 = QM 2
5. (a) Let and volume of hemisphere = πr
n −1 n−2 3
P (x) = anx + an − 1 x
n
+ an − 2x + ... ∴ ∠QGM = 90°
2 3
+ a1 x + a0 ∴Volume of solid = πr 2h + πr 9. (c) Let ∆ABC
3
a0 , a1 , a2 K ∈ I When height of cylinder is doubled, then BC = a
Given, P (2) = 2 and P (4) = 5 2 AC = b
volume of solid = 2 πr 2h + πr3
2 = an 2n + an − 1 2n − 1 + an − 2 2n − 2 + ... 3 AB = c
+ a1 2 + a0 … (i) 2
2 πr 2h + πr3 A
V2 3
5 = an 4n + an − 1 4n − 1 + an− 2 4n − 2 + . + ∴ = = 3
V1 2 2
4a1 + a0 … (ii) πr 2h + πr3 F E
2 3
On subtracting Eq. (i) from Eq. (ii), we get 2h + r
3 = 3 h r G
3 = an (4n − 2n ) + an − 1 (4n − 1 − 2n − 1 ) ⇒ ⇒ =
2 2 2 3
h+ r
+ ... + 2a1 3 B D C
Clearly, LHS is odd number and RHS is When the radius is doubled, then volume and median of ∆ABC
even number. 16 πr3 AD = l
of solid = 4 πr 2h +
∴No polynomials exists. 3 BE = m
6. (b) Four digits number which is 4h +
16
r CF = n
divisible by 7 are 1001, 1008, 1015, ...., V′ 3 = 4h + 8h = 6 Q r = h 
∴ 2= AD is median,
9996. V1 2 h+ h  3 2  AB + BC
h+ r ∴ AD <
Hence, total number of such numbers 3 2
= 1286 Hence, volume is increased by 500%. b+ c
∴ l<
th
N 8. (c) In ∆PQR 2
  observation a+ b a+ c
 2 Given, QR 2 + PR 2 = 5PQ 2 Similarly, m< and n <
th 2 2
+  + 1
N Median PM and QN intersect of G.
observation ∴ l+ m+ n< a+ b+ c
 2  P
Median = l+ m+ n
2 ⇒ <1 …(i)
a+ b+ c
[Q N is even] N
th Also in ∆BGC, BG + GC > BC
 1286  G 2
  observation ∴ (m + n ) > a
 2  3
th 2 2
+  + 1 Similarly, (n + l) > b and (m + l) > c
1286 Q M R
observation 3 3
 2  QG =
2 1
QN , GM = PM
Median = 4
2 3 3 Q (l + m + n ) > a + b + c
2 2 3
643th + 644th ⇒ QG 2 + GM 2 =  QN  +  PM 
2 1 l+ m+ n 3
= ⇒ > …(ii)
2 3  3 
a+ b+ c 4
(1001 + (642)7) + (1001) + (643)7) 4 1
= = QN 2 + PM 2 From Eqs. (i) and (ii), we get
2 9 9
2 (1001) + 7 (642 + 643) l + m+ n 3 
= 4  2PQ 2 + 2QR 2 − PR 2  ∈  , 1
2 =   a + b+ c 4 
9 4 
2 (1001) + 7 (1285) 10. (a) Let P (x0 , y0 )
= 1 2PQ + 2PR 2 − QR 2 
 2
2 +   Given x2 + y2 ≤ 1
9 4 
= 1001 + 4497.5 = 5498.5
Let any arbitrary point 8(x, y).
7. (c) Let the height and radius of  8PQ 2 + 8QR 2 − 4PR 2 
Y
 
cylinder are h are r, respectively. 1 + 2PQ 2 + 2PR 2 − QR 2  (0,1)
=  
P(x0,y0)
∴Volume of cylinder = πr 2h 9 4 Q
  (x,y)
 
r  
1
r 1  10PQ 2 + 7QR 2 − 2PR 2 
=   X′ X
9 4 O (1,0)

 2 (5PQ 2 − PR 2 ) + 7QR 2 
1
=  
h h 9 4
 
1  2QR + 7QR  1
2 2

r =   = QR = QM
2 2
Y′
9 4  4
https://iit-jeeacademy.blogspot.com

KVPY Question Paper 2018 Stream : SA 9

PQ 2 = (x − x0 )2 + ( y − y0 )2 14. (a) Given, PQR is an acute angle 16. (a) As temperature of water is
PQ 2 = (OP − OQ )2 triangle. increased from 0°C to 10°C, density of
PQ 2 = (OP − OQ )2 PQ < QR water initially increases upto a maximum
at 4°C and then it reduces.
PQ = (
2
x02 + x02 − 1) 2
[Q OQ = 1] Q
So, buoyant force on block of wood also
∴Mininimum value of PQ is 2
p increases till temperature reaches 4°C
( x02 + y02 − 1)2 r and then decreases from 4°C to 10°C.
Hence, volume of block above water also
11. (c) Given, in ∆PQR increases upto 4°C and then decreases
PQ = 3 P Q1 Q2 Q3 R from 4°C to 10°C.
Altitude RS = 3 q ∴Variation of V 0 versus t as shown below.
⇒ PS = QR ∠QRP < ∠QPR V0
1
R PQ3 = PR
2
PQ2 : Q2 R = r : p
 r 
PQ2 =   PR 0°C 4°C 10°C
t
 r + p
17. (c) Fraction of thickness of ice block
But r< p out of water is
P S Q 1
PQ2 < PR  ρ  0.9
x = 1 −  ice  = 1 − or x = 01
.
In ∆SQR, QR 2 = SR 2 + SQ 2 2  ρwater  1
PS 2 = ( 3 )2 + (QP − PS)2 Comparison between altitude and angle
[Q SQ = PQ − PS] bisector
PS 2 = 3 + (3 − PS)2 ∠QPQ2 + ∠PQ2Q + ∠PQQ2 = ∠RQQ2
PS 2 = 3 + 9 − 6PS + PS 2 ⇒ PS = 2 + ∠QQ2R + ∠QRQ2
∴∠PQQ2 = ∠RQQ2
In ∆PRS, 0.8 m
[since, QQ2 is angle bisector of ∠Q]
PR 2 = PS 2 + RS 2 = (2)2 + ( 3 )2 = 4 + 3
∠QPQ2 + ∠PQ2Q = ∠QQ2R + ∠QRQ2
PR = 7 8m
∴PQ < QR the ∠QPQ2 < ∠QRQ2
12. (d) Total number of students = 50 Hence, ∠QQ2P < ∠QQ2R
Average marks of student = 47.5 But ∠QQ2P + ∠QQ2R = 180° So, minimum length of rope required ≈
∴Total marks of students Hence, ∠QQ2P < 90° and ∠QQ2R > 90° thickness of ice × 01
. = 8 × 01
. = 0.8 m.
= 50 × 47.5 = 2375
Q Foot from Q to side PR lie inside ∆PQQ2 Hence, nearest option is 0.9 m.
Now, the student get integer marks
⇒ PQ1 < PQ2 < PQ3 18. (d) When box with hole is in free fall,
Hence, the maximum number of students
15. (a) Given, (a − 8)2 − (b − 7)2 = 5 both water and box cover equal distance
we will divide total mark by 48. downwards in equal time.
2375 ⇒ (a − 8 + b − 7) (a − 8 − b + 7) = 5
∴ = 49 Hence, no water comes out of hole in free
48 ⇒ (a + b − 15) (a − b − 1) = 5
fall of box.
13. (b) Given number, There are four case
a + b − 15 = 5; a − b − 1 = 1 … (i)
19. (c) Water evaporated in two hours
n = 152 × 518 = m = 2 h × 20 g/h
a + b − 15 = 1; a − b − 1 = 5 ... (ii)
n = 32 × 52 × 518 = 40 g = 40 × 10−3 kg
a + b − 15 = − 5; a − b − 1 = − 1 … (iii)
n = 9 × 520 Heat absorbed by water during
a + b − 15 = − 1; a − b − 1 = − 5 ... (iv)
Taking log base 10 both side evaporation is
On solving, we get
log10 n = log10 9 + log10 520 Q = Mass evaporated × Latent heat
(i) a = 11, b = 9 (ii) a = 11, b = 5
= 2 log10 3 + 20 log10 5 Q = mL …(i)
(iii) a = 5, b = 5 (iv) a = 5, b = 9
= 2 × 0.4771 + 20 × (1 − 0.3010) Assuming this heat is taken entirely from
= 14 characters value
D(11, 5) 4 C(11, 5) water in earthen pot, if ∆T is decrease of
temperature of pot then,
Hence, the number have 15 digits
Q = Ms∆T …(ii)
S = Sum of digits of the number
6 6 where, M = mass of water in pot
Now, n has last digit is 5.
and s = specific heat of water.
∴Minimum value of S = 1 + 5 = 6
Equating Eqs. (i) and (ii), we get
Maximum value of S = 9 × 14 + 5
A(5, 5) 4 B(5, 5) mL = Ms∆T
= 126 + 5 = 131
∴Perimeter = 2(4 + 6) = 20 m L 40 × 10−3
∴ 6 ≤ S < 140 or ∆T = × = × 540 = 5.4°C
M s 4
https://iit-jeeacademy.blogspot.com

10 KVPY Question Paper 2018 Stream : SA

20. (d) As emergent ray is parallel to 24. (a) A lunar eclipse occurs only on a 27. (No option is matching)
incident ray, deviation angle δ is 180°. full moon day. In late afternoon rainbow is visible in
But δ = 360°− 2θ So, option (a) is incorrect. east side when light of sun in west side is
where, θ = angle between inclined reflected and refracted by a layer of water
25. (d) Intensity of radiation (mainly droplets.
mirrors. visible light) emitted from surface of a
Rainbow is circular because locous of
star is proportional to its area.
reflected rays reaching eye of observer is
So, I ∝ A or I = kA a circle. Its roundness is not due to
where, k = constant. roundness of earth.
Now, if I 0 = intensity of parent star. There is no rainbow on moon due to lack
θ of atmosphere.
Then, I 0 = kπ (100 R )2 = k π R 2 × 10000
In case of a primary rainbow, violet
So, 360° − 2θ = 180° When exoplanet is in front of star, colour is on inside and red colour is on
observed intensity will be minimum. Let outside of arc.
or 2θ = 180° ⇒ θ = 90°
intensity minimum is I min , then In case of a secondary rainbow, red colour
21. (a) From principle of thermometry, is on inside and violet colour is on outside
T − TLFP I min = k [ π (100 R )2 − π R 2 ] of arc.
= a constant for every
TUFP − TLFP
W
Star radius R AINBO
thermometric scale. N DARY
100 R SECO
Now, for any temperature L on a ION
R K REG
thermometer designed with given liquid DAR
Exoplanet BO
W
and equivalent temperature C on RAIN
radius R ARY
PRIM
centigrade scale, we have V
R
 L − TLFP   C − TLFP 
  Liquid =   R V
 TUFP − TLFP  based  TUFP − TLFP  Centigrade ⇒ I min = kπR 2 (10000 − 1)
scale
scale
= kπR × 9999
2 42° 51°
L − (− 50) C−0 kπR 2 × 9999
Observer
⇒ = So,
I min
=
150 − (− 50) 100 − 0 I0 kπR 2 × 10000 East
L + 50 C
= ⇒ I min = I 0 × 0.9999 West
150 + 50 100
So, none of the option is correct. Option
⇒ L + 50 = 2C 26. (b) When current flows through a (b) is correct, if only secondary rainbow is
Now at 0°L, centigrade scale reading will conductor of tapered cross-section, considered.
be current flow through every section
remains constant. 28. (c) Consider the given diagram,
50
0 + 50 = 2 C or C = = 25° L Screen
2
I
and at 100° L, centigrade scale reading A1 A2 I
f=50 cm θ1
will be Camera

100 + 50 = 2 C or C =
150
= 75° L ⇒ I1 = I 2
θ2
2 ⇒ j1 A1 = j2A2
h=500 km
22. (b) An alpha-volt (α-V ) is the energy ⇒
j1 A2
= <1
acquired by an α-particle (charge 2e j2 A1
units) when accelerated by a potential ⇒ j1 < j2 Earth
difference of 1 V. Current density increases in the narrow
∴ 1 α-V = q (∆V ) region.
= 2 e × 1 V = 2 eV Also, j = nevd Assuming area observed and screen both
23. (d) If Q is charge contained in L ⇒ nevd1 < nevd2 circular, we have
d d d h
length of beam of area A, then ⇒ vd1 < vd2 θ1 = θ2 ⇒ 1 = 2 ⇒ 2 =
f h d1 f
L× A ×ρ=Q Drift velocity increases in the narrow
where, d1 = diameter of camera screen
where, ρ = charge density of beam. region.
E and d2 = diameter of area on earth.
Q Q /t I and j= area observed on earth A0
So, ρ= = = ρ Now, =
L × A L /t × A v × A area of screen A
where, ρ = resistivity of material.
500 × 10−6  π ⋅ d22 
= ⇒
E1 E2
< ⇒ E1 < E2  
3 × 107 × 1.50 × 10−6  4 
ρ ρ   d22
= =
5  π ⋅ d1  d12
2
= × 10−5 = 11
. × 10−5 Cm−3 Electric field magnitude increases in the  
3 × 1.5  4 
narrow region.  
https://iit-jeeacademy.blogspot.com

KVPY Question Paper 2018 Stream : SA 11

A0  h 
2
 500 × 10+3 
2
Solubility (in mol/L) × Molecular weight Here, the marked carbon (*) is chiral as
⇒ =   =  
A  f1  −2 
 50 × 10  = 233 × 10−5 = 2. 3 × 10−3 it has 4 different groups attached to it.
34. (b) Consider the following statements. CH2 CH2
= (10 × 103 × 102 )2 = (106 )2 = 1012 *
(I) According to Pauli’s exclusion CH3 CH3
29. (d) A cylindrical lens produces erect principle, no two electrons in an atom can
and laterally inverted image. Br
have the same set of four quantum
So, image appears as shown below. Y
numbers.
Thus, statement (a) is correct. Here, the marked carbon (*) is achiral
A
(II) The maximum number of electrons in as it has 2 similar ethyl group attached
B D the shell with principle quantum number, to it.
n= 2n 2.
C Thus, statement (b) is incorrect. 38. (b) O
30. (a) For the ball, we have (III) Electrons in an orbital must have CH3
1 −1 b c
−1
u = 45 ms , g = − 10 ms −2 opposite spin, i.e. ms = + and . Thus, N N
2 2 H H
a
Now using, v2 − u 2 = 2 gh, we have statement (c) is correct. N
v2 = (45)2 − 20h (IV) According to Aufbau principle, in the H
⇒ v = 2025 − 20h ground state of the atoms, the orbitals are In the given compound most acidic
2025 filled in order of their increasing energies. proton will be N b  H. This is because
At v = 0, h = ≈ 101 m
20 Thus, statement (d) is correct.
its conjugate base will get resonance
at h = 0, v = 45 ms−1 35. (d) From ideal gas equation stabilised and the most nucleophilic
As velocity decreases with height, slope of pV = nRT nitrogen will be N c . This is because the
v-h graph must be negative at all points. maximum number of moles in container, lone pair of electrons present on this N
Hence, correct graph is (a).
pV 2 × 2. 24 is localised over sp3 -hybrid orbital.
. g mL−1
31. (a) Given, density of water = 10 n= =
RT 0.0821 × 298 39. (a) The reaction between
volume of water = 250 mL. haloalkane and AgNO3 gives
= 018
. moles
∴ Mass of water = density × volume carbocation intermediate. So, more
Maximum weight of N 2 in container
= 10
. × 250 = 250 g = 0183
. × 28 = 5127
. g easily it will be formed, more readily it
18 g of water contains At 5.127 g exploding can occur. Thus, it will react to give precipitate.
= 6.023 × 1023 molecules must be less than 5.127. Thus, the In the given compound
∴ 250 g of water contains maximum amount of nitrogen that can be
a c
safely put in this container at 298 Cl Cl
6.023 × 1023
= × 250 temperature and exert pressure less than Cl
d
Cl b Cl
d
Cl b
18 2 atm will be closest to 4.2 g. AgNO3
= 83.65 × 1023 molecules 36. (a) NO2 O Cl a +
32. (a) Dil.aqueous solution of NH3 is O O
C
NH4 OH . + Cl
NH4 OH - NH+4 + OH− Cl
c

On adding solid ammonium chloride Benzene 2- nitrobenzoylchloride d


Cl Cl b
NH4 Cl → NH+4 + Cl − AlCl3
The reaction moves backward due to
common ion effect. The concentration of O NO2 O
+
OH− decreases and hence the pH
C
decreases. Cl a is easily lost from this compound
−10
33. (d) Given, Ksp = 1 × 10 and carbocation formation takes place
readily. This is because this Cl a is
BaSO4 - Ba 2+ + SO24− This reaction is Friedal-Craft acylation. In closest to electronegative atom O, which
S S this reaction, benzene reacts with acyl will attract the electron density towards
Let the solubity of Ba 2+ and SO2−
4 be S halide or acid anhydride in the presence itself and readily leaves C  Cl a bond.
∴ K sp = S 2 of Lewis acid like AlCl3 to yield Hence, it will most readily react with
acylbenzene.
1 × 10−10 = S 2 CH3
AgNO3 to give precipitate.
37. (c)
S = 10−5 mol/L CH2 * CH2 40. (d) The specie which follows
CH3 CH2
Thus, solubility of BaSO4 in pure water H
Huckel’s rule (4n + 2)π will be most
Br stable species.
(in g/L) X
https://iit-jeeacademy.blogspot.com

12 KVPY Question Paper 2018 Stream : SA


+ Among the given salt, KI is the experiences low oxygen level. The
(a) and strongest reducing agent. Thus, salt X resulting rise in red-blood cells increases
is KI. the oxygen carrying capacity of the blood.

It has 8πe s, doesn’t It has 4πe s, doesn’t 2KI + 4HNO3 → I2 + 2NO2 + 2KNO3 51. (a) Tendrils are the modifications of
follow Huckel’s rule follow Huckel’s rule stem or leaf. The tendril is a thread-like,
– + 2H2O twisting, clinging growth on the vines of
45. (b) The role of haemoglobin is to the plant that enables it to attach itself to
(b) +
another object or another plant for
transport oxygen from lungs or gills to
different parts of the body. There it support. Plants such as grapes, peas and
It has 8πe s, doesn’t It has 2πe s, cucumbers have tendrils.
follow Huckel’s rule follows Huckel’s rule releases the oxygen to permit aerobic


respirisation to provide energy to 52. (c) Ribosomes consist of two
power the functions of the organism in biomolecules (i.e. RNA and proteins).
(c) The small ribosomal subunits which read
the process called metabolism.
the RNA, and large subunits which join
It has 10πe s, follows It has 4πe s, doesn’t 46. (c) Penicillin is a secondary amino acids to form a polypeptide chain.
Huckel’s rule follow Huckel’s rule metabolite. Secondary metabolites are Each subunit comprises one or more
– +
organic compounds produced by ribosomal RNA (rRNA) molecules and a
(d) bacteria, fungi or plants which are not variety of ribosomal proteins (r-protein).
directly involved in the normal growth, 53. (d) Microtubule does not play a role
It has 10πe s, follows It has 2πe s, follows development or reproduction of the in skeletal muscle. Microtubules are
Huckel’s rule Huckel’s rule organism, e.g., antibiotics like hollow fibrous shafts whose main
As both the species in option (d) follow penicillin, streptomycin, etc. Rest function is to help support and give shape
Huckel’s rule. Thus, it is correct option. molecules like ethanol, lactate and to the cell. They also serve a
transportation function as they are the
41. (a) As the atomic number increases, citric acid are primary metabolites.
routes upon which organelles move
the energy of orbital decreases. This is 47. (b) Lecithin is a phospholipid that through the cell.
because the atomic radii decreases (nuclear is important in cell structure and 54. (b) In the stratosphere, ozone is
charge increases) with increase in atomic metabolism. Lecithins are composed of created primarily by ultraviolet
number. The atomic number of H, Li, Na phosphoric acid, cholines, esters of radiation. When high energy ultraviolet
and K respectively, are 1, 3, 11 and 19. glycerol and two fatty acids; the chain rays strike ordinary oxygen molecules
Thus, the correct order of energy of length, position and degree of (O2 ), they split the molecule into two
2s-orbitals is unsaturation of these fatty acids vary single oxygen atoms, known as atomic
K < Na < Li < H. and this variation results in different oxygen.
42. (c) The hybridisation of any compound lecithins with different biological O2 → O + O
can be calculated as, functions.
A freed oxygen then combines with
1 48. (a) Pure water at standard another oxygen molecule to form a
X = [Valence electrons
2 temperature and atmospheric pressure molecule of ozone (O3 ).
+ Number of monoatomic m Anion/cations] has a water potential of zero. As solute
55. (a) Trypsinogen is an inactive
1 is added, its value becomes more
∴ For XeF4 (X ) = (8 + 4 − 0) = 6 substance secreted by the pancreas, from
2 negative. This causes water potential
which the digestive enzyme trypsin is
∴ The hybridisation is sp3 d 2. to decrease. Water potential is the
formed in the duodenum. Trypsinogen is
measure of the potential energy in
43. (c) In Cr2O72− , converted into its active form trypsin by
water. It is denoted by the Greek letter
an enzyme enterokinase. This results in
Let the oxidation state of Cr be x ψ (Psi) and is expressed in units of the subsequent activation of pancreatic
∴ 2(x) + 7(−2) = − 2 pressure (pressure is a form of energy)
digestive enzymes.
called Mega Pascals (MPa).
2x − 14 = − 2 56. (b) Salamanders are a group of
2x = 12 49. (d) An action potential is amphibians typically characterised by a
generated by the rapid influx of
x=+ 6 lizard-like appearance with slender
Na + ions followed by a slightly slower
In ClO3− , bodies, blunt snouts, short limbs and a
efflux of K+ ions. The action potential
Let the oxidation state of Cl be x tail. Salamanders breath through their
is the mechanism by which nerve cells
skin and the thin membranes in the
∴ 1(x) + 3(−2) = − 1 communicate and conduct information
mouth and throat.
x− 6= −1 and muscle cells are induced to
contract. 57. (c) Mature human erythrocytes (Red
x=+ 5
blood cells) lack a nucleus. The absence of
44. (c) As, a filter paper soaked in salt X 50. (b) Erythropoietin (EPO) is a a nucleus is an adaptation of the red
turns brown when exposed to HNO3 hormone produced by the kidney that
blood cell for its role. It allows the RBC to
promotes the formation of red blood
vapour, then salt X must be a strong contain more haemoglobin and therefore
cells by the bone marrow. Chemically,
reducing agent which will reduce HNO3 to carry more oxygen molecules. It also
erythropoietin is a protein with an
NO2 (brown gas). attached sugar (a glycoprotein). It is allows the cell to have its distinctive
produced by kidney cells when the body biconcave shape which aids diffusion.
https://iit-jeeacademy.blogspot.com

KVPY Question Paper 2018 Stream : SA 13

58. (d) The first enzyme that the food 3, 4, 5 without repetitions such that sum 64. (d) Given, ABCD is a square.
encounters in the digestive system is of any two adjacent digits is odd. D C
amylase. Digestion begins in the mouth Sum of two digits are odd if one is even
with the secretion of saliva and its and other is odd. R
digestive enzymes. Saliva contains the Even = 2, 4
digestive enzyme amylase, which works N
Odd = 1, 3, 5
on carbohydrate, starch like breads,
Case I Digit is repeated. Q
potatoes or pasta to help break them
down into simple sugars. Two possibilities S

59. (c) Glycoproteins are formed in the (a) odd even odd even odd M
Golgi apparatus of the cell. = 3 × 2 × 3 × 2 × 3 = 108
Glycoproteins are proteins that contain A P B
(b) even odd even odd even
covalently attached sugar residues.
= 2 × 3 × 2 × 3 × 2 = 72 ∠CAP = ∠MAP = 45°
Glycoproteins are present at the surface
∴ m = 108 + 72 = 180 ∴ AM = MP = QN
of cells where they function as membrane
proteins and play a role in cell to cell Case II Digit is not repeated. PQRS is a square,
interactions. Q MN = PQ = PS
The possibility of arrangement is
60. (a) An example of nastic movement PS = 2PM = 2AM
odd even odd even odd
in plants is folding up of the leaves of AN = AM + MN = 3AM
= 3 × 2 × 2 × 1 × 1 = 12
Mimosa pudica. Nastic movements in In ∆ABQ,
plants are reversible and repeatable n = 12 AQ 2 = AN 2 + QN 2
m 180 1 = (3AM )2 + AM 2 [Q AQ = 1]
movements in response to a stimulus ∴ = = 15
whose direction is determined by the n 12 10AM 2 = 1
anatomy of the plant. The leaves of the 63. (b) Let height and radius of cone is h AM 2 = 1
Mimosa pudica fold up when touched and and r respectively, h , r ∈ I 1
AM 2 =
returns to full leaf in a few minutes. The Given volume of cone = Surface area of 10
leaves of the Mimosa achieve this rapid Area of square PQRS = PS 2
cone
folding by a change in turgor pressure. 4 2
1 2 = 4AM 2 = =
61. (b) Given, n 2 − 10n − 36 πr h = πrl + πr 2
3 10 5
Area square PQRS 2
n is a natural number. 1 2 ∴ =
⇒ πr h = πr h 2 + r 2 + πr 2 Area of square ABCD 5
∴Product of its digits is ≥ 0 3
∴ n 2 − 10n − 36 ≥ 0 1 65. (b) We have,
⇒ rh = h 2 + r 2 + r [r ≠ 0]
10 ± 100 + 144 3 ABCD is a trapezium.
n=
2 AB is parallel to CD.
⇒ rh − 3r = 3 h + r 2 2

n = 5± 61 Area of trapezium = 12
⇒ r 2h 2 + 9r 2 − 6hr 2 = 9h 2 + 9r 2
∴ n ∈ (− ∞ , 5 − 61) ∪ (5 + 61, ∞ ) D b C
⇒ h 2 (r 2 − 9) = 6 hr 2
But n is positive integer.
∴ n ≥ 13 6r 2
⇒ h=
When n is two digits numbers, then r2 − 9 h
maximum product = 9 × 9 = 81  r2 
⇒ h = 6  2 
∴ n 2 − 10n − 36 ≤ 81 r − 9 
n 2 − 10n − 117 ≤ 0 54 a
⇒ h = 6+ A b E B
∴ n ∈ [5 − 142 , 5 + 142 ] r2 − 9 1
⇒ × h (AB + CD ) = 12
n is taken two digit number. 2
h and r are integer.
∴ n ∈ [13,17) = 13, 14, 15, 16 ⇒ AB + CD =
24
Q r 2 − 9 is a factor of 54. h
∴Product of digits = 3, 4, 5, 6
∴r 2 − 9 = 1, 2, 3, 6, 9, 18, 27, 54 Sides and height of trapezium are
When put n = 13
132 − 10 × 13 − 36 = 169 − 166 = 3 r 2 = 10, 11, 12, 15, 18, 27, 36, 63 integer.
∴ r = 6 only possible value. ∴h is a factor of 24
n = 13 satisfies
54 h = 1, 2, 3, 4, 6, 8, 12, 24
62. (c) We have, m is 5-digits number ∴ h = 6+ AB + CD = 24, 12, 8, 6, 4, 3, 2, 1
using digits 1, 2, 3, 4, 5 with repetition 36 − 9
But AB + CD > h
such that sum of two adjacent digit is odd = 6+ 2= 8
and n is 5-digits number using digits 1, 2, AB + CD = 24, 12, 8, 6
∴ r = 6, h = 8
https://iit-jeeacademy.blogspot.com

14 KVPY Question Paper 2018 Stream : SA

In ∆BEC, So, we have dU


If = 0, when x = ± a.
BEC is a right angled triangle. 2x 60 dx
In case I, = …(i)
∴h must be 3 and 4 v 40 d 2U
Now, = km> 0
2(x − 90) 60 dx2
When h = 3, BE = 4, CE = 5 In case II, = …(ii)
v 60 So, particle is in unstable equilibrium at
AB + CD = 8
Substituting for x from Eq. (i) in Eq. (ii), x = ± a.
AE + BE + AE = 8
we get Hence, particle is unbounded for − a > x
2AE = 8 − BE = 8 − 4 2x − 180 = v and x > a.
AE = 2 ⇒
3
v − 180 = v In region, − a ≤ x ≤ a, time period of
∴ AB = 4 + 2 = 6, CD = 2 2
particle reduces from a maximum.
⇒ v = 360 ms−1
∴ |AB − CD| = |6 − 2| = 4 So, correct graph is (b).
68. (b) Apparent depth d in case of more
66. (b) Let m gram of steam is condensed 71. (c) The bond order can be calculated
than one medium is
in the process of heating mixture from as
d d
25°C to 70°C. d = 1 + 2 + ... …(i) 1
µ1 µ 2 B.O = (Nb − N a )
Then, 2
Heat lost by steam = Heat gained by where, d1 and d2 are the thickness of
slabs of medium with refractive index µ 1 where, Nb = electrons in bonding orbitals
mixture
and µ 2, respectively. N a = electrons in antibonding orbitals.
⇒ Heat of condensation of steam + Heat
Here, d1 = 5 cm, µ 1 = 133
. (a) CO and O2−
given by water formed = Heat gained by 2

mixture d2 = 2 cm, µ 2 = 15
. The electronic configuration of CO (14) is
⇒ m ⋅ L + msw ∆T = M ⋅ sm∆T Substituting these values in Eq. (i), σ1s2 σ * 1s2 σ 2s2 σ * 2s2 σ * 2 pz2 π 2 px2 π 2 p 2 y
⇒ mL + msw (100 − 70) we get 1 6
∴ B.O = (10 − 4) = = 3
5 2 2 2
= 500 × sw × (70 − 25) Apparent depth, d = +
500 × sw × 45 133
. 15
. The electronic configuration of O22− (18) is
⇒m =
L + 30 sw = 5.088 cm = 51
. cm σ1s2 σ * 1s2 σ 2s2 σ * 2s2σ 2 pz2 π 2 px2
500 × 45 69. (b) As α-particle is free to move, π 2 py2 π * 2 px2 π * 2 py 2
⇒m = initial kinetic energy of system will be
L  1
B.O = (10 − 8) = 1
 + 30 1
 sw  ki = µv2 2
2 −
(b) O2 and CO
500 × 45
⇒ m= ≈ 40 g where, µ = reduced mass of system The electronic configuration of O−2 (17) is
(540 + 30) m ⋅ 4m
= . σ1s2 σ * 1s2 σ 2s2 σ * 2s2 σ 2 pz2 π 2 px2
Now, in 1 min, 50 g of steam is m + 4m
condensed. π 2 py2 π * 2 px2 π * 2 p1y
Now, by energy conservation, we have 1 3
∴ 40 g of steam will be condensed in time B.O = (10 − 7) = = 1.5
Initial kinetic energy = Potential energy 2 2
t0 ,
at minimum separation r B.O of CO is 3
40 × 60
t0 = s = 48 s 1  m . 4m  2 1 2e2 [as calculated in option (a)]
50   v = .
2  m + 4m  4 πε0 r (c) B.O of O2− is 1
Nearest answer is 45 s. 2
5e2 [as calculated in option (a)]
67. (c) As drummer does not hear any ⇒ r= The electronic configuration of B2 (10) is
echo this means time between two 4 πε0 mv2
successive wavefronts is equal to time in σ1s2 σ * 1s2 σ 2s2 σ * 2s2 π 2 p1x π 2 p1y
70. (b) Given, potential function for the
which a wavefront reaches back to 1 2
oscillating particle is B.O = [6 − 4] = = 1
drummer. 2 2
 k (x + a ) 2
x  , x< 0 (d) B.O of CO is 3
V (x ) =  2
[as calculated in option (a)]
 k (x − a ) , x > 0
2

 2 Electronic configuration of N+2 (13) is


So, potential energy of the particle (mass σ1s2 σ * 1s2 σ 2s2 σ * 2s2 π 2 px2 π 2 py2 σ 2 p1z
m) is 1 5
B.O = [9 − 4] = = 2 .5
 km (x + a )2 2 2
 , x< 0
U (x ) =  2 Thus, option (c) is correct.
Distance covered by sound = 2x
 km(x − a ) , x < 0
2
72. (d) Given, mass of ethanol = 1kg
If v = speed of sound, then  2
2x = 1000 g
= time interval between two successive dU  km(x + a ), x < 0
v = Latent heat of vaporisation of ethanol
dx km (x − a ), x > 0
wavefronts. = 855 Jg −1
https://iit-jeeacademy.blogspot.com

KVPY Question Paper 2018 Stream : SA 15

Specific heat capacity of ethanol The geometry of PCl5 is triangular Yr YYRr YYrr YyRr Yyrr
= 2. 44 J / gk −1 pyramidal. yellow yellow yellow yellow
Heat, q = mc∆T + heat of vaporisation Cl round wrinkled round wrinkled
Cl
= 1000 × 2 .44 (351.45 − 293.45) yR YyRR YyRr yyRR yyRr
Cl P
+ 855 × 1000 J Cl yellow yellow green green
= 9.97 × 10 J
5 Cl round round round round

The axial bonds suffer more repulsions yr YyRr Yyrr yyRr yyrr
73. (b) Br
yellow yellow green wrinkled
Br Zn than equitorial bonds, so they are larger
MeOH
round wrinkled round green
in bond length.
1, 2-dibromopropane Prop - 1- ene The genotypic ratio is 1 : 2 : 1 : 2 : 4 : 2 : 1 :
(X) 76. (a) The genotype of child having 2:1
A O
blood group-O with parents having I I
Moles of 1, 2-dibromo propane The phenotypic ratio is 9 : 3 : 3 : 1
and IA IB blood groups can be represented
20.2 ∴ The number of different genotypes and
= = 01
. mole as
202 phenotypes obtained would be 9 and 4,
Parents IA IO × IA IB
3.58 respectively.
Moles of prop-1-ene =
42 78. (d) The H+ ion concentration of an
aqueous solution is 0.001 M or 1 × 10−3 M
= 0.085 mole Genotype IA IA IA IB IA IO IB IO
0.085 Blood Blood Blood Blood
Since we know that pH = − log[H+ ]
% yield = × 100 = 85%. Phenotype group –A group –AB group –A group –B
01
. Using this equation, by plugging in the
values
74. (a) The lower stability of ethyl anion Offsprings

(CH3 CH2 ) compared to methyl anion pH = − log 10−3 = − (− 3) log 10 = 3
From the above cross, it is shown that pH = 3
(CH3− ) is because of + I-effect of methyl
none of the offsprings will be of blood
group of ethyl anion. The higher stability We know that
group-O.
of ethyl radical compared to methyl pOH = 14 − pH = 14 − 3 = 11
∴ The probability of their first child
radical is due to σ − p-orbital conjugation having type-O blood is zero. ∴ pOH = 11
which is known as hyper conjugation in 79. (b) Hypermetropia is far
77. (c) In the given question, both
ethyl radical. sightedness. A vision condition in which
parents are heterozygous for two pairs of
genes. This means the cross is a dihybrid nearby objects are blurry. It is corrected
H H
H cross. by using convex lens.
C C H C CH2
H H Myopia is near sightedness. A condition
Lets assume a dihybrid cross
H H in which close objects appear clearly but
Hyper conjugation Pure breeding Yellow round , Wrinkled green far ones do not. It is corrected using
in ethyl radical traits seeds seeds
concave lens.
75. (d) The geometry of BrF5 is square (YYRR) (yyrr)
80. (c) The tendrils are sensitive to
pyramidal. Heterozygous Yellow round seeds touch. When they come in contact with
F trait (YyRr)
any support, the part of the tendril in
Gametes contact with the object does not grow
F F
rapidly as the part of the tendril away
YR Yr yR yr
Br from the object. This causes the tendril to
circle around the object and thus cling to
F F it. This process is known as positive
thigmotropism. Thigmotropism is the
YR Yr yR yr directional response of a plant organ to
Here, the lone pair occupies the axial + touch or physical contact with a solid
position and hence axial bonds will suffer
YR YYRR YYRr YyRR YyRr object. This differential response is
more repulsion than axial bonds.
yellow yellow yellow yellow generally caused by the induction of some
Thus, the axial Br—F bond length will be pattern differential growth.
round round round round
different than equitorial Br—F.
https://iit-jeeacademy.blogspot.com

16 KVPY Question Paper 2017 Stream : SA

KVPY
KISHORE VAIGYANIK PROTSAHAN YOJANA

QUESTION PAPER 2017


Stream : SA (Nov 19)
MM 100

Instructions
There are 80 questions in this paper.
This question paper contains two parts; Part I and Part II. There are four sections; Mathematics, Physics, Chemistry
and Biology in each part.
Out of the four options given with each question, only one is correct.

PART-I (1 Mark Questions)


MATHEMATICS on side BC such that MP is parallel to AB. If the area
5
1. Suppose BC is a given line segment in the plane and of the quadrilateral BNMP is equal to of the area
18
T is a scalene triangle. The number of points A in the of ∆ ABC, then the ratio AM / MC equals
plane such that the triangle with vertices A, B, C (in
(a) 5 (b) 6
same order) is similar to triangle T is 18 15
(c) (d)
(a) 4 (b) 6 (c) 12 (d) 24 5 2
2. The number of positive integers n in the set 5. Let n ≥ 4 be a positive integer and let l1 , l2 , ... , ln be
1
{2, 3, ... , 200} such that has a terminating decimal the lengths of the sides of arbitrary n sided
n
non-degenerate polygon P. Suppose
expansion is
l1 l2 ln − 1 ln
(a) 16 (b) 18 (c) 40 (d) 100 + + ... + + =n
l2 l3 ln l1
3. If a , b, c are real numbers such that a + b + c = 0 and
a 2 + b2 + c2 = 1, then (3a + 5b − 8c)2 + (− 8a + 3b + 5c)2 Consider the following statements:
I. The lengths of the sides of P are equal.
+ (5a − 8b + 3c)2 is equal to
II. The angles of P are equal.
(a) 49 (b) 98 III. P is a regular polygon if it is cyclic.
(c) 147 (d) 294
Then,
4. Let ABC be a triangle and M be a point on side AC (a) I is true and I implies II (b) II is true
closer to vertex C than A. Let N be a point on side AB (c) III is false (d) I and III are true
such that MN is parallel to BC and let P be a point
https://iit-jeeacademy.blogspot.com

KVPY Question Paper 2017 Stream : SA 17

6. Consider the following statements: For any integer n, 14. The least value of a natural number n such that
I. n 2 + 3 is never divisible by 17.  n − 1  n − 1  n  n n!
  +  <   , where   = , is
II. n 2 + 4 is never divisible by 17.  5   6   7  r  (n − r )! r !
Then, (a) 12 (b) 13
(a) both I and II are true (b) both I and II are false (c) 14 (d) 15
(c) I is false and II is true (d) I is true and II is false
15. In a Mathematics test, the average marks of boys is
7. Let S be the set of all ordered pairs (x, y) of positive x% and the average marks of girls is y% with x ≠ y. If
integers, with HCF (x, y) = 16 and LCM (x, y) = 48000. the average marks of all students is z%, the ratio of
The number of elements in S is the number of girls to the total number of students is
(a) 4 (b) 8 (c) 16 (d) 32 z−x z− y
(a) (b)
8. Consider the set A of natural numbers n whose units y−x y− x
digit is non-zero, such that if this units digit is z+ y z+ x
(c) (d)
erased, then the resulting number divides n. If K is y−x y− x
the number of elements in the set A, then
(a) K is infinite (b) K is infinite but K > 100
(c) 25 ≤ K ≤ 10 (d) K < 25 PHYSICS
9. There are exactly twelve Sundays in the period from 16. Particles used in the Rutherford’s scattering
January 1 to March 31 in a certain year. Then, the experiment to deduce the structure of atoms
day corresponding to February 15 in that year is (a) had atomic number 2 and were fully ionised
(a) Tuesday (b) Wednesday (b) had atomic number 2 and were neutral
(c) Thursday (c) had atomic number 4 and were fully ionised
(d) not possible to determine from the given data (d) had atomic number 4 and were neutral
10. Consider a three-digit number with the following 17. The number of completely filled shells for the
properties: element 16S32 is
I. If its digits in units place and tens place are
interchanged, the number increases by 36; (a) 1 (b) 2
II. If its digits in units place and hundreds place (c) 3 (d) 4
are interchanged, the number decreases by 198. 18. In an experiment on simple pendulum to determine
Now, suppose that the digits in tens place and the acceleration due to gravity, a student measures
hundreds place are interchanged. Then, the number the length of the thread as 63.2 cm and diameter of
(a) increases by 180 (b) decreases by 270 the pendulum bob as 2.256 cm. The student should
(c) increases by 360 (d) decreases by 540 take the length of the pendulum to be
11. Consider four triangles having sides (5, 12, 9), (5, 12, (a) 64.328 cm (b) 64.3 cm
11), (5, 12, 13) and (5, 12, 15). Among these, the (c) 65.456 cm (d) 65.5 cm
triangle having maximum area has sides. 19. A uniform metallic wire of length L is mounted in
(a) (5,12, 9) (b) (5, 12, 11) two configurations. In configuration 1 (triangle), it is
(c) (5, 12, 13) (d) (5, 12, 15) an equilateral triangle and a voltage V is applied to
12. In a classroom, one-fifth of the boys leave the class corners A and B. In configuration 2 (circle), it is bent
and the ratio of the remaining boys to girls is 2 : 3. in the form of a circle and the potential V is applied
If further 44 girls leave the class, then class the ratio at diametrically opposite points P and Q. The ratio of
of boys to girls is 5 : 2. How many more boys should the power dissipated in configuration 1 to
leave the class so that the number of boys equals that configuration 2 is
of girls? A
(a) 16 (b) 24 (c) 30 (d) 36
P Q
13. Let X ,Y , Z be respectively the areas of a regular
B
pentagon, regular hexagon and regular heptagon
(a) 2/3 (b) 9/8 (c) 5/4 (d) 7/8
which are inscribed in a circle of radius 1. Then,
X Y Z 20. Six objects are placed at the vertices of a regular
(a) < < and X < Y < Z
5 6 7 hexagon. The geometric centre of the hexagon is at the
X Y Z
(b) < < and X > Y > Z origin with objects 1 and 4 on the X-axis (see figure).
5 6 7 The mass of the kth object is mk = ki M |cos θ k |, where
X Y Z
(c) > > and X > Y > Z i is an integer, M is a constant with dimension of
5 6 7 mass and θ k is the angular position of the k th vertex
X Y Z
(d) > > and X < Y < Z measured from the positive X-axis in the
5 6 7 counter-clockwise sense.
https://iit-jeeacademy.blogspot.com

18 KVPY Question Paper 2017 Stream : SA

If the net gravitational force on a body at the centroid 24. Two students P and Q perform an experiment to
vanishes, the value of i is verify Ohm’s law for a conductor with resistance R.
3 2 They use a current source and a voltmeter with least
y counts of 0.1 mA and 0.1 mV, respectively. The plots
of the variation of voltage drop V across R with
4 x 1 current I for both are shown below.
12
10 P
5 6 8
(a) 0 (b) 1 (c) 2 (d) 3 6
4
21. A mirror is placed at an angle of 30° with respect to 2
Y -axis (see figure). A light ray travelling in the 0

V(mV)
–2
negative y-direction strikes the mirror. The direction –4
of the reflected ray is given by the vector –6
–8
y –10
–12
–6 –4 –2 0 2 4 6
I(mA)
O x 12
10 Q
30° 8
6
4
(a) $i (b) $i − 3$j 2
0
(c) 3 $i − $j (d) $i − 2 $j
V(mV)–2
–4
22. A total charge q is divided as q1 and q2 which are kept –6
at two of the vertices of an equilateral triangle of side –8
a. The magnitude of the electric field E at the third –10
–12
vertex of the triangle is to be depicted schematically –6 –4 –2 0 2 4 6
as a function of x = q1 / q. Choose the correct figure. I(mA)

(a) (b) The statement which is most likely to be correct?


(a) P has only random error(s)
E E (b) Q has only systematic error(s)
(c) Q has both random and systematic errors
(d) P has both random and systematic errors
0.0 0.5 1.0 0.0 0.5 1.0
x x 25. A cylindrical vessel of base radius R and height H
has a narrow neck of height h and radius r at one end
(c) (d)
(see figure). The vessel is filled with water (density ρw )
E
and its neck is filled with immiscible oil (density ρ0).
E
Then, the pressure at
2r
0.0 0.5 1.0 0.0 0.5 1.0
x x h

23. The refractive index of water in a biology laboratory


. + 0002
tank varies as 133 . / λ2, where λ is the
wavelength of light. Small pieces of organic matter H
of different colours are seen at the bottom of the tank N M
using a travelling microscope. Then, the image of the
organic matter appears 2R
(a) deeper for the violet pieces than the green ones r2
(a) M is g (hρ0 + Hρw ) (b) N is g (hρ0 + Hρw )
(b) shallower for the blue pieces than the orange ones R2
(c) at the same depth for both the blue and orange pieces ρw HR + ρ0 hr 2
2

(d) deeper for the green pieces than the red ones (c) M is gHρw (d) N is g
R2 + r 2
https://iit-jeeacademy.blogspot.com

KVPY Question Paper 2017 Stream : SA 19

26. Two cars S1 and S2 are moving in coplanar concentric Ignoring magnification effects, consider the following
circular tracks in the opposite sense with the periods statements.
of revolution 3 min and 24 min, respectively. At time (I) First image has been viewed from the planar
t = 0, the cars are farthest apart. Then, the two cars side of a plano-concave lens and second image
will be from the planar side of a plano-convex lens.
(a) closest to each other at t = 12 min and farthest at (II) First image has been viewed from the concave
t = 18 min side of a plano-concave lens and second image
(b) closest to each other at t = 3 min and farthest at from the convex side of a plano-convex lens.
t = 24 min
(III) First image has been viewed from the concave
(c) closest to each other at t = 6 min and farthest at
side of a plano-concave lens and second image
t = 12 min
from the planar side of a plano-convex lens.
(d) closest to each other at t = 12 min and farthest at
t = 24 min (IV) First image has been viewed from the planar
side of a plano-concave lens and second image
27. In the circuit shown below, a student performing from the convex side of a plano-convex lens.
Ohm’s law experiment accidently puts the voltmeter
Which of the above statements are correct?
and the ammeter as shown in the circuit below. The
reading in the voltmeter will be close to (a) Only statement III is correct
6V (b) Only statement II is correct
2kΩ
(c) Only statements III and IV are correct
(d) All statements are correct
8kΩ
V
CHEMISTRY
A 31. The IUPAC name for the following compound is
(a) 0 V (b) 4.8 V (c) 6.0 V (d) 1.2 V
28. The Bhagirathi and the Alaknanda merge at
Deoprayag to form the Ganga with their speeds in (a) 4,6-dimethylheptane (b) 1,3,5-trimethylhexane
the ratio 1 : 1: 5. The cross-sectional areas of the (c) 2,4-dimethylheptane (d) 2,4,6-trimethylhexane
Bhagirathi, the Alaknanda and the Ganga are in the
ratio 1 : 2 : 3. Assuming streamline flow, the ratio of 32. The stability of carbocations
the speed of Ganga to that of the Alaknanda is + + + +

(a) 7 : 9 (b) 4 : 3 (c) 8 : 9 (d) 5 : 3 (CH3)3C (CH3)2C(OCH3) CH3CH2CH2CH2 CH3CHCH2CH3


I II III IV
29. A long cylindrical pipe of radius 20 cm is closed at its
upper end and has an airtight piston of negligible follows the order
mass as shown. When a 50 kg mass is attached to (a) III < IV < II < I (b) III < IV < I < II
the other end of piston, it moves down by a distance (c) IV < III < II < I (d) IV < III < I < II
∆l before coming to equilibrium. Assuming air to be 33. The acidity of compounds I-IV in water
an ideal gas, ∆l / l (see figure) is close to (g = 10 m/ s2, I. ethanol II. acetic acid
atmospheric pressure is 105 Pa), III. phenol IV. acetonitrile
follows the order
l (a) IV < I < III < II (b) I < II < III < IV
(c) IV < I < II < III (d) IV < III < I < II
∆l
34. In the following reaction,
O

NH2 Br2
(a) 0.01 (b) 0.02 (c) 0.04 (d) 0.09 KOH

30. The word KVPY is written on a board and viewed


through different lenses such that board is at a the major product is
distance beyond the focal length of the lens. Br CO2H

KVPY (a) (b)


KVPY
Br
First image Second image
https://iit-jeeacademy.blogspot.com

20 KVPY Question Paper 2017 Stream : SA

NH2 CONH2 43. The oxidation states of P atom in POCl 3 , H2 PO3 and
(c) (d) H4 P2O6 , respectively are
(a) +5, +4, +4 (b) +5, +5, +4
(c) +4, +4, +5 (d) +3, +4, +5
Br
44. A solution (5 mL) of an acid X is completely
35. The reddish brown precipitate formed in the neutralised by y mL of 1M NaOH. The same volume
Fehling’s test for aldehydes (RCHO) is due to the (y mL) of 1M NaOH is required to neutralise 10 mL
formation of of 0.6 M of H2 SO4 completely. The normality (N) of
(a) Cu (b) Cu 2O the acid X is
(c) CuO (d) ( RCOO)2 Cu (a) 1.2 (b) 2.4 (c) 4.8 (d) 0.6
36. The reducing ability of the metals K, Au, Zn and Pb 45. 1.25 g of a metal (M) reacts with oxygen completely
follows the order to produce 1.68 g of metal oxide. The empirical
(a) K > Pb > Au > Zn (b) Pb > K > Zn > Au formula of the metal oxide is
(c) Zn > Au > K > Pb (d) K > Zn > Pb > Au [molar mass of M and O are 69.7 g mol −1 and 16.0 g mol −1 ,
respectively]
37. White phosphorus catches fire in air to produce dense (a) M2O (b) M2O3 (c) MO2 (d) M3 O4
white fumes. This is due to the formation of
(a) P4O10 (b) PH 3
(c) H 3 PO3 (d) H 3 PO2 BIOLOGY
38. The maximum number of electrons that can be filled 46. According to Watson-Crick model, hydrogen bonding
in the shell with the principal quantum number n = 4 in a double-stranded DNA occurs between
is (a) adenine and guanine (b) adenine and thymine
(a) 64 (b) 26 (c) 18 (d) 32 (c) cytosine and adenine (d) guanine and thymine
39. At a constant pressure p, the plot of volume (V ) as a 47. Which one of the following statements about mitosis
function of temperature (T ) for 2 moles of an ideal gas is correct?
gives a straight line with a slope 0.328 LK −1. The (a) One nucleus gives rise to 4 nuclei
value of p (in atm) is closest to (b) Homologous chromosomes synapse during anaphase
[Gas constant, R = 00821
. L atm mol −1 K −1 ] (c) The centromeres separate at the onset of anaphase
(a) 0.25 (b) 0.5 (d) Non-sister chromatids recombine
(c) 1.0 (d) 2.0 48. Gaseous exchange of oxygen and carbon dioxide
40. Which of the following transformations can be carried between alveolar air and capillaries takes place by
out by using HI as a reducing agent, under acidic (a) active transport
conditions? (b) diffusion
[Given : I2 (s) → 2I− ; E ° = 054
. V] (c) carrier-mediated transport
(d) imbibition
(i) Cu + → Cu (s); E ° = 052
. V
(ii) Cr3 + → Cr 2+ ; E ° = − 041
. V 49. Of the periods listed below, which one is the earliest
3+ 2+ ° period when ostracoderms, the jawless and finless
(iii) Fe → Fe ; E = 0.77 V
fishes, appeared?
(iv) Fe 2+ → Fe (s); E ° = − 044
. V (a) Devonian period (b) Cambrian period
(a) (i) and (iii) (b) (ii) and (iv) (c) Carboniferous period (d) Silurian period
(c) Only (iii) (d) Only (ii)
50. Scurvy is caused by the deficiency of
41. C 60 emerging from a source at a speed (v) has a (a) nicotinic acid (b) ascorbic acid
de Broglie wavelength of 11.0 Å. The value of v (c) pantothenic acid (d) retinoic acid
(in ms−1 ) is closest to 51. Optical activity of DNA is due to its
−34 (a) bases (b) sugars
[Planck’s constant h = 6626
. × 10 Js]
(c) phosphates (d) hydrogen bonds
(a) 0.5 (b) 2.5 (c) 5.0 (d) 30
52. The monarch butterfly avoids predators such as
42. The lattice energies of NaCl, NaF, KCl and RbCl birds by
follow the order (a) changing colour frequently
(a) KCl < RbCl < NaCl < NaF (b) flying away from the predator swiftly
(b) NaF < NaCl < KCl < RbCl (c) producing a chemical obnoxious to the predator
(c) RbCl < KCl < NaCl < NaF (d) producing ultrasonic waves
(d) NaCl < RbCl < NaF < KCl
https://iit-jeeacademy.blogspot.com

KVPY Question Paper 2017 Stream : SA 21

53. Filariasis is caused by 58. Match the diseases given in Column I with the
(a) Entamoeba histolytica (b) Plasmodium falciparum principal causal organisms in Column II and choose
(c) Trypanosoma brucei (d) Wuchereria bancrofti the correct combination.
54. Which one of the following conversions does not Column I Column II
happen under anaerobic conditions? (P) AIDS (i) HBV
(a) Glucose to ethanol by Saccharomyces (Q) Syphilis (ii) Neisseria sp.
(b) Lactose to lactic acid by Lactobacillus (R) Viral hepatitis (iii) Treponema sp.
(c) Glucose to CO2 and H 2O by Saccharomyces (S) Gonorrhoea (iv) HIV
(d) Cellulose to glucose by Cellulomonas
(a) P-iv, Q-iii, R-i, S-ii (b) P-iv, Q-ii, R-i, S-iii
55. An amount of 18 g glucose corresponds to (c) P-i, Q-ii, R-iv, S-iii (d) P-i, Q-iv, R-ii, S-iii
(a) 1.8 mole (b) 1 mole (c) 0.18 mole (d) 0.1 mole 59. Chromosomes are classified based on the position of
56. The number of electrons required to reduce one centromere. A chromosome having a terminal
molecule of oxygen to water during mitochondrial centromere is called
oxidation is (a) metacentric (b) telocentric
(a) 4 (b) 3 (c) 2 (d) 1 (c) sub-metacentric (d) acrocentric
57. Which one of the following molecules is derived from 60. Which one of the following options lists the primary
pantothenic acid? energy source(s) for all forms of life on the earth?
(a) Thiamine pyrophosphate (a) Light, inorganic substances
(b) Nicotinamide adenine dinucleotide phosphate (b) Inorganic substances, organic substances
(c) Flavin adenine dinucleotide phosphate (c) Light, organic substances
(d) AcetylCo-A (d) N2 , CO2

PART-II (2 Marks Questions)


MATHEMATICS 64. Let C1, C 2 be two circles touching each other
61. Let ABCD be a trapezium with parallel sides AB and externally at the point A and let AB be the diameter
CD such that the circle S with AB as its diameter of circle C1. Draw a secant BA3 to circle C 2,
touches CD. Further, the circle S passes through the intersecting circle C1at a point A1 (≠ A), and circle C 2
at points A2 and A3 . If BA1 = 2, BA2 = 3 and BA3 = 4,
mid-points of the diagonals AC and BD of the
then the radii of circles C1 and C 2 are respectively
trapezium. The smallest angle of the trapezium is
π π 30 3 30 5 7 5
(a) (b) (a) , (b) ,
3 4 5 10 2 10
π π 6 6 10 17 10
(c) (d) (c) , (d) ,
5 6 2 2 3 30
 a c 65. Let a , b, c, d be real numbers between − 5 and 5 such
62. Let S be the set of all points  ,  on the circle with
b d that
radius 1 centred at (0, 0) where a and b are relatively | a |= 4 − 5 − a ,|b| = 4 + 5 − b ,|c| = 4 − 5 + c ,
prime integers, c and d are relatively prime integers
(that is HCF (a , b) = HCF (c, d) = 1), and the integers |d | = 4 + 5 + d
b and d are even. Then, the set S Then, the product abcd is
(a) is empty (a) 11 (b) − 11
(b) has four elements (c) 121 (d) − 121
(c) has eight elements
(d) is infinite
PHYSICS
63. Suppose we have two circles of radius 2 each in the
plane such that the distance between their centers is 66. Persons A and B are standing on the opposite sides of
2 3 . The area of the region common to both circles a 3.5 m wide water stream which they wish to cross.
lies between Each one of them has a rigid wooden plank whose
(a) 0.5 and 0.6 (b) 0.65 and 0.7 mass can be neglected. However, each plank is only
(c) 0.7 and 0.75 (d) 0.8 and 0.9 slightly longer than 3 m. So, they decide to arrange
them together as shown in the figure schematically.
https://iit-jeeacademy.blogspot.com

22 KVPY Question Paper 2017 Stream : SA

With B (mass 17 kg) standing, the maximum mass of given charge Q, so that balls move away from each
A, who can walk over the plank is close to other with each thread making an angle of 45° from
A 3m the vertical. The value of Q is close to
B  1 
 = 9 × 109 in SI units
 4πε 0 
(a) 1 µC . µC
(b) 15 (c) 2 µC (d) 2.5 µC
3.5 m 70. Two parallel discs are connected by a rigid rod of
(a) 17 kg (b) 65 kg (c) 80 kg (d) 105 kg length L = 05. m centrally. Each disc has a slit
67. Two different liquids of same mass are kept in two oppositely placed as shown in the figure. A beam of
identical vessels, which are placed in a freezer that neutral atoms are incident on one of the discs axially
extracts heat from them at the same rate causing at different velocities v, while the system is rotated at
each liquid to transform into a solid. The schematic angular speed of 600 rev/second, so that atoms only
figure below shows that temperature T versus time t with a specific velocity emerge at the other end.
plot for the two materials. We denote the specific Calculate the two largest speeds (in metre/second) of
heat of materials in the liquid (solid) states to be C L1 the atoms that will emerge at the other end.
(C S1) and C L 2 (C S 2 ), respectively.
T
v ω

1
2 (a) 75, 25 (b) 100, 50 (c) 300, 100 (d) 600, 200
t
Choose the correct option given below. CHEMISTRY
(a) CL1 < CL 2 and CS1 < CS 2 (b) CL1 > CL 2 and CS1 < CS 2
(c) CL1 > CL 2 and CS1 > CS 2 (d) CL1 < CL 2 and CS1 > CS 2 71. Among the following compounds, E/Z isomerism is
possible for
68. A ray of light originates from inside a glass slab and (a) 2-methylbut-2-ene (b) 2-methylbut-1-ene
is incident on its inner surface at an angle θ as shown (c) 3-methylpent-1-ene (d) 3-methylpent-2-ene
below.
72. In the reaction,
Glass slab 1. NaNH2, ∆
CH3
H3C C C H 2. x
H3C
2
3. y
x
θ
0 x and y, respectively are
–2 (a) x = CH 3OH; y = Pd / BaSO4 , quinoline, H 2
(b) x = CH 3 I; y = Pd / BaSO4 , quinoline, H 2
(c) x = CH 3 I; y = Na in liq. NH 3
(d) x = CH 3OH; y = Na in liq. NH 3

Screen
73. Among the following molecules, the one with the
largest bond angle at the central atom is
In this experiment, the location x of the spot where
(a) ClF 3 (b) POCl 3 (c) BCl 3 (d) SO3
the ray hits the screen is recorded. Which of the
following correctly shows the plot of variation of x 74. A compound has the following composition by weight
with the angle θ ? : Na = 18.60%, S = 25.80%, H = 4.02% and O = 51.58%.
Assuming that all the hydrogen atoms in the
compound are part of water of crystallisation, the
0 0 0 0 correct molecular formula of the compound is
x x x x
(a) Na 2 S2O3 ⋅ 3H 2O (b) Na 2 SO4 ⋅ 5H 2O
(c) Na 2 SO4 ⋅ 10H 2O (d) Na 2 S2O3 ⋅ 5H 2O
θ θ θ θ
75. X g of ice at 0°C is added to 340 g of water at 20°C.
A. B. C. D. The final temperature of the resultant mixture is
(a) A (b) B (c) C (d) D 5°C. The value of X (in g) is closest to
69. Four identical pendulums are made by attaching a [Heat of fusion of ice = 333 J/g; specific heat of water
small ball of mass 100 g on a 20 cm long thread and = 4.184 J/g.K]
suspended from the same point. Now, each ball is (a) 80.4 (b) 52.8 (c) 120.6 (d) 60.3
https://iit-jeeacademy.blogspot.com

KVPY Question Paper 2017 Stream : SA 23

77. A 25,000 Da protein contains a single binding site for


BIOLOGY a molecule (ligand), whose molecular weight is 2,500
76. Considering ABO blood grouping system in humans, Da. Assuming high affinity and physiologically
during blood transfusion some combinations of blood irreversible binding, the amount of the ligand
groups are compatible (✓), whereas the others are required to occupy all the binding sites in 10 mg
incompatible (✗). Which one of the following options protein will be
is correct? (a) 0.1 mg (b) 1 mg
(c) 10 mg (d) 100 mg
Recipient
O A B AB 78. In an in vitro translation experiment, poly (UC) RNA
O ✗ ✗ P template produced poly (Ser-Leu), while poly (AG)

RNA template produced poly (Arg-Glu) polypeptide.
Donor

(a) A P ✗ P ✗
P Which one of the following options represents correct
B P ✗ ✗
interpretations of the codons assignments for Ser,
AB P P P P
Leu, Arg and Glu?
(a) Ser-UCU, Leu-CUC, Arg-AGA, Glu-GAG
Recipient
(b) Ser-CUC, Leu-GAG, Arg-UCU, Glu-AGA
O A B AB (c) Ser-AGA, Leu-UCU, Arg-GAG, Glu-CUC
O ✗ ✗ ✗ ✗ (d) Ser-GAG, Leu-AGA, Arg-CUC, Glu-UCU
Donor

A P ✗ P ✗
(b)
B ✗
79. A single bacterium is actively growing in a medium
P P ✗
that supports its growth to a number of 100 million.
AB P P P ✗
Assuming the division time of the bacterium as
3 hours and the lifespan of non-dividing bacteria as
Recipient
5 hours, which one of the following represents the
O A B AB maximum number of bacteria that would be present
O P ✗ ✗ ✗ at the end of 15 hours?
Donor

A P P ✗ ✗ (a) 10 (b) 64 (c) 24 (d) 32


(c)
B P ✗ P ✗ 80. A couple has two sons and two daughters. Only one
AB P P P P son is colourblind and the rest of the siblings are
normal. Assuming colourblindness is sex-linked,
Recipient which one of the following would be the phenotype of
O A B AB the parents?
O P P P P (a) Mother would be colourblind, father would be normal
A ✗ P ✗ P (b) Father would be colourblind, mother would be normal
Donor

(d) ✗
B ✗ P P (c) Both the parents would be normal
AB ✗ ✗ ✗ P (d) Both the parents would be colourblind

Answers
PART-I
1 (c) 2 (b) 3 (c) 4 (a) 5 (d) 6 (d) 7 (b) 8 (d) 9 (c) 10 (d)
11 (c) 12 (b) 13 (d) 14 (c) 15 (a) 16 (a) 17 (b) 18 (b) 19 (b) 20 (a)
21 (c) 22 (c) 23 (b) 24 (d) 25 (a) 26 (d) 27 (c) 28 (c) 29 (c) 30 (d)
31 (c) 32 (b) 33 (a) 34 (c) 35 (b) 36 (d) 37 (a) 38 (d) 39 (b) 40 (c)
41 (*) 42 (c) 43 (a) 44 (b) 45 (b) 46 (b) 47 (c) 48 (b) 49 (b) 50 (b)
51 (b) 52 (c) 53 (d) 54 (c) 55 (d) 56 (a) 57 (d) 58 (a) 59 (b) 60 (a)

PART-II
61 (d) 62 (a) 63 (c) 64 (a) 65 (a) 66 (c) 67 (b) 68 (a) 69 (b) 70 (d)
71 (d) 72 (c) 73 (a) 74 (d) 75 (d) 76 (d) 77 (b) 78 (a) 79 (d) 80 (c)

* No option is correct.
https://iit-jeeacademy.blogspot.com

24 KVPY Question Paper 2017 Stream : SA

Solutions
1. (c) Let triangle T is PQR and other ∆ABC ~ MPC We know,
triangle is ABC. Area of ∆ABC AC 2 Product of two number = HCF × LCM
= …(ii)
Area of ∆MPC MC 2 ∴ xy = 16 × 48000
P A
From Eqs. (i) and (ii), we get xy = 16 × 16 × (31 × 23 × 53 )
Area of ∆ANM AM 2 As HCF of (x, y) = 16
=
Area of ∆MPC MC 2 23 can be selected in 1 ways and 31 and 53
Area of ∆ANM + Area of ∆MPC can be selected in (1 + 1) (3 + 1) = 8 ways
Q R B C Area of ∆MPC
P ∴ Number of ordered pairs = 8
AM 2 + MC 2
B C = 8. (d) Let two digits number
MC 2
ab = 10a + b, b ≠ 0
Now, Area of ∆ANM + Area of ∆MPC
= Area of ∆ABC − Area of BNMP if b is erased.
Q R 13 (Area of ∆ABC ) AM 2 + MC 2 Then, the resulting number is a.
∴ =
A
18 (Area of ∆ MPC ) MC 2 ∴ab is divisible by a if ab is multiple of c.
∴Such number are 11, 12, 13, 14, 15, 16,
A can taken position if ∆ABC ~ ∆PQR. 13  (AC 2 )  AM 2 + MC 2
From Eq. (iii),   = 17, 18, 19, 22, 24, 26, 28, 33, 36, 39, 44,
We can arrange A , B , C in 3! ways 18  MC 2  MC 2 48, 55, 66, 77, 88, 99.
= 6 ways
⇒ 13 (AM + MC )2 = 18 (AM 2 + MC 2 ) ∴Total number are 23.
Total position of A can take = 3! × 2 = 12 AM
ways ⇒ =5 Hence, K < 25
MC
2. (b) We have, n ∈ {2, 3, 4, 5, 6, …, 200} 9. (c) There are 90 days from 1 January
5. (d) We have, l1 , l2 , l3 ..., ln be the to 31 March (Non-leap year)
1
has terminating decimal of n = 2a × 5b lengths of the side of arbitrary n sided If year 13 leap year, then total number of
n non-degenerate polygon P and days = 91(13 weeks)
∴ n = 2, 4, 5, 8, 10, 16, 20, 25, 32, 40, 50, l1 l l ln − 1 ln
+ 2 + 3 + ... + + = n, n ≥ 4 But we have 12 Sunday
64, 80, 100, 125, 128, 160, 200 l2 l3 l4 ln l1
∴ Total number of n = 18 ∴12 weeks
Using AM ≥ GM, we get ∴Ist Jan will be Monday as there will be
3. (c) We have, l1 l2 l3 ln
+ + + ... + 1/ n 90 days January 1 to 31 March.
a + b + c = 0 and a 2 + b2 + c2 = 1 l2 l3 l4 l1  l1 l2 ln 
≥  × × ... ×  ∴15th February will be Thursday.
Now (3a + 5b − 8c)2 + (− 8a + 3b + 5c)2 n  l2 l3 l1 
10. (d) Let three digits number be
+ (5a − 8b + 3c)2
l1 l ln
2 2 2
= 9a + 25b + 64c − 48ac + 30ab ∴ + 2 + ... + ≥n 100x + 10 y + z.
l2 l3 l1
− 80bc + 64a 2 + 9b2 + 25c2 − 80ac According to problem,
− 48ab + 30bc + 25a 2 + 64b2 + 9c2 ∴ n ≥ n ⇒n = n 100x + 10 y + z = 100x + 10z + y − 36
+ 30ac − 8ab − 48bc So, AM = GM ⇒ 9 y − 9z + 36 = 0
2 2 2
= 98(a + b + c ) − 98 (ab + bc + ca ) ∴ l1 = l2 = l3 … = ln ⇒ y− z + 4= 0 …(i)
= 98 (a 2 + b2 + c2 ) ∴ The length of sides of P are equal and ⇒ 100x + 10 y + z = 100z + 10 y + x + 198
 (a + b + c)2 − (a 2 + b2 + c2 )  P is regular polygon of it is cyclic. ⇒ x− z − 2= 0 …(ii)
− 98   Now, (100x + 10 y + z ) − (100 y + 10x + z )
 2  6. (d) Let n 2 + 3 is divisible by 17
= 90(x − y)
0 − 1 ∴ n 2 + 3 = 17K [K ∈ N ]
= 98(1) − 98  = 98 + 49 = 147 = 90(6) [Q from Eqs. (i) and (ii)]
 2  ⇒ n 2 = 17K − 3
= 540
4. (a) ∆ ABC ~ ∆ANM ⇒ n 2 = 3 (17m − 1) [Q K = 3 m]
∴ So, on interchanging for digit at tens
A 3 (17 m − 1) is a perfect square is not place and hundred place, the value of
possible. number is decreased by 540.
∴ n 2 + 3 is never divisible by 17. 11. (c) We have,
n 2 + 4 put n = 9 Four triangle having sides are
N M (9)2 + 4 = 81 + 4 = 85 is divisible by 17. (5, 12, 9), (5, 12, 11), (5, 12, 13), (5, 12, 15)
∴I is true and II is false. A right triangle has maximum area.
P 7. (b) We have, ∴ Among these the triangle whose sides
B C
Area of ∆ABC AC 2 HCF (x, y) = 16 (5, 12, 13) form a right angled triangle.
∴ = … (i)
Area of ∆ANM AM 2 LCM (x, y) = 48000 ∴ It has maximum area.
https://iit-jeeacademy.blogspot.com

KVPY Question Paper 2017 Stream : SA 25

12. (b) Let the number of boys and girls Now, given 2 L
⇒ RAB = × ×x
in classroom is x and y, respectively. Bx + Gy 3 3
=z
x − x/5 2 4x 2 B+G Power dissipated is
Given, = ⇒ =
y 3 5y 3 B z− y V2 9V 2
x 5 ⇒ B (x − z ) = G (z − y ) = = P1 = = ...(i)
⇒ = …(i) G x−z RAB 2Lx
y 6 G 1 1 x−z
Now, = = = In case II,
x − x/5 5 4x 5
Also, = ⇒ = B + G B + 1 z − y+1 x − y
y − 44 2 5 ( y − 44) 2 G x−z P Q
⇒ 8x = 25 y − 1100 … (ii) G z−x
⇒ =
From Eqs. (i) and (ii), we get B+G y−x
x = 50, y = 60 +
16. (a) Particles used in Rutherford’s –
Let z number of boy leaves so number of scattering experiment (Geiger-Marsden V
boys and number of girls are equal. experiment) are α-particles derived from πrx × πrx π 2r 2x2
∴ 50 − 10 − z = 60 − 44 a tube of radium emanation (or radon). RPQ = ( πrx|| πrx) = =
πrx + πrx 2 πrx
z = 40 − 16 = 24 α-particles are helium nuclei 42 He, they 1 1 L Lx
are fully ionised and have atomic = πrx = π × x=
13. (d) We have, 2 2 2π 4
X , Y , Z be respectively the area of a number 2.
So, power dissipated is
regular pentagon, regular hexagon and 17. (b) Atomic number of 16 S32 is 16. V2 4V 2
regular heptagon which are inscribed in Its electronic configuration using 2n 2 rule P2 = =
RPQ Lx
radius of unit circle. is
16 S = 1s2 , 2s2 p 6 , 3s2 p 4 Ratio of power dissipated in two cases is
14243 123
[Ne] Unfilled P1 9V 2 / 2Lx 9
1 = =
So, number of fully filled orbits or shells P2 4V 2 / Lx 8
2π is 2.
5 20. (a) For a mass m at centroid of
1 18. (b) Length of pendulum = Length of hexagon (at origin), net force is zero
thread + Radius of bob when ΣFx = 0 and ΣFy = 0.
2.256
= 63.2 + = 63.2 + 1128
.
2 3
1 2π = 64.328 cm 2
∴ X = 5× × 1 × sin
2 5 mk=ki⋅M⋅
But now the student must apply rule for
5 2π r m |cos θk|
X = sin taking significant digits in a
2 5 measurement. 4 O 1
Similarly,
In addition or subtraction,
6 2π 7 2π
Y = sin and Z = sin Number of digits after decimal in result
2 6 2 7 5 6
X 1 2π Y 1 2π Z 1 2π = Least number of digits after decimal in
= sin , = sin , = sin quantities added
5 2 5 6 2 3 7 2 7 Now, ΣFx = sum of all x-components of
2π 2π 2π So, length of pendulum = 64.3 cm. forces on m due to masses at vertices of
sin > sin > sin hexagon.
5 6 7 19. (b) Let a = side length of equilateral
Gm
X Y Z
∴ > > and X < Y < Z triangle, r = radius of circle and x = = 2 (Σ (ki M | cos θk | ⋅ cos θk ))
5 6 7 resistance per unit length of wire used. r
GmM i
14. (c) Given, L L = (1 | cos 0°|⋅ cos 0° + 2 i |cos 60° |
Then, L = 3a = 2πr or a = and r = r2
n −1
C5 + n − 1C6 < nC7 3 2π ⋅ cos 60°+ 3 i | cos 120° |⋅ cos 120°
n
C6 < nC7 Now, in case I, + 4 i |cos 180° |⋅ cos 180°
n+1 A
[Q nCr − 1 + nCr = Cr ] + 5 i |cos 240° |⋅ cos 240°
n! n! – + + 6 i |cos 300° |⋅ cos 300° )
⇒ < V
(n − 6)! 6! (n − 7)! 7!
GMm  i 2 i
3i 5i 6i 
= ⋅ 1 + − − 4i − + 
⇒ n − 6> 7 r 2
 4 4 4 4
⇒ n > 13 B
Equivalent resistance across AB is As ΣFx = 0, for net force on m to be zero.
∴ Least value of x = 14
ax × 2ax we have
15. (a) Let the number of boy = B RAB = (ax|| 2ax) =
ax + 2ax 2i 3i 5i 6i
1i + − − 4i − + =0
and number of girls = G 2a 2x2 2 4 4 4 4
= = ax
Sum of marks obtained by boys = Bx 3ax 3 Above equation is satisfied with i = 0.
∴Sum of marks obtained by girls = Gy
https://iit-jeeacademy.blogspot.com

26 KVPY Question Paper 2017 Stream : SA

21. (c) Following laws of reflection, Now, µ orange < µ blue 27. (c) The resistance of voltmeter is
reflected ray makes an angle of 30° with ⇒ (Apparent depth)blue very high and resistance of ammeter is
mirror as shown below. < (Apparent depth)orange very low. When ammeter is put in
parallel to 8 kΩ resistor, nearly whole of
24. (d) For P errors are both positive and
A Incident negative. current goes through the ammeter.
ray Hence, circuit is equivalent to following.
N For Q errors are only positive.
So, P has both random and systematic 2kΩ
30°

+ –
60°
errors. Low
O
x 25. (a) Pressure is same at all the points resistance
30°
of base.
30

i.e. Pressure at M = Pressure at N V


°

Reflected Also, pressure applied anywhere to the High resistance


ray fluid is equally transmitted in all
B directions. So, maximum potential drop occurs in the
So, if a vector x $i + y$j is along the So, pressure at base = pressure due to oil voltmeter (high resistance). Hence,
reflected ray, then column of height h + pressure due to reading of voltmeter is nearly 6 V.
y y 1 water column of height H. 28. (c) As water is not stored anywhere.
tan (−30° ) = or = −
x x 3 ⇒ ρo gh + ρw gH ⇒ g(ρ0⋅ h + ρw H ) So, volume flow rate of Ganga = volume
This is correct with option (c). 26. (d) T=24 min
flow rate of Bhagirathi + volume flow
22. (c) When q1 and q2 are the magnitudes S2 rate of Alaknanda
of charges at two vertices of an equilateral
triangle of side a, magnitude of electric Bha
girat
hi Ab
field at third vertex is vb
Ganga vg
E2 E E Ag
1 va
S1 Aa
da
nan
T=3 min Alak

Positions of cars at t = 0s
∴By equation of continuity, we have
At t = 12 min, car S1 has completed three ⇒ Ag vg = Ab vb + Aa va ...(i)
rounds and it is at its position. It is given that area of flow of Ganga,
q1 q2 At t = 12 min, car S2 completed half round Alaknanda and Bhagirathi are in ratio,
2 2 2 and it is at diametrically opposite point Ag : Aa : Ab = 3 : 2 : 1
E=  kq1  +  kq2  + 2k q1 q2 cos 60° as shown below.
    or Ag = 3x, Aa = 2x, Ab = x
 a   a  a2
k 2 2 Also, ratio of speeds of Bhagirathi and
= q1 + q2 + q1 q2
a Alaknanda is
k 3
= q12 + (q − q1 )2 + q1 (q − q1 ) vb : va = 1 :
a S1 2
 2  3
k   q1  q  or vb = y, va = y
=   + 1−  1   2
aq   q  q
  S2
Substituting these values in Eq. (i), we
 2  get
k   q1 1  3 Positions of cars at t = 12 min 3
=  −  +  3x ⋅ vg = x ⋅ y + 2x ⋅ y = 4xy
aq   q 2  4 So, cars are closest at t = 12 min. 2
 
4
q 1 At t = 24 min, cars S1 and S2 are both at So, vg = y
So, field is minimum when 1 = . their initial positions and so are farthest, 3
q 2
as shown below. ∴ Ratio of speed of Ganga to that of
This condition is satisfied in graph (c). S2 Alaknanda is
23. (b) As, refractive index, 4
y
vg 8
µ = 133
. +
0.002 = 3 =
3
λ2 va y 9
So, µ is more for small wavelengths. 2
i.e. µ orange < µ green < µ blue 29. (c) Initially pressure inside the
S1
cylinder is atmospheric pressure p0 .
real depth
As, µ= When mass m is attached to piston and it
apparent depth
comes down by a distance ∆l, let pressure
1 is p.
⇒ Apparent depth ∝ Hence, cars are farthest from each other
µ at t = 24 min.
https://iit-jeeacademy.blogspot.com

KVPY Question Paper 2017 Stream : SA 27

Then, in equilibrium, So, a plano-concave lens acts like a 34. (c) O


p0V 0 = pV ⇒ p0 (A ) (l) = pA (l + ∆l) diverging lens weather object is viewed NH2
from plane side or curved side. NH2 Br2

p0 Hence, image appears erect in both cases. KOH


l p
∴First image appears same when viewed Benzamide Aniline

from plane or curved side of a This reaction is known as Hofmann


∆l bromamide reaction. It is used for
plano-concave lens.
preparing amine containing one carbon
Similarly, a plano-convex lens is a
Initially less than the starting amide. In this
converging lens from both side view.
m reaction, migration of an alkyl or aryl
Finally So, second image appears always group takes place from carbonyl carbon of
inverted in both cases. the amide to N-atom.
So, final pressure will be
p0 Al p0 l 31. (c) The IUPAC name for the 35. (b) Fehling’s reagent is a mixture of
p= = following compound is
A (l + ∆ l) (l + ∆ l) aqueous copper sulphate and alkaline
sodium potassium tartarate. When an
In equilibrium, weight of mass m is 6 4 2 aldehyde is heated with Fehling’s reagent
balanced by force of suction due to 7 5 3 1
a reddish brown precipitate is obtained
reduced pressure p. 2, 4-dimethylheptane
and the aldehydes are oxidised to
Q ( p0 − p ) A = mg 32. (b) As the size of alkyl group goes on corresponding carboxylate anion. This
 p0 l  p A l increasing, the + I effect exerted by it reddish brown precipitate is due to the
⇒  p0 −  A = mg ⇒ 0 = +1
 l + ∆l  mg ∆l becomes strong and, thus the carbocation formation of copper oxide.
105 × π × (20 × 10−2 )2 l will be more stable. So, among 1°, 2° and ∆
⇒ = +1 3° carbocation, 3° will be most stable. RCHO + 2Cu2++5OH– RCOO–
50 × 10 ∆l (Fehling's solutions)
+
22 × 8 l Although (CH 3 )2 C (OCH 3 ) will be highly + Cu2O↓ +3H2O
⇒ − 1= Red brown
7 ∆l stable among the given carbocations due (ppt).
l 169 ∆l to resonance stabilisation.
⇒ = or ≈ 0.04 36. (d) The reducing ability of metals
∆l 7 l + can be determined by electrochemical
30. (d) For a plano-concave lens, when (CH3)2 C O CH3
series. In this series, various elements
view is from concave side. Radius of +
are arranged according to their
(CH3)2 C O CH3
curvature of surface 1 is R1 = ∞ and decreasing values of standard reduction
radius of curvature of surface 2 is R2 = R. Thus, the correct order of stability of potentials. The reducing ability of the
carbocations will be metal increases as you go up the series.
Object Image +
CH3CH2CH2CH2 < CH3CHCH2CH3 <
+
The increasing order of E° values of given
metals are,
(1°) (2°)
III IV K < Zn < Pb < Au.
(CH3)3C+ < (CH3)2C(OCH3)
+
Thus, the correct order of reducing ability
of metals K, Au, Zn and Pb follows the
(3°)
So, focal length of lens using I II order
1  1 1  3  K > Zn > Pb > Au.
= (µ − 1)  −  µ = for glass 33. (a) The acidity of compounds in
f  R1 R2   2  water depends upon the ease with which 37. (a) White phosphorus is highly
we have, when viewed from curved side, it can lose H + ions. Acetic acid is the reactive and catches fire when exposed to
−R air and produces white dense fumes of
f = strongest acid as the negative charge on
phosphorus oxide, P4 O10 .
(µ − 1) carboxylate ion (conjugate base) is
P4 + 5O2 → P4 O10
and R1 = − R , R2 = ∞. delocalised over two oxygen atoms.
Hence, H+ ion can be easily lost. The next 38. (d) The maximum number of
strongest acidic compound phenol. This is electrons that can be filled in the shell
with principle quantum number, n = 2n 2
because the phenoxide ion is resonance
stabilised. This easily allows the H to For n=4
Object Image leave as H+ ion. Among acetonitrile and Maximum number of electrons = 2(4)2 = 32
ethanol, ethanol is more acidic, this is 39. (b) According to ideal gas equation
because in ethanol the H-atom is directly V nR
When viewed from plane side, pV = nRT ⇒ = = slope
1 1 attached to more electronegative atom, O. T p
= (µ − 1)  −  Given, slope = 0.328, n = 2
f  R Thus, the correct order of acidity of
compound I-IV in water will be nR 2 × 0.0821
−R ∴ p= = = 0.500 atm
⇒ f = IV < I < III < II. slope 0.328
(µ − 1)
https://iit-jeeacademy.blogspot.com

28 KVPY Question Paper 2017 Stream : SA

40. (c) The more positive E° value of Number of equivalents of H 2 SO4 diffuse between the air in the alveoli and
metal, feasible transformation can be = 0.6 × 10 × 2 = 12 the blood in the capillaries. Diffusion is
carried out by using HI as reducing agent (Number of equivalents) NaOH the movement of gas from an area of high
under acidic conditions. concentration to an area of low
= (Number of equivalents) H 2 SO 4
concentration.
As E° = 0.77 V (Fe3 + → Fe2+ ) is more = y = 12 mL
positive than, E° = 0.54 (I2 (s) → 2I− ) thus Also, number of equivalents of acid
49. (b) The class Ostracodermi is
can be used for carrying out represented by the fossil vertebrates of
= Number of equivalents of NaOH
transformation as it is the strongest late Cambrian period. The earliest known
N × 5 = 1 × 12 × 1 [N = M × basicity] vertebrates to appear in fossil record
oxidising agent among the other given 12
options. N = = 2.4 were jawless primitive fish-like animals
5 collectively called ostracoderms. These
41. (*) According to de-Broglie 45. (b) M + O2 → MO2 animals resembled the present day
wavelength
1.25 cyclostomes (lampreys and hagfishes) in
λ=
h Percentage of M = × 100 = 74.4% many respects.
mv 168
.
Percentage of oxygen in oxide 50. (b) Scurvy is caused by the deficiency
Mass of C 60 = 12 × 60 = 720 g
of vitamin-C (Ascorbic acid) in the body.
o = 100 − 74.4% = 25.6%
. A = 11 × 10−10 m
Given, λ = 110 It can lead to anaemia, debility,
To calculate empirical formula exhaustion, spontaneous bleeding, pain
h = 6.626 × 10−34 Js
Eleme % of
At mass Moles Simplest Simplest in the limbs and especially the legs,
= 6.626 × 10−34 kg m 2s −1 -nt element
of of molar whole
swelling in some parts of the body and
element element ratio no.
h 6.626 × 10−34 kgm 2s −1 sometimes ulceration of the gums and
∴ v= = 74.4
mλ 720 × 10−3 kg × 11 × 10−10 m 74.4 69.7
1.06
=1 1× 2= 2 loss of teeth.
M 697.
1.06
= 0.8 × 10−18 m/s = 1.06 51. (b) DNA polymer is made up of
No option is correct in the given format 25.6 1.6 nitrogenous base, a sugar and one or
1.50 × 2
as the value of wavelength is given in Å O 25.6 16 16 1.06 more phosphate. Optical activity results
=3
which gives the large difference in = 1.6 = 150
. due to the molecular asymmetry. The
answer. nucleic acid bases have a plane of
∴Empirical formula of metal oxide is symmetry. Hence, they do not induce
42. (c) Lattice energy is the energy M2O3 . optical activity. Sugars are asymmetric
required to completely separate one mole 46. (b) In 1953, JD Watson and FHC and cause optical activity of DNA.
of a solid ionic compound into gaseous Crick proposed a 3-D model of
constituent. Lattice energy increases 52. (c) The monarch butterfly avoids
physiological DNA. They proposed that predators such as birds by producing a
with decrease in the size of ions. DNA is a double-stranded helical chemical obnoxious to the predator.
This is because as the size of ion is less, molecule. It consists of two Monarchs lay their eggs on milkweed
intermolecular distance will be less and sugar-phosphate backbones on the (swan plants), a member of the genus
so forces of attraction is greater. outside, held together by hydrogen bonds Asclepias. As the caterpillars eat the
Thus, the correct increasing order of between pairs of nitrogenous bases on the milkweed leaves, they ingest chemicals
lattice energies is, inside. The bases adenine (A) always called cardiac glycosides. Birds or other
pairs with thymine (T) by two hydrogen animals that eat the caterpillars (or
RbCl < KCl < NaCl < NaF.
bonds and guanine always pairs with milkweed itself) become sick and vomit.
43. (a) Let the oxidation state of P-atom cytosine (C) by three hydrogen bonds. The caterpillars sequester (hold onto) this
in POCl3 , H2PO3 and H4 P2O6 be x. This complimentarity is known as the toxin as they pupate and the toxins are
(i) POCl 3 base pairing rule. transferred to the adult butterflies. Birds
x + 1 (−2) + 3(−1) = 0 47. (c) In anaphase, sister chromatids or other creatures that eat the monarchs
x− 2− 3= 0 separate from centromeres so, number of become sick, so they learn to leave both
x=+5 chromosome becomes double. Other the butterflies and larvae alone.
(ii) H 2 PO3 statements about mitosis can be 53. (d) Filariasis is caused by
2(1) + x + 3(−2) = 0 corrected as Mitosis is a single nuclear Wuchereria bancrofti. It lives in
2+ x− 6= 0 division that results in two nuclei. lymphatic vessels and causes swelling of
Synapsis takes place during prophase-I of lower limbs and scrotum. Entamoeba
x=+4
histolytica causes amoebiasis.
(iii) H 4 P2O6 meiosis not during mitosis. Non-sister
Plasmodium falciparum causes malaria.
4(1) + 2x + 6(−2) = 0 chromatids recombine during prophase-I Trypanosoma brucei causes African
4 + 2x − 12 = 0 of meiosis. During mitosis, each sister sleeping sickness.
2x = 8 chromatid separates and moves to 54. (c) Conversion of glucose to CO2 and
x=+4 opposite pole of the cell at anaphase. H 2O by Saccharomyces is a reaction which
44. (b) Number of equivalents 48. (b) Gaseous exchange occurs at the takes place in aerobic conditions, i.e. in
alveoli in the lungs and takes place by the presence of oxygen.
= M × V × acidity/basicity
diffusion. The alveoli are surrounded by C6 H12O6 + 6O2 → 6CO2 + 6H2O
Number of equivalents of NaOH capillaries so, oxygen and carbon dioxide
= 1× y × 1= y
https://iit-jeeacademy.blogspot.com

KVPY Question Paper 2017 Stream : SA 29

55. (d) A mole is the quantity of a contain specific combinations of inorganic But 4 (k 2 + m2 + k + m) + 2 is not
substance whose weight in grams is equal elements including carbon, hydrogen, multiple of 4.
to the molecular weight of the substance. nitrogen and oxygen that combine to form ∴ Not possible.
1 mole is equal to 1 moles Glucose, or proteins and nucleic acids. ∴S is empty set.
180.15588 grams. 61. (d) Given, 63. (c) Given,
∴ 18 g of glucose = x mole × 180 g ABCD is a trapezium where AB is
18 Two circle each of radius is 2
x mole = = 01
. mole parallel to CD. A circle S with AB as
180 and difference between their centre is 2 3
diametre touch CD and also circle passes
1
∴An amount of 18 g glucose corresponds through the mid-points of diagonal AC AB = 2 3 ⇒ AC = AB
to 0.1 mole. and BD. 2
AR = RC AC = 3
56. (a) Four electrons are required to
reduce one molecule of oxygen to water ∠ARB = 90° P
during mitochondrial oxidation. ∴∆ABC is isosceles
O2 + 4e− + 4H+ → 2H2O AB = BC … (i)
2 2
This process mentioned above takes place θ C
A B
during oxidative phosphorylation. It is √3 √3
the metabolic pathway in which cells use
enzymes to oxidise nutrients, thereby A O B
Q
releasing energy which is used to produce
ATP. AC 3
Q R In ∆APC, cosθ = =
57. (d) Vitamin-B5 is pantothenic acid or AP 2
D M P C
pantothenate, that is required in the Similarly, in ∆ABD θ = 30°
synthesis of acetyl Co-A. In all living BQ = QD Area of common region
organisms, Co-A is synthesised in a five ∠AQB = 90° = 2 (Area of sector − Area of ∆APQ)
step process that requires four molecules 60 1
of ATP, pantothenate and cysteine.
∴∆ABD is isosceles. = 2  × π(2)2 − × (2)2 × sin 60°
∴ AB = AD … (ii)  360 2 
58. (a) HIV is the causative organism for From Eqs. (i) and (ii),  4π 4 3 
AIDS. = 2 − 
AB = BC = AD  6 4 
Syphilis is a bacterial infection caused by ∴Trapezium is isosceles. 2
Treponema sp. It spreads by sexual = 2  (314 . )
. ) − (173
In ∆ADM, 3 
contact that starts as a painless sore. AM OP OP
Viral hepatitis caused by HBV is an sin ∠ADM = = = = 2 (2.09 − 173
. ) = 2 (0.36) = 0.72
AD AD 2OP
infection that causes liver inflammation ∴Area of region lie between 0.7 and 0.75.
Q OP = 1 AB 
and damage organs.
 2  64. (a) Given,
Gonorrhoea is caused by Neisseria sp. It
1 AB is diameter of circle C1 .
is sexually transmitted bacterial infection ⇒ sin (∠ADM ) =
that if let untreated may cause infertility. 2 A3
π N
59. (b) Telocentric chromosome is a ⇒ ∠ADM = 30° = A1
A2
6 M
chromosome like a straight rod with the
centromere in terminal position. 62. (a) Let the equation of circle is B C
P A Q
x2 + y2 = 1,  ,  lie on circle.
a c
Metacentric chromosome is a X-shaped
chromosome, with the centromere in the  b d
middle so that the two arms of the a2 c2
chromosomes are almost equal. ∴ 2
+ =1 C1 C2
b d2 BA1 = 2
Acrocentric chromosome is a chromosome
c 1 2
in which the centromere is located quite ⇒ =± b − a2 BA2 = 3
near one end of the chromosome. d b
BA3 = 4
Sub-metacentric chromosome is a c and d are relatively prime.
c Let radius of circle C1 = r1 and radius of
chromosome whose centromere is located ∴ is rational. circle C2 = r2
near the middle. d
So, b2 − a 2 = λ2 [Q b is even;∴a is odd] ∴ BA = 2r1 and AC = 2r2
60. (a) Living organisms require energy 1
to grow, reproduce and respond to the b is even, a is odd. ⇒ BM = BA1 = 1
2
environment. Energy sources include ∴ λ2 is odd ⇒ b2 = λ2 + a 2
1
primarily light and inorganic compounds. ⇒ b2 = (2k + 1)2 + (2m + 1)2 ⇒ BN = BA2 + A2A3
The most common source of energy on the 2
⇒ b2 = 4k 2 + 4k + 1 + 4m2 + 4m + 1 1 7
earth is photosynthesis, which transforms = 3+ =
⇒ b2 = 4 (k 2 + m2 + k + m) + 2 2 2
sunlight into food. Life forms usually
∴b is even;∴b2 is multiple of 4.
https://iit-jeeacademy.blogspot.com

30 KVPY Question Paper 2017 Stream : SA

In ∆BMP and ∆ BNQ, 67. (b) Let Q = rate of heat removal.


∆BMP ~ ∆BNQ Then, Q ⋅ t = mcT 45° l
BM BP
∴ = Q
BN BQ ⇒ T = ⋅t
mc
1 r1 Fe
⇒ = Comparing this with y = mx,
x
7 / 2 2r1 + r2
1
Slope of T-t line ∝ mg
⇒ 2r2 = 3r1 … (i) Specific heat F
Now, BA2 × BA3 = BA × BC tan 45° = e
Slope of graph mg
⇒ 3 × 4 = 2r1 (2r1 + 2r2 ) T 1 in solid state
⇒ mg = Fe
⇒ 12 = 4 (r12 + r1 r2 ) is more
Slope of graph
2 2kq2
⇒ r1 + r1 r2 = 3 … (ii) 2 in liquid ⇒ mg = 2
state is more x
From Eqs. (i) and (ii), we get
1 ⇒ mgx2 = 2kq2
6 30 3 30 l
r1 = = and r2 = where, x = l sin θ =
5 5 10 2 2
t
65. (a) Given, So, substituting values, we get
From graph, 2
|a| = 4− 5− a  20 × 10−2 
∴ CS1 < CS 2 ⇒100 × 10−3 × 10 ×  
|b| = 4+ 5− b and CL1 > CL 2  2 

|c| = 4− 5+ c 68. (a) As θ increases, angle of incidence = 2 × 9 × 109 × q2


−2
(i = 90 − θ) decreases. Initially upto i = ic 10
|d| = 4+ 5+ d ⇒ q2 =
angle of critical incidence, reflection 9 × 109
On squaring, we get takes place and x is positive. Also, x ⇒ q2 > 10−12
a2 = 4 − 5 − a increases till θ is such that i = ic , after (slightly higher than 10−12 )
= a2 − 4 = − 5 − a that refraction takes place and x becomes ⇒ q > 10−6 C
Again squaring, we get negative. So, nearest answer is 1.5 µC.
a 4 − 8a 2 + 16 = 5 − a
70. (d) Time at which discs gaps are
⇒ a 4 − 8a 2 + a + 11 = 0 alined,
Similarly, squaring other given equation π 3π 5π
t= , , ,K
and solving we can say that ω ω ω
x θc O
a , b, − c, − d are roots of equation θ So, speeds of atoms that emerges on other
When θ is large x
When θ is less O side are
x4 − 8x2 + x + 11 = 0 and TIR clean occur
and TIR occur 0.5
v1 = = 600 ms −1
∴ The product of roots π
69. (b) Electrostatic force on any of the × 2π
i.e. abcd = 11 ball is (let x = separation between two 600
66. (c) When planks are arranged as 0.5
adjacent balls). and v2 = = 200 ms −1
given in question, we have following 3π
× 2π
situation. kq2 600
—2
Pivot x 71. (d) If alkenes have two different
m 17 kg x x substituents (x, y) at each end of the
A B 45° kq2
——2 C == C, then only they can show
45° (√2x) E/Z isomerism.
x x (a) 2-methylbut-2-ene
0.5 m 2.5 m kq2
—2
x x CH3 H x
kq2 2kq
Fe = + ⋅ cos 45° C C
3.5 m ( 2x)2 x2 y CH3 CH3 y
Let m = maximum mass of A. kq2 kq
Then, for safe crossing, = + 2 As x = y, this alkene will not show
2x2 x2 E/Z isomerism.
mg × 0.5 = 17 × g × 2.5
1 kq2 2kq2
17 × 2.5 =  + 2  2 ≈ 2 (b) 2-methylbut-1-ene
⇒ m= = 85 kg 2  x x
0.5 x H CH2CH3 x
As each ball is at an angle of 45° from C C
So, a man of mass upto 80 kg can pass
each other, so in equilibrium, we have y H CH3 y
over planks.
https://iit-jeeacademy.blogspot.com

KVPY Question Paper 2017 Stream : SA 31

Here also x = y 76. (d) Blood group-O individuals are called universal donor as
∴ It will not show E/Z isomerism. they can give blood to person with blood group-A, B, AB and O.
(c) 3-methylpent-1-ene Blood group-AB individuals can only give blood to persons with
x H H x
blood group-AB but can receive blood from all other blood
C C groups.
y H CH2(CH3)CH2CH3 y Therefore, the correct table for the blood transfusion
This compound will not show E/Z isomerism. compatibility for ABO blood group system in human is
(d) 3-methylpent-2-ene Recipient
x H CH2CH3 x O A B AB
C C O P P P P
y H3C CH3 y

Donor
A ✗ P ✗ P
(d)
As x ≠ y, thus this alkene will show E/Z is isomerism. B ✗ ✗ P P
72. (c) AB ✗ ✗ ✗ P
ρ σ σ ρ
(i) Na NH2
CH3 — C C—H CH3 C C Na 77. (b) Assuming x as the amount of ligand to occupy all the
CH3 (ii) CH3 — I (x)
δ – binding sites in 10 mg protein.
δ +

x mg Ligand molecular weight


HC==CH
(iii) Na/liq. NH3
CH3C CCH3 +NaI =
(y) Protein in grams Protein molecular weight
2500
CH3 x= × 10 = 1mg
(Trans form) 25000
Thus, in the given reaction, 78. (a) Serine is coded by UCU, UCC, UCA, UCG, AGU, AGC
x = CH3 I and y = Na / liq. NH3. Leucine is coded by CUU, CUC, CUA, CUG, UUA, UUG
73. (a) The bond angle depends upon the electronegativity of Arginine is coded by AGA, AGG, CGU, CGC, CGA, CGG
the central atom. More is the electronegativity of the central, Glutamic acid is coded by GAA, GAG
larger is the bond angle. Thus, among the given central atom Cl Therefore, option (a) is the correct interpretation of the assigned
has the highest electronegativity. Therefore, ClF3 has the amino acids.
largest bond angle at the central atom. Ser-UCU, Leu-CUC, Arg-AGA, Glu-GAG
74. (d) 79. (d) The number of bacteria after 15 hours will be 32. This
At mass Simplest Simplest happens as each bacterium doubles up after every 3 hours, i.e. it
% of Moles of 15
Elements
element
of
element
molar whole will double = 5 times in 15 hours.
element ratio no. 3
18.6 0.8 Therefore, the sequence in which the growth of bacteria taking
Na 18.6 23 = 0.8 =1 1 × 2= 2
23 0.8 place will be
3 hours 3 hours 3 hours 3 hours 3 hours
25.8 0.8 1 2 4 8 16 32
S 25.8 32 = 0.8 =1 1× 2 = 2
32 0.8 80. (c) Colourblindness is a X-linked recessive disease, i.e. an
5158
. heterozygous mother does not show the disease and is a carrier.
3.22
O 51.58 16 16 = 4 4× 2= 8 But a father cannot be a carrier of the disease as it has single
0.8
= 3.22 X-chromosome.
4.02 4.02 In the given question, the son is colourblind which means it had
H 4.02 1 1 = 5 5 × 2 = 10
0.8 inherited X c from the mother. But another son is normal. This
= 4.02 shows that the mother is heterozygous for the disease.
Thus, the empirical formula of compound is Na 2 S2 H10O8 . The expected cross for the question will be
As it is given the all the hydrogen atoms in the compound are XCX × XY
part of water of crystallisation, therefore molecular formula will Normal Normal
be Na 2 S2O3 ⋅ 5H 2O. carrier father
mother
75. (d) Given, latent heat of fusion of ice = 333 J/g
Specific heat of water = 4184
. J/g K
First X g of ice at 0°C melts and then its temperature increases
by gaining heat from 340 g of water at 20°C. XCX XCY XX XY
∴Energy gained by X g of ice = energy lost by 340 g of water Normal Colourblind Normal Normal
carrier son daughter son
[E = mc ∆T ] daughter
X (333) + X × 4184
. (278 − 273) = 340 × 4184
. (293 − 278) This shows that both the parents would be normal if they
333X + 20.92X = 21338.4 ⇒ 353.92X = 21338.4 have one colourblind son and one normal son.
X = 60.29 ≈ 60.3 g
https://iit-jeeacademy.blogspot.com

32 KVPY Question Paper 2017 Stream : SA

KVPY
KISHORE VAIGYANIK PROTSAHAN YOJANA

QUESTION PAPER 2017


Stream : SA (Nov 05)
MM 100

Instructions
There are 80 questions in this paper.
This question paper contains two parts; Part I and Part II. There are four sections; Mathematics, Physics, Chemistry
and Biology in each part.
Out of the four options given with each question, only one is correct.

PART-I (1 Mark Questions)


MATHEMATICS (b) S is the empty set
(c) S has exactly one element
1. A quadrilateral has distinct integer side lengths. If (d) S is a finite set and has at least two elements.
the second-largest side has length 10, then the
maximum possible length of the largest side is 5. Let A1 A2 A3 ... A9 be a nine-sided regular polygon
(a) 25 (b) 26 (c) 27 (d) 28 with side length 2 units. The difference between the
200 ! lengths of the diagonals A1 A5 and A2 A4 equals
2. The largest power of 2 that divides is
(a) 2 + 12 (b) 12 − 2 (c) 6 (d) 2
100 !
(a) 98 (b) 99 (c) 100 (d) 101 6. Let a1 , a 2 , ... , a n be n non-zero real numbers, of
3. Let a1 , a 2 , a3 , a 4 be real numbers such that which p are positive and remaining are negative. The
a1 + a 2 + a3 + a 4 = 0 and a12 + a 22 + a32+ = 1. Then,
a 42 number of ordered pairs ( j, k), j < k, for which a j a k is
positive, is 55. Similarly, the number of ordered pairs
the smallest possible value of the expression ( j, k), j < k, for which a j a k is negative, is 50. Then, the
(a1 − a 2 )2 + (a 2 − a3 )2 + (a3 − a 4 ) + (a 4 − a1 )2 lies in
value of p2 + (n − p)2 is
the interval
(a) 629 (b) 325 (c) 125 (d) 221
(a) (0, 1.5) (b) (1.5, 2.5) (c) (2.5, 3) (d) (3, 3.5)
7. If a , b, c, d are four distinct numbers chosen from the
4. Let S be the set of all ordered pairs (x, y) of positive a c
integers satisfying the condition x2 − y2 = 12345678. set {1, 2, 3,... ,9}, then the minimum value of + is
b d
Then, 3 1 13 25
(a) (b) (c) (d)
(a) S is an infinite set 8 3 36 72
https://iit-jeeacademy.blogspot.com

KVPY Question Paper 2017 Stream : SA 33

8. If 72x ⋅ 48 y = 6xy , where x and y are non-zero rational


PHYSICS
numbers, then x + y equals
(a) 3 (b)
10
(c) − 3 (d) −
10 16. Consider the following statements (X and Y stand for
3 3 two different elements):
65 65
9. Let AB be a line segment of length 2. Construct a (I) 32 X and 33 Y are isotopes.
semicircle S with AB as diameter. Let C be the (II) 86
and 85
are isotopes.
42 X 42 Y
mid-point of the arc AB. Construct another semicircle
174 177
T external to the ∆ ABC with chord AC as diameter. (III) and
85 X have the same number of
88 Y
The area of the region inside the semi-circle T but neutrons.
outside S is (IV) 235 235
π 1 π 1 92 X and 94 Y are isobars.
(a) (b) (c) (d)
2 2 2 2 Which of the above statements are correct?
(a) Only statements II and IV are correct
10. Let r (x) be the remainder when the polynomial
(b) Only statements I, II and IV are correct
x135
+x125
−x115
+ x + 1 is divided by x − x. Then,
5 3
(c) Only statements II, III and IV are correct
(a) r (x) is the zero polynomial (d) All statements are correct
(b) r (x) is a non-zero constant
(c) degree of r (x) is one (d) degree of r (x) is two 17. A student performs an experiment to determine the
acceleration due to gravity g. The student throws a
11. It is given that the number 43361 can be written as a steel ball up with initial velocity u and measures the
product of two distinct prime number p1 , p2. Further, height h travelled by it at different times t. The
assume that there are 42900 numbers which are less graph the student should plot on a graph paper to
than 43361 and are coprime to it. Then, p1 + p2 is readily obtain the value of g is
(a) 462 (b) 464 (c) 400 (d) 402 (a) h versus t (b) h versus t 2
12. Let ABC be a triangle with ∠C = 90° . Draw CD (c) h versus t (d) h / t versus t
perpendicular to AB. Choose points M and N on sides 18. A person goes from point P to point Q covering 1 / 3 of
AC and BC respectively such that DM is parallel to the distance with speed 10 km/h, the next 1/3 of the
BC and DN is parallel to AC. If DM = 5, DN = 4, then distance at 20 km/h and the last 1/3 of the distance at
AC and BC are respectively equal to 60 km/h. The average speed of the person is
41 41 39 39 38 38 37 37
(a) , (b) , (c) , (d) , (a) 30 km/h (b) 24 km/h (c) 18 km/h (d) 12 km/h
4 5 4 5 4 5 4 5
19. A person looks at the image of two parallel finite
13. Let A, G and H be the arithmetic mean, geometric length lines PQ and RS in a convex mirror (see
mean and harmonic mean, respectively of two figure).
distinct positive real numbers. If α is the smallest of
the two roots of the equation A(G − H )x2 + G(H − A) x P Q
+ H ( A − G) = 0 then,
(a) − 2 < α < − 1 (b) 0 < α < 1 R S
(c) − 1 < α < 0 (d) 1 < α < 2
14. In the figure, ABCD is a unit square. A circle is Which of the following represents schematically the
drawn with centre O on the extended line CD and image correctly?
passing through A. If the diagonal AC is tangent to
the circle, then the area of the shaded region is Note Letters P , Q, R and S are used only to denote
the endpoints of the lines.
O
Q P
P Q

A D R S
S R
X (A) (B)

B C Q P P Q
9− π 8− π 7− π 6− π
(a) (b) (c) (d)
6 6 4 4
S R R S
15. The sum of all non-integer roots of the equation
x5 − 6x4 + 11x3 − 5x2 − 3x + 2 = 0 is (C) (D)
(a) 6 (b) −11 (c) − 5 (d) 3 (a) A (b) B (c) C (d) D
https://iit-jeeacademy.blogspot.com

34 KVPY Question Paper 2017 Stream : SA

20. In Guericke’s experiment to show the effect of 25. The word KVPY is written on a board and viewed
atmospheric pressure, two copper hemispheres were through different lenses such that board is at a
tightly fitted to each other to form a hollow sphere distance beyond the focal length of the lens.
and the air from the sphere was pumped out to create
vacuum inside. If the radius of each hemisphere is R KVPY
and the atmospheric pressure is p, then the KVPY
minimum force required (when the two hemispheres
are pulled apart by the same force) to separate the First image Second image
hemispheres is Ignoring magnification effects, consider the following
p
(a) 2 pπR 2 (b) 4 pπR 2 (c) pπR 2 (d) πR 2 statements:
2
(I) First image has been viewed from the planar
21. Positive point charges are placed at the vertices of a side of a plano-convex lens and second image
star shape as shown in the figure. Direction of the from the convex side of a plano-convex lens.
electrostatic force on a negative point charge at the
centre O of the star is (II) First image has been viewed from the concave
side of a plano-concave lens and second image
2q
from the convex side of a plano-convex lens.
a
q q (III) First image has been viewed from the concave
side of a plano-concave lens and second image
O from the planar side of a plano-convex lens.
q 3q (IV) First image has been viewed from the planar
side of a plano-concave lens and second image
q from the convex side of a plano-convex lens.
(a) towards right (b) vertically up
Which of the above statements are correct?
(c) towards left (d) vertically down (a) All statements are correct
(b) Only statement III is correct
22. A total solar eclipse is observed from the earth. At (c) Only statement IV is correct
the same time an observer on the moon view’s the (d) Only statements II, III and IV are correct
earth. She is most likely to see (E denotes the earth)
26. A ball is dropped vertically from height h and is
E E
bouncing elastically on the floor (see figure). Which of
the following plots best depicts the acceleration of the
(A) (B) ball as a function of time.

E E h

(C) (D) t

(a) (b)
Acceleration

(a) A (b) B (c) C (d) D


Acceleration

23. Ice in a freezer is at −7°C. 100 g of this ice is mixed t t


with 200 g of water at 15°C. Take the freezing
temperature of water to be 0°C, the specific heat of
ice equal to 2.2 J/g °C, specific heat of water equal to
4.2 J/g°C and the latent heat of ice equal to 335 J/g. (d)
Assuming no loss of heat to the environment, the (c)
Acceleration
Acceleration

mass of ice in the final mixture is closest to


(a) 88 g (b) 67 g (c) 54 g (d) 45 g t t
24. A point source of light is placed at 2f from a
converging lens of focal length f. A flat mirror is
placed on the other side of the lens at a distance d
such that rays reflected from the mirror are parallel 27. A student studying the similarities and differences
after passing through the lens again. If f = 30 cm, between a camera and the human eye makes the
then d is equal to following observations:
(a) 15 cm (b) 30 cm (c) 45 cm (d) 75 cm I. Both the eye and the camera have convex lenses.
https://iit-jeeacademy.blogspot.com

KVPY Question Paper 2017 Stream : SA 35

II. In order to focus, the eye lens expands or 32. The stability of carbanions
contracts while the camera lens moves forward σ σ
CH3CH2CH2CH2 CH3CHCH2CH3
or backward.
I II
III. The camera lens produces upside down real σ σ
(CH3)3C CH3C(Ph)CH2CH3
images while the eye lens produces only
III IV
upright real images.
follows the order
IV. A screen in camera is equivalent to the retina
(a) III < IV < I < II (b) I < II < IV < III
in the eyes.
(c) III < II < I < IV (d) IV < III < II < I
V. A camera adjusts the amount of light entering
in it by adjusting the aperture of the lens. In 33. In the following reaction
+ –
the eye, the cornea controls the amount of light. N2Cl OH
Which of the above statements are correct? NaOH
+
(a) Statements I, II and IV are correct
(b) Statements I, III and V are correct
the major product is
(c) Statements I, II, IV and V are correct
(d) All statements are correct OH

28. A particle starts moving along a line from zero initial O


velocity and comes to rest after moving distance d. (a) (b)
During its motion, it had a constant acceleration f
over 2/3 of the distance and covered the rest of the OH
distance with constant retardation. The time taken to
NH2 N
cover the distance is
(c) N
(a) 2 d / 3 f (b) 2 d / 3 f (c) 3d / f (d) 3 d / 2 f (d)
29. If the image formed by a thin convex lens of power P
has magnification m, then image distance v is 34. In the reaction of 1-bromo-3-chlorocyclobutane with
1−m 1+ m m 1 + 2m
(a) v = (b) v = (c) v = (d) v = two equivalents of sodium in ether, the major product
P P P P is
30. A long cylindrical pipe of radius 20 cm is closed at its
upper end and has an airtight piston of negligible (a) Br Cl (b)
mass as shown. When a 50 kg mass is attached to n
the other end of the piston, it moves down. If the air
in the enclosure is cooled from temperature T to (c) Cl Cl (d)
T − ∆T , the piston moves back to its original position.
Then ∆T / T is close to (Assuming air to be an ideal 35. The order of basicity of
gas, g = 10 m/ s2, atmospheric pressure is 105 Pa) NH2 NH2

O2N
I II

N
N H
(a) 0.01 (b) 0.02 (c) 0.04 (d) 0.09 III IV
in water is
CHEMISTRY (a) IV < III < I < II (b) II < I < IV < III
(c) IV < I < III < II (d) II < III < I < IV
31. The structure of 3-methylpent-2-ene is
36. The first ionisation energy of Na, B, N and O atoms
follows the order
(a) (b) (a) B < Na < O < N (b) Na < B < O < N
(c) Na < O < B < N (d) O < Na < N < B
37. Among P2O5 , As2O3 , Sb2O3 and Bi2O3 , the most acidic
(c) (d) oxide is
(a) P2O5 (b) As 2O3 (c) Sb2O3 (d) Bi 2O3
https://iit-jeeacademy.blogspot.com

36 KVPY Question Paper 2017 Stream : SA

38. Among K, Mg, Au and Cu, the one which is extracted 50. Which one of the following statements is correct
by heating its ore in air is about the tobacco mosaic virus?
(a) K (b) Mg (c) Au (d) Cu (a) It affects all monocotyledonous plants
39. The metal ion with total number of electrons same as (b) It affects photosynthetic tissue of the infected plant
S2− is (c) It does not infect other species belonging to the
Solanaceae
(a) Na + (b) Ca 2 + (c) Mg2 + (d) Sr 2 +
(d) It infects gymnosperms
40. X g of Ca [atomic mass = 40] dissolves completely in
51. Which one of the following statements is correct
concentrated HCl solution to produce 5.04 L of H2 gas
about placenta?
at STP. The value of X is closest to
(a) Placenta is permeable to all bacteria
(a) 4.5 (b) 8.1 (c) 9.0 (d) 16.2
(b) Oxygen and carbon dioxide cannot diffuse through the
41. A 20 g object is moving with velocity 100 ms−1. The placenta
de Broglie wavelength (in m) of the object is (c) Waste products diffuse out of placenta into maternal
[Planck’s constant h = 6.626 × 10−34 Js] blood
(a) 3.313 × 10−34 (b) 6.626 × 10−34 (d) Placenta does not secrete chorionic gonadotropins
(c) 3.313 × 10−31 (d) 6.626 × 10−31 52. The respiratory quotient of the reaction given below is
42. In a closed vessel at STP, 50 L of CH4 is ignited with 2(C 51 H 98O6 ) + 145O2 → 102CO2 + 90H 2O + Energy
750 L of air (containing 20% O2 ). The number of (a) 0.703 (b) 0.725 (c) 0.960 (d) 1.422
moles of O2 remaining in the vessel on cooling to 53. Which one of the following statements is incorrect
room temperature is closest to about nucleosomes?
(a) 5.8 (b) 2.2 (c) 4.5 (d) 6.7
(a) They contain DNA
43. CO2 is passed through lime water. Initially the (b) They contain histones
solution turns milky and then becomes clear upon (c) They are membrane-bound organelle
continued bubbling of CO2. The clear solution is due (d) They are a part of chromosomes
to the formation of
54. The immediate precursor of thyroxine is
(a) CaCO3 (b) CaO
(c) Ca(OH)2 (d) Ca(HCO3 )2 (a) tyrosine (b) tryptophan
(c) pyridoxine (d) thymidine
44. The maximum number of electrons that can be filled
in the shell with the principal quantum number 55. The maximum number of oxygen molecules that can
n = 3 is bind to one molecule of haemoglobin is
(a) 18 (b) 9 (c) 8 (d) 2 (a) 8 (b) 6 (c) 4 (d) 2

45. The atomic radii of Li, F, Na and Si follow the order 56. Which one of the following biomolecules is
synthesised in smooth endoplasmic reticulum?
(a) Si > Li > Na > F (b) Li > F > Si > Na
(c) Na > Si > F > Li (d) Na > Li > Si > F (a) Proteins (b) Lipids
(c) Carbohydrates (d) Nucleotides
57. The products of light reaction during photosynthesis
BIOLOGY include
46. The major excretory product of birds is (a) ATP and NADPH (b) O2 and NADP +
(a) urea (b) uric acid (c) O2 and H2O (d) NADP + and H2O
(c) nitrates (d) ammonia 58. Hypothalamus directly controls the production of
47. Codon degeneracy means that which of the following hormones?
(a) several amino acids are coded by more than one codon (a) Glucocorticoid and insulin
(b) one codon can code for many amino acids (b) Insulin and glucagon
(c) one amino acid can be coded by only one codon (c) Atrial natriuretic factor and gastrin
(d) the codons are triplet nucleotide sequences (d) Glucocorticoids and androgens
48. In cell cycle, during interphase 59. Which one of the following drug is not obtained from
(a) two daughter cells are produced fungal or plant sources?
(b) the nucleus is divided into two daughter nuclei (a) Penicillin (b) Reserpine
(c) the chromosome condenses (c) Acetaminophen (d) Quinine
(d) the DNA is replicated
60. Jean Baptiste Lamarck explained evolution based on
49. Transfer of genetic material between population is (a) natural selection
best defined as (b) survival of the fittest
(a) gene flow (b) genetic drift (c) mutations
(c) genetic shift (d) speciation (d) inheritance of acquired characteristics
https://iit-jeeacademy.blogspot.com

KVPY Question Paper 2017 Stream : SA 37

PART-II (2 Marks Questions)


MATHEMATICS Choose the correct option.
(a) CL1 > CL 2 and U1 < U 2 (b) CL1 > CL 2 and U1 > U 2
61. Let S be the circle in XY -plane which touches the
(c) CL1 < CL 2 and U1 > U 2 (d) CL1 < CL 2 and U1 < U 2
X-axis at point A, the Y -axis at point B and the unit
circle x2 + y2 = 1 at point C externally. If O denotes 67. A long horizontal mirror is next to a vertical screen
the origin, then the angle OCA equals (seen figure).
5π π 3π 3π
(a) (b) (c) (d)
8 2 4 4

Screen
62. In an isosceles trapezium, the length of one of the
parallel sides, and the lengths of the non-parallel
α
sides are all equal to 30.In order to maximise the h
area of the trapezium, the smallest angle should be
π π π π d
(a) (b) (c) (d)
6 4 3 2 Parallel light rays are falling on the mirror at an
63. Let A1 , A2 , A3 be regions in the XY -plane defined by angle α from the vertical. If a vertical object of height
h is kept on the mirror at a distance (d > h ) tan α. The
A1 = {(x, y) : x2 + 2 y2 ≤ 1}
length of the shadow of the object on the screen
A2 = {(x, y) :|x3| + 2 2 | y|3 ≤ 1} would be
h
A3 = {(x, y) : max (| x|, 2 | y|) ≤ 1} (a) (b) h tanα
2
Then, (c) 2h (d) 4h
(a) A1 ⊃ A2 ⊃ A3 (b) A3 ⊃ A1 ⊃ A2
(c) A2 ⊃ A3 ⊃ A1 (d) A3 ⊃ A2 ⊃ A1
68. A spherical marble of radius 1 cm is stuck in a cir-
cular hole of radius slightly smaller than its own
64. Let ABCD be a square and E be a point outside radius (for calculation purpose, both can be taken
ABCD such that E , A, C are collinear in that order. same) at the bottom of a bucket of height 40 cm and
Suppose EB = ED = 130 and the areas to ∆EAB and filled with water up to 10 cm.
square ABCD are equal. Then, the area of square
ABCD is
(a) 8 (b) 10 (c) 120 (d) 125
65. Consider the set A = {1, 2, 3, ... , 30}. The number of
ways in which one can choose three distinct number
from A so that the product of the chosen numbers is
divisible by 9 is If the mass of the marble is 20 g, then the net force
(a) 1590 (b) 1505 (c) 1110 (d) 1025
on the marble due to water is close to
(a) 0.02 N upwards (b) 0.02 N downwards
(c) 0.04 N upwards (d) 0.04 N downwards
PHYSICS 69. In the circuit shown below (on the left) the resistance
66. Two different liquids of same mass are kept in two and the emf source are both variable.
identical vessels, which are placed in a freezer that R
extracts heat from them at the same rate causing A
each liquid to transform into a solid. The schematic V
figure below shows the temperature T versus time t
plot for the two materials. We denote the specific
heat in the liquid states to be C L1 and C L 2 for
materials 1 and 2, respectively and latent heats of
fusion U 1 and U 2, respectively. V
T 2V0 H

F G
V0
1 E
2 0
0 I0 2I0 I
t
https://iit-jeeacademy.blogspot.com

38 KVPY Question Paper 2017 Stream : SA

The graph of seven readings of the voltmeter and the H3 C CHO


ammeter (V and I, respectively) for different settings
of resistance and the emf, taken at equal intervals of (a) X = H3C CHO, Y=
time ∆t, are shown below (on the right) by the dots Ph
connected by the curve EFGH. Consider the internal
O O
resistance of the battery to be negligible and the
voltmeter an ammeter to be ideal devices. (Take,
V (b) X = H3C CH3 , Y= H3 C Ph
R0 ≡ 0 ).
I0 O
Then, the plot of the resistance as a function of time H3 C
(c) X = , Y=
corresponding to the curve EFGH is given by H 3C CH3 Ph
(a) R (b) R CH3
2R0 H X= , Y=
(d)
H3 C CHO H3 C Ph
F E F
R0 G R0 H
G 73. KMnO4 reacts with H2O2 in an acidic medium. The
E R0 /2
0 0 number of moles of oxygen produced per mole of
0 2∆t 4∆t 6∆t t 0 2∆t 4∆t 6∆t t
KMnO4 is
(c) R (d) R (a) 2.5 (b) 5 (c) 1.25 (d) 2
2R0 H 74. The photoelectric behaviour of K, Li, Mg and Ag
metals is shown in the plot below. If light of
E F E F wavelength 400 nm is incident on each of these
R0 R0 H
R0 /2 R0 /2
metals, which of them will emit photoelectrons?
G G
0 0 [Planck’s constant h = 6626
. × 10−34 Js; velocity of
0 2∆t 4∆t 6∆t t 0 2∆t 4∆t 6∆t t −1
. × 10−19 J]
light c = 3 × 10 m s ; 1 eV = 16
8

70. Stokes’ law states that the viscous drag force F


experienced by a sphere of radius a, moving with a (a) K (b) K and Li
speed v through a fluid with coefficient of viscosity η, (c) K, Li and Mg (d) K, Li, Mg and Ag
is given by F = 6πηav. If this fluid is flowing through 75. A piece of metal weighing 100 g is heated to 80°C and
a cylindrical pipe of radius r, length l and pressure dropped into 1 kg of cold water in an insulated
difference of p across its two ends, then the volume of container at 15°C. If the final temperature of the
water V which flows through the pipe in time t can be water in the container is 15.69°C, the specific heat of
a
V  p the metal in J/g.°C is
written as = k  ηbr c, where k is a dimensionless
t  l (a) 0.38 (b) 0.24
constant. Correct values of a , b and c are (c) 0.45 (d) 0.13
(a) a = 1, b = − 1, c = 4 (b) a = − 1, b = 1, c = 4
(c) a = 2, b = − 1, c = 3 (d) a = 1, b = − 2, c = − 4 BIOLOGY
76. The nucleus of a diploid organism contains 3 ng of
CHEMISTRY DNA in G1 -phase. Which one of the following state-
71. The reaction of an alkene X with bromine produces a ments describes the state of the cell at the end of S-
compound Y, which has 22.22% C, 3.71% H and phase?
74.07% Br. The ozonolysis of alkene X gives only one K Li Mg
Ag
product. The alkene X is,
kinetic energy of
photoelectron

[Given, atomic mass of C = 12; H = 1; Br = 80]


(a) ethylene (b) 1-butene
(c) 2-butene (d) 3-hexene
72. In the following reaction,
Hg 2 + dil. NaOH
H3 C — C ≡≡ C — H → X → Y
+ PhCHO
H 3O
2 3 4 5
X and Y, respectively, are Incident photon energy (eV)
https://iit-jeeacademy.blogspot.com

KVPY Question Paper 2017 Stream : SA 39

(a) The nucleus divides into two and each nucleus 78. The concentration of OH− ions in a solution with the
contains 3 ng of DNA . × 10−4 M is
H+ ions concentration of 13
(b) The nucleus does not divide and it contains 3 ng of
(a) 7.7 × 10−4 M (b) 1.3 × 10−4 M
DNA
−8
(c) The nucleus divides into two and each nucleus (c) 2.6 × 10 M (d) 7.7 × 10−11 M
contains 1.5 ng of DNA 79. Given that tidal volume is 600 mL, inspiratory
(d) The nucleus does not divide and it contains 6 ng of reserve volume is 2500 mL and expiratory reserve
DNA volume is 800 mL, what is the value of vital capacity
77. Three cellular processes are listed below. Choose the of lung?
correct combination of processes that involve proton (a) 3900 mL (b) 3300 mL
gradient across the membrane. (c) 3100 mL (d) 1400 mL
I. Photosynthesis II. Aerobic respiration 80. Which of the following organisms produces sperm
III. Anaerobic respiration without involving meiosis?
(a) II and III (b) I and II (a) Sandfly and fruitfly (b) Housefly and grasshopper
(c) Honeybee and ant (d) Zebra fish and frog
(c) I, II and III (d) I and III

Answers
PART-I
1 (b) 2 (c) 3 (b) 4 (b) 5 (d) 6 (c) 7 (d) 8 (d) 9 (b) 10 (c)
11 (a) 12 (a) 13 (b) 14 (d) 15 (d) 16 (c) 17 (d) 18 (c) 19 (b) 20 (c)
21 (a) 22 (c) 23 (b) 24 (c) 25 (d) 26 (b) 27 (c) 28 (c) 29 (a) 30 (c)
31 (a) 32 (c) 33 (d) 34 (d) 35 (c) 36 (b) 37 (a) 38 (d) 39 (b) 40 (c)
41 (a) 42 (b) 43 (d) 44 (a) 45 (d) 46 (b) 47 (a) 48 (d) 49 (a) 50 (b)
51 (c) 52 (a) 53 (c) 54 (a) 55 (c) 56 (b) 57 (a) 58 (d) 59 (c) 60 (d)

PART-II
61 (a) 62 (c) 63 (d) 64 (b) 65 (a) 66 (c) 67 (c) 68 (*) 69 (d) 70 (a)
71 (c) 72 (b) 73 (a) 74 (b) 75 (c) 76 (d) 77 (b) 78 (d) 79 (a) 80 (c)
https://iit-jeeacademy.blogspot.com

40 KVPY Question Paper 2017 Stream : SA

Solutions
2π 8π 4π ∴ a j and ak are both positive or negative.
1. (b) We have, side of quadrilateral has ∴ ∠A1OA5 = × 4= ⇒ ∠A2OA4 =
distinct integer second largest size has 9 9 9 ∴ P
C2 + n − P C2 = 55
length 10. OA1 = OA2 = r and a j ak is negative j < k
Let a = 8, b = 9, c = 10, (All are distinct) In ∆A1 OA2, a j ak = 50
We know, in quadrilateral Sum of three A6 any one of a j and ak are positive:
sides is greater than fourth side
∴ a + b + c > d ⇒ 8 + 9 + 10 > d ⇒ d < 27
A5 ∴ P
C1 × n − P C1 = 50 ⇒ P (n − P ) = 50
∴ Maximum length of 4th side is 26. A7 ⇒ P
C2 + n − P C2 = 55
2. (c) Exponent of 2 in 200!. ⇒ P (P − 1) + (n − P ) (n − P − 1) = 110
200   200   200   200   200  A4 ⇒ P 2 − P + (n − P )2 − (n − P ) = 110
= + + + +
 2   22   23   24   25  ⇒ P 2 + (n − P )2 − n = 110
+  6 +  7 +  8  ⇒ {P + (n − P )}2 − 2P (n − P ) − n = 110
200 200 200 A8 O 4π
 2   2   2  2π 9 ⇒ n 2 − 100 − n − 110 = 0 [Q P (n − P ) = 50]
9
= 100 + 50 + 25 + 12 + 6 + 3 + 1 = 197 A3 ∴ n 2 − n − 210 = 0
Exponent of 2 in 100! ⇒ (n − 15) (n + 14) = 0
100   100   100   100  +  100 
=
A9
+ + + n = 15, n ≠ − 14
 2   22   23   24   25 
A2 ∴ P (15 − p ) = 50
+  6 +  7 
100 100 A1
⇒ p 2 − 15 p + 50 = 0
 2   2  2 π OA12 + OA22 − A1 A22
cos = (P − 10) (P − 5) = 0, p = 5 or 10
= 50 + 25 + 12 + 6 + 3 + 1 = 97 9 2OA1OA2
∴ p 2 + (n − P )2 = 52 + 102
∴Exponent of 2. 2 π 2r 2 − 4
cos = = 25 + 100 = 125
200! 2197 9 2r 2
In = = 2100 7. (d) We have,
100! 297 2π
r 2 cos = r2 − 2 a , b, c, d are four distinct number from
∴ The largest power of 2 is 100. 9
the set {1, 2, 3, ..., 9}.
3. (b) Given, a1 + a2 + a3 + a4 = 0 2 2
r2 = = a + c is possible
2π π The minimum value of d
and a12 + a22 + a32 + a42 = 1 1 − cos 2 sin 2 b
9 9
It is possible only 1 when a = 2, b = 9, c = 1, d = 8
1 1 r= 2 1 16 + 9 25
when, a1 = a2 = and a3 = a4 = − π ∴ + = =
sin 9 8 72 72
2 2 9
∴ (a1 − a2 )2 + (a2 − a3 )2 + (a3 − a4 )2 8 π r 2 + r 2 − A1 A52 8. (d) Given, 72x ⋅ 48y = 6xy
+ (a4 − a1 )2 In ∆A1OA5 , cos =
9 2r 2 (23 ⋅ 32 )x ⋅ (24 ⋅ 3)y = 2xy ⋅ 3xy
2 2 2 2
1− 1  1 1  1 1  1 1 4π 23 x + 4 y ⋅ 32x + y = 2xy ⋅ 3xy
  +  +  + − +  + − −  ⇒ A1 A5 = 2r sin
2 2  2 2  2 2  2 2 9 Equating the exponent of 2 and 3, we get
0 + 1+ 0 + 1= 2 Similarly, ∆A2OA4 , 3x + 4 y = xy and 2x + y = xy
The value lies between (1.5, 2.5). 2π
A2A4 = 2r sin On solving these equation, we get
4. (b) x and y are positive integer 9 −15 5
4π 2π  x= and y =
x2 − y2 = 12345678 ∴ A1 A5 − A2A4 = 2r  sin − sin  3 3
 9 9
RHS 12345678 is and even number and − 15 5 − 10
π π ∴ x+ y= + =
last digit is 8. = 2r,  2 sin cos  3 3 3
 9 3
∴ The last digit of x be 3, 7 9. (b) Given,
 
and the last digit of y be 1, 9. 2 π 1  1  AB is diameter of circle S and C is the
= × 2 sin × r =
∴x and y must be odd and square of 9 2  
Q
π π
mid-point of arc length of AB.
difference is multiple of 8 but RHS is not sin
 sin 
9 9
multiple of 8. T C
=2
∴ S is the empty set. 3
5. (d) Given, A1 , A2 , A3 , ..., A9 are nine- 6. (c) Let p are positive number from
a1 , a2 , a3 , …, an 1
side regular polygon of each side 2 units.
∴ n − p are negative number. √2
∴ A1 A2 A2A3 A3 A4 A8 A9 2
= 2π = =. Given a j , ak is positive j < k and a j ak = 55
∠A1OA2 =.......................= = a j ak is positive.
9 A 1 O 1 B
https://iit-jeeacademy.blogspot.com

KVPY Question Paper 2017 Stream : SA 41

AC is diameter of circle T . In ∆DNC and ∆DMA, 15. (d) Given,


AB = 2 ∆DNC ~ ∆DMA x5 − 6x4 + 11x3 − 5x2 − 3x + 2 = 0
∴ OA = OB = OC = 1 ⇒
DN
=
NC (x − 1) (x − 2) (x3 − 3x2 + 1) = 0
Area of shaded region DM MA The sum of non-integer roots are sum of
= Area of semi-circle T + Area of 4 5 25
⇒ = ⇒ MA = roots of equation
∆OAC − Area of quadrant of circle S 5 MA 4 x3 − 3x2 + 1 = 0 i.e. 3.
25 41
π  2
2
1 π ∴ AC = MC + AM = 4 + = 16. (c) For a nucleus, ZA X
=   + × 1 × 1 − × (1)
2
4 4
2  2  2 4 13. (b) We have, A , G , H be arithmetic, Mass number, A = N + Z
π 1 π 1 where, N = number of neutrons
= + − = geometric and harmonic mean respectively
4 2 4 2 of two distinct positive real numbers. and Z = number of protons.
10. (c) Let p(x) = x135 + x125 − x115 + x5 + 1, A (G − H )x2 + G (H − A )x + H (A − G ) = 0
In 65 65
32 X and 33 Y , number of protons are
q(x) = x3 − x and p (x) = q(x)k + r (x) Let α and β be roots of the given equation different. So, these are not isotopes.
x135 + x125 − x115 + x5 + 1 α<β
− G (H − A ) H (A − G ) In 86 85
42 X and 42 Y , number of protons are
= (x3 − x)k + ax2 + bx + c ∴α + β = ⇒ αβ = equal. So, these are isotopes.
[Q r (x) = ax2 + bx + c] A (G − H ) A (G − H )
Put x = 0, In 174 177
85 X and 88 Y , number of neutrons are
β = 1is satisfied the equation,
∴ c=1 i.e. AG − AH + GA − GA + HA − HG = 0 174 − 85 = 89 and 177 − 88 = 89.
Put x = 1, 3 = a + b + c ⇒ 3 = a + b + 1 So, both have same number of neutrons.
∴ One of root is 1
⇒ a+ b= 2 … (i) In 235 235
H (A − G ) 92 X and 94 Y , both have same mass
Put x = − 1, − 1 = a − b + c ⇒ − 1 = a − b + 1 α=
A (G − H ) number, so these are isobars.
⇒ a− b= − 2 … (ii)
From Eqs. (i) and (ii), we get AH − GH 17. (d) For thrown ball,
⇒ α=
a = 0, b = 2 AG − AH h = ut −
1 2
gt ⇒
h 1
= u − gt
∴ r (x) = 2x + 1 G 2 − GH 2 t 2
α= [Q AH = G 2 ] h −g
∴ Degree of r (x) = 1 AG − AH ⇒ = ⋅t + u
G (G − H ) G t 2
11. (a) The distinct prime factor of α= ⇒α= [Q A > G ]
A (G − H ) A Comparing with y = mx + c, graph of
43361 = 131 × 331 h −g
where p1 = 131 and p2 = 331 α<1 versus t is a straight line with slope .
Hence, 0< α < 1 t 2
∴ p1 + p2 = 131 + 331 = 462 h
14. (d) Given, ABCD is a square —
t
12. (a) Given, ABC is right angled AB = CD = AD = BC = 1
triangle (0, u)
AC is tangent of circle Slope
∠C = 90° –g
∠OAC = 90° =tanθ = —
2
CD is perpendicular on AB, DN and DM ∠CAD = 45°
t
are parallel to AC and BC, respectively. ∴ ∠OAD = 45°
DN = 4 and DM = 5 ∴ OA = 2
From above graph, value of acceleration
B O due to gravity can be obtained by
θ 45° multiplying its slope with 2.
√2 1 18. (c) Let total distance is 3x km.
b y
A B
x x x
4 45° 1
D N A D For Pfirst part, time taken is t Q
45° x
1 =
h
10
a 5 5 x
90 – θ For second part, time taken is t2 = h
X 20
90 – θ θ 1
x
A x M 4 C For third part, time taken is t3 = h
60
In ∆DMC and ∆DNB, B C Average speed for PQ distance
∆DMC ~ ∆DNB ∴Area of shaded region Total distance 3x
DM MC = =
∴ = = Area of square + Area of Total time x x x
+ +
DN NB ∆AOD − Area of sector 10 20 60
5 4 16 1 45 3x
⇒ = ⇒ NB = = 1+ × − × ( 2 )2 ⋅ π = = 18 km/h
4 NB 5 360  6x + 3x + x 
1  
∴ BC = CN + NB = 5 +
16 41
= 1 π 3 π 6− π  60 
= 1+ 2 − = − =
5 5 2 4 2 4 4
https://iit-jeeacademy.blogspot.com

42 KVPY Question Paper 2017 Stream : SA

19. (b) For points P and Q ray diagram Hence, shadow of moon covers only a So, correct graph of acceleration and time
will be as shown below. small part of earth as shown in the figure will be as shown below.
given below. a
P Q
P′ Earth
Q′ Shadow
C F of moon
t

– 9.8 ms–2
Clearly image will be 23. (b) Let m gram of ice melt and water
reaches 0°C. 27. (c) Image produced on retina are real
P′ P Q and inverted. We are able to perceive
Then,
Q′ them as erect because of processing of our
Heat lost by ice = Heat gained by water brain.
⇒ miceLice + mice cice∆ Tice = mw cw ∆Tw Only statement III is incorrect all other
⇒ m(335) + m(2.2) (0 − (−7)) are correct.
S′
R′ R S = 200 (4.2) (15 − 0) 28. (c)
⇒ m(335 + 15.4) = 12600 Acceleration=f Retardation=a
20. (c) In Guericke’s experiment, 12600
⇒ m= ≈ 36 g
350.4 t=0 v1 d
A v2=0
So, ice left in mixture is 100 − 36 ≈ 64 g. u=0 B C
F F 2
Hence, nearest option is 67 g. —d
3
A=πR2 24. (c) Let S is the source placed at 2
Velocity of particle at the end of
If pressure difference between outside distance 2 f (= 60 cm) . 3
and inside is p, then 2f distance is v1 .
F Now, by equation of motion,
= p or F = pA = pπR 2
A S′′ M ⇒ v2 − u 2 = 2as, we have
21. (a) Net resultant force is due to S O S′ 2
⇒ v12 − 02 = 2f × d
unbalanced forces of 3q and 2q charges. 3
2q f f 4 2
⇒ v1 = fd ⇒ v1 =
2
. fd
2f 3 3
q q Image of S is formed at S′ (OS′ = 60 cm). As, final velocity is zero, so for next part
As light rays after reflection from mirror of journey is
O are parallel after passing through lens, v2 − u 2 = 2as
this is possible when they cross through
0 − v12 = 2a  d 
q 3q
1
focus as shown in figure. Gives,
3 
For plane mirror, MS ′′ = MS′ 4 2
q So, OM = d = OS ′′ + S ′′M ⇒ − fd = ad
3 3
After cancellation of equal forces, we f 30
= f + = 30 + = 45 cm or deacceleration, a = − 2f
have following configuration: 2 2
q Now, using v = u + at, for first part of
25. (d) A plano-concave lens is a
Unbalanced journey,
diverging lens and a plano-convex lens is
small force
a converging lens. v = u + at
Image formed by a plano-concave lens is ⇒ v1 = 0 + ft1
O Resultant is more always erect and virtual. 2 2 d
near to large force ⇒ fd = ft1 ⇒ t1 = .
Image formed by a plano-convex lens is 3 3 f
2q real and inverted when object is placed at
For second part of journey,
Unbalanced a distance larger than focal length of lens.
v = u + at
large force Plano-convex lens is a converging lens.
Hence, net force is towards right. Hence, image formed by a plano-convex ⇒ 0 = v1 − 2ft2
2
22. (c) Size of moon is very small lens is inverted. ⇒ fd = 2ft2
compared to that of earth also, moon is So, all statements are correct. 3
much nearer to earth in comparison to 1 d
26. (b) Acceleration of ball at all ⇒ t2 =
sun. instances is − 9.8 ms−2. 3 f
https://iit-jeeacademy.blogspot.com

KVPY Question Paper 2017 Stream : SA 43

So, total time is 32. (c) Among the given carbanions 2Na
Cl Br
t = t1 + t2 compound IV has maximum stability as Ether

2 d 1 d it is resonance stabilised, i.e. 1-bromo-3-chlorocyclobutane


= + –
3 f 3 f C CH2CH3 H3CC CH2CH3
CH3 + Cl Cl
d 3d
= 3 = Major
Minor
f f – bicyclo [1.1.0]

29. (a) Magnification is given by This reaction is an example of


v f −v Wurtz’ reaction.
m= =
u f H3C CCH2CH3 35. (c) Among the given compounds IV
⇒ fm = f − v will have the least basicity as the lone
pair on nitrogen takes part in resonance
⇒ v = f (1 − m)
and will not be available for donation.
1− m
⇒ v= – Compound I will have more basicity than
P IV, because of the availability of lone pair
30. (c) Initial pressure in cylinder is –
of NH 2 group. But its basicity will be less
H3C CCH2CH3 H3C CCH2CH3
atmospheric pressure p0 .
than II and III because of the − I effect of
When mass m is attached to piston, then – NO2 which decreases the basicity of aniline.
pressure = p − mg . Now between compounds II and III, II
0
A will be most basic. This is because it is an
As, temperature remains constant during Among the other given carbanions, aliphatic amine and also in III, the
expansion stability order of carbanion decreases as nitrogen is present within the ring, so its
electron will not be as much available as
⇒ i i = pf V f
pV we move from 1° to 3° anion because of
in II.
⇒ p0Vi =  p0 −
mg  + I effect of methyl groups. There is an
 . Vf Thus, the order of basicity will be,
 A  increased intensity of negative charge on
IV < I < III < II.
Vf p0 central carbon of 3° carbanion which
⇒ = 36. (b) As Na is an alkali metal it has
Vi  p − mg  further makes it unstable. Thus, the
 0  least ionisation energy due to its large
 A  correct increasing order is
– – size. Rest of the three elements B, N and
Vi mg mg V (CH3)3C < CH3CHCH2CH3
= 1− ⇒ = 1− i O are non-metal and lie in same period.
Vf p0 A p0 A Vf 3° 2° So, as we move from left to right in a
III II
mg V f − Vi ∆V period the ionisation energy increases
⇒ = = – –
p0 A Vf Vf <CH3CH2CH2CH2 <CH3C(Ph)CH2CH3 due to increased nuclear charge. But,
N has half-filled configuration which is
Now, when temperature is reduced by 1°
I IV stable, thus it will have maximum
∆T , the volume of gas again contracts to ionisation energy. Thus, the correct order
its original volume.
33. (d)
+ – is, Na < B < O < N.
V N2Cl OH
⇒ = constant 37. (a) P2O5 , As2O3 are acidic oxides,
T + Sb2O3 is an amphoteric while Bi 2O3 is
∆V ∆T
or = basic oxide but P2O5 is most acidic among
V T Diazonium Phenol them all. This is because down the group,
∆T ∆V mg salt
metallic character increases and metal
⇒ = = OH
T Vf p0 A oxides are more basic, thus the basicity of
∆T mg metallic oxides also increases.
⇒ = N
T p0 A NaOH N 38. (d) Metal with low reactivity can be
50 × 10 extracted by heating ore in air. This
= process of extraction is known as
10 5 × 314
. × (0.2)2 p-hydroxy azobenzene roasting. Among the given metals, Cu has
(azodye)
5 102 least reactivity, so it can be directly
= × This reaction is known as coupling
. × 4 10 × 10−2
314 5 extracted by heating its ore in air.
reaction. In this reaction electrophilic
= 0.4 × 10−1 = 0.04 2Cu 2S + 3O2 → 2Cu 2O + 2SO2 ↑
aromatic substitution takes place where
31. (a) The structure of aryl diazonium cation is the electrophile 39. (b) Total number of electrons in
3-methylpent-2-ene is and the activated arene is a nucleophile. S2− = 16 + 2 = 18
5 4 34. (d) In the reaction of Total number of electrons present in the
CH3 CH2 3 2 elements given in options are as follows :
1-bromo-3-chlorocyclobutane with two
C CH (i) Na +
1 equivalents of sodium in ether gives
CH3 CH3 bicyclo [1.1.0] as a major product. Total number of electrons in
Na + = 11 − 1 = 10
https://iit-jeeacademy.blogspot.com

44 KVPY Question Paper 2017 Stream : SA

(ii) Ca 2+ For n=3 50. (b) Tobacco Mosaic Virus (TMV)


Total number of electrons in ∴ Maximum number of electrons in shell affects photosynthetic tissue of the
Ca 2+ = 20 − 2 = 18 with (n = 3) = 2(3)2 = 18. infected plant. Other statements can be
(iii) Mg2 + 45. (d) Among the given elements Li and corrected as TMV affects all
Total number of electrons in Na belong to group 1, i.e. they are alkali dicotyledonous plants, of which most
Mg2+ = 12 − 2 = 10 metals. So, they will have maximum important are tobacco and tomato. But it
(iv) Sr2+ atomic radii. Between Li and Na, Na will does not affect any monocotyledonous
have the largest atomic radii. This is plant. TMV is a ssRNA virus, it infects a
Total number of electrons in
because as we move down the group wide range of plants, especially tobacco
Sr 2+ = 38 − 2 = 36
atomic radii increases. and other members of the family
Thus, S2− and Ca 2+ have same
number of electrons. Si belongs to group 14 and F belongs to Solanaceae. TMV does not infect
group 17. So, as we move from left to gymnosperms.
40. (c) Ca + 2HCl → CaCl 2 + H 2 right in period the atomic radii decreases. 51. (c) Placenta allows the foetus to
At STP 22.4 L of H 2 = 1 mole of H 2 Thus, Si will have large atomic radii than transfer waste products to the mother’s
1 F but less than Li.
∴ 5.04 of H2 = × 5.04 = 0.225 mole blood. Other statements can be corrected
22.4 So, the correct order of atomic radii of as Placenta gives protection against most
X given elements is, Na > Li > Si > F.
Number of moles of Ca = bacteria and does not allow infections to
40 46. (b) The major excretory product of enter the foetus.
1 mole of Ca reacts to produce 1 mole of H 2 . birds is uric acid. Nitrogenous wastes in Placenta allows gaseous exchange via the
X
∴ moles of Ca reacts to produce 0.225 the body of animals tend to form toxic mother’s blood supply, i.e. it allows
40 ammonia, which must be excreted. diffusion of O2 and CO2 . Placenta
moles of H2 gas. Mammals such as humans excrete urea,
X produces hormones like human Chorionic
∴ = 0.225 while birds, reptiles and some terrestrial
Gonadotropin (hCG), progesterone,
40 invertebrates produce uric acid as waste
oestrogen and human Placental Lactogen
X = 9.000 excretory product.
(hPL).
41. (a) Given, mass of an object = 20 g 47. (a) There are 64 codons present in
52. (a) Respiratory Quotient (RQ)
Velocity of an object = 100 ms−1 each living organism, out of which, 61
measures the ratio of the volume of
According de Broglie codons represent or code for amino acids
carbon dioxide (VC ) produced by an
and rest three are stop codons. Thus,
h 6.626 × 10−31 organism to the volume of oxygen
λ= = there are more codon combinations than
mv 20 × 10−3 × 100 consumed (VO ). The RQ for the given
there are amino acids. The genetic code is
= 3.313 × 10−34 m described as degenerate because more
equation is
CO2 produced 102
42. (b) CH4 + 2O2 → CO2 + 2H2O than one codon sequence can code for the RQ = = = 0.703
O2 consumed 145
of O2 in 750 L of air (containing 20% same amino acid.
of O2) RQ = 0.703
48. (d) In cell cycle, during interphase,
20 the DNA is replicated. Interphase begins 53. (c) Nucleosome is a structural unit of
= × 750 = 150 L
100 with G1 -phase. During this phase, the cell a eukaryotic chromosome, consisting of a
Some of CH4 is ignited with 100 mL of O2. length of DNA coiled around a core
makes a variety of proteins that are
histone. Thus, nucleosome is not a
∴Remaining volume of O2 in vessel = 50 L needed for DNA replication. G1 -phase is
membrane bound organelle of a cell.
22.4 L contains 1 mole of O2 followed by synthetic or S-phase. This
∴ 50 L contains = 1/22.4 × 50 = 2.2 mole. phase is responsible for the synthesis or
54. (a) Tyrosine is the immediate
precursor of the thyroxine hormone.
43. (d) CO2 is passed through lime water replication of DNA. The aim of this
Thyroxine is produced in the thyroid
which initially turns the solution milky process is to produce double the amount
gland from tyrosine and iodine.
because of formation of calcium of DNA, providing the basis for the Thyrotropin Releasing Hormone (TRH) is
carbonate. On continuous bubbling of CO2 chromosome sets of the daughter cells. produced by the hypothalamus which
the solution becomes clear due to the
49. (a) Gene flow is the transfer of induces the release of thyroxine.
formation of calcium bicarbonate.
genetic variation from one population to 55. (c) The haemoglobin molecule has
Ca(OH)2 + CO2 → CaCO3 ↓ + H 2O
Lime water (Milky) another. Genetic drift is a change in the four binding sites for oxygen molecules.
frequency of an allele within a population Thus, each Hb tetramer can bind four
CaCO3 + CO2 + H 2O → Ca(HCO3 )2 ( aq)
Excess Calcium bicarbonate over time. Speciation is the formation of oxygen molecules. Haemoglobin is the
(Soluble in water) new and distinct species in the course of oxygen transporting protein of red blood
44. (a) Maximum number of electrons evolution. Genetic shift is a major change cells and is a globular protein with
that can be accommodated in the shell. within a population which changes the quaternary structure. Haemoglobin
Principal quantum with number, population altogether. consists of four polypeptide subunits,
2α chains and 2β chains.
n = 2n 2
https://iit-jeeacademy.blogspot.com

KVPY Question Paper 2017 Stream : SA 45

during his life in order to adapt to its 3θ π θ π


56. (b) The smooth endoplasmic ⇒ = or =
reticulum functions in lipid synthesis and environment, those changes are passed 2 2 2 2
metabolism, the production of steroid on to its offsprings. π
⇒ θ = or θ = π
hormones and detoxification. Smooth 61. (a) OC = 1radius of circle x2 + y2 = 1 3
Endoplasmic Reticulum (SER) is a π
OA = AP For maximum θ =
meshwork of five disc-like tubular 3
∴ ∠AOP = ∠OPA = 45°
membrane vesicles, part of a continuous 63. (d) Given, A1 = {(x, y) : x2 + 2 y2 ≤ 1}
membrane organelle within the
A2 = {(x, y) :|x|3 + 2 2|y|3 ≤ 1}
cytoplasm of eukaryotic cells.
B P A3 = {(x, y) : max (|x|, 2 y1 ≤ 1)}
57. (a) The light dependent reaction of
photosynthesis uses light energy to make Graph of A1 , A2 and A3 are
C
two molecules needed for the next stage A
O
of photosynthesis. These include the
energy storage molecule ATP and the
1
y = √2 ( 0, √12 (
x=–1
reduced electron carrier NADPH. The
x2+2y2=1
light reaction takes place in the thylakoid x=1
membranes of chloroplasts. –1, 0
AP = PC radius of circle Max(|x|, 2√2/|y|)≤1) (1, 0)
58. (d) Hypothalamus directly controls
In ∆PCA,
the production of glucocorticoids and
∴∠PCA + ∠PAC + ∠CPA = 180°
androgens. These hormones are secreted y=– 1
in response to ACTH (Adrenocorticotropic
⇒ 2∠PCA + 45° = 180°
135°
√2 ( 0, – √12 (
Hormone), which is secreted from the ⇒ ∠PCA =
2
anterior pituitary gland. The ACTH is Clearly from graph A1 ⊂ A2 ⊂ A3
⇒ ∠OCA = 180° − ∠PCA
released in response to corticotropin 64. (b) Given, area of ∆EAB = area of
135° 3π
releasing hormone from the ⇒ ∠OCA = 180° − = π−
2 8 square ABCD
hypothalamus. The pathway can be
5π EB = ED = 130
explained as ⇒ ∠OCA =
8 Let side of square = x
Hypothalamus ACTH-releasing hormone 62. (c) Given ABCD is trapezium x
BM = = AM
AD = BC = CD = 30 2
Let the smallest angle be θ. D C
Pituitary ACTH D C
gland

30 30 x
Adrenal gland x
θ M √2
A M N B
Glucocorticoids
Androgens ∠DAB = θ A x B
In ∆AMD,
59. (c) Acetaminophen, also known as AM
paracetamol is not produced by plant or cosθ =
fungi, it is artificially formed. The ⇒ AM = 30cosθ
DM
30 E
starting material for the manufacturing = DM = 30
sin θ = 30 sin θ Area of ∆AEB = Area of ∆BEM − area of
of paracetamol is phenol which is
1
Area of trapezium = (AB + CD ) DM (∆AMB )
nitrated to give a mixture of the ortho
2 1 1
and para-nitrotoluene. = EM × BM − AM × BM
1 2 2
Other drugs are obtained as, penicillin is ⇒ A = (60 + 60 cosθ) 30 sin θ
2 1
an antibiotic obtained from ascomycetous = BM (EM − AM )
fungi Penicillium notatum. Reserpine is ⇒ A = 900 (sin θ + sin θ cosθ) 2
an alkaloid derived from the roots of ⇒
dA
= 900 (cosθ − sin 2 θ + cos2 θ) 1 x  x2 x 
= 130 − −
Rauwolfia serpentina plant. Quinine dθ 2 2  2 2 
dA
comes from the bark of the Cinchona For maximum or minimum, put =0
tree. dθ 1x  x2 x 
∴ . 130 − − = x2
∴ cosθ − sin 2 θ + cos2 θ = 0  2 2 
60. (d) Lamarck is best known for his  
theory of Inheritance of Acquired ⇒ cosθ + cos 2θ = 0 2 2 2
3θ θ x x
Characteristics’, first presented in 1801. ⇒ 2 cos cos = 0 ⇒ 130 − = 2 2x +
It states that if an organism changes 2 2 2 2
https://iit-jeeacademy.blogspot.com

46 KVPY Question Paper 2017 Stream : SA

x2  5x 
2
From similar triangles ∆BGF and ∆DEF, From E to F,
⇒ 130 − = 
2  2 DE FE V 0 = I 0 R0
we have =
x2 25 x2 BG GF ⇒ s lope = R0
⇒ 130 − = V0
2 2 h′  d − x ∴ At F, resistance = R0 =
=  I0
⇒ 13x2 = 130 ⇒ x2 = 10 h  x 
∴Area of square = 10 d h′ + h
From F to G,
⇒ = …(i) V = V 0 = constant
65. (a) Given, A = {1, 2, 3, …, 30} x h
But current increases, so resistance must
Case I All three number are multiple of 3 Now, from similar triangles ∆ABG and
decreases.
then product of three number are ∆ACE, we have R V
divisible by 9. At G, resistance = 0 = 0 .
CE BG 2 2I 0
∴ 10
C3 = 120 =
AE AG
Case II Two number are multiple of 3 From G to H, current is constant but
and other are not multiple of 9. ∴ ∆ABG ≅ ∆FBG voltage increases, so resistance
decreases.
i.e. 10
C2 × 20C1 = 900 and AG = GF = x 2V 0 V 0
At H, resistance = = = R0
Case III One are multiple of 9 and other H + h′ h
two are not multiple of 3. ⇒ = 2I 0 I0
d+ x x
3
C1 × 20C2 = 570 So, correct option is (d).
d H + h′ − h a
⇒ = = k   ηb r c ,
∴Total number of ways = 120 + 900 + 570 …(ii) V p
70. (a) From
x h t  l
= 1590
Equating Eqs. (i) and (ii), we get we have
66. (c) We have, heat extracted from a a
h′ + h H + h′ − h  ML−1 T −2 
liquid during solidification, = [L3 T −1 ] =  −1 −1 b c
h h  [ML T ] [L]
U = Qt = mL ⇒ L ∝ U  L 
Also, heat extracted from liquid during ⇒ 2h = H
Equating powers of M, L and T, we get
cooling, Hence, height of shadow on wall is 2h.
a + b = 0 ⇒ − 2a − b + c = 3
H = Qt = mc∆T 68. (No option is matching) −2a − b = − 1
Temperature of liquid, Net force on marble due to water is
Solving, we get a = 1, b = − 1 and c = 4
Q
T = ⋅ t + Ti 71. (c) Skeletal diagram for the given
mc
information can be shown as:
Slope of T versus t line is inversely 9 cm
proportional to specific heat c. X + Br2 Y
Now, from given graph, we get 1 cm O3/Zn, H2O

T One product
U1>U2 Fnet = (Force of water column of height ~
1 Slope of 2 is more 9 cm) − (Buoyant force on marble) Emperical formula for Y can be
⇒ CL1<CL2 calculated as,
= πr 2 ρw gh − Volume of marble
2 under water × ρw × g Elem- % of At Moles Simplest Simplest
3 ents elem mass of molar whole
= πr ρw gh −
2
πr ρw g ent element ratio no.
2
t C 22.22% 12 22.22/12 1.85/0.92 2 × 2
= πr 2ρw g  h − r 
2
= 1.85 = 2.01
we get, U1 > U 2 and CL1 < CL 2  33 
H 3.71% 1 3.71/1 3.71/0.92 4 × 2
67. (c) From geometry of figure, shadow . × (1 × 10−2 )2 × 1000 × 10
≈ 314 = 3.71 = 4.03
length is CD (= H ).  9 − 2  × 10−2 Br 74.07% 80 74.07/80 0.92/0.92 1 × 2
 
 3 = 0.92 =1
C −2
= 26 × 10 = 0.26 N (downwards)
α ∴The emperical formula of Y is C4 H8 Br2.
lig m o nt
a e

69. (d) In given V -I graph,


ht f
be cid

According to retero synthesis.


In

H V
CH3 CH CH — CH3+Br2
B 2V0 H
(X)
D
2-butene
hα α α h′ V0
F G CH3 CH CH CH3
90–α 90–α 90–α 90–α

A G F E Br Br
x d–x E (Y)
d I 2, 3-dibromobutane
I0 2I0
https://iit-jeeacademy.blogspot.com

KVPY Question Paper 2017 Stream : SA 47

Also, ∴ (Number of eq.)KMnO4 cell with 3 ng of DNA in G1 -phase will


CH3 CH CH CH3 + O3 = (Number of eq.)H 2O2 now have 6 ng of DNA. G 2 -phase comes
1× 5 = x × 2 after S-phase. It is second growth phase
(X)
5 but here the DNA content will remain
O x = = 2. 5 6 ng.
2
CH3 CH CH CH3 77. (b) In photosynthesis,
74. (b) Given, wavelength λ = 400 nm
photophosphorylation and in aerobic
O O = 400 × 10−9
m respiration, oxidative phosphorylation
hc
Energy of photon E occurs that requires proton gradient. The
= sunlight-driven production of ATP from
6.626 × 10−34 Js × 3λ × 108 m / s
O Zn/H2O
ADP and inorganic phosphate is called
2CH3C H E= 400 × 10−9 m photophosphorylation. It occurs in the
chloroplast. Oxidative phosphorylation is
Acetaldehyde E = 4.97 × 10−19 J the process in which ATP is formed by
1 eV
72. (b) = 4.97 × 10−19 × the transfer of electrons from NADH or
O
1.6 × 10−19 J FADH 2 to O2 by a series of electron
Hg2+ E = 31
. eV carriers. It occurs inside the
CH3 C C H H3C C H3 C mitochondria.
H3O+ (X) If the energy of incident light ≥ work
PhCHO dil. NaOH
function of light. Then photoelectrons will 78. (d) [H + ] [OH − ] = 10−14
(aldol
condensation) be ejected. 1.3 × 10−4 × [OH − ] = 10−14
Thus, K and Li will emit photoelectrons 1
O [OH − ] = × 10+4 × 10−14
as their threshold energy (obtained from 1.3
CH3 C CH CH Ph graph) is less than 3.1 eV. 1
(Y) = × 10−10 = 0.769 × 10−10
(α, β-unsaturated ketone) 75. (c) Given, 1.3
In first step one molecule of water adds to weight of metal = 100 g = 0.77 × 10−10 = 7.7 × 10−11 M
alkyne on warming with mercuric
T2 = 15.69° C, T1 = 80° C 79. (a) Vital capacity = Inspiratory
sulphate and dilute sulphuric acid to
reserve volume + Tidal volume +
form carbonyl compound, i.e. acetone. In Specific heat of water = 4184
. J/g°C
Expiratory reserve volume
second step 2 molecules of acetone Heat gained by 100 g of metal = Heat lost
condense in presence of dil. NaOH to = 2500 mL + 800 mL + 600 mL = 3900 mL
by 1000 g of water.
form α , β-unsaturated ketone. This We know, Q = mc ∆T
Vital capacity is the volume of air
reaction is known as aldol condensation. breathed out after the deepest inhalation.
∴ 100 × x × (80 − 15.69)
73. (a) For the reaction, 80. (c) All haploid sexually reproducing
= 1000 × 4184
. (15. 69 − 15)
+7 −4 organisms would produce sperms/male
2MnO−4 + 5H2 O 2 + 6H+ → = 100 x (64.31) = 2886.96 gametes without the process of meiosis,
−2 2886.96
2Mn 2+ + 8H2 O + 5O2 x= = 0.448 ≈ 0.45 J/g.°C e.g. Honeybee (Apis) and Ant (Formica).
6431 Haploid parents produce gametes by
Let the number of moles of oxygen mitotic division. This happens because
76. (d) In the G1 -phase of cell cycle, the
produced per mole KMnO4 be x. meiosis is reductional division in which
cell grows in size, i.e. the cell synthesises
Number of equivalent = Number of moles the daughter cells contain half the
various enzymes and nutrients that are
× change in oxidation state needed later on for DNA replication. The
number of chromosomes as the parent
Number of equivalent of KMnO4 = 1 × 5 cell. Therefore, haploid organisms do not
next phase of cell cycle is S-phase during
show meiosis to further disturb their
Number of equivalent of H 2O2 = x × 2 which the DNA amount doubles up, i.e. a
ploidy.
https://iit-jeeacademy.blogspot.com

48 KVPY Question Paper 2016 Stream : SA

KVPY
KISHORE VAIGYANIK PROTSAHAN YOJANA

QUESTION PAPER 2016


Stream : SA
MM 100

Instructions
There are 80 questions in this paper.
This question paper contains two parts; Part I and Part II. There are four sections; Mathematics, Physics, Chemistry
and Biology in each part.
Out of the four options given with each question, only one is correct.

PART-I (1 Mark Questions)


MATHEMATICS 5. Let a1 , a 2 , ... , a100 be non-zero real numbers such that
1. Suppose the quadratic polynomial P (x) = ax + bx + c 2 a1 + a 2 + ... + a100 = 0
Then,
has positive coefficients a , b, c in arithmetic − ai
(a) Σ100 100
i = 1 ai 2 > 0 and Σi = 1 ai 2
ai
<0
progression in that order. If P (x) = 0 has integer roots
− ai
α and β. Then, α + β + αβ is equal to (b) Σ100 ai 100
i = 1 ai 2 ≥ 0 and Σi = 1 ai 2 ≥0
− ai
(a) 3 (b) 5 (c) 7 (d) 14 (c) Σ100 100
i = 1 ai 2 ≤ 0 and Σi = 1 ai 2
ai
≤0
− ai
2. The number of digits in the decimal expansion of (d) The sign of Σ100 100
i = 1 ai 2 or Σi = 1 ai 2
ai
depends on the
5 16 choice of ai ’ s
16 5 is
(a) 16 (b) 17 (c) 18 (d) 19 6. Let ABCD be a trapezium, in which AB is parallel to
3. Let t be real number such that t 2 = at + b for some CD, AB = 11, BC =4, CD = 6 and DA = 3. The distance
positive integers a and b. Then, for any choice of between AB and CD is
positive integers a and b, t3 is never equal to (a) 2 (b) 2.4 (c) 2.8
(d) Not determinable with the data
(a) 4t + 3 (b) 8t + 5 (c) 10t + 3 (d) 6t + 5
4. Consider the equation (1 + a + b)2 = 3(1 + a 2 + b2) , 7. The points A, B, C , D, E are marked on the
where a , b are real numbers. Then, circumference of a circle in clockwise direction such
that ∠ABC = 130° and ∠CDE = 110°. The measure of
(a) there is no solution pair (a , b)
∠ACE in degree is
(b) there are infinitely many solution pairs (a , b)
(a) 50° (b) 60°
(c) there are exactly two solution pairs (a , b)
(c) 70° (d) 80°
(d) there is exactly one solution pair (a , b)
https://iit-jeeacademy.blogspot.com

KVPY Question Paper 2017 Stream : SA 49

8. Three circles of radii 1, 2 and 3 units respectively


touch each other externally in the plane. The cir-
PHYSICS
cumradius of the triangle formed by joining the 16. A person walks 25.0° north of east for 3.18 km. How
centers of the circles is far would she have to walk due north and then due
(a) 1.5 (b) 2 (c) 2.5 (d) 3 east to arrive at the same location?
9. Let P be a point inside a ∆ ABC with ∠ABC = 90°. Let (a) Towards north 2.88 km and towards east 1.34 km
(b) Towards north 2.11 km and towards east 2.11 km
P1 and P2 be the images of P under reflection in AB
(c) Towards north 1.25 km and towards east 1.93 km
and BC respectively. The distance between the
(d) Towards north 1.34 km and towards east 2.88 km
circumcenters of ∆ ABC and P1PP2 is
AB AP + BP + CP 17. The length and width of a rectangular room are
(a) (b) . ± 005
measured to be 395 . m and 305 . ± 005
. m,
2 3
AC AB + BC + AC respectively. The area of the floor is
(c) (d) (a) 12.05 ± 0.01 m2 (b) 12.05 ± 0.005 m2
2 2
(c) 12.05 ± 0.34 m2 (d) 12.05 ± 0.40 m2
10. Let a and b be two positive real numbers such that
a + 2b ≤ 1. Let A1 and A2 be respectively the areas of 18. A car goes around uniform circular track of radius R at
a uniform speed v once in every T seconds. The mag-
circles with radii ab3 and b2. Then, the maximum
A nitude of the centripetal acceleration is a c. If the car
possible value of 1 is now goes uniformly around a larger circular track of
A2
radius 2R and experiences a centripetal acceleration of
1 1
(a) (b) magnitude 8a c. Then, its time period is
16 64 (a) 2T (b) 3T (c) T /2 (d) 3/2T
1 1
(c) (d)
16 2 32 19. The primary and the secondary coils of a transformer
contain 10 and 100 turns, respectively. The primary
11. There are two candles of same length and same size. coil is connected to a battery that supplies a constant
Both of them burn at uniform rate. The first one voltage of 1.5 V. The voltage across the secondary coil
burns in 5 hr and the second one burns in 3 h. Both is
the candles are lit together. After how many minutes (a) 1.5 V (b) 0.15 V (c) 0.0 V (d) 15 V
the length of the first candle is 3 times that of the
20. Water falls down a 500.0 m shaft to reach a turbine
other?
which generates electricity. How much water must
(a) 90 (b) 120 (c) 135 (d) 150 . × 109 W of
fall per second in order to generate 100
12. Consider a cuboid all of whose edges are integers and power ? (Assume 50% efficiency of conversion and
whose base is a square. Suppose the sum of all its g =10 ms−2)
edges is numerically equal to the sum of the areas of (a) 250 m3 (b) 400 m3
all its six faces. Then, the sum of all its edges is (c) 500 m3 (d) 200 m3
(a) 12 (b) 18 (c) 24 (d) 36 21. The diagram below shows two circular loops of wire
13. Let A1 A2 Am (A and B) centred on and perpendicular to the X-axis
, ,......, be non-empty subsets of and oriented with their planes parallel to each other.
{ 1, The Y -axis passes vertically through loop A (dashed
1. 2,The
3, …, 100} satisfying
numbers| A |,| A |,the
1 2
following
K ,| conditions:
A |are distinct.
m line). There is a current I B in loop B as shown in the
2. A1 , A2 ,... , Am are pairwise disjoint. diagram. Possible actions which we might perform on
(Here| A|donotes the number of elements in the set A) loop A are
Y
Then, the maximum possible value of m is
(a) 13 (b) 14 (c) 15 (d) 16
IB
14. The number of all 2-digit numbers n, such that n is
equal to the sum of the square of digit in its tens –X X
place and the cube of the digit in units place is
(a) 0 (b) 1 (c) 2 (d) 4
A
15. Let f be a function defined on the set of all positive B
integers such that f (xy) = f (x) + f ( y) for all positive
integers x, y. If f (12) = 24 and f (8) = 15. The value of
f (48) is (I) move A to the right along X-axis closer to B
(a) 31 (b) 32 (c) 33 (d) 34 (II) move A to the left along X-axis away from B
https://iit-jeeacademy.blogspot.com

50 KVPY Question Paper 2016 Stream : SA

(III) as viewed from above, rotate A clockwise about nucleus. The lead nucleus has A =206. The
Y -axis electrostatic force between two protons in this
(IV) as viewed from above, rotate A anti-clockwise nucleus is approximately
about Y -axis (a) 102 N (b) 107 N (c) 1012 N (d) 1017 N
Which of the actions will induce a current in A only 26. A hollow lens is made of thin glass and in the shape
in the direction shown? of a double concave lens. It can be filled with air,
(a) Only (I) (b) Only (II) water of refractive index 1.33 or CS2 of refractive
(c) Only (I) and (IV) (d) Only (II) and (III) index 1.6. It will act as a diverging lens, if it is
(a) filled with air and immersed in water
22. A rigid ball rolls without slipping on a surface shown
(b) filled with water and immersed in CS2
below:
(c) filled with air and immersed in CS2
(d) filled with CS2 and immersed in water
27. A stone thrown down with a speed u takes a time t1
to reach the ground, while another stone thrown
Which one of the following is the most likely upwards from the same point with the same speed
representation of the distance travelled by the ball takes time t2. The maximum height the second stone
versus time graph? reaches from the ground is
1
Distance Distance Distance Distance (a) gt1 t2 (b) g /8 (t1 + t2 )2
2
1 2
(a) (b) (c) (d) (c) g /8(t1 − t2 )2 (d) gt2
2

Time Time Time


28. An electric field due to a positively charged long
Time
straight wire at a distance r from it is proportional to
23. In an experiment, set up A consists of two parallel r −1 in magnitude. Two electrons are orbiting such a
wires which carry currents in opposite directions as long straight wire in circular orbits of radii 1 A and
shown in the figure. A second set up B is identical to 2A . The ratio of their respective time periods is
set up A, except that there is a metal plate between (a) 1 : 1 (b) 1 : 2 (c) 2 : 1 (d) 4 : 1
the wires.
29. Two particles of identical mass are moving in circular
orbits under a potential given by V (r ) = Kr − n , where
K is a constant. If the radii of their orbits are r1. r2
and their speeds are v1 ⋅ v2, respectively. Then,
(a) v12r1n = v22r2n (b) v12r1− n = v22r2− n
Set up A Set up B (c) v12r1 = v22r2 (d) v12r12 − n = v22r22 − n

Let FA and FB be the magnitude of the force between 30. Mercury is often used in clinical thermometers.
the two wires in setup A and setup B, respectively. Which one of the following properties of mercury is
(a) FA > FB ≠ 0 (b) FA < FB
not a reason for this ?
(c) FA = FB ≠ 0 (d) FA > FB = 0 (a) The coefficient of the thermal expansion is large
(b) It is shiny
24. In the circuit, wire 1 is of negligible resistance. Then, (c) It is a liquid at room temperature
R1 R2 (d) It has high density

Wire 1 CHEMISTRY
+ – + –
31. One mole
having
of carbon
of one
carbon of the sodium
content
dioxide upon close to salts
heating 14.3% listed
(atomic
below,
produces
mass of1 mole
ε1 ε2 Na = 23, H = 1, C = 12, O(b)
COONa = 16). The salt is (a) C H
NaHCO
2 5 3
(a) current will flow through wire 1, if ε1 ≠ ε2
ε ε
(b) current will flow through wire 1, if 1 ≠ 2
R1 R2
ε +ε ε −ε (c) HCOONa (d) CH3COONa
(c) current will flow through wire 1, if 1 2 ≠ 1 2
(R1 + R2 ) (R1 − R2 )
32. Among formic acid, acetic acid, propanoic acid and
(d) no current will flow through wire 1 phenol, the strongest acid in water is
25. The radius of a nucleus is given by r0 A1/ 3 , where (a) formic acid (b) acetic acid
−15 (c) propanoic acid (d) phenol
r0 = 13
. × 10 m and A is the mass number of the
https://iit-jeeacademy.blogspot.com

KVPY Question Paper 2017 Stream : SA 51

33. According to Graham’s law, the rate of diffusion of 44. The electronic configuration, which obeys Hund’s rule
CO, O2 , N2 and CO2 follows the order for the ground state of carbon atom is
(a) CO == N2 > O2 > CO2
(b) CO == N2 > CO2 > O2
2p 2p
(c) O2 > CO == N2 > CO2 (a) Energy
2s (b) Energy 2s
(d) CO2 > O2 > CO == N2
34. The major product formed when 2-butene is reacted 1s 1s
with O3 followed by treatment with Zn/H2O is
(a) CH3 COOH (b) CH3 CHO
(c) CH3 CH2OH (d) CH2 == CH2 2p 2p
(c) Energy
35. The IUPAC name for the following compound is 2s (d) Energy
2s
CH3 — CH2— CH2— CH2— C— CH2— CH2— CH3 1s
|| 1s
CH2
(a) 2-propylhex-1-ene (b) 2-butylpent-1-ene 45. The graph that depicts Einstein’s photoelectric effect
(c) 2-propyl-2-butylethene (d) Propyl-1-butylethene for a monochromatic source of frequency above the
threshold frequency is
36. The major products obtained in the reaction of oxalic
acid with conc. H2SO4 upon heating are

Photoelectric

Photoelectric
(a) CO, CO2 , H2O (b) CO, SO2 , H2O

current

current
(c) H2S, CO, H2O (d) HCOOH, H2S, CO (a) (b)

37. LiOH reacts with CO2 to form Li2CO3 (atomic mass of


Li = 7). The amount of CO2 (in g) consumed by 1 g of
LiOH is closest to Intensity of Intensity of
(a) 0.916 (b) 1.832 radiation radiation
(c) 0.544 (d) 1.088
Photoelectric

Photoelectric
38. The oxidation number of sulphur is −4 in
current

current
(a) H2S (b) CS2 (c) (d)
(c) Na 2SO4 (d) Na 2SO3
39. Al2O3 reacts with
(a) only water (b) only acids Intensity of Intensity of
(c) only alkalis (d) both acids and alkalis radiation radiation

40. The major product formed in the oxidation of


acetylene by alk. KMnO4 is BIOLOGY
(a) ethanol (b) acetic acid
(c) formic acid (d) oxalic acid 46. What is the length of human DNA containing
. × 109 bp?
66
41. In a closed vessel, an ideal gas at 1 atm is heated
(a) 22 nm (b) 0.22 mm
from 27°C to 327°C. The final pressure of the gas will
approximately be (c) 2.2 m (d) 22 m
(a) 3 atm (b) 0.5 atm 47. The Diphtheria, Pertussis, Tetanus (DPT) vaccine
(c) 2 atm (d) 12 atm consists of
(a) live attenuated strains of diphtheria, pertussis,
42. Among the elements Li, N, C and Be, one with the
Tetanus
largest atomic radius is
(b) toxoid of diphtheria, tetanus and heat-killed whole
(a) Li (b) N cells of Pertussis
(c) C (d) Be (c) whole cell lysate of diphtheria, pertussis, tetanus
43. A redox reaction among the following is (d) heat-killed strains of diphtheria, pertussis, tetanus
(i) CdCl2 + 2KOH → Cd(OH)2 + 2KCl 48. Which of the following is not an enzyme?
(ii) BaCl2 + K 2SO4 → BaSO4 + 2KCl (a) Lipase (b) Amylase
(iii) CaCO3 → CaO + CO2 (c) Trypsin (d) Bilirubin
(iv) 2Ca + O2 → 2CaO 49. The pH of the avian blood is maintained by
(a) (i) (b) (ii) (c) (iii) (d) (iv) (a) HCO3− (b) H2PO4− (c) CH3 COO− (d) Cl −
https://iit-jeeacademy.blogspot.com

52 KVPY Question Paper 2016 Stream : SA

50. Podocyte layer that provides outer lining to the 56. Which one of the following animals is a connecting
surface of glomerular capillaries are found in link between reptiles and mammals?
(a) Bowman’s capsule (b) loop of Henle (a) Platypus (b) Bat
(c) renal artery (d) ureter (c) Armadillo (d) Frog
51. If a dsDNA has 20% adenine, what would be its 57. What is the number of chromosomes in an individual
cytosine content? with Turner’s syndrome?
(a) 20% (b) 30% (a) 44 (b) 45
(c) 40% (d) 80% (c) 46 (d) 47
52. Which one of the following is incapable of curing 58. ‘Chipko Movement’ in the year 1974 in Garhwal
pellagra? Himalayas involved (a) protecting tigers
(a) Niacine (b) Nicotine (b) preventing soil erosion by planting trees (c)
(c) Nicotinamide (d) Tryptophan preventing pollution by closing down industries
53. In Escherichia coli, how many codons code for the (d) hugging trees to prevent the contractors from felling
standard amino acids? them
(a) 64 (b) 60 59. Which of the following amino acids is not involved in
(c) 61 (d) 20 gluconeogenesis?
54. Bombyx mori (silkworm) belongs to the order (a) Alanine (b) Lysine
(a) Lepidoptera (b) Diptera
(c) Hymenoptera (d) Coleoptera (c) Glutamate (d) Arginine
55. The source of mammalian hormone ‘relaxin’ is 60. Which of the following entities causes syphilis?
(a) ovary (b) stomach (a) Treponema pallidum (b) Neisseria gonorrhoeae
(c) intestine (d) pancreas (c) HIV (d) Hepatitis-B

PART-II (2 Marks Questions)


MATHEMATICS 64. Let S1 be the sum of areas of the squares whose sides
are parallel to coordinate axes. Let S2 be the sum of
61. Suppose a is a positive real number such that areas of the slanted squares as shown in the figure.
a5 − a3 + a = 2 . Then, S
Then, 1 is equal to
(a) a 6 < 2 (b) 2 < a 6 < 3 S
2
(c) 3 < a 6 < 4 (d) 4 ≤ a 6
62. Consider the quadratic equation nx2 + 7 nx + n = 0,
where n is a positive integer. Which of the following
statements are necessarily correct?
I. For any n, the roots are distinct.
II. There are infinitely many values of n for which
both roots are real.
III. The product of the roots is necessarily an
integer.
(a) III only (b) I and III
(c) II and III (d) I, II and III 1
(a) 2 (b) 2 (c) 1 (d)
63. Consider a semicircle of radius 1 unit constructed on 2
the diameter AB and let O be its centre. Let C be a 65. If a 3-digit number is randomly chosen. What is the
point on AO such thatAC : CO = 2 : 1. Draw CD probability that either the number itself or some
perpendicular to AO with D on the semi-circle. Draw permutation of the number (which is a 3-digit
OE perpendicular to AD with E on AD. Let OE and number) is divisible by 4 and 5?
CD intersect at H. Then, DH equals 1 29 11 1
1 1 1 5 −1 (a) (b) (c) (d)
(a) (b) (c) (d) 45 180 60 4
5 3 2 2
https://iit-jeeacademy.blogspot.com

KVPY Question Paper 2017 Stream : SA 53

70. A V-shaped rigid body has two identical uniform


PHYSICS arms. What must be the angle between the two arms,
66. Which one of the following four graphs best depict so that when the body is hung from one end the other
the variation with x of the moment of inertia I of a arm is horizontal?
uniform triangular lamina about an axis parallel to (a) cos−1 (1/3) (b) cos−1 (1/2)
its base at a distance x from it? (c) cos−1 (1/4) (d) cos−1 (1/6)

CHEMISTRY
h
71. In the following reaction, X, Y and Z are
x
CH3 CH3
NO2
l l Z
+ X Y

(a) (b)
(a) X = CH3Cl; Y = Anhydrous AlCl 3; Z = HNO3 + H2SO4
x x (b) X = CH3COCl; Y = Anhydrous AlCl 3; Z = HNO3 + H2SO4
h h
(c) X = CH3Cl; Y = Conc. H2SO4; Z = HNO3 + H2SO4
l l (d) X = CH3Cl; Y = Dil. H2SO4; Z = HNO3

(c) (d) 72. 2,3-dibromobutane can be converted to 2-butyne in a


two steps reaction using
(a) (i) HCl and (ii) NaH
x h
x (b) (i) alc.KOH and (ii) NaNH2
h
(c) (i) Na and (ii) NaOH
67. A rectangular block is composed of three different (d) (i) Br2 and (ii) NaH
glass prisms (with refractive indices µ1 , µ 2 and µ3 ) 73. Given, NO( g) + O3 ( g) → NO2 ( g) + O2 ( g);
as shown in the figure below. A ray of light incident
normal to the left face emerges normal to the right ∆H = − 1989
. kJ/mol
face. Then, the refractive indices are related by O3 ( g) → 3 / 2 O2 ( g); ∆H = − 1423
. kJ/mol
O2 ( g) → 2O ( g); ∆H = + 4950
. kJ/mol
µ1 µ3
µ2 The enthalpy change (∆H ) for the following reaction is
45°
45° NO( g) + O( g) → NO2 ( g)
(a) −3041
. kJ/mol
(a) µ 12 + µ 22 = 2µ 32 (b) µ 12 + µ 22 =µ 32 (b) +304.1 kJ/mol
(c) µ 12 + µ 32 = 2µ 22 (d) µ 22 + µ 32 = 2 µ 12 (c) −4031
. kJ/mol
68. A uniform metal plate shaped like a triangle ABC (d) +403.1 kJ/mol
has a mass of 540 g. The length of the sides AB, BC
74. A 1.85 g sample of an arsenic containing pesticide
and CA are 3 cm, 5 cm and 4 cm, respectively. The
plate is pivoted freely about the point A. What mass was chemically converted to As O34− (atomic mass of
4 2.
must be added to a vertex, so that the plate can hang As20
If = 74.9)
mL ofand
0.1 titrated
M Pb2+ iswith Pb2+ totoform
required reach
Pbthe
3 (AsO )
with the long edge horizontal? equivalence point, the mass percentage of arsenic in
(a) 140 g at C (b) 540 g at C the pesticide sample is closest to
(c) 140 g at B (d) 540 g at B
(a) 8.1 (b) 2.3
69. A 20 g bullet whose specific heat is 5000 J kg°C and (c) 5.4 (d) 3.6
moving at 2000 m/s plunges into a 1.0 kg block of 75. When treated with conc. HCl. MnO2 yields a gas (X)
wax whose specific heat is 3000 J kg°C. Both bullet which further reacts with Ca(OH)2 to generate a
and wax are at 25°C and assume that (i) the bullet
white solid (Y). The solid Y reacts with dil. HCl to
comes to rest in the wax and (ii) all its kinetic energy
produce the same gas X. The solid Y is
goes into heating the wax. Thermal temperature of
the wax (in °C) is close to (a) CaO (b) CaCl 2
(a) 28.1 (b) 31.5 (c) 37.9 (d) 42.1 (c) Ca(OCl)Cl (d) CaCO3
https://iit-jeeacademy.blogspot.com

54 KVPY Question Paper 2016 Stream : SA

BIOLOGY

Reaction rate

Reaction rate
76. The atmospheric pressure is 760 mm Hg at the sea P. Q.
level. Which of the following ranges is nearest to the
partial pressure of CO2 in mm Hg?
(a) 0.30-0.31 (b) 0.60-0.61
(c) 3.0-3.1 (d) 6.0-6.1 Temperature Temperature

77. A breeder crossed a pure bred tall plant having white


flowers to a pure bred short plant having blue

Reaction rate

Reaction rate
flowers. He obtained 202 F1 progeny and found that
R. S.
they are all tall having white flowers. Upon selfing
these F1 plants, he obtained a progeny of 2160 plants.
Approximately, how many of these are likely to be
short and having blue flowers? Temperature Temperature
(a) 1215 (b) 405 (a) P and P (b) P and S
(c) 540 (d) 135 (c) P and R (d) R and R
78. Match the different types of heart given in Column I
80. Match the enzymes in Column I with the reactions in
with organisms given in Column II. Choose the
Column II. Select the correct combination.
correct combination.
Column I Column I Column II
i. Column
Human II P. Hydrolase i. Inter-conversion of optical isomers
P. Neurogenic
Q. heart
Bronchial heart ii. King crab Q. Lyase ii. Oxidation and reduction of two
R. Pulmonary heart iii. Shark substrates
R. Isomerase iii. Joining of two compounds
(a) P-ii, Q-iii, R-i
(b) P-iii, Q-ii, R-i S. Ligase iv. Removal of a chemical group from a
(c) P-i, Q-iii, R-ii substance
(d) P-ii, Q-i, R-iii v. Transfer of a chemical group from one
79. Given below are the four schematics that describe the substrate to another
dependence of the rate of an enzymatic reaction on
temperature. Which of the following combinations is (a) P-iv, Q-ii, R-iii, S-i (b) P-v, Q-iv, R-i, S-iii
true for thermophilic and psychrophilic organisms? (c) P-iv, Q-i, R-iii, S-v (d) P-i, Q-iv, R-v, S-ii

PART-I
Answers
1 (c) 2 (c) 3 (b) 4 (d) 5 (a) 6 (b) 7 (b) 8 (c) 9 (c) 10 (b)
11 (d) 12 (c) 13 (a) 14 (c) 15 (d) 16 (d) 17 (c) 18 (c) 19 (c) 20 (b)
21 (a) 22 (d) 23 (c) 24 (d) 25 (a) 26 (d) 27 (b) 28 (b) 29 (a) 30 (d)
31 (b) 32 (a) 33 (a) 34 (b) 35 (a) 36 (a) 37 (a) 38 (*) 39 (d) 40 (d)
41 (c) 42 (a) 43 (d) 44 (a) 45 (c) 46 (c) 47 (b) 48 (d) 49 (a) 50 (a)
51 (b) 52 (b) 53 (c) 54 (a) 55 (a) 56 (a) 57 (b) 58 (d) 59 (b) 60 (a)

PART-II
61 (c) 62 (b) 63 (c) 64 (a) 65 (b) 66 (a) 67 (c) 68 (c) 69 (c) 70 (a)
71 (a) 72 (b) 73 (a) 74 (c) 75 (c) 76 (a) 77 (d) 78 (a) 79 (d) 80 (b)
* No option is correct.
https://iit-jeeacademy.blogspot.com

KVPY Question Paper 2017 Stream : SA 55

Solutions
1. (c) We have, p(x) = ax2 + bx + c, where (iii) 10t + 3 × ×
= 3 4 …(i)
a , b, c are in AP and a , b, c are positive a 2 + b = 10, ab = 3 1
real. a = 3, b = 1 it is possible Also, area of
α , β are root of p (x) = 0, where α and β are 1 2
(iv) 6t + 5 ∆BCE = × BE × h
integers. 2
a 2 + b = 6, ab = 5
× ×
p (x) = ax2 + bx + c = 0 a = 1, b = 5 it is also possible = 5 h …(ii)
−b c 1
α+β= , αβ = Hence, option (b) is correct.
a a From Eqs. (i) and (ii),
4. (d) Given, 1 1
α , β are integer. × 3 ×24 = × 5 × h
−b (1 + a + b)2 = 3 (1 + a 2 + b2 ) 2 2
∴ α+β= = − λ, λ ∈ I 1 + a + b2 + 2a + 2b + 2ab
2
a ⇒ h = 2.4
⇒ b = aλ = 3 + 3a 2 + 3b2 7. (b) Given,
a , b, c are in AP. ⇒ 2a 2 + 2b2 − 2a − 2b − 2ab + 2 = 0 ∠ABC = 130°
a+ c a+ c ⇒ (a 2 − 2a + 1) + (b2 − 2b + 1) ∠CDE = 110°
∴ b= ⇒ = aλ
2 2 ABCE is a cyclic quadrilateral.
+ (a 2 + b2 − 2ab) = 0
⇒ c = a (2λ − 1)
∴ ax2 + aλx + a (2λ − 1) = 0 ⇒ (a − 1)2 + (b − 1)2 + (a − b)2 = 0 A

⇒ x2 + λx + (2λ − 1) = 0 [Q a ≠ 0] ∴ a − 1 = 0, b − 1 = 0, a − b = 0
70°
2
D = λ − 4 (2λ − 1) is a perfect square for ⇒ a = 1, b = 1, a = b
integral roots. ∴ a= b=1 E 50° B
130°
Q λ2 − 8λ + 4 = k 2 Exactly one pair.
⇒ (λ − 4)2 − 12 = k 2 5. (a) We have, a1 , a2 , a3 , ..., a100 be
⇒ (λ − 4 − k ) (λ − 4 + k ) = 2 × 6 non-zero real number and 60°
⇒ λ − 4 − k = 2 and λ − 4 + k = 6 a1 + a2 + a3 + ... + a100 = 0 110°

Q λ = 8 and k = 2 ai ⋅ 2ai > ai and ai ⋅ 2− ai < ai D C


−b c 100 100 100 100
∴ ∠ABC + ∠AEC = 180°
∑ a1 ⋅ 2 ∑ ai and ∑ a1 ⋅ 2 ∑ ai
∴ α + β + αβ = + − ai
∴ ai
> <
a a i =1 i =1 i =1 i =1 ⇒ ∠AEC = 50°
− aλ + a (2λ − 1)
= 100 100
ACDE is also cyclic quadrilateral.
a ⇒ ∑ a1 ⋅ 2ai > 0 and ∑ a1 ⋅ 2− ai < 0
a (λ − 1) i =1 i =1 ∴ ∠CDE + ∠EAC = 180°
=
a Hence, option (a) is correct. ⇒ ∠EAC = 70°
= λ − 1= 8 − 1= 7 6. (b) ABCD is a trapezium. In ∆AEC,
2. (c) We have, AB is parallel to CD. ∠EAC + ∠AEC + ∠ACE = 180°
165 ⋅ 516 = 16 ⋅ 164 ⋅ 516 AB = 11, BC = 4, CD = 6 and DA = 3 ⇒ 70° + 50° + ∠ACE = 180°
= 16 × 216 ⋅ 516 Construct CE is parallel to DA. ⇒ ∠ACE = 60°
= 16 × (10)16 D 6 C 8. (c) Given, radii of circle are 1, 2 and 3.
∴ Total number of digits in 165 ⋅ 516 = 18
3. (b) Given, C
90°
t 2 = at + b, where a , b are positive 3 3 4 1
h 1
integers. t3 = at 2 + bt 2
⇒ t3 = a (at + b) + bt 3
2 B
⇒ t3 = a 2t + bt + ab A 6 E 5
B
3
11 A
⇒ t3 = (a 2 + b)t + ab
(i) 4t + 3 ∴ CE = 3
a 2 + b = 4, ab = 3 BC = 4 ∴Side of ∆ ABC are
a = 1, b = 3 it is possible BE = 5 AB = 5
(ii) 8t + 5 ∴ ∠BCE is a right angled triangle. BC = 3
1
a 2 + b = 8, ab = 5 ∴Area of ∆BCE = EC × BC AC = 4
2
It is not possible
https://iit-jeeacademy.blogspot.com

56 KVPY Question Paper 2016 Stream : SA

∴∆ABC is formed a right angled triangle 11. (d) We have, length and size of two A1 ∩ A2 ∩ A3 ... ∩ Am = φ
where AB is hypotenuse of triangle. candles are same. Let L be the length of ∴ A1 ∩ A2 ∪ A3 ... ∪ Am = {1, 2, 3, ..., 100}
We know circumradius of a right angled candles. Let|A1| = 1|A2| = 2 ....|Am| = M
triangle is the half of the hypotenuse. Given, first candle burns in 5 h and A1 , A2 , A3 …, Am are disjoint set.
1 second candle burns in 3 h.
∴Circumradius = × AB ∴|A1| + |A2|... + |Am|= 100
2 L
In one hours length of candles are and
× = 1 + 2 + 3 ... + m = 100
= 5 2.5 5
L m (m + ) =
1 , respectively. 100
3 1
9. (c) ABC is a right angled triangle, m2 + m − 200 = 0
∠ABC = 90° 2 Let after time t h the length of candles
are L1 and L2. b2 − 4ac2 −1 ± 1 + 4 ⋅ 1⋅ 200
−b ±
Circumcentre of ∆ ABC is mid-point of AC = =
i.e. M. ∴
L
L1 = L − t and L2 = L − t
L 2a 2⋅ 1
5 3 −1 ± 1 + 800 −1 + 801
= =
A According to the problem, 2 2
L1 = 3L2 −1 + 28.30 27.30
= = = 16.65
2
L − t = 3  L − t 
L L
∴ 1 + 28
5  3  m= 2 .30 = 29.30 = 14.65
M
1 1 2 2
P1 ⇒ 1 − t = 3 − t ⇒ t  1 −  = 3 − 1 ∴m < 14
P 5  5
4t 5 ∴Maximum possible of m is 13.
⇒ = 2 ⇒t = h
5 2 (14th set will have same size as that of
B C 5 previous size)
⇒ t = × 60 = 150 min
2 14. (c) Let two-digits number be
P2 12. (c) Given, a cuboid has all edges are n = 10a + b
integers and base is square. Given, n = a 2 + b3
Circumcentre of ∆ P1 PP2 is mid-point of
P1 P2. Let the length, breadth and height of ∴ 10a + b = a 2 + b3
cuboid is x, x, y. ⇒ a 2 − 10a + b3 − b = 0
AB is perpendicular bisector of PP1 and
BC is perpendicular bisector of PP2. ⇒ a (a − 10) + b (b + 1) (b − 1) = 0
Perpendicular bisector of PP1 and PP2 ⇒ b (b + 1) (b − 1) = a (10 − a )
intersect at B. b ≠ 1if b = 1then a = 10 not possible
∴ B is circumcentre of ∆P1 PP2. if b = 2, a (10 − a ) = 6, no value of a
∴Distance between b = 3, a (10 − a ) = 24, a = 4, 6
BM = AM = MC =
AC
.
y Numbers are 43 and 63.
x
2 If b = 4, a (10 − a ) = 60 no value of a
x
10. (b) Given, a + 2b ≤ 1 a, b are positive If b = 5, a (10 − a ) = 120 not possible
Sum of all edges of cuboid = 4x + 4x + 4 y
real number. ∴Numbers are 43 and 63.
Sum of area of all faces = 2x2 + 2xy + 2xy
Radius of circle C1 = ab3 15. (d) Given, f (xy) = f (x) + f ( y)
Given,
Radius of circle C2 = b2 f (12) = 24 ⇒ f (8) = 15
Sum of all edges of cuboid = Sum of area
∴Area of circle C1 = A1 = πa 2b6 f (8) = f (2 ⋅ 2 ⋅ 2) = f (2) + f (2) + f (2)
of all faces
and area of circle C2 = A2 = πb4 ⇒ 15 = 3f (2) ⇒ f (2) = 5
∴ 4x + 4x + 4 y = 2 (x2 + xy + xy)
A1 πa 2b6
Now, = = a 2b2 ⇒ 4 (2x + y) = 2 (x2 + 2xy) ∴ f (48) = f (12 ⋅ 2 ⋅ 2) = f (12) + f (2) + f (2)
A2 πb4 2 = 24 + 5 + 5 = 34
⇒ x + 2xy − 4x − 2 y = 0
a + 2b ≤ 1 16. (d) Displacement of person is
⇒ x2 + 2x( y − 2) − 2 y = 0
a + 2b
⇒ ≥ 2ab
2 − 2 ( y − 2) ± 4( y − 2)2 + 4(2 y) N
⇒ x=
[Q AM ≥ GM] 2
B
⇒ 1 ≥ (a + 2b)2 ≥ 8ab ⇒ x = y − 2± 2
y − 2y + 4
km

⇒ 8ab ≤ 1
18
3.

1 x is integer, when y = 2 25°


⇒ a 2b2 ≤ E
64 ∴ y = 2, x = 2 W O A
A1 1 64 Hence, sum of edges = 8x + 4 y = 16 + 8 = 24

A2 ≤ 13. (a) We have, A1 , A2 , A3 , Am are
A
∴Maximium value of 1 = 1 non-empty subsets of {1, 2, 3, …, 100}
A 64
2 |A1|,|A2|,...,|Am|are distincts. S
https://iit-jeeacademy.blogspot.com

KVPY Question Paper 2017 Stream : SA 57

From above figure, distance travelled 21. (a) Current in loop is shown 25. (a) Taking protons at dimetrically
along north direction is anti-clockwise. opposite points of nucleus,
AB = OB sin 25° = 318
. × sin 25° = 1.34 km A
Distance travelled along east direction is
IA IB
OA = 318
. × cos 25° = 2.88 km B r
17. (c) Area, A = l × b P1 P2

= 3.95 × 3.05 = 12.05 m 2


Now, A=l×b
dA dl db Hence to induce a anti-clockwise current
⇒ = +
A l b in A, flux going into A must be increased
 ∆l ∆b  and by bringing A closer to B, we get a
Lead
⇒ ∆A =  +  ×A nucleus
 l b anti-clockwise current in A. This is in
0.05 0.05  accordance with Lenz’s law. Force of electrostatic repulsion,
=  +  × 12.05
 3.95 3.05  k (e) (e) ke2 k ⋅ e2
22. (d) As ball moves from point A to F= 2
= 1/3 2
= 2 2 /3
≈ 0.34 point D, r (r0 A ) r0 ⋅ A
So, area of floor is A = 12.05 ± 0.34 m 2. Substituting values in above equation, we
18. (c) When car goes around track of get
A B
radius R, then . × 10−19 )2
9 × 109 × (16
F=
Time period, T =
2πR C D . × 10−15 )2 (206)2/3
(13
v Velocity of ball increases rapidly in = 0.039 × 102 N
v2 region BC. Then, from C to D, it moves
Centripetal acceleration, ac = 26. (d) When medium outside a lens is
R with a constant velocity. So, graph is denser than medium of lens, then a
When car goes around circular track of parabolic in region BC and slope of AB is concave lens will acts like a convex lens
radius 2R, then less than slope of CD, as shown below. and vice-versa.
Centripetal acceleration, x Now, when hollow lens is filled with
v′2 8v2 D CS2 (η = 16. ) and immersed in water
ac′ = = ⇒ v′ = 4v
2R R Straight line (η = 133
. ) , its nature remains diverging as
C refractive index of medium of lens is
So, time period of car is
more than refractive index of
2 πR ′ 2 π (2R ) B Parabolic curve
′= = surrounding medium.
v′ 4v A
T 1 2 πR T Straight line 27. (b) For first stone,
= × =
2 v 2 t=0
Note Velocity change is not very abrupt. u
19. (c) As primary voltage is constant,
there is no change of magnetic flux of 23. (c) Metal plate between wires may
secondary coil. modify field pattern within the metal h
volume but number of field lines is not
So, there is no induction and hence
changed.
voltage across secondary is zero.
So, force between wires is same in both
20. (b) Power output, P = 50% of 1
cases and is non-zero. − h = − ut1 + (− g ) t12
potential energy of water 2
i.e. FA = FB ≠ 0. 2
Received by generator per second or h = ut1 + gt1 ...(i)
24. (d) 1
50  mgh 
= × 
100  t  R1 R2 For second stone,
2
= 0.5 ×   ρ × g × h
V
...(i)
 t
i2
Here, P = 1 × 109 W, u
t=0
g = 10 ms −2, ρ = 1000 kgm −3 i1 i1 i2
E2
E1
and h = 500 m. h
Current leaving the cell must be equal to
Substituting these values in Eq. (i),
current going into the cell. t2
we get
V So, current going from first loop to second 1 2
Volume flow rate of water = loop must be zero for any value of E or R. − h = ut2 − gt2 ...(ii)
t 2
9 Hence, there is no current through the
1 × 10 Adding Eqs. (i) and (ii), we get
= = 400 m3 s −1 wire connecting loops.
0.5 × 1000 × 10 × 500 1
0 = u (t1 + t2 ) − g (t12 − t22 )
2
https://iit-jeeacademy.blogspot.com

58 KVPY Question Paper 2016 Stream : SA

g 30. (d) Fluid used in a thermometer


⇒ u= (t1 − t2 ) O

O O
2 must be easily visible, expands uniformly
Maximum height attained by second and significantly and it must be a liquid –
stone is at room temperature.
u2
H =h+ So, density of mercury is not a feature
2g –
for selecting mercury in clinical Phenoxide ion
1 u2
⇒ H = ut1 + gt12 + thermometers.
2 2g –
31. (b) NaHCO3 which is a sodium salt O O
Substituting for u and rearranging, only produces carbon dioxide on heating

we get while the other salts produce ions on
g heating. Also, its carbon content which is
H = (t1 + t2 )2
8 close to 14.3%.
28. (b) Electron revolve around wire due ∆
2NaHCO3 → Na 2CO3 + H2O + CO2 Thus, the order of acidic strength of given
to its electrostatic force of attraction.
compounds is
As field E ∝ r −1 ⇒ E = kr −1 Molecular mass of NaHCO3
= 23 + 1 + 12 + 16 × 3 = 84 OH
+ 12
% of C = × 100 = 14.28%
84 HCOH>CH3COOH >CH3CH2COOH>
32. (a) Key Idea Acetic Propanoic
+ O acid acid Phenol
r • The substituents attached to benzoic Formic
+ acid having + I effects tends to acid

decrease its acidity. Hence, formic acid is a strongest acid.


+ • Carboxylic acids are more acidic than 33. (a) According to Graham’s law, rate
phenol as they are more resonance of diffusion is inversely proportional to
stabilised.
So, force on electron is the square root of molar mass, i.e.
Among the three carboxylic acids given in 1
F = eE = ker −1 the options, the + I effect of CH3 group r∝
M
This force is necessary centripetal force. intensifies the negative charge on the
carboxylate ion thereby making acetate Thus, rate of diffusion decrease with
So,
ion less stable than formate ion. As a increase in molecular weight. Therefore,
mv2 ke the order of rate of diffusion will be
= result, the release of H+ ion from acetic
r r
acid will become more difficult as CO == N2 > O2 > CO2
ke
⇒ v= compared to formic acid. Hence, formic (28 g) (28 g) (32 g) (44 g)
m acid is a stronger acid than acetic acid. 34. (b) The major product, formed when
As velocity of electron is independent of 2-butene is reacted with O3 followed by
O O
radius of paths, CH3 C

H C

treatment with Zn / H2O is acetaldehyde.
⇒ v1 = v2 O O This reaction is known as reductive
Acetate ion Formate ion
Now, ratio of time periods of rotation are ozonolysis.
 2 πr1  Further since + I effect of alkyl groups
  CH3CH CH CH3 + O3
T increases in the order CH3 —< CH3 CH2— 2 butene
 v1  r1 1Å 1
1
2π = = = the relative acid strength will decrease in
T  r  r 2 Å 2
2 =
 2 2 the same order, i.e., O
 v2  HC OH > CH3 COOH > CH3 CH2COOH
CH3CH CH CH3

29. (a) Given, || O O


O
Potential, V = Kr − n
Ozonide
Now between carboxylic acids and
So, magnitude of field, phenol, carboxylic acids are stronger Zn/H2O
dV d acids than phenol because carboxylate
E=− =− (Kr − n )
dr dr ion is more resonance stabilised. This is 2CH3 C H
⇒ E = n+1
nK because the negative charge on the
r carboxylate ion is delocalised over two O
Acetaldehyde
mnK
Hence, force, F = mE = n + 1 electronegative oxygen atoms.
r – 35. (a) The IUPAC name of the following
O O
As particles are rotating in circular path, is
C C 6 5 4 3 2
mv2 mnK O– O CH3 CH2 CH2 CH2 C CH2 CH2CH3
F= = n+1
r r Carboxylate ion (More resonance ||
1CH2
⇒ v2r n = nK = constant stabilised)
2 propylhex-1-ene
So, v12r1n = v22r2n
https://iit-jeeacademy.blogspot.com

KVPY Question Paper 2017 Stream : SA 59

36. (a) The major products obtained in 41. (c) According to ideal gas equation, 45. (c) According to photoelectric effect,
the reaction of oxalic acid with pV = nRT the number of electrons ejected is
conc. H2SO4 upon heating are carbon nRT proportional to the intensity of radiation.
monoxide, carbon dioxide and water. p=
V Thus on increasing the intensity of
COOH radiation, the value of photoelectric
| →∆
CO + CO2 + H2O At constant volume and number of moles
H SO
current also increases, i.e. photoelectric
COOH 2 4 p ∝T current ∝ intensity of radiation. Hence,
Oxalic acid p1 T1 its graph would be linear.
∴ =
37. (a) 2LiOH + CO2 → Li 2CO3 + H2O p2 T2

Photoelectric
1 T1 = 27°C = 27 + 273 = 300K
Number of moles of LiOH = moles

current
24 T2 = 327°C
2 moles of LiOH reacts with 1 mole of = 327 + 273 = 600K
CO2 to form 1 mole of Li 2CO3 and H2O. 1 300
∴ =
1 p2 600
∴ Number of moles of CO2 = Intensity of
24 × 2
⇒ p2 = 2 atm radiation
1
= moles 46. (c) The distance between
48 42. (a) As the given elements Li, N, C
and Be belong to same period i.e. 2nd 2 nucleotides/nitrogen bases is
1 mole of CO2 = 44 g
period, so on moving from left to right in 0.34 × 10−9 m or 3.4 Å.
1 1
moles of CO2 = × 44 = 0.916 g a period the atomic radius decreases Therefore the length of human DNA
48 48
because the effective nuclear charge containing 6.6 × 109 bp would be
38. (*) The oxidation number of S in increases. Thus, Li has the largest atomic = 0.34 × 10−9 m × 6.6 × 109 bp
given compounds are as follows : radius among them all. = 2.244 m or 2.2 m
(i) H2S 43. (d) A redox reaction is one in which 47. (b) DPT is a class of combination
Let the oxidation state of S be x the oxidation and reduction reactions vaccines against three infectious diseases
∴ 2+ x= 0 occur simultaneously. in humans, i.e. diphtheria, pertussis
x= −2 (i) CdCl 2 + 2KOH → Cd(OH)2 + 2KCl (whooping cough) and tetanus. The
(ii) CS2 This is an example of double vaccine components include diphtheria
displacement reaction. and tetanus toxoids and killed whole cells
4 + 2x = 0 ⇒ x = − 2
of the bacterium that causes pertussis.
(iii) Na 2SO4 (ii) BaCl 2 + K2SO4 → BaSO4 + 2KCl
This is a double displacement reaction.
48. (d) Bilirubin is an orange yellow
2(+1) + x + 4(−2) = 0 pigment formed in the liver by the
Reduction
2+ x− 8= 0 breakdown of haemoglobin and excreted
x=+6 (iii) CaCO3 → CaO + CO2 in bile. It is not an enzyme. Other options
(iv) Na 2SO3 Reduction like lipase, amylase and trypsin are lipid
2(+1) + x + 3(−2) = 0 0 0 +2 –2 digesting, starch digesting and
(iv) 2Ca + O2 → 2CaO endopeptidase enzymes.
2+ x− 6= 0
Oxidation 49. (a) The pH of the avian blood is
x=+4
maintained by bicarbonate (HCO3− ) ions.
No, option is correct. As both oxidation and reduction reaction
A variety of buffering systems exist in the
occurs simultaneously, so it is a redox
39. (d) Al 2O3 is an amphoteric oxide body of birds that helps to maintain the
reaction.
(those oxides which show both the pH between 7.35 and 7.45. Since the
44. (a) According to Hund’s rule, ‘pairing bicarbonate (HCO3− ) ion is a base, it helps
properties of acids and bases), so it can
of electrons in the orbitals belonging to neutralise the acid in the blood and
react both with acids and alkalis, e.g.
same subshell does not take place until increases the pH above 7.
Al 2O3 + HCl → AlCl3 + H2O each orbital belonging to that subshell
Al 2O3 + NaOH + H2O → Na[Al(OH)4 ] 50. (a) Podocytes are cells of squamous
has got one electron each, i.e. it is singly
epithelium of Bowman capsule of
40. (d) The major product formed in the occupied.
nephron. The Bowman’s capsule filters
oxidation of acetylene by alk. KMnO4 is The electronic configuration, which obeys the blood, retaining large molecules such
oxalic acid. Hund’s rule for ground state of carbon is as proteins while smaller molecules such
CH CH O 1s2 2s2 2 p 2. as water, salts and sugars are filtered as
Alk. KMnO4 2[O]
+2[O] the first step in the formation of urine.
Alk. KMnO4
CH CH O 51. (b) If dsDNA has 20% adenine, then
Acetylene 2p
Glyoxal COOH according to the Chargaff’s rule, it would
2s have 20% thymine. The remaining 60%
COOH represents both G + C. Since guanine and
Oxalic acid
1s cytosine are always present in equal
(Major product) numbers, the percentage of cytosine
molecule is 30%.
https://iit-jeeacademy.blogspot.com

60 KVPY Question Paper 2016 Stream : SA

52. (b) Pellagra cannot be cured by 60. (a) Causative agent of syphilis is In ∆OCD,
nicotine. Instead, pellagra can be cured Treponema pallidum. Syphilis is a 1 2 2
CD = OD 2 − OC 2 = 1 − =
by giving niacine, nicotinamide and bacterial infection usually spreads by 9 3
tryptophan. Pellagra is a disease caused sexual contact that starts as a painless
In ∆ACD,
by a lack of the vitamin niacin (vitamin- sore.
8 4
B3 ) which includes both nicotinic acid and Neisseria gonorrhoeae causes gonorrhoea. AD = CD 2 + AC 2 = +
nicotinamide and its precursors, i.e. the 9 9
HIV causes AIDS. Hepatitis-B virus 2 3 2
amino acid tryptophan. The main causes hepatitis, i.e. a liver infection. = =
symptoms of pellagra are dermatitis, 3 3
dementia and diarrhoea. 61. (c) Given, a5 − a3 + a = 2 In ∆DEH ~ ∆OEA
53. (c) In all living organisms, there are ⇒ a5 − a3 + a − 2 = 0 DE DH 1/ 3 DH
Let f (a ) = a5 − a3 + a − 2 = ⇒ =
64 codons and out of which 3 codons are OE OA 2/ 3 1
stop or termination codons, i.e. UAA, f ′ (a ) = 5a 4 − 3a 2 + 1
Q DE = 1 AD 
UAG and UGA which do not code for any f ′(a ) > 0, ∀ a ∈ R  
1 2
amino acids. Therefore, 61 codons code ∴ a5 − a3 + a − 2 = 0 has only one roots. ⇒ DH =  
2
for the standard 20 amino acids. for a 6 = 3 ⇒ a = (3)1/ 6 = 12 . [by calculation] 2  2 1  
OE = OA −  AD  
54. (a) The order of silkworm (Bombyx f (4 / ) 0 = 4, a = (4)1/ 6  2  
mori) is Lepidoptera. It is the order of So 1one
6 root lies in (3, 4).
> and at 64. (a) Here,
insects that includes butterflies and ∴ 3a<6 a 6 < 4
moths. About 1,80,000 species of the
Lepidoptera are described till now.
62. (b) Given, nx2 + 7 n x + n = 0
55. (a) Relaxin hormone is produced by D = 49n − 4n 2 = n (49 − 4n )
the ovary and the placenta with D ≠ 0 ; ∴∀ n ∈ I +
important effects in the female ∴ Roots are distinct.

a
2
2√
reproductive system and during For roots are real D ≥ 0
pregnancy. In preparation for childbirth, a/2
49 a/2 a
it relaxes the ligaments in the pelvis and ∴ n (49 − 4n ) ≥ 0 ⇒ n ≤
4 √2
softens and widens the cervix.
So, n ∈ {1, 2, 3, 4, ..., 12} a
56. (a) Platypus is a connecting link 2 2
So, x have finite value. S1 = a 2 +   +   + ...
a a
between reptiles and mammals. They
n  2  4
have few mammalian characters such as Product of roots is = 1
n 2 2
hair, mammary glands, diaphragm
S2 = 
a   a  + ...
whereas it lays eggs with yolk and egg ∴Products of root is necessarily integer.  +  
 2  2 2
shell similar to reptiles. Hence, option (b) is correct.
a2 a2 a2 4a 2
57. (b) The people with Turner’s 63. (c) Given, S1 = a 2 + + + ... = =
4 16 1 3
syndrome have 44 + XO chromosomes, 1−
D 4
so there are a total of 45 chromosomes
only in each cell. Such persons are sterile a2 a2 a2 a 2 / 2 4a 2
E S2 = + + + ... = =
females who have rudimentary ovaries, 2 8 32 1 6
1 1−
undeveloped breasts, small uterus, short 4
H
stature and abnormal intelligence. 4a 2
58. (d) ‘Chipko Movement’ was headed S1 3 =2
A 2 C 1 O B
∴ =
by social activist Sunder Lal Bahuguna S2 4a 2 / 6
3 3
in Uttarakhand to save trees from felling. 65. (b)The 3-digit number which is
The movement got its name due to AB is diameter of circle.
O is centre of circle. divisible by 4 and 5 both.
people’s action of hugging trees in order
= i.e. last digits are 00, 20, 40, 60, 80
to prevent them from cutting down by ∴ OA = OB = AB 1
state forest contractors. 1 Now ending with 00 are (100, 200, 300 …
C is a point on AO such that 900) = 9
59. (b) Lysine is not involved in AC 2
gluconeogenesis. Gluconeogenesis is the = ⇒ AC 2 = 2OC If digit repeat other than 0′ then they are
biosynthesis of new glucose from certain OC 1 (220, 440, 660, 880) but 220 number can
non-carbohydrate carbon substrates like CD is perpendicular to AO. be permuted according to condition as
amino acids, etc. Out of the 20 amino ∴ OD = OA radius of circle (220, 202).
acids, 18 are glucogenic (i.e. can be OE is perpendicular on AD. Similarly, for 440 as (440, 404), 660 and
converted to glucose upon ∴ AOD is isosceles triangle. 880, so there are 8 favourable cases.
gluconeogenesis) while the remaining two ∴ E is mid-point of AD. If the number have no digit repeated like
amino acids, i.e. lysine and leucine are 1 2 120, 120 can be permuted in 4 ways.
∴ OA = 1, OC = , AC = , OD = 1
purely ketogenic (i.e. can be degraded 3 3 So, such number are 8 × 4 × 4 = 128
into acetyl-Co-A).
https://iit-jeeacademy.blogspot.com

KVPY Question Paper 2017 Stream : SA 61

Total favourable cases = 9 + 8 + 128 = 145 Weight of lamina acts through its Substituting values in above equation,
145 29 centroid G to prevent tilting of lamina, let we get
Required probability = =
900 180 a mass m1 is added at vertex B. From A, 20 × 10−3 × (2000)2
perpendicular AE is dropped on BC. AD ∆T =
66. (a) By parallel axes theorem, 2(20 × 10−3 × 5000 + 1 × 3000)
moment of inertia of triangular lamina is medium and G is centroid of ∆ABC.
400
about a parallel axes, which passes below Now, consider ∆ABC and ∆EBA. ⇒ ∆T = = 12.9 ⇒ Tf − Ti = 12.9
31
its centre of mass is A or Tf = 25 + 12.9 = 37.9° C
2h 70. (a) Let length of each of rod is l and
3 CM 4 angle between them is θ.
h 3
–x
3 x H
G y
2
I = ICM + M  − x
h D
3  C 5/2 E x B
When axis of rotation of lamina, passes ∆ABC ~ ∆EBA
above its centre of mass and its moment
AB BC AB 2 9
of inertia is = ⇒ EB = x = ⇒ EB = x = θ
2 EB AB BC 5
I = ICM + M  x − 
h
So, DE = BD − EB
 3 l
5 9 25 − 18 7
Clearly, I versus x is a parabolic graph. = − = = cm
2 5 10 10
Also, I first reduces axis of rotation comes
Now, consider ∆ADE, G is centroid of
closer to centre of mass and then it again
∆ABC.
increases.
AG 2 2 P
So, correct variation of I with x is So, = or AG = AD
GD 1 3 Rod 1
I Also, GH is parallel to DE. A
AG GH
So, =
AD DE
AG DE θ B Rod 2
⇒ GH = 2
ICM AD × DE C D
×
= 3
h AD mg
x
x=h/3 AD mg
67. (c) Ray diagram of ray through the 2 7 14 7 Let the lower rod is horizontal and upper
= × = = cm
composition of prisms will be 3 10 30 15 rod makes θ angle with horizontal.
For BC to remain horizontal, torque of Weights of rods acts vertically
A
µ1 µ3 m1 g about A must be balanced by torque downwards from their centres A and B as
α
°

shown in above figure.


45

µ2 of mg about A.
1i =

⇒ mg × GH = m1 g × BE Now, perpendicular distance of weight


45° θ acting through point A from point D is
α–θ 7 9
⇒ 540 × = m1 × l
45° 45° 15 5 CD = l cos θ − cos θ
B C 540 × 7 × 5 2
⇒ m1 = = 140 g l
By Snell’s law on surface AB and AC, we 15 × 9 CD = cos θ
have µ 1 sin 45° = µ 2 sin θ ...(i) 2
So, mass of 140 g must be added to vertex
and µ 3 sin 45° = µ 2 cos θ ...(ii) and perpendicular distance of weight
B, so that BC remains horizontal.
As α − θ = 45°, from figure acting through B from point D is
69. (c) As kinetic energy of bullet is used l l
Squaring and adding Eqs (i) and (ii), up in heating and melting the wax. BD = − l cos θ = (1 − 2 cos θ)
2 2
µ 12 µ 32
we get + = µ 22 By energy conservation, we have At equilibrium torque of these two
2 2 1
mb vb2 = mw cw (∆Tw ) + mb cb (∆Tb ) weights about D must balance each other,
68. (c) Given situation is 2 l l
i.e. mg × cos θ = mg × (1 − 2 cos θ)
As both bullet and wax initially are at 2 2
A
same temperature (Ti = 25° C). 3 1
⇒ cos θ =
So, ∆Tw = ∆Tb = ∆T (say) 2 2
1 1
G Then, mb vb2 = (mw cw + mb cb ) ∆T ⇒ cos θ =
2 3
53° B mb vb2 1
C
D or ∆T = or θ = cos−1  
E
m 1g 2(mw cw + mb cb )  3
https://iit-jeeacademy.blogspot.com

62 KVPY Question Paper 2016 Stream : SA

71. (a) For the given reaction, 74. (c) 3Pb2+ + 2AsO4 → Pb3 (AsO4 )2 77. (d) TTWW × ttww
CH3 3 moles of Pb 2+
reacts with 2 moles AsO4 Tall plant Short plant
with white with blue
∴1 mole of Pb will react with flowers flowers
Anhyd. AlCl3 (Y) 2
+ CH3Cl Friedel-Craft's
=
(X) reaction moles of AsO4 TtWw
Benzene Toluene 3 (Tall plant with white flower) (202 plants)
Selfing
CH3 Normality = Molarity × Volume
20
+ = 01
. × = 2 × 10−3 Obtained 2160 plants total
NO2 N 2 1000
HNO3/H2SO4;(Z)
Pb According to dihybrid phenotypic ratio
2
Nitration ∴ N AsO 4 = × 2 × 10−3 = 0.00133 9 : 3 : 3 : 1,
3
2-nitro toluene N As = N 3− = 0.00133 TW — 9
When benzene reacts with CH3Cl in
AsO
4 Tw — 3
WAs = N As × MassAs
presence of anhyd.AlCl3 , then toluene is tW — 3
formed. This reaction is known as = 0.00133 × 74.9
tw — 1
Friedel-Craft’s reaction. The formed = 0.0996
The total number of short and blue
toluene then undergoes nitration to give 0.0996
% of As = × 100 flowered plants is
2-nitrotoluene. 1.85 × = = 135
≈ 5.38 1 1080
72. (b) For the conversion of 2130
2,3-dibromobutane to 2-butyne following ≈ 5.4% 16 Q–iii, R–i 8
78. (a) P–ii,
steps can be used 75. (c) When treated with conc.HCl, – A neurogenic heart requires
Step 1 MnO2 yields a chlorine gas (X), which nervous input to contract. It is seen
Br Br in crustaceans like king crab.
further reacts with Ca(OH)2 to generate
Alc.KOH calcium oxychloride CaOCl 2(Y ), which is – Bronchial hearts are myogenic
H3C CH CH CH3 –HBr accessory pumps found in coleoid
2, 3-dibromobutane a white solid that then reacts with dil.
cephalopods like shark that
Br HCl to produce again chlorine gas (X ).
supplement the action of the main,
The equations can be written as systemic heart.
CH3 CH C CH3
2-bromo but-2-ene MnO2 + HCl (conc.) → Cl 2 ( g ) – Pulmonary heart is found in humans
In this step, dehydrohalogenation occurs X
where the portion of the circulatory
where 2,3-dibromobutane gets converted Ca(OH)2 + Cl 2 → CaOCl 2
Y system carries deoxygenated blood
into 2-bromobut-2-ene. away from the right ventricle of the
CaOCl 2 + dil. HCl → Cl 2 + CaCl 2 + H2O
Step 2 (X) heart to the lungs and returns
Br Thus, solid Y is Ca(OCl)Cl. oxygenated blood to the left atrium
and ventricle of the heart.
–NaBr 76. (a) pCO2 = 0.30 − 0.31 mm Hg in air.
CH3 CH C — CH3 + NaNH2 –NH3 79. (d) Both thermophiles and
Air contains 0.04% of carbon dioxide. psychrophiles will show same enzymatic
CH3 C C CH3 This means that in every 100 molecules reaction graph. Mostly proteinaceous
In this step, also dehydrohalogenation of air, 0.04 will be CO2 molecules. enzymes are labile to temperature.
occurs where alkenyl halide on treatment The number of moles of carbon dioxide in Thermophiles live at very high
with soda amide gives 2-butyne. 100 molecules of air will be temperature while psychrophiles live in
73. (a) Given, nCO2 = 0.04 molecules × N A = 0.04 × N A the range of −20° C to +10°C. In either
NO( g ) + O3 ( g ) → NO2 ( g ) + O2 ( g ); The total number of moles in the sample case, rising temperature will first raise
of air will be the rate of reaction but if temperature is
∆H1 = − 198.9 kJ/mol
3 still raised continuously, enzymes get
O3 → O2 ( g ) ∆H 2 ntotal = 100 molecules × N A = 100N A
denatured, hence reaction rate decreases.
2 This means that mole fraction of carbon
dioxide in the mixture will be 80. (b) P–v, Q–iv, R–i, S–iii
= − 142.3 kJ/mol
0.04N A – Hydrolases catalyse transfer of a
O2 → 2O2 ( g ) ∆H3 = + 495.0 kJ/mol XCO2 = = 0.0004
100N A chemical group from one substrate
For the reaction, to another.
NO( g ) + O( g ) → NO2 ( g ) Carbon dioxide’s partial pressure in air
– Lyase catalyses removal of chemical
1 will thus be
∆H = ∆H1 − ∆H 2 − ∆H3 groups from a substrate.
2 pCO2 = 0.0004 × 760 mm Hg – Isomerase catalyses interconversion
1 = 0.304 mm Hg of optical, geometric or positional
= − 198.9 − (−142.3) − ×
∴We can say it ranges between isomers.
0.30-0.31 mm Hg. – Ligase catalyses linking together of
two compounds.
495
2
= − 3041
. kJ/mol
https://iit-jeeacademy.blogspot.com

KVPY Question Paper 2015 Stream : SA 63

KVPY
KISHORE VAIGYANIK PROTSAHAN YOJANA

QUESTION PAPER 2015


Stream : SA
MM 100

Instructions
There are 80 questions in this paper.
This question paper contains two parts; Part I and Part II. There are four sections; Mathematics, Physics, Chemistry
and Biology in each part.
Out of the four options given with each question, only one is correct.

PART-I (1 Mark Questions)


MATHEMATICS 4. In the figure given below, a rectangle of perimeter
76 units is divided into 7 congruent rectangles.
1. Two distinct polynomials f (x) and g(x) are defined as
follows:
f (x) = x2 + ax + 2; g(x) = x2 + 2x + a.
If the equations f (x) = 0 and g(x) = 0 have a common
root, then the sum of the roots of the equation
f (x) + g(x) = 0 is
1 1
(a) − (b) 0 (c) (d) 1 What is the perimeter of each of the smaller
2 2
rectangles?
2. If n is the smallest natural number such that (a) 38 (b) 32 (c) 28 (d) 19
n + 2n + 3n + ... + 99n is a perfect square, then the
number of digits of n 2 is 5. The largest non-negative integer k such that 24k
(a) 1 (b) 2 (c) 3 (d) more than 3 divides 13! is
(a) 2 (b) 3 (c) 4 (d) 5
3. Let x, y, z be positive reals. Which of the following
6. In a ∆ ABC, points X and Y are on AB and AC, re-
implies x = y = z ?
spectively, such that XY is parallel to BC. Which of
I. x3 + y3 + z3 = 3xyz II. x3 + y 2z + yz 2 = 3xyz the two following equalities always hold? (Here [PQR]
III. x3 + y 2z + z 2x = 3xyz IV. ( x + y + z )3 = 27 xyz denotes the area of ∆PQR).
(a) I, IV only (b) I, II and IV only I. [BCX ] = [BCY ]
(c) I, II and III only (d) All of them II. [ ACX ] ⋅ [ ABY ] = [ AXY ] ⋅ [ ABC ]
https://iit-jeeacademy.blogspot.com

64 KVPY Question Paper 2015 Stream : SA

(a) Neither I nor II (b) Only I 15. How many ways are there to arrange the letters of
(c) Only II (d) Both I and II the word EDUCATION so that all the following
7. Let P be an interior point of a ∆ ABC. Let Q and R be three conditions hold?
the reflections of P in AB and AC, respectively. If — the vowels occur in the same order (EUAIO),
Q, A, R are collinear, then ∠A equals — the consonants occur in the same order (DCTN),
(a) 30° (b) 60° (c) 90° (d) 120° — no two consonants are next to each other.
8. Let ABCD be a square of side length 1, and Γ a circle (a) 15 (b) 24 (c) 72 (d) 120
passing through 3B and C, and touching AD. The
radius of Γ is (a)
8 PHYSICS
1 1 5
(b) (c) (d)
2 2 8 16. In an experiment, mass of an object is measured by
9. Let ABCD be a square of side length 1. Let P , Q, R, S applying a known force on it, and then measuring its
acceleration. If in the experiment, the measured val-
be points in the interiors of the sides AD, BC , AB, CD
ues of applied force and the measured acceleration
respectively, such that PQ and RS intersect at right
. ± 02
are F = 100 . N and a = 100
. ± 001
. m/s 2,
3 3
angles. If PQ = respectively. Then, the mass of the object is
, then RS equals (a) 10.0 kg (b) 10.0 ± 01
. kg
4 3 3
2 (b) (c) 10.0 ± 0.3 kg (d) 10.0 ± 0.4 kg
(a) 4
23 + 1 17. A hollow tilted cylindrical vessel of negligible mass
(c) (d) 4 − 2 2
2 rests on a horizontal plane as shown. The diameter of
the base is a and the side of the cylinder makes an
10. In the figure given below, if the areas of the two angle θ with the horizontal.Water is then slowly
regions are equal then which of the following is true?
poured into the cylinder. The cylinder topples over
45° when the water reaches a certain height h, given by

2y 2y 45°
y y y 45°
45°
x x h

(a) x = y (b) x = 2 y
(c) 2x = y (d) x = 3 y θ
a
11. A man standing on a railway platform noticed that a
train took 21 s to cross the platform (this means the (a) h = 2a tanθ (b) h = a tan 2 θ
time elapsed from the moment the engine enters the a
(c) h = a tanθ (d) h
platform till the last compartment leaves the = tanθ
platform) which is 88 m long, and that it took 9 s to 2
18. An object at rest at the origin begins to move in the
pass him. Assuming that the train was moving with + x-direction with a uniform acceleration of 1 m/s 2 for
uniform speed, what is the length of the train in 4 s and then it continues moving with a uniform
meters? velocity of 4 m/s in the same direction.The x -t graph
(a) 55 (b) 60 for object’s motion will be
(c) 66 (d) 72
x x
12. The least positive integer n for which
1
3 n + 1 −3 n < is
12
(a) (b)
(a) 6 (b) 7 (c) 8 (d) 9
13. Let n > 1 be an integer. Which of the following sets of
t t
numbers necessarily contains a multiple of 3? 4s 4s
x x
(a) n19 − 1, n19 + 1 (b) n19 , n38 − 1
(c) n38 , n38 + 1 (d) n38 , n19 − 1
(c) (d)
14. The number of distinct primes dividing 12 ! + 13 ! + 14 !
is
(a) 5 (b) 6 (c) 7 (d) 8 t t
4s 4s
https://iit-jeeacademy.blogspot.com

KVPY Question Paper 2015 Stream : SA 65

19. If the axis of rotation of the earth were extended into 26. A light bulb of resistance R = 16 Ω is attached in
space, then it would pass close to (a) the moon series with an infinite resistor network with identical
(b) the sun (c) resistances r as shown below. A 10 V battery drives
the pole star current in the circuit. What should be the value of r
(d) the centre of mass of all the planets in the solar system such that the bulb dissipates about 1 W of power.
20. Methane is a greenhouse gas because (a) R
it absorbs longer wavelengths of the electromagnetic r r r
spectrum while transmitting shorter wavelengths
(b) it absorbs shorter wavelengths of the electromagnetic r r r
10 V
spectrum while transmitting longer wavelengths
(c) it absorbs all wavelengths of the electromagnetic
spectrum (a) 14.8 Ω (b) 29.6 Ω (c) 7.4 Ω (d) 3.7 Ω
(d) it transmits all wavelengths of the electromagnetic
27. A ball is launched from the top of Mt. Everest which
spectrum
is at elevation of 9000 m. The ball moves in circular
21. A parachutist with total weight 75 kg drops vertically orbit around earth. Acceleration due to gravity near
onto a sandy ground with a speed of 2 ms −1 and the earth’s surface is g. The magnitude of the ball’s
comes to halt over a distance of 0.25 m. The average acceleration while in orbit is
force from the ground on her is close to (a) close to g / 2 (b) zero
(a) 600 N (b) 1200 N (c) much greater than g (d) nearly equal to g
28. A planet is orbiting the sun in an elliptical orbit. Let
(c) 1350 N (d) 1950 N 22. The β- U denote the potential energy and K denote the
particles of a radioactive metal originate from kinetic energy of the planet at an arbitrary point on
(a) the free electrons in the metal the orbit. Choose the correct statement.
(b) the orbiting electrons of the metal atoms (a) K <|U|always
(c) the photons released from the nucleus (b) K >|U|always
(d) the nucleus of the metal atoms (c) K =|U|always
(d) K =|U|for two positions of the planet in the orbit
23. An optical device is constructed by fixing three
identical convex lenses of focal lengths 10 cm each 29. One mole of ideal gas undergoes a linear process as
inside a hollow tube at equal spacing of 30 cm each. shown in the figure below. Its temperature expressed
One end of the device is placed 10 cm away from a as a function of volume V is
point source. How much does the image shift when
p0
the device is moved away from the source by another
10 cm?
(a) 0 (b) 5 cm (c) 15 cm (d) 45 cm
24. An isosceles glass prism with base angles 40° is
clamped over a tray of water in a position such that (0, 0) V0
the base is just dipped in water. A ray of light p0V 0 p0V
incident normally on the inclined face suffers total (a) (b)
R R
internal reflection at the base. If the refractive index  2
of water is 1.33, then the condition imposed on the p0V  V  p0V 0 1 − V  
(c) 1 −  (d)  
refractive index µ of the glass is R  V0  R   V0  
 
(a) µ < 2.07 (b) µ > 2.07
(c) µ < 174
. (d) µ > 174
. 30. The international space station is maintained in a
−1 nearly circular orbit with a mean altitude of 330 km
25. A point source of light is moving at a rate of 2 cms and a maximum of 410 km. An astronaut is floating
towards a thin convex lens of focal length 10 cm
in the space station’s cabin. The acceleration of
along its optical axis. When the source is 15 cm away
astronaut as measured from the earth is
from the lens, the image is moving at
(a) zero
(a) 4 cms−1 towards the lens
(b) nearly zero and directed towards the earth
(b) 8 cms−1 towards the lens
(c) nearly g and directed along the line of travel of the
(c) 4 cms−1 away from the lens station
(d) 8 cms−1 away from the lens (d) nearly g and directed towards the earth
https://iit-jeeacademy.blogspot.com

66 KVPY Question Paper 2015 Stream : SA

42. Among Mg, Cu, Fe, Zn the metal that does not
CHEMISTRY produce hydrogen gas in reaction with hydrochloric
31. The percentage of nitrogen by mass in ammonium acid is
sulphate is closest to (atomic masses of H = 1, N = 14, (a) Cu (b) Zn
O = 16, S = 32) (c) Mg (d) Fe
(a) 21% (b) 24% 43. The maximum number of isomeric ethers with the
(c) 36% (d) 16% molecular formula C4 H10 O is
32. Mendeleev’s periodic law states that the properties of (a) 2 (b) 3 (c) 4 (d) 5
elements are a periodic function of their 44. The number of electrons required to reduce chromium
(a) reactivity of elements (b) atomic size completely in Cr2 O72− to Cr 3+ in acidic medium, is
(c) atomic mass (d) electronic configuration
(a) 5 (b) 3 (c) 6 (d) 2
33. Maximum number of electrons that can be
accommodated in the subshell with azimuthal
45. At constant pressure, the volume of a fixed mass of a
gas varies as a function on temperature as shown in
quantum number l = 4, is
the graph
(a) 10 (b) 8 (c) 16 (d) 18
34. The correct order of acidity of the following
compounds is 500
OCH3 NO2
400

V(cm3)
300

200
COOH COOH COOH
1 2 3 100
(a) 1 > 2 > 3 (b) 1 > 3 > 2 (c) 3 > 1 > 2 (d) 3 > 2 > 1
0 100 200 300
35. Reaction of 2-butene with acidic KMnO4 gives T/°C
(a) CH3CHO (b) HCOOH The volume of the gas at 300°C is larger than that at
(c) CH3CH2OH (d) CH3COOH 0°C by a factor of
36. The gas released when baking soda is mixed with (a) 3 (b) 4 (c) 1 (d) 2
vinegar is
(a) CO (b) CO2 (c) CH4 (d) O2 BIOLOGY
37. The element which readily forms an ionic bond has
the electronic configuration (a) 1 2 2
46. Excess salt inhibits bacterial growth in pickles by
2 2 (a) endosmosis (b) exosmosis
s s p3 (b) 1s2 2s2 2 p1
(c) oxidation (d) denaturation 47. Re-
(c) 1s2 2s2 2 p 2 (d) 1s2 2s2 2 p 6 3s1
striction endonucleases are enzymes that are used
38. The major products of the following reaction, by biotechnologists to
ZnS (s) + O2 ( g ) → ……… are
Heat
(a) cut DNA at specific base sequences
(b) join fragments of DNA
(a) ZnO and SO2 (b) ZnSO4 and SO3
(c) digest DNA from the 3′ end
(c) ZnSO4 and SO2 (d) Zn and SO2
(d) digest DNA from the 5′ end
39. If Avogadro’s number is A0, the number of sulphur
48. Enzyme X extracted from the digestive system
atoms present in 200 mL of 1N H2SO4 is hydrolyses peptide bonds. Which of the following is
A0 A0 A0
(a) (b) (c) (d) A0 probable candidate to be enzyme X ?
5 2 10
(a) Amylase (b) Lipase
40. The functional group present in a molecule having (c) Trypsin (d) Maltase
the formula C12 O9 is
(a) carboxylic acid (b) anhydride 49. A person with blood group AB has
(c) aldehyde (d) alcohol (a) antigen A and B on RBCs and both anti-A and anti-B
antibodies in plasma
41. A sweet smelling compound formed by reacting acetic (b) antigen A and B on RBCs, but neither anti-A nor
acid with ethanol in the presence of hydrochloric acid anti-B antibodies in plasma
is (c) no antigen on RBCs but both anti-A and anti-B
(a) CH3 COOC2H5 (b) C2H5 COOH antibodies are present in plasma
(c) C2H5COOCH3 (d) CH3OH (d) antigen A on RBCs and anti-B antibodies in plasma
https://iit-jeeacademy.blogspot.com

KVPY Question Paper 2015 Stream : SA 67

50. Glycolysis is the breakdown of glucose to pyruvic 56. Widal test is prescribed to diagnose
acid. How many molecules of pyruvic acid are formed (a) typhoid (b) pneumonia
from one molecule of glucose? (c) malaria (d) filaria
(a) 1 (b) 2 (c) 3 (d) 4
57. Which among grass, goat, tiger and vulture in a food
51. The process of the transfer of electrons from glucose chain, will have the maximum concentration of
to molecular oxygen in bacteria and mitochondria is harmful chemicals in its body due to contamination
known as of pesticides in the soil?
(a) TCA cycle (b) oxidative phosphorylation (a) Grass since it grows in the contaminated soil
(c) fermentation (d) glycolysis (b) Goat since it eats the grass
52. Which one of the following cell types is a part of (c) Tiger since it feeds on the goat which feeds on the grass
innate immunity? (d) Vulture since it eats the tiger, which in turn eats the
(a) Skin epithelial cells (b) B-cells goat, which eats the grass
(c) T-lymphocytes (d) Liver cells 58. Considering the average molecular mass of a base to
53. Deficiency of which one of the following vitamins can be 500 Da, what is the molecular mass of a
cause impaired blood clotting? double-stranded DNA of 10 base pairs?
(a) Vitamin-B (b) Vitamin-C (a) 500 Da (b) 5 kDa (c) 10 kDa (d) 1 kDa
(c) Vitamin-D (d) Vitamin-K 59. Which of the following pairs are both
54. Which one of the following is detrimental to soil polysaccharides?
fertility? (a) Cellulose and glycogen
(a) Saprophytic bacteria (b) Nitrosomonas (b) Starch and glucose
(c) Nitrobacter (d) Pseudomonas (c) Cellulose and fructose
(d) Ribose and sucrose
55. In which one of the following phyla is the body
segmented? 60. Which one of the following is a modified leaf ?
(a) Porifera (b) Platyhelminthes (a) Sweet potato (b) Ginger
(c) Annelida (d) Echinodermata (c) Onion (d) Carrot

PART-II (2 Marks Questions)


MATHEMATICS 64. Given are three cylindrical buckets X , Y , Z whose
61. A triangular corner is cut from a rectangular piece of circular bases are of radii 1, 2, 3 units, respectively.
paper and the resulting pentagon has sides 5, 6, 8, 9, Initially water is filled in these buckets upto the
12 in some order. The ratio of the area of the same height. Some water is then transferred from Z
pentagon to the area of the rectangle is to X so that they both have the same volume of
11 13 15 17 water. Some water is then transferred between X and
(a) (b) (c) (d)
18 18 18 18 Y so that they both have the same volume of water. If
62. For a real number x, let [x] denote the largest integer hY , hZ denote the heights of water at this stage in
less than or equal to x, and let { x} = x − [x]. The h
the buckets Y , Z , respectively, then the ratio Y
number of solutions x to the equation [x] { x} = 5 with hZ
0 ≤ x ≤ 2015 is equals
(a) 0 (b) 3 (c) 2008 (d) 2009 4 9 81
(a) (b) 1 (c) (d)
63. Let ABCD be a trapezium with AD parallel to BC. 9 4 40
Assume there is a point M in the interior of the 65. The average incomes of the people in two villages are
segment BC such that AB = AM and DC = DM. Then, P and Q, respectively. Assume that P ≠ Q. A person
the ratio of the area of the trapezium to the area of moves from the first village to the second village. The
∆ AMD is new average incomes are P′ and Q′, respectively.
(a) 2 Which of the following is not possible?
(b) 3 (a) P ′ > P and Q ′ > Q (b) P ′ > P and Q ′ < Q
(c) 4 (c) P ′ = P and Q ′ = Q (d) P ′ < P and Q ′ < Q
(d) not determinable from the data
https://iit-jeeacademy.blogspot.com

68 KVPY Question Paper 2015 Stream : SA

70. Stokes’ law states that the viscous drag force F


PHYSICS experienced by a sphere of radius a, moving with a
66. A girl sees through a circular glass slab (refractive speed v through a fluid with coefficient of viscosity η,
index 1.5) of thickness 20 mm and diameter 60 cm to is given by F = 6πηav.
the bottom of a swimming pool. Refractive index of If this fluid is flowing through a cylindrical pipe of
water is 1.33. The bottom surface of the slab is in radius r, length l and a pressure difference of p across
contact with the water surface. its two ends, then the volume of water V which flows
through the pipe in time t can be written as
a
v  p
= k   ηbr c
t  l
where, k is a dimensionless constant. Correct value of
a , b and c are
(a) a = 1, b = − 1, c = 4
The depth of swimming pool is 6 m. The area of (b) a = − 1, b = 1, c = 4
bottom of swimming pool that can be seen through (c) a = 2, b = − 1, c = 3
the slab is approximately (d) a = 1, b = − 2, c = − 4
(a) 100 m 2 (b) 160 m 2 (c) 190 m 2 (d) 220 m 2
67. 1 kg of ice at − 20°C is mixed with 2 kg of water at CHEMISTRY
90°C. Assuming that there is no loss of energy to the
71. When 262 g of xenon (atomic mass = 131) reacted
environment, what will be the final temperature of
completely with 152 g of fluorine (atomic mass = 19),
the mixture? (Assume, latent heat of ice = 3344 . kJ/kg,
a mixture of XeF2 and XeF6 was produced. The molar
specific heat of water and ice are 4.18 kJ kg −1K −1 and ratio XeF2 : XeF6 is
2.09 kJ kg −1-K −1, respectively.)
(a) 1 : 2 (b) 1 : 4
(a) 30°C (b) 0°C (c) 80°C (d) 45°C (c) 1 : 1 (d) 1 : 3
68. A rigid body in the shape of a V has two equal arms 72. Reaction of ethanol with conc. sulphuric acid at
made of uniform rods. What must the angle between 170°C produces a gas which is then treated with
the two rods be so that when the body is suspended bromine in carbon tetrachloride. The major product
from one end, the other arm is horizontal? obtained in this reaction is
(a) cos−1   (b) cos−1  
1 1
(a) 1,2-dibromoethane
 3  2
(b) ethylene glycol
(c) cos−1   (d) cos−1  
1 1
(c) bromoethane
 4  6
(d) ethyl sulphate
69. A point object is placed 20 cm left of a convex lens of 73. When 22.4 L of C4 H8 at STP is burnt completely,
focal length f = 5 cm (see in the below figure). The
89.6 L of CO2 gas at STP and 72 g of water are
lens is made to oscillate with small amplitude A
produced. The volume of the oxygen gas at STP
along the horizontal axis. The image of the object will
consumed in the reaction is closest to
also oscillate along the axis with
(a) 89.6 L (b) 112 L
(c) 134.4 L (d) 22.4 L
f 74. The amount of Ag (atomic mass = 108) deposited at
the cathode when a current of 0.5 amp is passed
through a solution of AgNO3 for 1 h is closest to
(a) 2 g (b) 5 g
(c) 108 g (d) 11 g
75. The major product of the reaction is
H+/H2O
A Product
(a) amplitude A / 9 , out of phase with the oscillations of OH
the lens OH OH
(b) amplitude A / 3, out of phase with the oscillations of the
lens
HO
(c) amplitude A / 3, in phase with the oscillations of the I II III IV
lens
(d) amplitude A / 9, in phase with the oscillations of the (a) I (b) II (c) III (d) IV
lens
https://iit-jeeacademy.blogspot.com

KVPY Question Paper 2015 Stream : SA 69

BIOLOGY

Population

Population
76. Genomic DNA is digested with Alu I, a restriction (c) (d)
enzyme which is a four base-pair cutter. What is the
frequency with which it will cut the DNA assuming a
random distribution of bases in the genome?
(a) 1/4 (b) 1/24 (c) 1/256 (d) 1/1296 Time Time
77. If rice is cooked in a pressure cooker on the Siachen
glacier at sea beach and on Deccan plain, which of 79. What is the advantage of storing glucose as glycogen
the following is correct about the time taken for in animals instead of a monomeric glucose?
cooking rice? (a) Energy obtained from glycogen is more than that from
(a) Gets cooked faster on the Siachen glacier the corresponding glucose monomers
(b) Gets cooked faster at sea beach (b) Glucose present as monomers within the cell exerts
(c) Gets cooked faster on Deccan plain more osmotic pressure than a single glycogen
(d) Gets cooked at the same time at all the three places molecule, resulting in loss of water from the cells
(c) Glucose present as monomers within the cell exerts
78. A few rabbits are introduced in an uninhabited
more osmotic pressure than a single glycogen
island with plenty of food. If these rabbits breed in
molecule, resulting in excess water within the cells
the absence of any disease, natural calamity and
(d) Glycogen gives more rigidity to the cells
predation, which one of the following graphs best
represents their population growth? 80. A line is drawn from the exterior of an animal cell to
the centre of the nucleus, crossing through one
mitochondrion. What is the minimum number of
membrane bilayers that the line will cross?
Population

Population

(a) (b) (a) 4


(b) 3
(c) 8
(d) 6
Time Time

Answers
PART-I
1 (c) 2 (c) 3 (b) 4 (c) 5 (b) 6 (d) 7 (c) 8 (d) 9 (b) 10 (b)
11 (c) 12 (c) 13 (b) 14 (a) 15 (a) 16 (c) 17 (c) 18 (b) 19 (c) 20 (a)
21 (c) 22 (d) 23 (a) 24 (b) 25 (d) 26 (a) 27 (d) 28 (a) 29 (c) 30 (d)
31 (a) 32 (c) 33 (d) 34 (c) 35 (d) 36 (b) 37 (d) 38 (a) 39 (c) 40 (b)
41 (a) 42 (a) 43 (b) 44 (c) 45 (d) 46 (b) 47 (a) 48 (c) 49 (b) 50 (b)
51 (b) 52 (a) 53 (d) 54 (d) 55 (c) 56 (a) 57 (d) 58 (c) 59 (a) 60 (c)

PART-II
61 (d) 62 (d) 63 (b) 64 (d) 65 (c) 66 (b) 67 (a) 68 (a) 69 (a) 70 (a)
71 (c) 72 (a) 73 (c) 74 (a) 75 (a) 76 (c) 77 (b) 78 (a) 79 (c) 80 (a)
https://iit-jeeacademy.blogspot.com

70 KVPY Question Paper 2015 Stream : SA

Solutions
1. (c) We have, IV. (x + y + z )3 = 27xyz ∴(Area of ∆ACX) (Area of ∆ABY )
f (x) = x2 + ax + 2 and g (x) = x2 + 2x + a x= y=z 1 1
= (AX ) (AC ) sin A × (AY )(AB ) sin A
Let α be the common root of f (x) = 0 and Then, (3x)3 = 27x3 2 2
g (x) = 0. 1 1
Hence option (iv) is also true. = (AX )(AY ) sin A × (AB )(AC ) sin A
∴ α 2 + aα + 2 = 0 2 2
4. (c) Given,
and α 2 + 2α + a = 0 Perimeter of rectangle is 76 units. = (Area of ∆AXY ) (Area of ∆ABC)
α2 −α 1 x x x x Hence, I and II both are true.
∴ = =
a − 4 a− 2 2− a
2
7. (c) ABC is a triangle. P be interior
α2 α y y y y y point of a ∆ ABC, Q and R be the
= reflections of P in AB and AC
a −4
2
2 −a
y y y respectively.
a 2 − 4 (a + 2) (a − 2)
⇒α = = = − (a + 2) x x x x R
2− a − (a − 2)
α 1 y y y
and = ⇒α = 1 A
φ
2− a 2− a Let x and y are sides of each rectangles. Q θ
θφ
∴ − (a + 2) = 1 ∴Perimetre of rectangle = 6x + 5 y = 76
a + 2 = − 1⇒ a = − 3 …(i)
P
Now f (x ) + g (x ) = 0 and 4x = 3 y …(ii)
∴x − 3x + 2 + x2 + 2x − 3 = 0
2
On solving Eqs. (i) and (ii), we get
2x2 − x − 1 = 0 x = 6, y = 8
Sum of roots =
1 Q α + β = − b  ∴Perimeter of each rectangle
B C
2  a  QAR are collinear
= 2 (x + y) = 2 (6 + 8) = 28 units
2. (c) We have, ∴ ∠QAR = 180°
5. (b) 13! = 2 × 3 × 4 × 5 × 6 × 7
n + 2n + 3n + ... + 99n is a perfect square Q is reflection of P on AB
× 8 × 9 × 10 × 11 × 12 × 13
n × 99 × 100 ∴ ∠QAB = ∠PAB
n (1 + 2 + ... + 99), = 210 × 35 × 52 × 7 × 11 × 13
2 R is reflection of P on AC
24k = (23 × 3)k
n × 11 × 9 × 2 × 25 ∴ ∠RAC = ∠PAC
When 13! is divide by 24k
= (3)2 × (5)2 × 2 × 11 × n is a perfect square ∠QAR = 180°
210 × 35 × 52 × 7 × 11 × 13
∴ n must be 22. ∴ ∴2 (∠PAB + ∠PAC ) = 180°
23 k ⋅ 3k ∠PAB + ∠PAC = 90°
∴ n 2 = (22)2 = 484
= 210 − 3 k ⋅ 35 − k ⋅ 52 × 7 × 11 × 13 ⇒ ∠BAC = 90°
Number of digit of n 2 is 3.
∴ 10 − 3k = integer
3. (b) We have, 8. (d) ABCD is a square
Then, maximum value of k = 3
x = y = z, x, y, z positive reals. AB = BC = CD = AD = 1unit
6. (d) ABC is a triangle points X and Y
I. x3 + y3 + z3 = 3xyz C
on AB and AC respectively. D
We know,
XY is parallel to BC.
x3 + y3 + z3 − 3xyz = (x + y + z ) r 1
A 2
(x2 + y2 + z 2 − xy − yz − zx)
r
1 N M
= (x + y + z ) [(x − y)2 O 1–r
2
+ ( y − z ) 2 + (z − x ) 2 ] X Y
When x = y = z
A B
Then, (x − y)2 + ( y − z )2 + (z − x)2 = 0
∴ x3 + y3 + z3 = 3xyz B C A circle Γ passing through B and C and
II. x + y z + yz 2 = 3xyz
3 2 I. Area of BCX : Area of BCY touching AD.
Put x = y = z It is true because same base between BC is chord of circle.
same parallels. ∴OM bisects the chord AB
Then, LHS = RHS
1
III. Put x = z = 1and y = 2 II. Area of ∆ACX = (AX )(AC ) sin A ∴
1
CM = MB = BC =
1
2 2 2
Then, it is also true. 1
Area of ∆ABY = (AY ) (AB ) sin A ⇒ OM = MN − ON = 1 − r
So, we cannot say only for x = y = z for true 2
https://iit-jeeacademy.blogspot.com

KVPY Question Paper 2015 Stream : SA 71

In ∆OMC, OC 2 = OM 2 + CM 2 Area of fig. (ii) 14. (a) We have, 12! + 13! + 14!
2
Area of ABCG + Area of DEFG
r = (1 − r ) +   12!(1 + 13 + 13 × 14)
1
⇒ 2 2
 2 = 2xy + (2x − y) y 12! (1 + 13 (1 + 14))
⇒ r 2 = 1 − 2r + r 2 +
1
⇒r =
5 = 2xy + 2xy − y2 = 4xy − y2 12! × 196
4 8 7xy 1
∴ = 4xy − y2 ⇒ y2 = xy ⇒ 2y = x The number of distinct prime of 12! × 196
9. (b) ABCD is square 2 2 is 2, 3, 5, 7, 11.
AB = BC = CD = AD = 1 11. (c) Let the length of trains be 15. (a) Given, EDUCATION
PQ is perpendicular to RS x meter.
Vowel occurs in same order
D(0, 1) S(5, 1) C(1, 1) Time taken by train h cross person = 9 s
_E_U_A_I_O_
x
∴ Speed of trains = m/s There are 6 place for letter DCTN
9
Time taken by train to cross platform = 21s ∴ Total number of arrangement is
P(0, p)
x x + 88
6
C4 = 15.
∴ =
9 21 16. (c) As, force F = ma
Q(0, q)
[Q length of plateform = 88 m] ∆F ∆m ∆a ∆m ∆F ∆a
⇒ = + ⇒ = −
⇒ 21x = 9x + 9 × 88 F m a m F a
A(0, 0) R(r, 0) B(1, 0)
∆m  ± 0.2   ± 0.01
Q Slope of PQ × Slope of RS = − 1 ⇒ 12x = 9 × 88 ⇒ =  − 
m  10   1 
q − p 1− 0 9 × 88
∴ × = −1 ⇒ x= = 66 m  ∆m  0.2 0.01
1− 0 s − r 12 So,   = + = 0.03
 m  max 10 1
⇒ q− p= r − s …(i) 12. (c) We have,
1 Maximum error in mass occurs when
⇒ (PQ )2 = (1 − 0)2 + (q − p )2 3 n + 1−3 n < error in force and acceleration are of
2 12
 3 3 different signs.
⇒   = 1 + (q − p )
2 1
3 n + 1< 3 n + So, ∆m = 0.03 × m = 0.03 × 10 ⇒ ∆m= 0.3 kg
 4  12
27 11 Hence, mass of object is m = 10 ± 0.3 kg.
⇒ (q − p ) 2 = − 1= Cubing both sides, we get
16 16 1 1 1 17. (c) Cylinder will topple when centre
n + 1 < n + 3(n )23
/
× + 33 n × +
11 12 144 1728 of mass of filled cylinder lies outside the
⇒ (r − s)2 = [Q q − p = r − s] right edge of base. As centre of mass of
16 3n1/3  1/3 1 1
⇒ 1< n +  + filled cylinder lies at its mid-point.
⇒ RS = (1 − 0)2 + (r − s)2 12  12  1728
Cylinder
11 n1/3  1/3 1 1
⇒ RS = 1 + ⇒ n +  > 1− C
16 4  12  1728
⇒ n1/3  n1/3 +
1  1727
∴ RS =
27 3 3
= > h
/2
h
 12 
θ)

16 4 432 ——
sin

sinθ
Put n = 8 only possible least positive
(h

10. (b) Given, θ θ


integers. A B
Area of both figures are equal a/2
13. (b) Let n = 3q + r
Now, from above diagram, we have
0≤ r < 3 BC h
sinθ = ⇒ AC =
∴ n = 3q, 3q + 1, 3q + 2 AC sinθ
If n is multiple of 3  a 
 
i.e. n = 3q So, cosθ =  2  or h = a
Then, n19 is also multiple of 3.  sin θ 
h
tanθ
2
1 When n = 3q + 1and 3q + 2
Area of fig. (i) = 2xy + . x (3 y ) 18. (b) Initially, the velocity is
2 n38 = (3q + 1)38
increasing, so the (x - t ) graph must be
3xy 7xy = (3q + 1)36 (3q + 1)2
= 2xy + = with increasing slope or parabolic.
2 2 = (36k + 1) (9q2 + 6q + 1) 1
F 2x – y E For first 4-s, x = ut + at 2
[Q (x + 1)x = nk + 1] 2
45°
= 36k (9q + 6q + 1) + 9q2 + 6q + 1
2 ⇒ x = t 2 / 2 (parabola).
45° D = 3k + 3λ + 1 After 4 s, particle is moving with a
2y G C
2x constant velocity, so its graph is a
∴ n38 − 1 = 3k + 3λ + 1 − 1 = 3m
y 45° straight line of constant slope after 4 s.
45° ∴ n38 − 1 is multiple of 3 After 4 s, velocity is constant.
A 2x B Similarly, when n = 3q + 2 ∴ x = vt = (4 + at )t = 4t (straight line)
(ii) n38 − 1 is also multiple of 3. Hence, best suited option is (b).
https://iit-jeeacademy.blogspot.com

72 KVPY Question Paper 2015 Stream : SA

19. (c) Axis of rotation of earth as shown 23. (a) Initially given situation is Adding or removing one of repeating
below. 10 cm 30 cm 30 cm 20 cm member does not alters the resistance of
Polaris an infinite network. Let RAB = x, then
5° (The north star) RAB = RCD .
23 .
rx
Axis of rotation f f 2f ⇒ x=r+
O I r+ x
⇒ x2 − rx − x = 0
Earth’s
orbit 10 20 20
From sridharacharya formula, we have
cm cm cm
Perpendicular to When device is moved away from source −b ± b2 − 4ac
x=
plane of orbit O, then situation is as shown below. 2a
The southern 20 cm 30 cm 30 cm 10 cm
cross (− r ) ± r 2 + 4r 2
⇒ x=−
Axis of rotation of earth passes close to 2
polaris, the polar star. r (1 + 5 )
2f 2f
I
⇒ x=
O 2
20. (a) Heat through sun reaches earth
in form of infrared radiations of higher f Now, power consumed by bulb of
frequency range approx 1014 Hz. This resistance R is 1 W,
So, distance between object and image in
heat is absorbed by solids on earth’s 1 1
both cases is 90 cm. Hence, there is no i 2R = 1 ⇒ i 2 = ⇒ i= A
surface and they re-radiate this heat in 16 4
shift in image’s position.
form of infrared radiations of lower Now, current in circuit is
frequency range approx 1010 Hz. These 24. (b) Base angles of prism is given 40°.
V V
radiations are absorbed by greenhouse So, angle of prism A = 180° − 80° = 100°. i= ⇒i =
Rtotal R + RCD
gases like methane and does not escapes
1 10
into space causing warming of earth’s 100° ⇒ =
atmosphere. 4 16 + r (1 + 5 )
40° 2
21. (c) As parachutist lands on earth’s 40°
r
surface, there are two forces acting on her. ⇒ 16 + (1 + 5 ) = 40 ⇒ r = 14.8 Ω
µw=1.33 2
FResistance
27. (d) Let orbital radius of ball is r then
orbital velocity of ball is
For TIR, i > θc ⇒ sin 40° > sin θc
GM
µ 133
. v=
FGravitation ⇒ sin 40° > w ⇒ µ g > ≈ 2.07 r
µg sin 40°
Here, r=R+ h
Now, before writing Newton’s second law or µ g > 2.07
equation, we calculate acceleration of ⇒ r = 6400 km + 9 km
25. (d) From mirror formula, we have
parachutist using 1 1 1 or r ≈ 6400 km
− = …(i)
v2 − u 2 = 2as v u f ⇒ r = R (radius of earth)
0 − (− 2)2 = 2 (a ) (− 0.25) Here, f = + 10 cm, u = − 15 cm Now, acceleration of ball in orbit is
So, retardation of parachutist is 10 × − 15 v2 GM GM
⇒ v= = 30 cm a= = 2 ≈ 2 or a ≈ g
a = 8 ms−2 (directed upwards) −15 + 10 r r R
Now, using Fnet = ma, we have Now differentiating Eq. (i) with respect to So, acceleration of ball is nearly equal
FR − Fg = ma ⇒ FR − mg = ma time, we get to g.
or FR = m( g + a ) ⇒ FR = 75(10 + 8) = 1350 N dv v2  du 
=  
28. (a) For a satellite or planet, if total
So, resistive force of ground on
dt u 2  dt  energy is E, then
parachutist is 1350 N.
dv (+ 30)2 kinetic energy, K = − E
22. (d) β − -particles are emitted from ∴ = × (+ 2 cm s−1 )
dt (− 15)2 and potential energy,U = 2E
following nuclear reaction:
dv where, E is negative.
0 n → 1 p + −1 e + ν
1 1 0
⇒ = + 8 cm s−1
dt So, |U|> K .
A neutron in nucleus is converted into a
proton with emission of a β − -particle and So, image is moving away from lens. 29. (c) Process given is
an antineutrino. This converts emitting 26. (a) p
nucleus into another nucleus of higher p0
R C r A r r
proton number.
Z X → z + 1Y + −1 e + ν
A 4 0
r r
This decay is characteristics of nuclii for
N V
which, < 1. V0
Z D B
https://iit-jeeacademy.blogspot.com

KVPY Question Paper 2015 Stream : SA 73

To find process equation, we use two NO2 M = 0.5 mol/L


point form of equation of straight line, ∴No. of moles of H2SO4 = No. of moles of S
y − y1 0.5 × 200
y − y1 = 2 (x − x1 ) base of the compound 3 and atom = = 01
. mol
x2 − x1 1000
Here, (x, y,) = (0, p0 ) and (x2 , y2 ) = (V 0 , 0) [No. of moles = Volume × Molarity]
Process equation is COO– 1 mol of S = A0 atoms
p 1 A0
p = p0 − 0 . V hence increases the acidity. Thus, the ∴ 0.1 mole of S = A0 × = atoms.
V0 correct order of acidity of the given 10 10
RT compounds are 40. (b) The structure of C12O9 is as
As, p=
V NO2 OCH3 follows
RT p
⇒ = p0 − 0 . V
V V0 O
> >
O
p0V  V 
⇒ T = 1 −  C O Anhydride
R  V0  C group
COOH COOH COOH
C O
30. (d) At height h, acceleration due to 3 1 2 O
GM
gravity is gh = 35. (d) Acidic potassium permanganate
(R + h ) 2 oxides alkenes to ketones or acids de-
C C O
C O
For h < < R, pending upon the nature of the alkene. O
GM Thus, reaction of 2-butene with acidic O
gh ≈ 2 = g
R KMnO4 gives acetic acid. Mellitic anhydride
KMnO4 /H+
Direction is towards centre of earth. 2 CH3COOH Thus, the functional group present in a
Strong
2 butene Acetic acid molecule having C12O9 is an anhydride
31. (a) Total mass of ammonium oxidising
agent group.
sulphate (NH 4 )2 SO4 36. (b) When baking soda is mixed with
= 2 × 18 + 32 + 16 × 4 vinegar aqueous solution of sodium
41. (a) When acetic acid reacts with
ethanol in the presence of hydrochloric
= 36 + 96 = 132 acetate is formed with the evolution of
acid then ethyl acetate (ester) is formed
Mass of nitrogen in (NH 4 )2 SO4 = 28 carbon dioxide gas
which is a sweet smelling compound.
28
% of N by mass in (NH4 )2 SO4 = × 100 CH3COOH + NaHCO3 CH3COONa (aq)
– +
CH3COOH + C2H5OH CH3COOC2H5
132
Vinegar Baking soda Sodium Acetic acid Ethanol Ethyl acetate
= 21.2% acetate (sweet smelling
compound)
32. (c) According to Mendeleev’s periodic + H2O (l)+CO2(g) + H2O
Carbon dioxide gas
law, the physical and chemical properties 42. (a) The metals that are present
of the elements are a periodic function of 37. (d) Alkali metals have the highest below hydrogen in reactivity series will
their atomic mass. tendency to form ionic bond as they have not produce hydrogen gas in reaction
33. (d) Maximum number of electrons low ionisation energy. The general with hydrochloric acid. Among the given
that can be accommodated in a subshell electronic configuration of alkali metal is metals, Cu is present below H in
= 2(2l + 1) ns1 . Among the given electronic reactivity series, i.e. it is less reactive
configuration, 1s2 2s2 2 p 6 3s1 corresponds to
When l = 4 than H, will not produce H 2 gas in
the configuration of Na, which is an alkali reaction with HCl acid.
Maximum number of electrons metal and hence forms ionic bond readily.
= 2(2 × 4 + 1) = 18 43. (b) Isomers of compound with
38. (a) molecular formula C4 H10 O are as follows
34. (c) Electron donating substituents ∆
2ZnS( s) + 3O 2 ( g ) → 2ZnO + 2SO 2 CH3 CH2CH2CH2OH
tends to decrease the acidic strength
while electron withdrawing substituents This process is known as roasting where CH3 CH(OH)CH2CH3
tends to increase the acidic strength of the sulphide ore is heated in a regular CH3 — O — CH2CH2CH3
substituted benzoic acids relative to supply of air to give its oxide form at a
CH3 CH2OCH2CH3
benzoic acid. OCH 3 exerts + M effect temperature below the melting point of
the metal. CH3 — O — C H — CH3
which destabilises the conjugate base |
OCH3 39. (c) Given, CH3
Normality of H2SO4 = 1 N Thus, there are 3 isomeric ethers with
Avogadro’s number = A molecular formula C4 H10 O.
of the compound 2 and hence +6
Volume of H2SO4 = 200 mL 44. (c) C r2O72− + 14H+ → 2Cr3 + + 7H2O
Normality = Basicity × Molarity
As in the above reaction, there are net
COO– For H 2 SO4 , basicity = 2 twelve positive charges on the left side
decreases the acidity, whereas NO2 exerts ∴ 1= 2 × M and only six positive charges on right
−M effect and stabilises the conjugate side.
https://iit-jeeacademy.blogspot.com

74 KVPY Question Paper 2015 Stream : SA

Therefore, 6 electrons are required to pyruvate (pyruvic acid) molecules, a total chemicals in its body. Since vulture
reduce chromium completely in of four ATP molecules and two molecules occupies the top level as it eats the tiger,
Cr2O72− to Cr3 + in acidic medium of NADH. which eats the goat, which eats the grass
Cr2O72− (aq) + 14H+ (aq) + 6e− → in the food chain, it will have the
51. (b) Oxidative phosphorylation is the
maximum concentration of harmful
2Cr3 + (aq) + 7H2O process in which ATP is formed as a chemicals in its body.
45. (d) result of the transfer of electrons from
NADH or FADH 2 (produced during
58. (c) Molecular mass of a base = 500
glycolysis from glucose) to molecular Da
500
oxygen (O2 ) by a series of electron Number of base in a dsDNA = 10 BP or
400 carriers. It takes place in the 20 bases
mitochondria in eukaryotes and in Thus, molecular mass of a dsDNA with
V(cm3)

300
cytoplasm in prokaryotes. 20 bases = 20 × 500 = 10 kDa
200 52. (a) Innate immunity refers to 59. (a) A carbohydrate (e.g. starch,
non-specific defence mechanisms that cellulose or glycogen) is a molecule
100
0 100 200 300 come into play immediately or within consisting of a number of sugar molecules
T/°C hours of an antigen’s appearance in the bonded together by glycosidic linkages
Volume of gas at 0°C body. These mechanisms include physical and on hydrolysis give its constituent
barriers such as skin epithelial cells, monosaccharides or oligosaccharides.
V1 = 250 cm3
chemicals in the blood and immune Cellulose is a polymer of β, D-glucose and
Volume of gas at 300°C system cells that attack foreign cells in glycogen of α, D-glucose. Glucose, fructose
V 2 = 500 cm3 the body. and ribose are monosaccharides.
V 2 500
∴ = =2 53. (d) Vitamin-K is a cofactor for the 60. (c) Onion is a bulb, i.e. it is a
V1 250 enzyme responsible for chemical modified leaf. A bulb is an underground
Thus, the volume of the gas at 300°C is reactions that maintains blood clotting pyriform-spherical structure that
larger than that at 0°C by a factor of 2. factors : prothrombin; factor VII, IX, X; possesses a reduced convex or slightly
conical disc-shaped stem and several
46. (b) Excess salt inhibits growth in and proteins. Thus vitamin-K plays a key
fleshy scales enclosing a terminal bud.
pickles by exosmosis. Salt kills and role in helping the blood clot thereby
In Onion, the fleshy scales represent leaf
inhibits the growth of microorganisms by preventing excessive bleeding. bases in the outer part and scale leaves
drawing water out of the cells of both the 54. (d) Pseudomonas is denitrifying in the central region.
microbe and the food through osmosis (or bacteria. Denitrifying bacteria are 61. (d) We have,
more specifically exosmosis). Due to microorganisms whose action results in A rectangular corner is cut form a
hypertonic solution outside the bacterial the conversion of nitrates in soil to free rectangular piece of paper.
cell, bacteria will die by plasmolysis. atmospheric nitrogen, thus depleting soil
47. (a) Restriction endonuclease is an fertility and reducing agricultural D 4 Q 8 C
enzyme that cuts dsDNA into fragments productivity. 3
at or near specific recognition sites 5
55. (c) Annelida shows metameric P 9
(palindromic sequence) within the segmentation. It is the repetition of
molecule known as restriction sites. 6
organs and tissues at intervals along the
These enzymes are found in bacteria and body of an animal, thus dividing the body A 12 B
archaea and provide a defence into a linear series of similar parts or Area of rectangle
mechanism against invading viruses. segments (metameres). = 12 × 9 = 108 sq units
48. (c) In duodenum, trypsin enzyme 56. (a) The widal test is one method Area of pentagon
catalyses the hydrolysis of peptide bonds, used to diagnose enteric fever also known = Area of rectangle − Area of triangle
breaking down proteins into smaller as typhoid fever. Typhoid is caused by
peptides. Amylase hydrolyses starch into = 108 − 6 = 102
Salmonella typhi bacteria. Widal test was
102 17
maltose inside the mouth. Lipase breaks based on demonstrating the presence of ∴ Ratio = =
down dietary fats into fatty acids and agglutinin (antibody) in the serum of an 108 18
glycerol. Maltase hydrolyses maltose into infected patient, against the ‘H’ (flagellar) 62. (d) We have,
simple sugar glucose. and ‘O’ (somatic) antigens of Salmonella [x ] {x } = 5
49. (b) Person with blood group AB have typhi. x ∈[0, 2015]
both A and B antigen in the membrane of 57. (d) The increase in concentration of 5
⇒ {x } =
his red blood cell but lacks both harmful chemical substance like [x ]
antibodies (a, b) in his plasma. Due to pesticides in the body of living organisms
at each trophic level of a food chain is {x} ∈ [0,1)
this reason, blood group AB is called 5
universal recipient. called biological magnification. The ∴ <1
organism which occurs at the highest [x ]
50. (b) Glycolysis starts with one trophic level in the food chain will have [x ] > 5
molecule of glucose and ends with two the maximum concentration of harmful
∴ Total number of solution is 2009.
https://iit-jeeacademy.blogspot.com

KVPY Question Paper 2015 Stream : SA 75

63. (b) Given, 65. (c) Let the number of people in two ⇒ mw sw ∆T = mi si (0 − (− 20° C))
ABCD is a trapezium. villages are x and y respectively. + mi L + mi sw (T − 0)
AD is parallel to BC Given, average income of x people = P ⇒ 2 × 418
. × (90 − T ) = 1 × 2.09 × 20
and average income of y people = Q + 1 × 334.4 + 1 × 418
. ×T
M is point on BC
∴Total income of people in two villages ⇒ 752.4 − 376.2 = 3 × 418. ×T
such that AB = AM and DC = DM
are Px and Qy respectively.
A N D ⇒ T = 30°C
One person moves from first village to
So, final temperature of mixture is 30°C.
second village.
Then, number of people in first village 68. (a) Let length of each of rod is l and
B C
P M Q = x − 1and second village = y + 1. angle between them is θ.
In ∆AMD, Average income = P ′ and Q′
Area of ∆AMD = Area of ∆AMN ∴Total income = P ′ (x − 1) and Q ′ ( y + 1)
+ Area of∆DMN Total income in both cases are same
Area of ∆AMN = Area of ∆AMP ∴ Px + Qy = P ′ (x − 1) + Q ′ ( y + 1)
= Area of ∆ABP ⇒ Px − P ′ (x − 1) = Q ′ ( y + 1) − Qy θ
Area of ∆DMN = Area of ∆DQM ⇒ x(P − P ′ ) + P ′ = y(Q ′ − Q ) + Q ′
= Area of ∆DQC ∴ P ′ ≠ P and Q ′ ≠ Q l
∴Area of trapezium ABCD Hence, option (c) is correct.
= Area of ∆ABM + Area of ∆AMD 66. (b) Girl can observe only those light P
+ Area of ∆MDC rays which are refracted and leaves the Rod 1
glass slab at angle of 90° or less as shown
= 3 [Area of ∆APM + Area of ∆DMN) A
below.
= 3 Area of ∆AMD To eye of observer Rod 2
Area of trapezium ABCD θ B
∴ Glass 1
C
Area of ∆AMD slab D
3 [Area of ∆ADM ] 3 2
= = = 3:1 3
mg mg
Area of ∆ADM 1 Pool r h=6m
bottom Let the lower rod is horizontal and upper
64. (d) Let hX , hY and hZ are height of rod makes θ angle with horizontal.
cylindrical bucket of X , Y and Z Weights of rods acts vertically
x 0.6 x
respectively and rx , ry and rz are radii of downwards from their centres A and B as
Now, from Snell’s law in layer 1 and 3, shown in the above figure.
bases of cylindrical bucket X , Y and Z
we have
respectively. Now, perpendicular distance of weight
n1 sin i = n2 sin r acting through A from point D is
∴ V X = πrX2 × hX 4
⇒ 1 × sin 90° = × sin r l
CD = l cosθ − cosθ
VY = πrY2 × hY 3 2
V Z = πrZ2 × hZ 3
sin r = l
CD = cosθ
V X = π hX [Q rx = 1] 4 2
VY = 4πhY [Q ry = 2] Now, from Pythagoras theorem, we have and perpendicular distance of weight
V Z = 9πhZ [Q rz = 3] ⇒ tan r =
3 acting through B from point D is
At initial stage hX = hY = hZ = h 7 l l
So, from figure, we have BD = − l cosθ = (1 − 2 cosθ)
∴ V X = πh, VY = 4πh, V Z = 9πh x 2 2
tan r =
At second stage some water transfer Z to h At equilibrium torque of these two
X, then volume are equal 6× 3 weights about D must balance each other.
⇒ x = h tan r = = 6.8 m
∴ V X = V Z = 5πh [Q V X + VY = 10πh ] 7 i.e.
l l
mg × cosθ = mg × (1 − 2 cosθ)
At third stage some water is transferred Hence, area of pool visible through glass 2 2
between x and y. πd 2 π × (2x + 0.6)2 3 1 1
slab is A = = ≈ 160 m 2 ⇒ cosθ = ⇒ cosθ =
9 πh
V X = VY = [Q V X + VY = 9πh ] 4 4 2 2 3
2 −1  1 
Volume of water at third stage 67. (a) Let final temperature of mixture or θ = cos  
 3
9 is T °C. Then,
VY = 4 πhY = πh
2 Heat lost by 2 kg water at 90°C to cool 69. (a) From lens equation, we have
and V Z = 9 πhZ = 5 πh down at T°C = Heat gained by 1 kg ice at 1 1 1
9 − =
πh − 20°C to reach at 0°C + Heat gained by v u f
VY 4 πhY 81
= = 2 = 1 kg ice at 0°C to change its state from ice Now, differentiating above equation with
VZ 9 π hZ 5 πh 10 to water + Water 1 kg formed at 0°C is respect to time, we get
hY 81 dv v2  du 
= m2 
⇒ = now absorbs heat to reach temperature of dv du 
=   or 
hZ 40 T °C dt u 2  dt  dt  dt 
https://iit-jeeacademy.blogspot.com

76 KVPY Question Paper 2015 Stream : SA

v 152 So, it is evident that wherever there will


As, = magnification (m) No. of initial moles of F2 = = 4 mol
u 38 be the above sequence in the DNA
⇒ ∆v = m2 ⋅ ∆u …(i) 2 moles of Xe react completely with fragment, Alu I will make blunt cuts over
i.e. if object oscillates with an amplitude 4 moles of F2 to give 1 mol of XeF2 and there.
∆u, then image also oscillates with 1 mol of XeF6. Now according to question, if there is
amplitude ∆v given by i. Thus, the molar ratio of XeF2 : XeF6 is random distribution of bases in the
Also, magnification, 1 : 1. genome, the probability of occurrence of
v  f  72. (a) When ethanol reacts with conc. the above cut sides will be
m= =  …(ii) sulphuric acid at 170°C produces ethene 1 1
u  f + u = [since Alu I is a 4 base
gas which is then treated with bromine in 4 × 4 × 4 × 4 256
Now, in given question, carbon tetrachloride to give pair cutter]
u = − 20 cm, f = 5 cm 1,2-dibromoethane as a major product. 1
f 5 1 Conc. H2SO4 So, the frequency will be .
So, m= = ⇒ m= − CH3CH2OH CH2 CH2 256
f + u 5 − 20 3 Ethanol
170°C
Ethene
(Dehydration 77. (b) The cooking of rice in open
From Eq. (i), we have of alcohol)
CCl4 vessels is favoured at low temperatures
2 Br2 (Addition
∆v =  −  × ∆u
1 reaction) and higher altitudes due to the
 3 CH2—CH2 atmospheric pressure. When the rice is
1 cooked in the pressure cooker, then the
or ∆v = × A [given, ∆u = A] rice will be cooked faster at the sea beach
9 Br Br
1, 2 dibromoethane because the temperature is higher and
As object is placed between ∞ and 2f (major)
pressure is lower at sea level than higher
distance, so on moving object near to
lens, its image moves away from lens. 73. (c) C4 H8 ( g ) + 6O2 ( g ) → 4CO2 ( g ) altitude. This will allow the water to boil
At STP 22.4 L 89.6 L faster inside the pressure cooker and the
So, oscillations of object and image are
+ 4H2O( g ) rice will be cooked faster.
out of phase.
72 L 78. (a) In the absence of disease, natural
70. (a) By Stokes’ law, 72
No. of moles of water at STP = = 4 mol calamities and predation growth of rabbit
F = 6 π η av 18 is exponential. When resources are
F
We have, η= 1 mole of C4H8 burns completely with 6 unlimited, populations exhibit
6 πav moles of O2 to give 4 moles of CO2 and 4 exponential growth, resulting in a
Dimensions of viscosity index η are moles of H2O. J-shaped curve (i.e. option a).
 MLT −2  At STP 1 mole of O2 contains = 22. 4 L
⇒ [ η] =  −1 
= [ML−1 T −1 ] 79. (c) Glucose is a monosaccharide and
 L ⋅ LT  6 moles of O2 contain = 22.4 × 6 = 134.4 L an osmotically active molecule which
74. (a) Given, current, I = 0.5 A increases osmotic pressure in cell. So,
Now, given relation of volume flow rate is
a water enters in cell while glycogen is
Time, t = 1hr = 3600 s
= k   ⋅ ηb ⋅ r c
V p osmotically inert molecule does not
t  l According to Faraday’s IInd law of change the osmotic pressure. This is the
Substituting dimensions of physical electrolysis, reason why glucose is not stored in the
quantities and equating dimensions on Atomic mass × I × t cell instead glycogen is stored in the
W =
both sides of equation, we have 96500 animal body.
[L3 ] 108
= [ML−2T −2 ]a ⋅ [ML−1 T −1 ]b ⋅ [L]c = × 0.5 × 36000 = 2 g 80. (a) There will be four membrane
[T ] 96500 bilayers that the line will cross
⇒[M0 L3 T −1 ] = [Ma + b L− 2a − b + c T − 2a − b ] 75. (a) 1 = Cell membrane
Equating dimensions, we have 2 = Mitochondrion
a+ b= 0 …(i) 1 = Nucleus
− 2a − b + c = 3 …(ii)
− 2a − b = − 1 …(iii)
From Eqs. (ii) and (iii), we have Mitochondrion
c=4
From Eqs. (i) and (iii), we have
The addition of water to alkenes in the 1 2 3 4
b= −1 presence of an acid form alcohols which
Substituting b in Eq. (i), we have occurs through electrophilic addition
a=1 mechanism and follows Markownikoff
Nucleus
So, a = 1, b = − 1 and c = 4. rule.
Hence, option (a) is correct.
71. (c) 2Xe + 4F2 → XeF2 + XeF6
76. (c) Alu I has the cut site 5′ AGCT3′
262
No. of initial moles of Xe = = 2 mol 3′ TCGA5′
Cell membrane
131
https://iit-jeeacademy.blogspot.com

KVPY Question Paper 2014 Stream : SA 77

KVPY
KISHORE VAIGYANIK PROTSAHAN YOJANA

QUESTION PAPER 2014


Stream : SA
MM 100

Instructions
There are 80 questions in this paper.
This question paper contains two parts; Part I and Part II. There are four sections; Mathematics, Physics, Chemistry
and Biology in each part.
Out of the four options given with each question, only one is correct.

PART-I (1 Mark Questions)


MATHEMATICS coefficient of the highest degree term of k(x) is 1,
then sum of the roots of (x − 1) + k(x) is
1. Let r be a root of the equation x + 2x + 6 = 0. The
2
(a) 4 (b) 5 (c) 6 (d) 7
value of (r + 2) (r + 3) (r + 4) (r + 5) is equal to
6. In a quadrilateral ABCD, which is not a trapezium, it
(a) 51 (b) − 51 (c) − 126 (d) 126
is known that ∠DAB = ∠ABC = 60°. Moreover,
2. Let R be the set of all real numbers and let f be a ∠CAB = ∠CBD. Then,
function from R to R such that (a) AB = BC + CD (b) AB = AD + CD
 1 (c) AB = BC + AD (d) AB = AC + AD
f (x) +  x +  f (1 − x) = 1, for all x ∈ R. Then
 2
7. A semi-circle of diameter 1 unit sits at the top of a
2 f (0) + 3 f (1) is equal to semi-circle of diameter 2 units.
(a) 2 (b) 0 (c) − 2 (d) − 4
3. The sum of all positive integers n for which
13 + 23 + ... + (2n )3
is also an integers is
12 + 22 + ... + n 2 1 unit

(a) 8 (b) 9 (c) 15 (d) Infinite


4. Let x and y be two 2-digit numbers such that y is
obtained by reversing the digits of x. Suppose they
2 units
also satisfy x2 − y2 = m2 for some positive integer m.
The shaded region inside the smaller semi-circle but
The value of x + y + m is
outside the larger semi-circle is called a lune. The
(a) 88 (b) 112 (c) 144 (d) 154
area of the lune is
5. Let p(x) = x2 − 5x + a and q(x) = x2 − 3x + b, where a
π 3 3 π 3 π 3 π
and b are positive integers. Suppose HCF (a) − (b) − (c) − (d) −
6 4 4 24 4 12 4 8
( p(x), q(x)) = x − 1 and k(x) = 1 cm ( p(x), q(x)) If the
https://iit-jeeacademy.blogspot.com

78 KVPY Question Paper 2014 Stream : SA

8. The angle bisectors BD and CE of a ∆ ABC are di- 13. The number of 6-digit numbers of the form ababab
vided by the incentre I in the ratios 3 : 2 and 2 : 1 (in base 10) each of which is a product of exactly 6
respectively. Then, the ratio in which I divides the distinct primes is
angle bisector through A is (a) 8 (b) 10 (c) 13 (d) 15
(a) 3 : 1 (b) 11 : 4 14. The houses on one side of a road are numbered using
(c) 6 : 5 (d) 7 : 4 consecutive even numbers. The sum of the numbers
9. Suppose S1 and S2 are two unequal circles, AB and of all the houses in that row is 170. If there are at
CD are the direct common tangents to these circles. A least 6 houses in that row and a is the number of the
transverse common tangent PQ cuts AB in R and CD sixth house, then
in S. If AB = 10, then RS is (a) 2 ≤ a ≤ 6 (b) 8 ≤ a ≤ 12
A (c) 14≤ a ≤ 20 (d) 22≤ a ≤ 30

R 15. Suppose a 2, a3 , a 4 , a5 , a 6 , a7 are integers such that


B
Q 5 a 2 a3 a 4 a5 a 6 a7
= + + + + + ,
7 2! 3! 4! 5! 6! 7!
where 0 ≤ a j < j for j = 2, 3, 4, 5, 6, 7. The sum of
P
D a 2 + a3 + a 4 + a5 + a 6 + a7 is
S (a) 8 (b) 9 (c) 10 (d) 11
C
(a) 8 (b) 9 (c) 10 (d) 11 PHYSICS
10. On the circle with center O, points A and B are such
16. In the following displacement x versus time t graph,
that OA = AB. A point C is located on the tangent at
at which among the points P, Q and R is the object’s
B to the circle such that A and C are on the opposite speed increasing?
sides of the line OB and AB = BC. The line segment
x
AC intersects the circle again at F. Then, the ratio
P
∠BOF : ∠BOC is equal to
Q
A
(0, 0) t
R

O (a) R only (b) P only


(c) Q and R only (d) P, Q and R 17.
F A box when hung from a spring balance shows a
reading of 50 kg. If the same box is hung from the
same spring balance inside an evacuated chamber,
C
the reading on the scale will be (a)
(a) 1 : 2 (b) 2 : 3 (c) 3 : 4 (d) 4 : 5
50 kg because the mass of the box remains unchanged.
11. In a cinema hall, the charge per person is ` 200. On (b) 50 kg because the effect of the absence of the at-
the first day, only 60% of the seats were filled. The mosphere will be identical on the box and the spring
owner decided to reduce the price by 20% and there balance
was an increase of 50% in the number of spectators (c) less than 50 kg because the weight of the column of air
on the next day. The percentage increase in the on the box will be absent
revenue on the second day was (d) more than 50 kg because the atmospheric buoyancy
(a) 50 (b) 40 (c) 30 (d) 20 force will be absent
12. The population of cattle in a farm increases so that 18. Two positively charged spheres of masses m1 and m2
the difference between the population in year n + 2 are suspended from a common point at the ceiling by
and that in year n is proportional to the population in identical insulating massless strings of length l.
year n + 1. If the populations in years 2010, 2011 and Charges on the two spheres are q1 and q2,
2013 were 39, 60 and 123, respectively,then the respectively. At equilibrium, both strings make the
population in 2012 was same angle θ with the vertical. Then
(a) 81 (b) 84 (c) 87 (d) 90 (a) q1 m1 = q2m2 (b) m1 = m2
(c) m1 = m2 sinθ (d) q2m1 = q1 m2
https://iit-jeeacademy.blogspot.com

KVPY Question Paper 2014 Stream : SA 79

19. A box when dropped from a certain height reaches 26. A bar magnet falls with its north pole pointing down
the ground with a speed v. When it slides from rest through the axis of a copper ring. When viewed from
from the same height down a rough inclined plane above, the current in the ring will be (a) clock-
inclined at an angle 45° to the horizontal, it reaches wise, while the magnet is above the plane of the
the ground with a speed v / 3. The coefficient of ring and counter clockwise, while below the plane of
the ring
sliding friction between the box and the plane is
(b) counter clockwise throughout (c) coun-
(Take, acceleration due to gravity is 10 ms −2)
8 1 2 1 ter clockwise, while the magnet is above the plane
(a) (b) (c) (d) of the ring and clockwise, while below the plane of the
9 9 3 3
ring
20. A thin paper cup filled with water does not catch fire (d) clockwise throughout 27.
when placed over a flame. This is because (a) Two identical bar magnets are held perpendicular to
the water cuts off oxygen supply to the paper cup each other with a certain separation, as shown below.
(b) water is an excellent conductor of heat (c) The area around the magnets is divided into four
the paper cup does not become appreciably hotter than zones.
the water it contains
(d) paper is a poor conductor of heat
21. Ice is used in a cooler in order to cool its contents.
Which of the following will speed up the cooling N S
process?
(a) Wrap the ice in a metal foil N
(b) Drain the water from the cooler periodically
S
(c) Put the ice as a single block
(d) Crush the ice
22. The angle of a prism is 60°. When light is incident at Given that there is a neutral point it is located in
an angle of 60° on the prism, the angle of emergence (a) zone I (b) zone II (c) zone III (d) zone IV
is 40°. The angle of incidence i for which the light ray
28. A large number of random snap shots using a camera
will deviate the least is such that
are taken of a particle in a simple harmonic motion
(a) i < 40° (b) 40° < i < 50°
(c) 50°< i < 60° (d) i > 60° between x = − x0 and x = + x0 with origin x = 0 as the
mean position. A histogram of the total number of
23. A concave lens made of material of refractive index times the particle is recorded about a given position
1.6 is immersed in a medium of refractive index 2.0. (Event no.) would most closely resemble.
The two surfaces of the concave lens have the same
radius of curvature 0.2 m. The lens will behave as a
Event no.

Event no.
(a) divergent lens of focal length 0.4 m (a) (b)
(b) divergent lens of focal length 0.5 m
(c) convergent lens of focal length 0.4 m
(d) convergent lens of focal length 0.5 m
x x
24. A charged particle initially at rest at –x0 0 x0 –x0 0 x0
O,when released follows a trajectory as
shown alongside. Such a trajectory is O
Event no.

Event no.

possible in the presence of (a) elec- (c) (d)


tric field of constant magnitude and varying
direction
(b) magnetic field of constant magnitude and varying
direction –x0 0 x0
x
–x0 0 x0
x
(c) electric field of constant magnitude and constant
direction 29. In 1911, the physicist Ernest Rutherford discovered
(d) electric and magnetic fields of constant magnitudes and that atoms have a tiny, dense nucleus by shooting
constant directions which are parallel to each other positively charged particles at a very thin gold foil. A
25. Two equal charges of magnitude Q each are placed at key physical property which led Rutherford to use
gold was that it was
a distance d apart. Their electrostatic energy is E. A
(a) electrically conducting (b) highly malleable
third charge −Q / 2 is brought midway between these
(c) shiny (d) non-reactive
two charges. The electrostatic energy of the system is
30. Consider the following statements:
now
I. All isotopes of an element have the same number of
(a) − 2E
(b) − E (c) 0 (d) E neutrons.
https://iit-jeeacademy.blogspot.com

80 KVPY Question Paper 2014 Stream : SA

II. Only one isotope of an element can be stable and 39. At room temperature, the average speed of helium is
non-radioactive. higher than that of oxygen by a factor of
III. All elements have isotopes. (a) 2 2 (b) 6 / 2 (c) 8 (d) 6
IV. All isotopes of carbon can form chemical compounds
with oxygen-16. 40. Ammonia is not produced in the reaction of
Choose the correct option regarding an isotope. (a) NH4Cl with KOH (b) AlN with H2O
(c) NH4Cl with NaNO2 (d) NH4Cl with Ca(OH)2
(a) Statements III and IV are correct
(b) Statements II, III and IV are correct 41. The number of isomers which are ethers and having
(c) Statements I, II and III are correct the molecular formula C 4 H10O, is
(d) Statements I, III and IV are correct (a) 2 (b) 3 (c) 4 (d) 5
42. The major product of the reaction of 2-butene with
alkaline KMnO4 solution is
CHEMISTRY
31. The isoelectronic pair is (a) (b)
(a) CO, N 2 (b) O2 , NO (c) C 2 , HF (d) F2 , HCl O OH

32. The numbers of lone pairs and bond pairs in O


hydrazine are, respectively (c) (d)
(a) 2 and 4 (b) 2 and 6 (c) 2 and 5 (d) 1 and 5 OH
33. The volume of oxygen at STP required to burn 2.4 g OH
of carbon completely is 43. Among the compounds I-IV, the compound having
(a) 1.12 L (b) 8.96 L (c) 2.24 L (d) 4.48 L the lowest boiling point is
34. The species that exhibits the highest Rf value in a HO
thin layer chromatogram using a non-polar solvent OH OH
on a silica gel plate is I II

O OH
OH
(a) (b)
III IV
N (a) I (b) II (c) III (d) IV
44. Of the following reactions
(c) (d)
+ (i) A r B, ∆G° = 250 kJ mol −1
N
(ii) D r E, ∆G° = − 100 kJ mol −1
(iii) F r G, ∆G° = − 150 kJ mol −1
OH
35. The number of C—C sigma bonds in the compound (iv) M r N , ∆G° = 150 kJ mol −1
O
The reaction with the largest equilibrium constant is
(a) (i) (b) (ii) (c) (iii) (d) (iv)
is 45. The first ionisation enthalpies for three elements are
1314, 1680 and 2080 kJ mol −1, respectively. The cor-
(a) 16 (b) 17 (c) 18 (d) 11 rect sequence of the elements is
36. If the radius of the hydrogen atom is 53 pm, the (a) O, F and Ne (b) F, O and Ne
radius of the He + ion is closest to (c) Ne, F and O (d) F, Ne and O
(a) 108 pm (b) 81 pm
(c) 27 pm (d) 13 pm BIOLOGY
37. The diamagnetic species is 46. Individuals of one kind occupying a particular
(a) NO (b) NO2 (c) O2 (d) CO2 geographic area at a given time are called
38. The pH of 0.1 M aqueous solutions of NaCl, (a) community (b) population
CH COONa
(a) 3NaCl andCOONa
< CH NH4Cl <will
NHfollow
Cl the order (c) species (d) biome
3 4

(b) NH4 Cl < NaCl < CH3 COONa 47. What fraction of the assimilated energy is used in
(c) NH4 Cl < CH COONa < NaCl respiration by the herbivores?
3 (a) ~ 10 per cent (b) ~ 60 per cent
(d) NaCl < NH4 Cl < CH3 COONa (c) ~ 30 per cent (d) ~ 80 per cent
https://iit-jeeacademy.blogspot.com

KVPY Question Paper 2014 Stream : SA 81

48. Athletes are often trained at high altitude because 55. The auditory nerve gets its input from which of the
(a) training at high altitude increases muscle mass (b) following?
training at high altitude increases the number of red (a) The sense cells of the cochlea
blood cells (b) Vibration of the last ossicle
(c) there is less chance of an injury at high altitude (c) Eustachian tube
(d) athletes sweat less at high altitude (d) Vibration of the tympanic membrane
49. In human brain, two cerebral hemispheres are 56. Which of the following organelles contain circular
connected by a bundle of fibres which is known as DNA?
(a) medulla oblongata (b) cerebrum (a) Peroxisomes and mitochondria
(c) cerebellum (d) corpus callosum 50. (b) Mitochondria and Golgi complex
Which one of the following hormones is produced by (c) Chloroplasts and lysosomes
the pancreas? (d) Mitochondria and chloroplast
(a) Prolactin (b) Glucagon 57. A reflex action does not involve
(c) Luteinising hormone (d) Epinephrine (a) neurons (b) brain
51. The stalk of a plant leaf is derived from which one of (c) spinal cord (d) muscle fibre
the following types of plant tissue?
58. Which one of the following options is true in
(a) Sclerenchyma (b) Parenchyma photosynthesis?
(c) Chlorenchyma (d) Collenchyma 52. (a) CO2 is oxidised and H 2O is reduced
Which of the following muscle types cannot be used (b) H 2O is oxidised and CO2 is reduced
voluntarily? (c) Both CO2 and H 2O are reduced
(a) Both striated and smooth (d) Both CO2 and H 2O are oxidised
(b) Both cardiac and striated 59. Human mature Red Blood Cells (RBCs) do not contain
(c) Both smooth and cardiac (a) iron (b) cytoplasm
(d) Cardiac, striated and smooth (c) mitochondria (d) haemoglobin

53. The pulmonary artery carries 60. A person was saved from poisonous snake bite by
(a) deoxygenated blood to the lungs anti-venom injection.
(b) oxygenated blood to the brain Which of the following immunities explains this form
(c) oxygenated blood to the lungs of protection?
(a) Naturally acquired active immunity
(d) deoxygenated blood to the kidney
(b) Artificially acquired active immunity
54. Both gout and kidney stone formation
(b) uric acid is caused by (c) Naturally acquired passive immunity
(a) creatinine
(c) calcium oxalate (d) potassium chloride (d) Artificially acquired passive immunity

PART-II (2 Marks Questions)


MATHEMATICS (a) 0 (b) 1007 (c) 2013 (d) 2014

61. Let a , b, c be non-zero real numbers such that 64. In a ∆ ABC with ∠A = 90°, P is a point on BC such
a + b + c = 0, let q = a + b + c and r = a + b + c .
2 2 2 4 4 4 that PA : PB = 3 : 4. If AB = 7 and AC = 5, then
BP : PC is
Then,
(a) 2 : 1 (b) 4 : 3 (c) 4 : 5 (d) 8 : 7
(a) q2 < 2r always
(b) q2 = 2r always 65. The number of all 3-digit numbers abc (in base 10) for
(c) q2 > 2r always which (a × b × c) + (a × b) +
(d) q2 − 2r can take both positive and negative values (b × c) + (c × a ) + a + b + c = 29 is
1947 (a) 6 (b) 10 (c) 14 (d) 18
1
62. The value of ∑ is equal to
n =0 2n + 21947
487 1946 1947 1948 PHYSICS
(a) (b) (c) (d)
1945 1947 1947 1947
2 2 2 2 66. A uniform square wooden sheet of side a has its
centre of mass located at point O as shown in the
63. The number of integers a in the interval [1, 2014] for
figure below on the left. A square portion of side b of
which the system of equations x + y = a ,
this sheet is cut out to produce an L-shaped sheet as
x2 y2
+ = 4 has finitely many solutions is shown in the figure on the right.
x−1 y−1
https://iit-jeeacademy.blogspot.com

82 KVPY Question Paper 2014 Stream : SA

a b If the refractive index of the material of the prism is


µ, then
(a) µ > 5 (b) 3 < µ < 5
(c) 2 < µ < 3 (d) µ < 2
70. Consider the circuit shown below where all resistors
O O are 1 kΩ.
P P

X
Q
The centre of mass of the L-shaped sheet lies at the
point P (in the above diagram), when If a current of magnitude 1 mA flows through the
(a) a / b = ( 5 − 1) / 2 (b) a / b = ( 5 + 1) / 2 resistor marked X, what is the potential difference
(c) a / b = ( 3 − 1) / 2 (d) a / b = ( 3 + 1) / 2 measured between points P and Q?
(a) 21 V (b) 68 V
67. A machine is blowing spherical soap bubbles of (c) 55 V (d) 34 V
different radii filled with helium gas.It is found that,
if the bubbles have a radius smaller than 1 cm, then
they sink to the floor in still air. Larger bubbles float CHEMISTRY
in the air. Assume that the thickness of the soap film 71. 10 moles of a mixture of hydrogen and oxygen gases
in all bubbles is uniform and equal. Assume that the at a pressure of 1 atm at constant volume and
density of soap solution is same as that of water temperature, react to form 3.6 g of liquid water. The
(= 1000 kg m −3 ). The density of helium inside the pressure of the resulting mixture will be closest to
bubbles and air are 0.18 kg m−3 and 1.23 kg m−3 , (a) 1.07 atm (b) 0.97 atm
respectively. Then, the thickness of the soap film of (c) 1.02 atm (d) 0.92 atm
the bubbles is (Note 1 µm = 10−6 m) 72. The ammonia evolved from 2 g of a compound in
(a) 0.50 µm (b) 1.50 µm Kjeldahl’s estimation of nitrogen neutralises
(c) 7.00 µm (d) 3.50 µm 10 mL of 2 M H2 SO4 solution. The weight percentage
of nitrogen in the compound is
68. An aluminium piece of mass 50 g initially at 300°C is (a) 28 (b) 14
dipped quickly and taken out of 1 kg of water, (c) 56 (d) 7
initially at 30°C. If the temperature of the
aluminium piece immediately after being taken out 73. Complete reaction of 2.0 g of calcium (at. wt. = 40) with
of the water is found to be 160°C, what is the excess HCl produces 1.125 L of H2 gas. Complete re-
temperature of the water? Then, specific heat action of the same quantity of another metal M with
capacities of aluminium and water are 900 Jkg −1K −1 excess HCl produces 1.85 L of H2 gas under identical
and 4200 Jkg −1K −1, respectively. conditions. The equivalent weight of M’ is closest to
(a) 165°C (b) 45°C (c) 31.5°C (d) 28.5°C (a) 23 (b) 9
(c) 7 (d) 12
69. A ray of light incident parallel to the base PQ of an
isosceles right-angled triangular prism PQR suffers 74. A compound X formed after heating coke with lime
two successive total internal reflections at the faces reacts with water to give Y which on passing over
PQ and QR before emerging reversed in direction as red-hot iron at 873 K produces Z. The compound Z is
shown below.
(a) (b)
R

(c) (d)

75. In the following reaction sequence,


Br
Br 1. Alc. KOH 3. HgSO4/dil, H 2SO 4, Heat
2. NaNH2
X 4. Conc. HNO /H SO
Y
Q P Ph
3 2

4
https://iit-jeeacademy.blogspot.com

KVPY Question Paper 2014 Stream : SA 83

X and Y respectively are, 77. A woman heterozygous for colourblindness marries a


O colourblind man. What would be the ratios of carrier
daughters, colourblind daughters, normal sons and
colourblind sons in the F1 -generation?
(a) Ph C C H and (a) 1 : 2 : 2 : 1
NO2 (b) 2 : 1 : 1 : 2
O (c) 1 : 1 : 1 : 1
NH2 (d) 1 : 1 : 2 : 2
OH
(b) NH2 and 78. Two semipermeable bags containing 2% sucrose are
Ph placed in two beakers, ‘P’ containing water and ‘Q’
NO2 containing 10% sucrose. Which one of the following
H2N outcomes is true?
O (a) Bag in ‘P’ becomes flaccid due to exosmosis
NH2
(b) Bag in ‘P’ becomes turgid due to endosmosis
(c) OH
and (c) Bag in ‘Q’ becomes turgid due to endosmosis
Ph (d) Concentration of sucrose remains unchanged in both

NO2 79. Children suffering from phenylketonuria are given


O food low in phenylalanine and supplemented with
tyrosine. This is because they
OH
(d) NH2 (a) are unable to utilise phenylalanine
and (b) do not require phenylalanine
Ph
(c) have increased tyrosine anabolism
(d) have increased tyrosine catabolism
NO2
80. Two bottles were half-filled with water from Ganga
(‘P’) and Kaveri (‘Q’) and kept under identical airtight
BIOLOGY conditions for 5 days. The oxygen was determined to
be 2% in bottle (‘P’) and 10% in bottle (‘Q’). What
76. In which of the following cellular compartments do could be the cause of this difference?
respiratory reactions occur? (a) Ganga is more polluted than Kaveri
(a) Cytoplasm and endoplasmic reticulum (b) Both the rivers are equally polluted
(b) Mitochondria and Golgi complex (c) Kaveri is more polluted than Ganga
(c) Mitochondria and cytoplasm (d) Kaveri has more minerals than Ganga
(d) Only mitochondria

Answers
PART-I
1 (c) 2 (c) 3 (a) 4 (d) 5 (d) 6 (c) 7 (b) 8 (b) 9 (c) 10 (b)
11 (d) 12 (b) 13 (c) 14 (c) 15 (b) 16 (a) 17 (d) 18 (b) 19 (a) 20 (c)
21 (d) 22 (b) 23 (d) 24 (a) 25 (b) 26 (c) 27 (a) 28 (c) 29 (b) 30 (a)
31 (a,d) 32 (c) 33 (d) 34 (a) 35 (b) 36 (c) 37 (d) 38 (b) 39 (a) 40 (c)
41 (b) 42 (d) 43 (c) 44 (c) 45 (a) 46 (b) 47 (c) 48 (b) 49 (d) 50 (b)
51 (d) 52 (c) 53 (a) 54 (b) 55 (a) 56 (d) 57 (b) 58 (b) 59 (c) 60 (d)

PART-II
61 (b) 62 (a) 63 (d) 64 (a) 65 (d) 66 (b) 67 (d) 68 (c) 69 (a) 70 (d)
71 (b) 72 (a) 73 (d) 74 (a) 75 (a) 76 (c) 77 (c) 78 (b) 79 (a) 80 (a)
https://iit-jeeacademy.blogspot.com

84 KVPY Question Paper 2014 Stream : SA

Solutions
1. (c) We have, r be root of the equation 4. (d) We have, ∆AEB is an equilateral.
x2 + 2x + 6 = 0 x and y be two-digit numbers. Q AB = BE = AE
Q r 2 + 2r + 6 = 0 Let x = 10a + b, where b is units place and E
r = − (2r + 6)
2 a is ten’s place.
60°
Now, (r + 2) (r + 3) (r + 4) (r + 5) Q y = 10b + a C
= (r 2 + 5r + 6) (r 2 + 9r + 20) x2 − y2 = m2 D

= (− 2r − 6 + 5r + 6) (− 2r − 6 + 9r + 20) Q (10a + b)2 − (10b + a )2 = m2


60°
⇒ (10a + b + 10b + a )
= (3r ) (7r + 14) = 21 (r 2 + 2r ) 60°
(10a + b − 10b − a ) = m2 A B
= 21 × (− 6) [Q r 2 + 2r = − 6] In ∆BDE and ∆ABC,
⇒ 11(a + b) ⋅ 9 (a − b) = m2
= − 126 ∠BED = ∠ABC
⇒ 99(a 2 − b2 ) = m2
⇒ ∠DBE = ∠CAB given,
2. (c) Given, ⇒ 3 × 11 (a 2 − b2 ) = m2
2
[Q∠DBE = ∠DBC ]
⇒ f (x) +  x +  f (1 − x) = 1 Now, 3 × 11(a 2 − b2 ) is a perfect square.
1 2
… (i) Q ∆BED ~ ∆ABC
 2 Q a 2 − b2 = 11 ⇒ (a + b)(a − b) = 11 × 1 BE BD ED BE ED
Q = = ⇒ =
Put x = 1 − x, we get a + b = 11and a − b = 11 AB AC BC AB BC
On solving, we get a = 6, b = 5 AB AE − AD
f (1 − x) +  1 − x +  f (1 − (1 − x) = 1
1 ⇒ =
 2 Q Number x = 60 + 5 = 65 AB BC
⇒ AE − AD = BC ⇒ AB = AD + BC
⇒ f (1 − x) +  − x f (x) = 1 y = 56
3
…(ii)
2  and m2 = (65)2 − (56)2 = (65 + 56) (65 − 56) 7. (b) Area of lune = Area of semi-circle
ACBD − Area of segment AEBA
Eq. (ii) multiply by  x +  , we get
1 ⇒ m2 = 121 × 9 ⇒ m = 33
 2 C
Q x + y + m = 65 + 56 + 33 = 154
 − x   x +  f (x ) +  x + 1 
3 1
      5. (d) We have,
2   2  2
p (x) = x2 − 5x + a and q(x) = x2 − 3x + b
1 B E A
f (1 − x) = x + …(iii) Given, (x − 1) is HCF of p (x) and q(x). 1D
2
Q p(1) = 0 and q(1) = 0
On subtracting Eq. (iii) from Eq. (i), we get 1 1
Q p (1) = 0 = 1 − 5 + a and q(1) = 0
f (x) 1 −  + x − x2   = 1 − x −
3 1
  4 = 1− 3 + b
  2
60°
⇒ a = 4 and b = 2
1
−x Q p (x) = x2 − 5x + 4 and q(x) = x2 − 3x + 2 1 O 1
⇒ f (x ) = 2 ⇒ f (0) = 2 and f (1) = − 2 2
1 ⇒ p (x) = (x − 1) (x − 4) 1  1
2
 60° 3 2
x2 − x + = π  −  × π × (1)2 − () 
4 and q(x) = (x − 1) (x − 2) 2  2  360 1 
Q 2f (0) + 3f (1) − 2(2) + 3 (− 2) = 4 − 6 = − 2 LCM of p (x) and q(x) = k (x) 4
π  π 3  π π 3 3 π
p (x) ⋅ q(x) = − − = − + = −
3. (a) Given,
13 + 23 + 33 + ... + (2n )3 Q k (x ) = 8 6 4  8 6 4 4 24
12 + 22 + 32 + ... + n 2 HCF of p (x) and q(x)
(x − 1) (x − 4)⋅ (x − 1) (x − 2) 8. (b) Given,
(2n )2 (2n + 1)2  n 2 (n + 1)2  k (x ) =
Q Σn =
3
 x−1 In ∆ABC
= 4  4  A
n (n + 1)(2n + 1) k (x) = (x − 1) (x − 2) (x − 4)
 Σn 2 = n ( n + 1)( 2n + 1) 
6   Now, x − 1 + k (x)
6
= x − 1 + (x − 1) (x − 2) (x − 4)
4n 2 (2n + 1)2
= (x − 1) (1 + x2 − 6x + 8)
4 6n (2n + 1) D
= = = (x − 1) (x − 3) (x − 3)
E
n (n + 1)(2n + 1) n+1 1
2
6 Q Roots of x − 1 + k (x) are 1, 3, 3. b
I
12n 2 + 6n 6 Sum of roots are 1 + 3 + 3 = 7
= = (12n − 6) + 3 2
n+1 n+1 6. (c) ABCD is a quadrilateral
Q
6
is an integer if n + 1is factor of 6. ∠DAB = ∠ABC = 60°
B F C
n+1 and ∠CAB = ∠CBD The angle bisector BD and CE are divided
Q n + 1 = 1, 2, 3, 6 ⇒ n = 1, 2, 5 Construction, AD and BC produced to by incentre I in the ratio 3 : 2 and 2 : 1
Sum of n = 1 + 2 + 5 = 8 meet at such that respectively.
https://iit-jeeacademy.blogspot.com

KVPY Question Paper 2014 Stream : SA 85

BI 3
Q = 10. (b) Given, 12. (b) Given,
ID 2 OA = OB = OF radii of circle Population in year 2010, 2011 and 2013
CI 2 were 39, 60 and 123 respectively.
and = BC is tangent on circle at B
IC 1 A According to problems,
AI b + c BI a + c
We know, = , = , The population of cattle in farm increases
IF a ID b such that difference between in year
CI a + b n + 2 and that in year n is proportional to
=
IE c the year n + 1.
B
BI a + c 3 Q (n + 2) − (n ) = k (n + 1)
Q = = ⇒ 2(a + c) = 3b …(i) O
ID b 2 Q Let population in year 2012 = x
CI a + b 2 F
and = = ⇒ a + b = 2c …(ii) Year Population
IE c 1 2010 39
On solving Eqs. (i) and (ii), we get C
2011 60
3 5
b = a and c = a 2012 x
2 4 AB = BC
2013 123
3 5 OA = AB
a+ a x − 39
AI b + c 2 4 = 11 In ∆AOB, =k … (i)
= =
Q
Q 60
IF a a 4 AO = OB = AB
123 − 60
Hence, ratio = 11 : 4. Q ∠ABO = 60° and =k … (ii)
x
9. (c) Given, [Q ABC is an equilateral triangle] From Eqs. (i) and (ii), we get
AB and CD are direct common tangents In ∆ABC, x − 39 123 − 60
on circle PQ is transverse common =
AB = BC 60 x
tangent PQ cuts AB in R and CD in S.
Q ∠BAC = ∠BCA ⇒ x2 − 39x − 3780 = 0
A ⇒ ∠BAC + ∠BCA + ∠ABC = 180° ⇒ (x − 84) (x + 40) = 0
⇒ 2∠BAC = 180° − 90° − 60° ∴ x = 84
R
B ⇒ ∠BAC = 15°
Q 13. (c) We have,
In ∆OBC, OB = OC 6-digits number are ababab .
Q ∠BOC = ∠BCO = 45° Q ababab = 105 a + 104 b + 103 a + 102 b
D ⇒ ∠BOF = 2∠BAF + 10a + b
P
[Q angle subtend on centre of circle is = (105 + 103 + 10) a + (104 + 102 + 1) b
S
C twice the angle subtend an arc of circle] = (104 + 102 + 1) (10a + b)
∠BOF = 30° = (10000 + 100 + 1) (10a + b)
AB = 10 ∠BOF 30° 2 = (10101) (10a + b)
Q = =
RP = RA ∠BOC 45° 3 = 3 × 7 × 13 × 37 (10a + b)
[Q tangents from external point 11. (d) Let total seats = x Since, 6-digit number are product of
on a circle are equal] Ticket price of each seat = ` 200 exactly 6 primes.
Similarly On first day 60% of seats over filled Q10a + b is product of 2 primes,
RQ = RB 60 10a + b is lie between 10 to 99.
Q Total revenue = x × 200 = 120x
SP = SC 100 Q 10a + b = 10 = 2 × 5
RS = SP + PQ + RQ On second day 22 = 2 × 11
RS = SP + RP Ticket price = 200 − 20% of 200 = 160 34 = 2 × 17
RS = SP + RA Total seat filled on 2nd day 38 = 2 × 19
RS = SP + AB − RB …(i) =
60
x+
50
×
60 x 90x
= 46 = 2 × 23
Also, SQ = SD
100 100 100 100 55 = 5 × 11
RS − QR = CD − CS
Total revenue on 2nd day 58 = 2 × 29
90 x 62 = 2 × 31
RS = QR + CD − CS = × 160 = 144x
100 74 = 2 × 37
RS = RB + AB − SP …(ii)
Percentage increase in revenue on 2nd 82 = 2 × 41
From Eqs. (i) and (ii), we get day
SP = RB 85 = 5 × 17
144x − 120x 
=   × 100 94 = 2 × 47
Q RS = SP + AB − RB  120x 
RS = AB = 10 24 95 = 5 × 19
= × 100 = 20%
120 Q 13, 6-digits number.
https://iit-jeeacademy.blogspot.com

86 KVPY Question Paper 2014 Stream : SA

14. (c) Let the number of houses be 16. (a) In given displacement-time 19. (a) When box is dropped from height
x, x + 2, x + 4, x + 6, x + 8, x + 10, ... graph, velocity at a particular point is h, its speed when it reaches the ground is
6th number of house is a. given by the slope of tangent to curve v= 2 gh
drawn at that point.
Q x + 10 = a ⇒ x = a − 10 When block slides down the inclined
Speed is the magnitude of velocity, so
∴ x > 10 plane θ = 45°,
magnitude of slope gives speed.
n
Now, Sn = (2x + (n − 1)2) x f
2 θ1= 0°
Sn = n (x + n − 1) P θ2
⇒ 170 = n (a − 10 + n − 1) θ h
Q θ3 sin
⇒ n 2 + (a − 11)n − 170 = 0 mg

(a − 11) ± (a − 11)2 + 680 R


θ=45°
⇒ n=−
2 As angle of tangent at R(θ3 ) is maximum,
(11 − a ) ± (a − 11)2 + 680 so slope’s magnitude|m| = |tan θ3|is Net downward acceleration of block is
⇒ n= mg sinθ − f
2 maximum at R. Hence, speed is a=
increasing at point R. m
n≥6
17. (d) In an evacuated chamber, where, f = friction force.
(11 − a ) ± (a − 11)2 + 680
≥6 buoyant force of air is absent, so reading mg sin θ − µmg cosθ
Q
2 ⇒ a=
of spring balance is more than 50 kg. m
800
⇒ a≤ ≤ 33.33 18. (b) In given situation, forces on each = g (sin θ − µ cosθ)
24 of charged sphere are g
Q 12 ≤ a ≤ 32 (i) gravitational pull (mg ) = (1 − µ)
2
a = 12, 14, 16, 18, … (ii) electrostatic repulsion  12 2 
kq q
Q sin θ = cosθ = 1 , when θ = 45°
When, a = 18, n = 10, then Sn = 170  r   
2
Q a = 18 (iii) tension of string (T )
Velocity of block when it reaches bottom
15. (b) We have, as shown below. of inclined plane is
a2 , a3 , a4 , a5 , a6 , a7 are integers.
v′ = 2as
5 a2 a3 a a a a θ
= + + 4 + 5 + 6 + 7 T1 θ T2 where, s = slope length of inclined plane.
7 2! 3! 4! 5! 6! 7! kq1q2 kq1q2
r2 r2 ⇒ v′ = 2 ah / sinθ
and 0 ≤ a j < j m1 m2
5 2520a 2 + 840a 3 + 210a 4 + 42a 5 + 7a 6 + a7  2 gh (1 − µ ) ×
= r =  2 
7 7!  2 
m 1g m2 g
⇒ 3600 = 2520a2 + 840a3 + 210a4
If we resolve tension in horizontal and = 2 gh (1 − µ )
+ 42a5 + 7a6 + a7 vertical directions, we have following v
= (given)
0 ≤ aj < j situation in equilibrium. 3
Q a2 = 1 T cos θ 1
T So, 2 gh (1 − µ ) = 2 gh
a3 ∈ {1, 2} 3
If a3 = 2, then 2520 + (840) × 2 > 3600 θ 1
⇒ 1− µ =
Q a3 must be 1 kq1q2 9
T sin θ r2 1 8
a4 ∈ {1, 2, 3} ⇒ µ = 1− =
9 9
If a4 = 2, then 2520 + 840 + 210(2) > 3600
Q a4 must be 1 mg
20. (c) Thermal resistance of thin layer
of paper is quite less, so heat reaches
Q 3600 = 2520 + 840 + 210 + 42a5 So, T sinθ =
kq1 q2
and T cosθ = mg across the paper and water absorbs that
+ 7a6 + a7 r2
heat. Temperature of paper does not rises
⇒ 30 = 42a5 + 7a6 + a7 ⇒ tanθ =
kq1 q2
beyond 100°C and upto its burning
a5 ∈ {1, 2, 3, 4} r 2 ⋅ mg
temperature.
If a5 = 1 As angle θ is same for both spheres, we
30 < 42 + 7a6 + a7 ∴ Paper cup does not catches fire.
have
Q a5 = 0 tan θ1 = tan θ2
21. (d) When ice is crushed, total surface
Put a5 = 0, then 30 = 7a6 + a7 kq1 q2 kq q area of ice that comes in contact with
or = 21 2 surrounding air increases. As a result
Q a6 = 4 and a7 = 2 2
r m1 g r m2 g crushing the ice speed up the cooling
Q a2 + a3 + a4 + a5 + a6 + a7 ⇒ m1 = m2 process.
= 1+ 1+ 1+ 0 + 4 + 2= 9
https://iit-jeeacademy.blogspot.com

KVPY Question Paper 2014 Stream : SA 87

22. (b) Graph of deviation δ versus angle kQ 2 kQ 2 / 2 kQ 2 / 2 As gold is very malleable, it is possible to
=− −
of incidence i is as shown below (for an d d /2 d /2 produce a foil which is only few atoms
3 thick.
equilateral glass, µ = prism). kQ 2
2 =− =−E
d 30. (a) Isotopes have same number of
δ [from Eq. (i)] protons. All isotopes of few elements are
stable and non-radioactive. Also, all
26. (c) Direction of current in ring is
isotopes of few elements are unstable and
given by Lenz’s law,
radioactive. All elements have isotopes.
40° S All isotopes of carbon can form
δm=38° compounds with oxygen.
N
Current flows A north pole
So, only statements III and IV are correct.
i anti-clockwise in appears
40° 45° 50° 60°
ring when on this face
31. (a, d) Isoelectronic species are those
Clearly, angle of incidence for least viewed from of ring species, which have same number of
above
deviation lies between 40° < i < 50°. electrons. The total number of electrons
23. (d) In air, focal length f of concave in each pair given in the options are as
follows
lens is given by When viewed
(a) CO, N2
1  1 1 from above
= (µ − 1)  −  Current flows current appears No. of electrons in CO = 6 + 8 = 14
fair  R1 R2  anti-clockwise in clockwise
ring when No. of electrons in N 2 = 7 + 7 = 14
Here, nga = 16. , R1 = − 0.2 m viewed from North pole
(b) O2, NO
bottom appears on
and R2 = + 0.2 m this face of No. of electrons in O2 = 8 + 8 = 16
− 2  − 0.6 × 2 ring
. − 1) 
1 No. of electrons in NO = 7 + 8 = 15
∴ = (16 = S
fair  0.2  0.2 (c) C2, HF
1 N
⇒ fair = − m No. of electrons in C 2 = 6 + 6 = 12
6 No. of electrons in HF = 1 + 9 = 10
27. (a) Neutral point appears in region
When this lens is dipped in a medium of in which fields of magnets are in opposite (d) F2, HCl
refractive index nea = 2.0, then directions. No. of electrons in F2 = 9 + 9 = 18
fliquid nga − 1 nga − 1
= = No. of electrons in HCl = 1 + 17 = 18
fair nge − 1  nga 
 − 1 Thus, CO and N 2 , F2 and HCl are
 nea  isoelectronic pairs.
fe 1. 6 − 1 f 0.6 32. (c) The molecular formula of
⇒ = ⇒ e = N S
fa  1.6
− 1 fa 0.8 − 1 hydrazine is NH 2 NH 2 .

 2  I II
1 0.6 = From the structure, it is clear that it has
⇒ fe = × = m 0.5 m 2 lone pairs and 5 bond pairs (4 N — H
6 0.2 1 Magnetic fields are in opposite
directions in region I and 1 N–N).
Hence, lens acts like a convergent lens
of 0.5 m.
33. (d) C (s) + O2 ( g ) → CO2 ( g )
2 N
24. (a) As particle is initially at rest, so 1 mole of carbon reacts completely with
to move the charged particle an electric S 1 mole of oxygen to produce 1 mole of CO2.
field is required. III IV 12 g of C reacts = 1mole of O2
1
As path of particle is a curve, so direction 2.4 g of C reacts with = × 2. 4
of electric field must be changing with From above figure, we can conclude that 12
distance. magnetic fields cancel each other in = 0.2 mole of O2
25. (b) Electrostatic energy of two equal region I only. At STP 1 mole of O2 contains = 22. 4 L
charges of magnitude Q placed d distance 28. (c) In a simple harmonic motion, ∴ 0.2 mole of O2 contains = 22. 4 × 0.2
apart is oscillating particle passes extreme = 4.48 L
kq q kQ 2 positions two times, while it crosses 34. (a) The most often used stationary
E= 1 2= … (i)
r12 d mean position once in each half of phase gel and alumina are polar material.
oscillation. Consequently, the least polar compound
Now, when a third charge  −  is placed
Q
So, graph (c) most closely resembles this will have the highest Rf value, as they
 2
situation. will be least bounded to the stationary
at mid-point of these charges, phase and moves quickly up the TLC
29. (b) In Geiger-Marsden experiment,
then electrostatic energy of system is plate.
kq q kq2q3 kq q objective is to target α-particles towards
E′ = 1 2 + + 1 3 an atom. This is possible only when Among the given compound, compound (a)
r12 r23 r13 is least polar, so its Rf value will be
target is a very thin metal foil.
maximum.
https://iit-jeeacademy.blogspot.com

88 KVPY Question Paper 2014 Stream : SA

O 1 constant. Thus, the reaction with largest


35. (b) ∴ Average speed ∝
1 6 7 8 12 14 M equilibrium constant will be
13
F r G, ∆G° = − 150 kJ mol −1
16 17
2 5 V He MO 2 32
= = =
9
15 8
3 4 11 10 VO 2 MHe 4 45. (a) As O, F and Ne belong to same
period, i.e. 2nd period, the ionisation
Thus, the number of C — C sigma bonds V He
⇒ =2 2 energies increases on moving from left to
in the above given compound are 17. VO 2 right. This increase in ionisation energy
36. (c) According to Bohr’s radius of an is due to decrease in the atomic radii
atom Thus, the average speed of helium is
across a period. Thus, the first ionisation
52. 9n 2 higher than of oxygen by a factor of 2 2.
rn = pm enthalpies for O, Fe and Ne are 1314,
Z 40. (c) The products formed in each 1680 and 2080, respectively. So, the
where, n = charge on atom reaction given in the options are as correct sequence of the element is O, F
Z = atomic number follows and Ne.
52.9 × 12
rHe+ = = 26.45 ≈ 27 pm (i) NH 4 Cl + KOH → KCl + NH 3 + H 2O 46. (b) Population is a group of
2 individuals belonging to same species
(ii) AlN + 3H 2O → Al(OH)3 + NH 3
Thus, the radius of He+ ion is closest to occupying a particular geographic area in
27 pm. (iii) NH 4 Cl + NaNO2 → NaCl + N 2 + 2H 2O
a given time. A community is a group of
37. (d) Diamagnetic species are those (iv)NH4Cl + Ca(OH)2 → CaCl 2 + NH3 + H2O people living in the same place or having
species which have paired electrons in Thus, ammonia is not produced in a particular characteristic in common.
their molecular orbitals. reaction given in option (c). Biome is a large naturally occurring
The electronic configurations of molecules 41. (b) Isomers of compound of molecular community of flora and fauna occupying a
given in the options are as follows formula C4H10O are as follows major habitat. Species is a group of living
(i) NO organisms consisting of similar
CH3CH(OH)CH2CH3
Total number of electrons = 7 + 8 = 15 individuals capable of interbreeding.
CH3CH2CH2CH2OH
The electronic configuration of NO will be 47. (c) The energy assimilated by the
σ1s2 , σ * 1s2 , σ 2s2 , σ * 2s2 , σ 2 pz2 , π 2 px2 CH3 — O — CH2 — CH2CH3
herbivores is used in respiration and a
= πzpy2 , π * 2 p1x CH3 — CH2OCH2CH3 fraction of unassimilated energy is
CH3 transferred to decomposers (e.g. faecal
(ii) NO2 |
matter). With increasing trophic levels,
Total number of electrons = 7 + 8 + 8 = 23 CH3 — O — C H — CH3
the respiration cost also increases
The electronic configuration of NO2 will Thus, there are 3 isomers which are sharply. On an average, producers
be [18 Ar] σ * 2 pz2 , σ 3s2 , σ * 3s1 ethers having the molecular formula consume about 20% of their gross
(iii) O2 C4 H10 O. productivity in respiration. The
Total number of electrons in O2 = 16 42. (d) The major product of the reaction herbivores consume about 30% of
Electronic configuration of O2 will be of 2-butene (alkene) with alk. KMnO4 assimilated energy in respiration. In
σ1s2 σ * 1s2 σ 2s2 σ * 2s2 σ 2 pz2 solution is a vicinal glycol, i.e. butane-2,3 carnivores, the proportion of assimilated
π 2 px2 π 2 py2 π * 2 p1x π * 2 p1y diol. energy consumed in respiration rises to
(iv) CO2 OH about 60%.
Total number of electrons in 2 4 Alk. KMnO4 4 48. (b) Athletes are often trained at high
2 3
CO2 = 6 + 8 + 8 = 22 1
3
(Syn addition) 1 altitude because the air is ‘thinner’ at
2-butene high altitudes, means there are fewer
The electronic configuration of CO2 will be OH
[18 Ar] σ * 2 pz2 σ 3s2 Butane-2, 3-diol oxygen molecules per volume of air.
Thus, CO2 is a diamagnetic species. 43. (c) The boiling point of a compound Every breath taken at high altitude
38. (b) NaCl (NaOH + HCl) is a neutral depends upon the extent of H-bond delivers less of what working muscles
present in it. As compound I, II and IV require. To compensate for the decrease
salt, NH4 Cl (HCl + NH4 OH) is an acidic
are alcohols, so they can easily form in oxygen there occurs more production of
salt while CH3 COONa
H-bonds while compound III is an ether red blood cells to aid in oxygen delivery to
(CH3 COOH + NaOH) is a basic salt.
and cannot form H-bonds. Thus the muscles.
The value of pH of acidic salt is less than
7, while for neutral salt pH value is equal compound III, i.e. O has 49. (d) Corpus callosum is nervous band
to 7 and for basic salt pH value is greater lowest boiling point. which attaches both cerebral
than 7. Thus, the increasing order of pH 44. (c) The relation between Gibbs free hemispheres of mammals. It is a thick
0.1 M aqueous solutions of NaCl, energy and equilibrium constant can be band of nerve fibres that divides the
CH3 COONa and NH4 Cl will be given as cerebral cortex lobes into left and right
NH4 Cl < NaCl < CH3 COONa. ∆G ° = − 2.303RT log K eq hemispheres. It connects the left and
39. (a) At room temperature log K eq = − ∆G ° / 2.303RT right sides of the brain allowing for
The reaction having most negative value communication between both
8RT
V avg = of ∆G° will have the largest equilibrium hemispheres.
πM
https://iit-jeeacademy.blogspot.com

KVPY Question Paper 2014 Stream : SA 89

50. (b) Glucagon is secreted from alpha Its pathway is as discussed below 62. (a) We have,
cells of pancreas. Prolactin is secreted by Receptor → Sensory neuron → 1947
1
the anterior pituitary. Luteinising (Skin)
Integration centre ∑
hormone is secreted by the gonadotropic (Spinal cord) n=0 2n + 21947
cells in the anterior pituitary. | 1
↓ Let f (n ) =
Epinephrine (or adrenaline) is secreted 2 + 21947 n
by the medulla of the adrenal gland. Effector ← Motor neuron
(muscle) 1 1
51. (d) The stalk of plant leaf (petiole) is f (0) + f (1947) = 1947
+ 1947
58. (b) In photosynthesis,
derived from collenchyma. Collenchyma 1+ 2 2 21947 + 2 2
cells are elongated cells with irregular Light reaction → Photolysis of water (H 2O
1 1
thick cell walls that provide structural is oxidised) = +
1947 1947  1947 
2H 2O → O2 + 4 [H] requires light reaction
support, particularly in growing shoots 1+ 2 2 2 2  2 2 + 1
and leaves. Their thick cell walls are Dark reaction → CO2 is reduced for sugar  
 
composed of the compounds cellulose and formation 1947
pectin. +1
4 [H] + CO2 → (CH 2O) + H 2O (Reduction) =
2 2
=
1
52. (c) Both smooth muscle (unstriated 1947  1947  1947
59. (c) A mature RBC lacks nucleus,
muscle) and cardiac muscle are 2 2 2 2 + 1 2 2
mitochondria and endoplasmic reticulum.  
functionally involuntary. Muscles that
In humans, mature RBCs are flexible and  
are under our conscious control are called 1
oval biconcave disks. They lack a cell Similarly, f (1) + f (1946) =
voluntary muscles, while muscles that 1947
nucleus and most organelles, in order to
are not under our conscious control are 2 2
accommodate maximum space for
called involuntary muscles. Striated 1947
∑ f (x) = f (0) + f (1) + f (2) + f (3) +
haemoglobin.
muscles are voluntary muscles. Q
60. (d) Antivenom injection provides n=0
53. (a) Pulmonary artery arises from left
artificial acquired passive immunity. + f (1947)
ventricle and carries deoxygenated blood
Passive immunity is the transfer of active
to the lungs. = (f (0) + f (1947) + (f (1) + f (1946)) + ...
humoral immunity of readymade
The pulmonary artery begins in the heart antibodies. It can occur naturally, when ... + (f (973) + f (974))
at the base of the right ventricle. 1
maternal antibodies are transferred to = 974 × 1947
54. (b) Gout is caused by the deposition foetus through placenta and it can be
of uric acid in joints. Composition of induced artificially when high level of 2 2
1947
2 × 487 487
kidney stone is calcium oxalate, calcium
phosphate, uric acid, xanthine and indigo
antibodies specific to a pathogen or toxin
are transferred to non-immune persons
∑ f (n ) = 1945
=
n=0 21945
calculi. Researches have shown that through blood products that contain 2 ×2 2
kidney stones are a complication of gout antibodies, such as antivenom injections 63. (d) Given,
because extra uric acid can collect in the are given. x + y = a ⇒ a∈[1, 2014]
urinary tract and crystallise into stones. 61. (b) Given, a + b + c = 0 ⇒ a, b, c ∈ R x2 y2
55. (a) Cochlea is the main hearing and + =4
⇒ a 2 + b2 + c2 = q x−1 y−1
organ. It is composed of sensory cells
⇒ a 4 + b4 + c4 = r
called hair cells, which convert vibrations x2 y2
into neural messages. These messages ⇒ (a 2 + b2 + c2 )2 = a 4 + b4 + c4 ∴ + =4
x −1 y −1
are then passed to the auditory nerve and + 2 (a 2b2 + b2c2 + c2a 2 )
carried up to the brain. ⇒ x2 y − x2 + xy2 − y2 = 4(x − 1)( y − 1)
⇒ (a 2 + b2 + c2 )2 = a 4 + b4 + c4 + 2
56. (d) Mitochondria and chloroplast [(ab + bc + ca )2 − 2abc (a + b + c)] ⇒ x2 y + xy2 = 4(xy − (x + y) + 1) + x2 + y2
contain circular DNA. Even though both
⇒ q = r + 2[(ab + bc + ca )2 − 2(abc)(0)]
2
⇒ xy (x + y) = 4(xy − a + 1)
organelles are found in eukaryotic cell,
both mitochondria and chloroplast have ⇒ q2 = r + 2 [ab + bc + ca ]2 + (x + y)2 − 2xy
characteristics often found in prokaryotic ⇒ q2 = r + 2 ⇒ xy(a ) = 4xy − 4a + 4 + a 2 − 2xy
2
cells. These prokaryotic cell’s  (a + b + c)2 − (a 2 + b2 + c2 ) 
  ⇒ xya − 2xy = a 2 − 4a + 4
characteristics include enclosed double 2
 
membrane, circular DNA and bacteria 2 ⇒ xy (a − 2) = (a − 2)2
 0 − q
like ribosomes. ⇒ q = r + 2
2
 ⇒ (a − 2)2 − xy (a − 2) = 0
 2 
57. (b) Reflex actions do not involve the
2q2 ⇒ (a − 2) (a − 2 − xy) = 0
brain in the decision making process. ⇒ q2 = r +
Reflex action is a rapid, spontaneous and 4 ⇒ a = 2 or xy = a − 2
involuntary activity that is produced in 1 x (a − x ) = a − 2 [Q y = a − x]
⇒ q − q2 = r
2 or
response to a stimulus. It is controlled by 2
⇒ x2 − ax + a − 2 = 0
spinal cord. ⇒ q2 = 2r
https://iit-jeeacademy.blogspot.com

90 KVPY Question Paper 2014 Stream : SA

Since, x ∈ R ⇒ (a × b)(c + 1) + b (c + 1) + a (c + 1) Rearranging, we get


D≥0 + (c + 1) = 30 2
⇒   −   − 1 = 0
Q a a
⇒ (c + 1) (a × b + b + a + 1) = 30  b  b
Q a 2 − 4 (a − 2) ≥ 0
⇒ (c + 1) (b(a + 1) + 1(a + 1)) = 30
a − 4a + 8 ≥ 0, ∀ a ∈ R
2 a 1± 1+ 4
⇒ (a + 1)) (b + 1) (c + 1) = 30 ⇒ =
b 2
Q a ∈ [1, 2014] 1 ≤ a ≤ 9, 0≤ b, c ≤ 9
a 5+1
64. (a) Given, Q Total number of solution = 18 ⇒ =
b 2
ABC is right angled triangle. 66. (b) Centre of mass of square wooden Note Choice of a different origin gives a
B plate with respect to chosen axis is at a
different value of .
centre of plate. b
4x O 67. (d) For a soap bubble floating in air,
x
a Gravitational force = Buoyant force
P 2 ⇒ g (mass of helium + mass of soap film)
√7
a/2 = Weight of air displaced by bubble …(i)
3x
t

y
C √5 A r

∠A = 90° Coordinates of centre of mass are


a a
AC = 5 x1 = , y1 =
2 2
AB = 7 Mass of square plate, m1 = ka 2 Let r = inner radius of soap bubble and
PA 3
= where, k = mass per unit area. t = thickness of film.
PB 4 Coordinates of centre of mass of removed Then, from Eq. (i), we have
In ∆ABC portion and its mass with respect to axes 4 3
⇒ πr × ρHe + 4 πr 2 × t × ρsoap
BC 2 = AB 2 + AC 2 = 5 + 7 = 12 chosen are 3
BC = 12 = 2 3 4
x O = πr3 × ρair
In ∆ABP, b/2 3
AB 2 + PB 2 − AP 2 Substituting values in above equation, we
cosB = b/2 get
2AB ⋅ PB
4
7 7 + 16x2 − 9x2 ⇒ × π × (10−2 )3 × 018 . + 4 π × (10−2 )2
⇒ = 3
2 3 2 ⋅ 7 × 4x
× t × 1000
 AB 7 y
4
Q cosB = BC = 2 3  x2 =
b
, y2 =
b = × π × (10−2 )3 × 1. 23
  2 2 3
28x Rearranging, we get
⇒ = 7 + 7x2 ⇒ 28x = 7 3 (x2 + 1) and mass of removed portion,
4
3 m2 = kb2 ⇒ 4 π (10−2 ) ⋅ t ⋅ 1000 = π (10−6 ) (108 . )
⇒ 3 x2 − 4x + 3 = 0 3
Now, centre of mass of remaining
⇒ 3x − 3x − x + 3 = 0
2 ⇒ (10 5 ) t = 0.35
L-shaped portion is given at point
⇒ ( 3x − 1) (x − 3 ) = 0 or t = 3.5 × 10−6 m
P whose coordinates are (b, b) with
1 respect to chosen axes. = 3.50 µm
Q x = 3, ,x≠ 3
3
x
68. (c) As heat lost by aluminium piece =
Q PB = 4 / 3 heat gained by water
4 2 {ms (Ti − Tf )}aluminium = {ms(Tf − Ti )}water
PC = BC − BP = 2 3 − =
3 3 P(b,b)
Substituting given values, we get
BP 4 / 3
Q = = 2:1 ⇒ 50 × 10−3 × 900 × (300 − 160)
PC 2 / 3
= 1 × 4200 × (T − 30)
65. (d) abc is three-digits number y ⇒ 6300 = 4200 (T − 30)
abc = 100a + 10b + c m1 x1 − m2x2
100 ≤ abc < 999, a ∈ {1, 2, 3, ..., 9} b, c∈{0, Now, using XCM = , we have ⇒ T = 30 + 1.5
m1 − m2
1, 2, 3, ..., 9} or T = 31.5°C
ka 2   − kb2  
a b
Now, (a × b × c) + (a × b) + (b × c) So, temperature of water after taking out
 2  2 a3 − b3
+ (c × a ) + a + b + c = 29 b= ⇒b = aluminium piece is 31.5°C.
ka − kb
2 2
2(a 2 − b2 )
https://iit-jeeacademy.blogspot.com

KVPY Question Paper 2014 Stream : SA 91

i4 = i3   ∴ Resulting moles of gases in mixture


69. (a) From the geometry of given figure, 5
 3 = x − 0.2 + 10 − x − 01. = 9.7
5 8
45° So, current, i5 = i4 + i3 = i3 + i3 = i3 At constant temperature and volume,
3 3 p1 p
r = 2
45°– r 8
45°– r
=
(3i1 ) = 8 i1 n1 n2
3
45°+ r
45°+ r

1 p
⇒ = 2
Similarly, across section bb, 10 9.7
45°
i7 b i5 c ⇒ p2 = 0.97 atm
We have, for total internal reflections i6 R
R 5 72. (a) 2NH3 + H2SO4 → (NH4 )2 SO4
45° + r > θc …(i) 8
R

and 45° − r > θc …(ii) Eq. of H2SO4 = 2 × 2 × 10 × 10−3


b c [basicity of H2SO4 = 2]
or 90° > 2θc ⇒ sin 45° > sin θc
Ri6 =  R  i5 or i6 =
13 13

1 1
> or µ > 2 ⇒ i5 Eq. of H2SO4 = Eq. of NH3 = No. of moles of
…(iii)  8  8
2 µ ammonia = 4 × 10−2
So, current,
From Eq. (ii), we have 1 mole of NH3 = 17 g
13 21 21
45 − θc > r i7 = i5 + i5 = i5 = (8i1 ) = 21 i1
8 8 8 ∴ 4 × 10−2 moles of NH3 = 17 × 4 × 10−2
sin(45 − θc ) > sin r
Now, for section aa, we have ∴ WNH 3 = 17 × 4 × 10−2 g
1 1 sin 45°
⇒ cosθc − sin θc > i a i7 14
2 2 µ
b
⇒ WN = × 17 × 4 × 10−2
i8 R 17
µ2 − 1 1 1 R 13
⇒ − > ⇒ µ2 − 1> 2 21
R = 0.56 g
µ µ µ
0.56
⇒ µ> 5 …(iv) a b % of N = × 100 = 28%
2
Ri8 = i7  1 R
13
Common solution of Eqs. (iii) and (iv) is +
 21  73. (d) Let the equivalent weight of
µ > 5.
metal M = x
Ri8 = i7  R 
34
70. (d) Let current through resistor X is
 21  (eq )Ca (eq )H 2 , released
be i. =
34
P a b c d i1 ⇒ i8 = i7 × (eq )M (eq )H 2 , released
21
2 1125
.
X Hence, current i is
( eq vol )H 2
i = i7 + i8 ⇒ 20 =
34 55 2 185
.
Q a b c d
= i7 + i7 = i7 x (eq vol. )H 2
21 21
Now, we consider section dd,
55
i3 d i1 R = × 21i1 ∴
x
=
1.125
21 20 1.85
i2
= 55i1 = 55 × 10−3 A 1.125
R R
(Q i1 = 1 mA, given) ⇒ x= × 20
1.85
Total resistance across PQ is
d = 12.16 ≈ 12
34
Equating potential across dd, we get Req = kΩ
55 74. (a) When coke is heated with lime
i2R = i1 (2R )
34 × 1000 (CaO), then CaC2(X ) is formed which
or i2 = 2i1 = Ω
55 then reacts with water to form acetylene
Hence, current i is (Y ) as a major product. This acetylene on
So, potential drop across, PQ
i3 = i1 + i2 = i1 + 2i1 = 3i1 34 passing over red hot iron at 873 K
Now, we consider section cc, = i Req = 55 × 10−3 × × 103 V produces benzene (Z).
55
i5 c i3 d = 34 V CaO + C

CaC2
i4 R X
2R × R 2R 71. (b) 18 g of H2O = 1 mole (Coke)
Calcium carbide
=
R 2R + R 3 1
3.6 of H2O = × 3.6 = 0.2 mole H2O
18 Fe (red hot)
c d
2H2 ( g ) + O2 ( g ) → 2H2O(l) 373 K
CH CH + Ca(OH)2
Equating potentials, we get (Trimerisation) Y
Initially x mol (10 − x ) mol 0 Acetylene
i4 R = i3  R + R 
2 0.2 Z
After reaction x − 0.2 10 − x −
 3  2
0.2 mol Benzene
https://iit-jeeacademy.blogspot.com

92 KVPY Question Paper 2014 Stream : SA

75. (a) The ETC occurs in the plasma membrane On the other hand, the second beaker
Br of prokaryotes and the inner contains 10% sucrose solution which is
(i) Alc. KOH more concentrated than the
mitochondrial membrane of eukaryotes.
semipermeable bag’s sucrose
Br Ph (ii) NaNH2 77. (c) Colourblindness is X-linked
(Dehydrohalogenation) concentration. Hence, water will move
recessive disorder.
out of the bag to the beaker and make the
H C C Ph XXC × XCY bag flaccid (exosmosis). Thus, the answer
(X) Heterozygous Colourblind (b) is correct.
woman man
HgSO4/dil. H2SO4, 79. (a) Phenylketonuria is an autosomal
∆ (Acidic hydration
of alkyne) recessive disorder with mutation in gene
for enzyme Phenylalanine Hydroxylase
Tautomerism
CH3 C H2C C Ph XXC XY XCXC XCY (PAH), making it non-functional.
PAH
Ratio Phenylalanine → Tyrosine
O OH X

Conc. HNO3/H2SO4 Genotype F1 ratio Such person cannot metabolise the above
nitration
Carrier daughter XXC 1 reaction leading to accumulation of
NO2 phenylalanine. So, are given food low in
Colourblind daughter XC XC 1
phenylalanine and supplemented with
Normal son XY 1 tyrosine.
Colourblind son XC Y 1
80. (a) Ganga is more polluted than
O (m-isomer)
78. (b) Osmosis is the net movement of Kaveri → lower DO [Dissolved Oxygen]
(Y)
solvent molecules into a region of higher indicates polluted water
76. (c) Respiratory reactions occur in the solute concentration through a 2% Ganga water (P)
mitochondria and cytoplasm. Respiratory semipermeable membrane. In the given
DO
question, one of the 2 sucrose containing
reactions include glycolysis, Kreb’s cycle
bags (semipermeable) is placed in a water 10% Kaveri water (Q)
and Electron Transport Chain (ETC). The
containing beaker. Clearly, the Dissolved oxygen refers to the level of
glycolysis always occurs in the cytoplasm concentration in the bag is more than the
of all living cells. The Kreb’s cycle occurs free oxygen present in water levels that
beaker and as a result, water will move are too high or too low can harm aquatic
in the cytoplasm of all prokaryotes and in through the semipermeable membrane life and affect water quality.
the mitochondrial matrix in eukaryotes. (endosmosis) and make the bag turgid.
https://iit-jeeacademy.blogspot.com

KVPY Question Paper 2013 Stream : SA 93

KVPY
KISHORE VAIGYANIK PROTSAHAN YOJANA

QUESTION PAPER 2013


Stream : SA
MM 100

Instructions
There are 80 questions in this paper.
This question paper contains two parts; Part I and Part II. There are four sections; Mathematics, Physics, Chemistry
and Biology in each part.
Out of the four options given with each question, only one is correct.

PART-I (1 Mark Questions)


MATHEMATICS 5. Let x, y, z be non-zero real numbers such that
x y z y z x x3 y3 z3
1. Let x, y, z be three non-negative integers such that + + = 7 and + + = 9, then 3 + 3 + 3 − 3
x + y + z = 10. The maximum possible value of y z x x y z y z x
xyz + xy + yz + zx is is equal to
(a) 52 (b) 64 (c) 69 (d) 73 (a) 152 (b) 153 (c) 154 (d) 155
2. If a , b are natural numbers such that 2013 + a = b ,
2 2 6. In a ∆ ABC with∠A < ∠B < ∠C, points D, E , F are on
then the minimum possible value of ab is the interior of segments BC , CA, AB respectively.
(a) 671 (b) 668 Which of the following triangles cannot be similar to
(c) 658 (d) 645 ∆ABC?
(a) ∆ ABD (b) ∆ BCE (c) ∆CAF (d) ∆ DEF
3. The number of values of b for which there is an
isosceles triangle with sides of lengths b + 5, 3b − 2 7. Tangents to a circle at points P and Q on the circle
and 6 − b is intersect at a point R. If PQ = 6 and PR = 5, then the
(a) 0 (b) 1 (c) 2 (d) 3 radius of the circle is
13
4. Let a , b be non-zero real numbers. Which of the (a) (b) 4
3
following statements about the quadratic equation 15 16
(c) (d)
ax2 + (a + b)x + b = 0 is necessarily true? 4 5
I. It has at least one negative root. 8. In an acute angled ∆ ABC, the altitudes from A, B, C
II. It has at least one positive root. when extended intersect the circumcircle again at
III. Both its roots are real. points A1 , B1 , C1 respectively. If ∠ABC = 45°, then
(a) I and II only (b) I and III only ∠A1B1C1 equals
(c) II and III only (d) All of them (a) 45° (b) 60° (c) 90° (d) 135°
https://iit-jeeacademy.blogspot.com

94 KVPY Question Paper 2013 Stream : SA

9. In a rectangle ABCD, points X and Y are the


mid-points of AD and DC, respectively. Lines BX and
PHYSICS
CD when extended intersect at E, lines BY and AD 16. A man inside a freely falling box throws a heavy ball
when extended intersect at F. If the area of ABCD is towards a side wall. The ball keeps on bouncing
60, then the area of BEF is between the opposite walls of the box. We neglect air
(a) 60 (b) 80 (c) 90 (d) 120 resistance and friction. Which of the following figures
10. In the figure given below, ABCDEF is a regular depicts the motion of the centre of mass of the entire
hexagon of side length 1, AFPS and ABQR are system (man, the ball and the box)?
squares. Then, the ratio ar ( APQ) /ar (SRP ) equals
A B (a) (b) (c) (d)

F C
17. A ball is thrown horizontally from a height with a
R Q certain initial velocity at time t = 0. The ball bounces
repeatedly from the ground with the coefficient of
P restitution less than 1 as shown below.
E D
2+1 3 3
(a) (b) 2 (c) (d) 2
2 4
11. A person X is running around a circular track
completing one round every 40 s. Another person Y
running in the opposite direction meets X every 15 s. Neglecting air resistance and taking the upward
The time, expressed in seconds, taken byY to complete direction as positive, which figure qualitatively
one round is depicts the vertical component of the ball’s velocity vy
(a) 12.5 (b) 24 (c) 25 (d) 55 as a function of time t?
vy vy
12. The least positive integer n for which
n + 1 − n − 1 < 02
. is
(a) 24 (b) 25 (a) t (b) t
(c) 26 (d) 27
13. How many natural numbers n are there such that
n ! + 10 is a perfect square? vy vy
(a) 1 (b) 2
(c) 4 (d) infinitely many
(c) t (d) t
14. Ten points lie in a plane so that no three of them are
collinear. The number of lines passing through
exactly two of these points and dividing the plane
into two regions each containing four of the 18. A tall tank filled with water has an irregular shape
remaining points is as shown. The wall CD makes an angle of 45° with
(a) 1 the horizontal, the wall AB is normal to the base BC.
(b) 5 The lengths AB and CD are much smaller than the
(c) 10 height h of water (figure not to scale).
(d) dependent on the configuration of points
15. In a city, the total income of all people with salary
below ` 10000 per annum is less than the total
income of all people with salary above ` 10000 per
annum. If the salaries of people in the first group
increases by 5% and the salaries of people in the
second group decreases by 5%, then the average h
income of all people p3
(a) increases A p1 D
p2
(b) decreases
(c) remains the same
(d) cannot be determined from the data B C
https://iit-jeeacademy.blogspot.com

KVPY Question Paper 2013 Stream : SA 95

Let p1, p2 and p3 be the pressures exerted by the 23. In a car, a rear view mirror having a radius of
water on the wall AB, base BC and the wall CD curvature 1.50 m forms a virtual image of a bus
respectively. Density of water is ρ and g is located 10.0 m from the mirror. The factor by which
acceleration due to gravity. Then, approximately the mirror magnifies the size of the bus is close to
1 (a) 0.06 (b) 0.07 (c) 0.08 (d) 0.09
(a) p1 = p2 = p3 (b) p1 = 0, p3 = p2
2
1
24. Consider the following circuit shown below.
(c) p1 = p3 = p2 (d) p1 = p3 = 0, p2 = hρg
2 I
I′
19. The accompanying graph of position x versus time t
represents the motion of a particle. If p and q are
both positive constants, the expression that best
describes the acceleration a of the particle is
x
E
All the resistors are identical. The ratio of I / I ′ is
(a) 8 (b) 6 (c) 5 (d) 4
25. The figure shows a bar magnet and a metallic coil.
Consider four situations:
(I) Moving the magnet away from the coil.
(II) Moving the coil towards the magnet.
t (III) Rotating the coil about the vertical diameter.
(a) a = − p − qt (b) a = − p + qt (IV) Rotating the coil about its axis.
(c) a = p + qt (d) a = p − qt
20. Two stones of masses m1 and m2 (such that m1 > m2)
are dropped ∆t time apart from the same height
towards the ground. At a later time t, the difference An emf in the coil will be generated for the following
in their speed is ∆v and their mutual separation is ∆s. situations.
While both stones are in flight (a) I and II only (b) I, II and IV only
(a) ∆v decreases with time and ∆s increases with time (c) I, II, and III only (d) I, II, III, and IV
(b) Both ∆v and ∆s increase with time 26. A current of 0.1 A flows through a 25 Ω resistor
(c) ∆v remains constant with time and ∆s decreases with represented by the circuit diagram. The current in
time 80 Ω resistor is
(d) ∆v remains constant with time and ∆s increases with
80Ω
time
0.1 A
21. The refractive index of a prism is measured using 25Ω
three lines of a mercury vapour lamp. If µ1 , µ 2 and µ3
are the measured refractive indices for these green,
blue and yellow lines respectively, then V 20Ω
20Ω 60Ω
(a) µ 2 > µ 3 > µ 1 (b) µ 2 > µ 1 > µ 3
(c) µ 3 > µ 2 > µ 1 (d) µ 1 > µ 2 > µ 3
22. A horizontal parallel beam of light passes through a
(a) 0.1 A (b) 0.2 A (c) 0.3 A (d) 0.4 A
vertical convex lens of focal length 20 cm and is then
reflected by a tilted plane mirror, so that it converges 27. Solar energy is incident normally on the earth’s
to a point I. The distance PI is 10 cm. surface at the rate of about 1.4 kW m −2. The distance
I between the earth and the sun is 15 . × 1011 m. Energy
E and mass m are related by Einstein equation
E = mc2, where c = 3 × 108 ms−1 is the speed of light in
free space. The decrease in the mass of the sun is
P (a) 109 kg s−1 (b) 1030 kg s−1
M (c) 1026 kg s−1 (d) 1011 kg s−1
28. If the current through a resistor in a circuit increases
M is a point at which the axis of the lens intersects by 3%, then the power dissipated by the resistor
the mirror. The distance PM is 10 cm. The angle
(a) increases approximately by 3%
which the mirror makes with the horizontal is
(b) increases approximately by 6%
(a) 15° (b) 30° (c) 45° (d) 60°
(c) increases approximately by 9%
(d) decreases approximately by 3%
https://iit-jeeacademy.blogspot.com

96 KVPY Question Paper 2013 Stream : SA

29. An ideal gas filled in a cylinder occupies volume V . 36. Which of the following molecules has no dipole moment?
The gas is compressed isothermally to the volume (a) CH 3 Cl (b) CHCl 3 (c) CH 2 Cl 2 (d) CCl 4
V / 3. Now, the cylinder valve is opened and the gas is 37. The decay profiles of three radioactive species A, B
allowed to leak keeping temperature same. What and C are given below :
percentage of the number of molecules should escape
to bring the pressure in the cylinder back to its
original value?

Concentration
(a) 66% (b) 33% (c) 0.33% (d) 0.66%
30. An electron enters a chamber in which a uniform
A
magnetic field is present as shown below.
C B
Time

These profiles imply that the decay constants


kA , kB and kC follow the order
e– (a) kA > kB > kC (b) kA > kC > kB
(c) kB > kA > kC (d) kC > kB > kA
38. A specific volume of H2 requires 24 s to diffuse out of
a container. The time required by an equal volume of
Magnetic field O2 to diffuse out under identical conditions, is
(a) 24 s (b) 96 s (c) 384 s (d) 192 s
An electric field of appropriate magnitude is also
applied, so that the electron travels undeviated 39. Acetic acid reacts with sodium metal at room
without any change in its speed through the temperature to produce
chamber. We are ignoring gravity. Then, the (a) CO2 (b) H 2 (c) H 2O (d) CO
direction of the electric field is 40. The equilibrium constant, kc for
(a) opposite to the direction of the magnetic field 3 C2H2(g) C6H6(g)
(b) opposite to the direction of the electron’s motion 2 −2
(c) normal to the plane of the paper and coming out of the is 4 L mol . If the equilibrium concentration of
plane of the paper benzene is 0.5 mol L−1, that of acetylene in mol L−1
(d) normal to the plane of the paper and into the plane of must be
the paper (a) 0.025 (b) 0.25 (c) 0.05 (d) 0.5
41. The weight per cent of sucrose (formula weight = 342
g mol −1 ) in an aqueous solution is 3.42. The density of
CHEMISTRY the solution is 1 g mL−1, the concentration of sucrose
31. The molecule having a formyl group is in the solution in mol L−1 is
(a) acetone (b) acetaldehyde (a) 0.01 (b) 0.1 (c) 1.0 (d) 10
(c) acetic acid (d) acetic anhydride 42. The order of reactivity of K, Mg, Au and Zn with
32. The structure of cis-3-hexene is water is
(a) K > Zn > Mg > Au (b) K > Mg > Zn > Au
(a) (b) (c) K > Au > Mg > Zn (d) Au > Zn > K > Mg
43. Which of the following is an anhydride?
O O O
(c) (d)
(a) (b) H3C CH3
H 3C O CH3 O
2
33. The number of sp -hybridised carbon atoms in
O O
O CH3
(c) O (d)
H 3C CH3 H3C O
HC C CH2 C CH2 CH CH2, is
O
(a) 3 (b) 5 (c) 4 (d) 6 44. Which of the following metals will precipitate copper
34. The number of valence electrons in an atom with from copper sulphate solution?
electronic configuration 1s22s22 p63s23 p3 is (a) Hg (b) Sn (c) Au (d) Pt
(a) 2 (b) 3 (c) 5 (d) 11 45. The radii of the first Bohr orbit of H (rH ), He + (rHe + )
35. The pair of atoms having the same number of and Li 2+ (rLi 2 + ) are in the order
neutrons is (a) rHe + > rH > r 2 + (b) rH < rHe + < rLi 2 +
Li
(a) 12 24 23 19 23 24 23 39
6 C, 12 Mg (b) 11 Na, 9 F (c) 11 Na, 12 Mg (d) 11 Na, 19 K (c) rH > rHe + > rLi 2 + (d) rHe + < rH > r 2 +
Li
https://iit-jeeacademy.blogspot.com

KVPY Question Paper 2013 Stream : SA 97

53. Animal cells after removal of nuclei still contained


BIOLOGY DNA. The source of this DNA is
46. The Bowman’s capsule, a part of the kidney is the (a) nucleosomes (b) mitochondria
site of (c) peroxisomes (d) lysosome
(a) filtration of blood constituents 54. Which one of the following combinations is found in
(b) reabsorption of water and glucose DNA?
(c) formation of ammonia (a) Guanine and guanidine (b) Guanidine and cytosine
(d) formation of urea (c) Guanine and cytosine (d) Adenine and guanidine
47. In human brain, the sensation of touch, pain and 55. Which one of the following is not a mode of asexual
temperature is controlled by the reproduction?
(a) parietal lobe of cerebrum (a) Binary fission (b) Multiple fission
(b) limbic lobe of cerebrum (c) Budding (d) Conjugation
(c) temporal lobe of cerebrum
(d) frontal lobe of cerebrum 56. Which one of the following classes of animals
constitutes the largest biomass on the earth?
48. A pathogen which cannot be cultured in an artificial (a) Insects (b) Fishes
medium is
(c) Mammals (d) Reptilians
(a) protozoan (b) virus (c) bacterium (d) fungus
57. In the digestive system, the pH of the stomach and
49. Meiosis-I and meiosis-II are characterised by the the intestine, respectively are
separation of
(a) alkaline, acidic (b) acidic, alkaline
(a) homologous chromosomes; sister chromatids
(c) acidic, neutral (d) acidic, acidic
(b) sister chromatids; homologous chromosomes
(c) centromere; telomere 58. The major nitrogenous excretory product in
(d) telomere; centromere mammals is
(a) amino acids (b) ammonia
50. People suffering from albinism cannot synthesise
(c) urea (d) uric acid
(a) suberin (b) melanin (c) keratin (d) collagen
59. Which of the following plant traits (characters) is not
51. Short-sightedness in humans can be corrected by using an adaptation to dry (xeric) habitats?
(a) concave lens (b) convex lens
(a) Sunken stomata on leaves
(c) cylindrical lens (d) plain glass
(b) Highly developed root system
52. A person with blood group ‘A’ can (i) donate blood to (c) Thin epidermis without a cuticle on stem and leaves
and (ii) receive blood from (d) Small leaves and photosynthetic stem
(a) (i) person with blood group ‘AB’ and (ii) persons with 60. Biological diversity increases with the productivity of
any blood group
an ecosystem. In which of the following habitats do
(b) (i) person with blood group ‘A’ or ‘AB’ and (ii) ‘A’ or ‘O’
blood groups
we see the greatest diversity of species?
(c) (i) person with blood group ‘B’ or ‘AB’ and (ii) ‘B’ or ‘O’ (a) Tropical dry grasslands
blood groups (b) Temperate deciduous forests
(d) (i) person with any blood group and (ii) ‘O’ blood group (c) Alpine grasslands
only (d) Tropical evergreen forests

PART-II (2 Marks Questions)


MATHEMATICS are 16, 24 and 31. The volume of the cuboid lies
between
61. Let a , b, c, d , e be natural numbers in an arithmetic (a) 7 and 14 (b) 14 and 21
progression such that a + b + c + d + e is the cube of (c) 21 and 28 (d) 28 and 35
an integer and b + c + d is square of an integer. The
least possible value of the number of digits of c is 63. Let ABCD be a square and let P be a point on
segment CD such that DP : PC = 1 : 2. Let Q be a point
(a) 2 (b) 3
on segment AP such that ∠BQP = 90°. Then, the ratio
(c) 4 (d) 5
of the area of quadrilateral PQBC to the area of the
62. On each face of a cuboid, the sum of its perimeter and square ABCD is
its area is written. Among the six numbers so 31 37 39 41
(a) (b) (c) (d)
written, there are three distinct numbers and they 60 60 60 60
https://iit-jeeacademy.blogspot.com

98 KVPY Question Paper 2013 Stream : SA

64. Suppose the height of a pyramid with a square base Note That irrespective of speed of P, ball always
is decreased by p% and the lengths of the sides of its leaves P’s hand with speed 2 ms −1 with respect to the
square base are increased by p% (where, p > 0). If the ground. Ignore gravity. Balls will be received by Q.
volume remains the same, then (a) One every 2.5 s in case (I) and one every 3.3 s in
(a) 50 < p < 55 (b) 55 < p < 60 case (II)
(c) 60 < p < 65 (d) 65 < p < 70 (b) One every 2 s in case (I) and one every 4 s in case (II)
(c) One every 3.3 s in case (I) and one every 2.5 s in
65. There are three kinds of liquids X , Y , Z . Three jars case (II)
J1 , J 2 , J3 contains 100 ml of liquids X , Y , Z (d) One every 2.5 s in case (I) and one every 2.5 s in
respectively. By an operation we mean three steps in case (II)
the following order
68. A 10.0 W electrical heater is used to heat a container
– stir the liquid in J1 and transfer 10 ml from J1 filled with 0.5 kg of water.It is found that the
into J 2, temperature of the water and the container rose by
– stir the liquid in J 2 and transfer 10 ml from J 2 3 K in 15 min. The container is then emptied, dried
into J3 , and filled with 2 kg of an oil. It is now observed that
– stir the liquid in J3 and transfer 10 ml from J3 the same heater raises the temperature of the
into J1. container-oil system by 2 K in 20 min. Assuming no
After performing the operation four times, let x, y, z other heat losses in any of the processes, the specific
be the amounts of X , Y , Z respectively, in J1.Then, heat capacity of the oil is
(a) x > y > z (b) x > z > y (c) y > x > z (d) z > x > y (a) 2.5 × 10 3 JK−1 kg −1
(b) 51. × 10 3 JK−1 kg −1
(c) 3.0 × 10 3 JK−1 kg −1
PHYSICS . × 10 3 JK−1 kg −1
(d) 15

66. Two identical uniform rectangular blocks (with 69. A ray of light incident on a transparent sphere at an
longest side L) and a solid sphere of radius R are to angle π / 4 and refracted at an angle r, emerges from
be balanced at the edge of a heavy table such that the the sphere after suffering one internal reflection. The
centre of the sphere remains at the maximum total angle of deviation of the ray is
3π π
possible horizontal distance from the vertical edge of (a) − 4r (b) − 4r
the table without toppling as indicated in the figure. 2 2
π 5π
(c) − r (d) − 4r
4 2
. × 106 ms −1 is
70. An electron with an initial speed of 40
brought to rest by an electric field. The mass and
charge of an electron are 9 × 10−31 kg and 16
. × 10−19 C,
respectively. Identify the correct statement.
(a) The electron moves from a region of lower potential
to higher potential through a potential difference of
Table 11.4 µV
(b) The electron moves from a region of higher potential
L R to lower potential through a potential difference of
x 11.4 µV
If the mass of each block is M and of the sphere is M / 2, (c) The electron moves from a region of lower potential to
then the maximum distance x that can be achieved is higher potential through a potential difference of 45 V
(a) 8L / 15 (b) 5L / 6 (d) The electron moves from a region of higher potential to
lower potential through a potential difference of 45 V
(c) (3L / 4 + R ) (d) (7L / 15 + R )
67. Two skaters P and Q are skating towards each other.
Skater P throws a ball towards Q every 5 s such that CHEMISTRY
it always leaves her hand with speed 2 ms −1 with 71. The degree of dissociation of acetic acid (0.1 mol L−1 )
respect to the ground. Consider two cases:
in water (K a of acetic acid is 10−5 ) is
(I) P runs with speed 1 ms −1 towards Q, while Q
(a) 0.01 (b) 0.5
remains stationary.
(c) 0.1 (d) 1.0
(II) Q runs with speed 1 ms −1 towards P, while P
remains stationary.
https://iit-jeeacademy.blogspot.com

KVPY Question Paper 2013 Stream : SA 99

72. Compound X on heating with Zn dust gives 77. A diabetic individual becomes unconscious after
compound Y which on treatment with O3 followed by self-administering insulin. What should be done
reaction with Zn dust gives propionaldehyde. The immediately to revive the individual?
structure of X is (a) Provide him sugar
Br Br (b) Give him higher dose of insulin
(a) (c) Provide him salt solution
(b)
(d) Provide him lots of water
Br Br 78. A regular check on the unborn baby of a lady towards
Br Br the end of her pregnancy showed a heart rate of 80
(c) (d) beats per minute. What would the doctor infer about
the baby’s heart condition from this?
Br Br
(a) Normal heart rate
73. The amount of metallic Zn (atomic weight = 65.4) (b) Faster heart rate
required to react with aqueous sodium hydroxide to (c) Slower heart rate
produce 1 g of H2 , is (d) Defective brain function
(a) 32.7 g (b) 98.1 g (c) 65.4 g (d) 16.3 g 79. Three uniformly watered plants i, ii and iii were kept
12
74. Natural abundances of C and C isotopes of carbon 13 in 45% relative humidity, 45% relative humidity with
are 99% and 1%, respectively. Assuming they only blowing wind and 95% relative humidity,
contribute to the mol. wt. of C2F4, the percentage of respectively. Arrange, these plants in the order
C2F4 having a molecular mass of 101 is (fastest to slowest) in which they will dry up.
(a) 1.98 (b) 98 (c) 0.198 (d) 99 (a) i → ii → iii (b) ii → i → iii
75. 2,3-dimethylbut-2-ene when reacted with bromine (c) iii → ii → i (d) iii → i → ii
forms a compound which upon heating with alcoholic 80. Many populations colonising a new habitat show a
KOH produces the following major product. logistic population growth pattern over time, as
shown in the figure below
(a) (b)
Br OH
Population size

OH
OH
(c) (d)

OH OH

Time
BIOLOGY
In such a population, the population growth rate
76. Sister chromatids of a chromosome have
(a) stays constant over time
(a) different genes at the same locus
(b) increases and then reaches a asymptote
(b) different alleles of the same gene at the same locus
(c) decreases over time
(c) same alleles of the same gene at the same locus
(d) increases to a maximum and then decreases
(d) same alleles at different loci

Answers
PART-I
1 (c) 2 (c) 3 (c) 4 (b) 5 (c) 6 (a) 7 (c) 8 (c) 9 (c) 10 (d)
11 (b) 12 (c) 13 (a) 14 (b) 15 (b) 16 (a) 17 (b) 18 (a) 19 (d) 20 (c)
21 (b) 22 (d) 23 (b) 24 (a) 25 (c) 26 (c) 27 (a) 28 (b) 29 (a) 30 (c)
31 (b) 32 (c) 33 (a) 34 (b) 35 (c) 36 (d) 37 (d) 38 (b) 39 (b) 40 (d)
41 (b) 42 (b) 43 (a) 44 (b) 45 (c) 46 (a) 47 (a) 48 (b) 49 (a) 50 (b)
51 (a) 52 (b) 53 (b) 54 (c) 55 (d) 56 (a) 57 (b) 58 (c) 59 (c) 60 (d)

PART-II
61 (b) 62 (d) 63 (d) 64 (c) 65 (b) 66 (a) 67 (a) 68 (a) 69 (a) 70 (d)
71 (a) 72 (c) 73 (a) 74 (a) 75 (b) 76 (c) 77 (a) 78 (c) 79 (b) 80 (d)
https://iit-jeeacademy.blogspot.com

100 KVPY Question Paper 2013 Stream : SA

Solutions
1. (c) We have, x + y + z = 10 4. (b) We have, In ∆PRM,
Let three number x + 1, y + 1, z + 1 ⇒ ax2 + (a + b)x + b = 0 RM 2 = PR 2 − PM 2 = 25 − 9 = 16
⇒ ax2 + ax + bx + b = 0 RM = 4
AM ≥ GM
⇒ (ax + b) (x + 1) = 0 PM 3
(x + 1) + ( y + 1) + (z + 1) In ∆PRM, tanθ = = …(i)
≥ b RM 4
3 ⇒ ,−1 x=− OP r
a In ∆POR, tanθ = = …(ii)
[(x + 1) ( y + 1) (z + 1)]1/3 PR 5
It has at least one negative root, i.e. − 1.
x+ y+ z+ 3
⇒ ≥ So, it has both roots are real. From Eqs. (i) and (ii), we get
3 3 r 15
∴Option (b) is correct. = ⇒r =
(xyz + xy + yz + xz + x + y + z + 1)1/3 4 5 4
3
5. (c) Given,
 13  ≥ xyz + xy + yz + xz + 11 x y z y z x 8. (c) Given, ABC is an acute angle
⇒   ⇒ + + = 7⇒ + + = 9 triangle.
 3 y z x x y z
∠B = 45°
Now, x, y, z are integer. We know that,
A
∴ xyz + xy + yz + xz + 11is also integer. a3 + b3 + c3 − 3abc = (a + b + c)
3 (a 2 + b2 + c2 − ab − bc − ca )
∴  is also integer.
13 C1 B1
 3 ∴a3 + b3 + c3 − 3abc
  13  3    13  3  = [(a + b + c)2 − 3(ab + bc + ca ))]
∴     = 81 Q   = 8137
.   x
3 3
x y z
3
  3     3  ⇒   +   +   − 3 =  + + 
y z

 y  z  x y z x
∴xyz + xy + yz + xz + 11 ≤ 81
 x y z
2
x y z  45°
⇒ xyz + xy + yz + xz ≤ 70  + +  − 3  + +   B D
C
  y z x z x y 

∴Maximum value of xyz + xy + yz + xz is
69. x3 y3 z3 A1
∴ 3
+ 3
+ − 3 = (7) [72 − 3 × 9]
2. (c) Given, 2013 + a = b 2 2 y z x3 ∠ADC = 90°
= 7 (49 − 27) = 7 × 22 = 154
⇒ b2 − a 2 = 2013 [Q AD is altitude]
6. (a) In ∆ABC, ∠A < ∠B < ∠C ∠BAD = 45° = ∠BAA ′
⇒ (b − a ) (b + a ) = 3 × 11 × 61
B Similarly, ∠BCC1 = 45°
ab is minimum. ∴ ∠BAA1 = ∠BB1 A1
When b − a = 33 and b + a = 61 D [Q angle on same segment are equal]
On solving, we get a = 14 and b = 47 ∠BCC1 = ∠BB1C1
A C
∴Minimum value of ab = 14 × 47 = 658 [Q angle on same segment are equal]
In ∆ABD, ∠D > ∠C
3. (c) We have sides of triangle are, ∴ ∠A1 B1C1 = ∠BB1 A1 + ∠BB1C1
So, ∆ABD not similar to ∆ABC.
b + 5, 3b − 2, 6 − b = 45° + 45° = 90°
7. (c) Given, 9. (c) Given, ABCD is rectangle.
Triangle are isosceles.
PR and QR are tangents. ∴ AB = CD, BC = AD
∴Two sides are equal.
Case I b + 5 = 3b − 2 P 2x
A B
7
∴ b= 5 r
2 y
17 17 5 θ 3
So, sides are , , . R O 2y
2 2 2 4 6 M X
Case II 3b − 2 = 6 − b ⇒ b = 2 r y
5
∴Sides are 7, 4, 4 Q E
1 2x D x Y x C
Case III b + 5 = 6 − b ⇒ b =
2 PQ = 6
11 1 11 2y
Sides are , − , which is not PR = 5
2 2 2 1
possible. PM = PQ
2 F
∴ Only for two values of b, triangles are 1
∴ PM = × 6 = 3 X and Y are mid-point of AD and CD
isosceles. 2 respectively.
https://iit-jeeacademy.blogspot.com

KVPY Question Paper 2013 Stream : SA 101

Let AB = 2x , BC = 2 y 1 13. (a) Given, n! + 10


× AQ × AP × sin 30°
Area of ∆PAQ 2
∴AX = XD = y ∴ = Put n = 1, 2, 4, 5
Area of ∆RSP 1
× RS × PS × sin 30°
DY = YC = x 2 n! + 10 is not a perfect square.
Area of rectangle ABCD = 4xy = 60 2× 2 Put n = 3, 3! + 10 = 6 + 10 = 16 is a perfect
= =2
⇒ xy =15 1 square.
In ∆ABX and ∆DEX, [Q AQ = AP = 2 , RS = PS = 1] If n>5
∆ABX − ~ ∆DEX 11. (b) Given, n! is multiple of 10.
∴ DE = AB = 2x X complete one round in 40 s. ∴ n ! = 10k
Similarly, ∆CBY ~
− ∆DFY ∴ 2 π = 40 s n ! + 10 = 10k + 10 = 10(k + 1)
∴FD = BC = 2 y (when, k is even)
= 10 (2m + 1)
∴Area of ∆BEF
= 2 × 5 (2m + 1)
= Area of ∆EFX + Area of ∆ BFX
θ ∴Product of odd and even is not a perfect
1 1
= FX ⋅ DE + FX ⋅ AB square.
2 2
1 1 14. (b) We have 10 points lie a plane
= × 3 y × 2x + × 3 y × 2x such that no three of them are collinear.
2 2
= 6xy 2 3
2π  4
= 6 × 15 = 90 [Qxy =15] In one second, he complete,   round 5
10. (d) Given,  40  1
2 π × 15 
In 15 s, he complete 
6
A 1 B  round
 40 
10 7
S 1 LetY complete one round in t s 9 8
1
1 ∴ 2π = t According to question only 5 ways are
F C possible i.e. 1- 6, 2 - 7, 3 - 8, 4 - 9 and 5 - 10.
In one second Y complete 
R 2π 
1 Q  round
1  t  15. (b) Let total number of people whose
1 1 salary less than ` 10000 per annum = x
2π 
In 15 s,Y complete 
P
 × 15 round
 t  and annual salary of each person = a
E 1 D
ABCDEF is a regular hexagon of side Since, both are move in opposite direction. ∴Total salary = ax
length 1. 2π 2π
∴ × 15 + × 15 = 2 π and total number of people whose salary
ABQR and AFPS is a square of each side 40 t more than ` 10000 per annum = y
15  +  = 1
1 1
length also 1. ⇒ and annual salary of each person = b
 40 t 
ADCDEF is a regular hexagon ∴Total salary = bx
1 1 1
∴ ∠FAB = 120° ⇒ = − When 5% increase of salary of people x
t 15 40
In square ABQR, 1 8− 3 105ax
⇒ = i.e. x (a + 5% of a) =
AB = BQ = 1 t 120 100
5 1 and 5% decrease of salary of people y
AQ is a diagonal of square = = 95 by
∴ AQ = AB 2 + BQ 2 = 2
120 24 i.e. y (b − 5% of b) =
∴ t = 24 s 100
⇒ ∠BAS = ∠FAB − ∠FAS 105ax 95by
12. (c) We have, +
Average salary after
= 120° − 90° = 30° n + 1 − n − 1 < 0.2, n ∈ N = 100 100
Average salary before ax + by
⇒ ∠SAR = ∠BAR − ∠BAS ⇒ n + 1 < 0.2 + n − 1
= 90° − 30° = 60° 5  ax − by 
On squaring both side, we get = 1+  
100  ax + by 
⇒ ∠ASR = 60° n + 1 < 0.04 + n − 1 + 0.4 n − 1
[Q ∆ARS is an equilateral triangle] ⇒ n + 1 − n + 1 − 0.04 < 0.4 n − 1 ax − by < 0
⇒ ∠RSP = ∠ASP − ∠ASR 2 − 0.04 ∴Average salary after be decreases.
⇒ < n −1
= 90° − 60° = 30° 0.4 16. (a) As centre of mass is subjected to
⇒ ∠FAB = ∠FAP + ∠PAQ + ∠QAB
⇒ 4.9 < n − 1 a downward external force only, so its
⇒ n − 1 > (4.9)2 motion is along the direction of external
⇒ 120° = 45° + ∠PAQ + 45°
⇒ n > 1 + 24.01 force, i.e. downwards. Any internal force
[Q ∠FAP = ∠QAB = 45° does not change position of centre of
FA = FP and AB = BQ] ⇒ n > 25.01
mass.
∴ ∠PAQ = 30° ∴Minimum value of n = 26
https://iit-jeeacademy.blogspot.com

102 KVPY Question Paper 2013 Stream : SA

17. (b) When ball is released, vertical As both g and ∆t are constants. 23. (b) Rear view mirror is a convex
component of ball’s velocity first ∴∆v is constant and its value does not mirror.
increases in negative direction changes with time t. Here, u = − 10 m
(downwards), then on collision with floor, The mutual separation ∆s of the stones is = +   m
R 1.5
∴ f =
its velocity is reversed (upwards).
∆s = s1 − s2 2  2
As acceleration remains constant, so lines
= − gt 2 −  − g (t − ∆t )2 
1 1 Now, from mirror equation,
are parallel to each other as given in  2  1 1 1 1 1 1 2 1
2
option (b). + = or = − = −
1 v u f v f u 1.5 (−10)
18. (a) Pressure of a fluid column = g ((t − ∆t )2 − t 2 )
2 4 1 43 30
depends only on height of fluid column = + = or v = m
1
and as pressure is scalar, its magnitude = g (t 2 + ∆t 2 − 2t∆t − t 2 ) 3 10 30 43
2 Now, magnification,
does not depend on orientation of surface
1
= g (− 2t∆t + ∆t 2 ) −  
over which pressure acts. 30
2 −v  43 
19. (d) We have following observations 1 m= = = 0.069 or m = 0.07
from position x versus time t graph. ⇒ ∆s = g (∆t 2 − 2t∆t ) u − 10
2
Velocity or slope
is again negative
Clearly, ∆s decreases with time and 24. (a) first we distribute current in
x becomes zero when 2t = ∆t. circuit given as
21. (b) Refractive index of a material is I2–I4
Slope or velocity
is negative inversely proportional to wavelength of I2
I4 I5
x Slope or velocity light. I3
decreases is positive 1 3 I2–I4 –I5
2
⇒ µ∝
Slope or velocity changes λ 1
4
I1
from negative to positive
t
Now, λ yellow > λgreen > λ blue
⇒ µ yellow < µ green < µ blue I2+I3
From above graph we can draw following or µ3 < µ1 < µ 2 Current distribution must follows
velocity v versus time t graph.
22. (d) As focal length of lens is 20 cm, Kirchhoff’s junction rule.
v
point of Now, from closed loops marked 1, 2, 3
Slope or acceleration I and 4, we have following set of equations
is positive Slope or by application of Kirchhoff’s loop rule,
acceleration
I1 = I 2 + I3 …(i)
is negative
I3 = I 2 + I 4 …(ii)
M I′
P I 4 = I 2 − I 4 + I5
t ⇒ 2I 4 = I 2 + I5 …(iii)
convergence of a parallel beam of light is I5 = 2 (I 2 − I 4 − I5 )
So, acceleration of given particle is also 20 cm.
⇒ I5 = 2I 2 − 2I 4 − 2I5 …(iv)
initially positive but with time it becomes Now, given PM = 10 cm
3I5 = 2I 2 − 2I 4 …(v)
negative. So, PI ′ = PM + MI = 20 cm
From Eqs. (iii) and (v), we have
Hence, a = p − qt is best suited option. or MI = 20 − 10 = 10 cm
3I5 = 2I 2 − (I 2 + I5 )
20. (c) Let first stone mass m1 is dropped As, PI = 10 cm
⇒ 4I5 = I 2 …(vi)
at instant t = 0. ∴∆PMI is an equilateral triangle of side
From Eqs. (iii) and (vi), we have
Then at time t, its velocity and 10 cm.
5
displacement respectively, are Now, if MN is normal to mirror, as angle 2I 4 = 4I5 + I5 ⇒ I 4 = I5 …(vii)
2
1 of incidence and reflection are equal, we
v1 = − gt and s1 = − gt 2 From Eqs. (ii), (vi) and (vii), we have
2 have following situation
5 13
As, second stone mass m2 is dropped ∆t I I3 = 4I5 + I5 = I5 …(viii)
2 2
time after, so its velocity and displacement
at instant t respectively, are 30° Now, marked currents I and I ′ in the
N α
given circuit are
v2 = − g (t − ∆t ) 30°
I′
I ′ = (I 2 − I 4 − I5 ) =  4I5 − I5 − I5 
1 P α M 5
and s2 = − ( g ) (t − ∆t )2  2 
2
8 − 5 − 2
= 
I
Difference in speeds of stones is From above figure, we have  I5 = 5 …(ix)
 2  2
∆v = v1 − v2 α + 30° = 90° ⇒α = 60°
= (− gt ) − (− g (t − ∆t )) And I = I 2 = 4I5
Hence, mirror makes an angle of 60° with
= − gt + gt − g∆t = − g∆t Hence, ratio of I / I ′ = (4I5 ) / (I5 / 2) = 8.
the horizontal.
https://iit-jeeacademy.blogspot.com

KVPY Question Paper 2013 Stream : SA 103

25. (c) An emf is induced in the coil 4 × 22 × 14


. × 103 × (15
. × 1011 )2 31. (b) A formyl group is one which
=
when there is a flux change in the coil or 7 × (3 × 108 )2 consists of a carbonyl group attached to a
when field lines are cut by the coil. hydrogen, i.e.,
≈ 109 kg s−1
When coil rotates about its axis, there is R — C — H group.
So, mass reduction per second is around ||
no change in flux as no field line is cut by
109 kg. O
the coil. The structure of the given organic
28. (b) Power dissipated by resistor is
ω compounds are as follows
∆P 2∆I
B P = I 2R ⇒ =
P I (i) Acetone CH3 C CH3
∆P 2∆I
So, no emf is generated in coil when it is or × 100 = × 100
P I O
rotated about its axis. In all other cases
an emf is induced in the coil. Per cent change in power dissipation (ii) Acetaldehyde CH3 C H
= 2 × Per cent change in current
26. (c) Given circuit is O
A = 2 × 3% = 6%
80 Ω 0.1A 29. (a) Initially let pressure is p, then (iii) Acetic acid CH3OH
+ 25 Ω pV = n1 RT
V (iv) Acetic anhydride CH3 C O C CH3
– 20 Ω V
Finally pressure is p and volume is .
60 Ω 3 O O
20 Ω
Let number of moles of gas left is n2, then
As acetaldehyde has R  C  H group,
B V
p = n2RT ||
20 × 60 3 O
Req = = 15 Ω
20 + 60 Dividing both equations, we get Thus, the correct option is (b).
We can redraw the circuit as n2 1 n 1 n − n2 2
= or 1 − 2 = 1 − or 1 = 32. (c) The structure of cis-3-hexene is
A I2 n1 3 n1 3 n1 3 2 5
1
0.1A n1 − n2 2 6
80 Ω Hence, × 100 = × 100 = 66% 3 4
+ 25 Ω n1 3 . H H
V
20 Ω
– or percentage of number of molecules 33. (a) Generally a sp 2-hybridised
15 Ω escaped = 66%. carbon atom is one which has a double
bond.
B 30. (c) Using right hand rule, direction O
Let current through 20 Ω resistor is I 2, then of magnetic force can be found. It acts
outward from the plane of paper in given HC C CH2 C CH2 CH CH2
V AB = 01. (25 + 15) = I 2 × 20 case. sp sp sp3 sp2 sp3 sp2 sp2
. × 40
01
or I2 = = 0.2 A ∧
20 j B Thus, the given structure has
So, by Kirchhoff’s junction rule, current 3 sp 2-hybridised carbon atoms.
through 8 Ω resistance is 34. (b) Valence electrons are those
I = 01
. + 0.2 = 0.3 A. e electrons which are present in the
outermost shell of an element.
27. (a) Energy from sun is radiated in a
O v∧ In the given electronic configuration,
sphere of radius (r = 15
. × 1011 m).
∧ i 1s2 2s2 2 p6 3s2 3 p3 , 3p is the outermost
–k
Earth Fm shell, thus the number of valence
To avoid deflection of electron, electric electrons are 3.
r field must be applied normal to the plane 35. (c) Number of neutrons in the pairs
Sun
of paper (XOY -plane in diagrams) and given in options are as follows
pointing outward. 12 24
(a) 6
C, 12
Mg
y
B No. of neutrons in C = 12 − 6 = 6
No. of neutrons in Mg = 24 − 12 = 12
So, total energy radiated per second from Fe 23 19
sun is E (b)
11
Na , 9
F
∆E = 4 πr 2 × 14
. × 103 No. of neutrons in Na = 23 − 11 = 12
22 No. of neutrons in F = 19 − 9 = 10
= 4× × 14
. × 103 × (15
. × 1011 )2 O
x v
7 23 24
From E = mc2 , we have Fm (c)
11
Na , 12
Mg
∆E Magnitude of electric field applied is such No. of neutrons in Na = 23 − 11 = 12
⇒ ∆m = 2
c that Fm = Fe . No. of neutrons in Mg = 24 − 12 = 12
https://iit-jeeacademy.blogspot.com

104 KVPY Question Paper 2013 Stream : SA

23 39
(d) Na , 19
K 40. (d) For the reaction, 45. (c) According to Bohr’s radius of an
11
atom
No. of neutrons in Na = 23 − 11 = 12 3C2H2 ( g ) r C6 H6 ( g )
n2 1
No. of neutrons in K = 39 − 19 = 20 [C6 H6 ] 0.5 r = 0.529 × ⇒ r∝
Kc = ⇒4= Z Z
Thus, option (c) is correct. [C2H2 ]3 [C2H2 ]3
where, Z is the atomic number.
36. (d) A molecule which has a 0.5 1
[C2H2 ]3 = = Thus, more is the atomic number lesser
symmetrical geometry will have no dipole 4 8 will the Bohr’s radius. Therefore, the
moment, as the magnitude of all the bond 1 correct order is
[C2H2 ] = = 0.5 mol/L
moments cancel each other. 2 rH > rHe + > rLi 2 +
The structure of compounds given in 46. (a) In Bowman’s capsule
41. (b) Given,
options are as follows ultrafiltration of blood occurs. Bowman’s
3.42
Cl H Weight of sucrose = = 0.0342 g capsule is a cup-like sack at the
100 beginning of the tubular component of a
C , C Molar mass of sucrose = 342 g nephron in the mammalian kidney that
H H Cl Cl Mass of solution = 100 g performs the first step in the filtration of
H Cl
CH3Cl CHCl3 Density of solution = 1 g mL−1 blood to form urine. Fluids from blood in
(µ = 0) (µ = 0) 0.0342 the glomerular are collected in the
∴ No. of moles of sucrose = Bowman’s capsule (i.e. glomerular
Cl 342
H H Mass filtrate) and further processed along the
Also, density = nephron to form urine. This process is
C , C Volume
100 known as ultrafiltration.
Cl Cl Cl Cl
∴ Volume = = 100 mL
Cl 1 47. (a) Parietal lobe is sensory lobe for
CCl4 touch, pain and temperature. The
Symmetrical molecule, µ =0 Concentration of solution is calculated in
cerebral cortex is divided into four
terms of molarity
Thus, CCl 4 has no dipole moment. sections, called ‘lobes’. Out of these, the
No. of moles of sucrose
Molarity = parietal lobe is associated with
37. (d) As the species are radioactive, so Vol. of solution in litres movement, orientation, recognition and
they follow Ist order kinetics. 0.0342 0.0342
= = × 1000 = 01 . mol/L perception of stimuli. Thus, the parietal
For Ist order Ct = C0 e− kt 100 100 lobe functions in registration of sensory
1 1000
∴ k∝ perception of touch, pain, heat and cold,
t 42. (b) The reactivity of K, Mg, Au and knowledge about position in space, taking
From the graph it can be concluded that Zn with water can be determined by the in information from environment,
species A takes maximum time to decay reactivity series of metals. organising it and communicating to rest
while species C takes least time. Thus, According to reactivity series, the of brain.
decay constant follows the order decreasing order of reactivity will be 48. (b) Virus cannot be cultured in an
kC > kB > kA . K > Mg > Zn > Au artificial medium. It multiples only in
living cells. Viruses are obligate
38. (b) The ratio of rate of diffusion of 43. (a) An anhydride is a compound that
intracellular parasites.
two gases, O 2 and H 2 can be given as has two acyl groups bonded to same
oxygen atom, i.e. They lack metabolic machinery to
rO 2 MH 2
= generate energy or to synthesise proteins,
rH 2 MO 2 R C O C R
instead they rely on their host cells to
rO 2 2 1 O O carry out these functions.
= =
rH 2 32 4 49. (a) Meiosis-I Reduction division
This type of structure is given in option (a). (2n → n ), separation of homologous
rO 2 : rH 2 = 1 : 4 O O chromosomes results in reduction of
Also, rate of diffusion chromosome ploidy to half.
O
Volume of diffused gas CH3 CH3 Meiosis-II Similar to mitosis where
=
Time of diffused gas 2- acyl groups sister chromatids separate.
rO 2 tH 2 1 24 attached to O-atoms 50. (b) Melanin pigment synthesised
∴ = = = = 96 s from tyrosine amino acid, imparts colour
rH 2 tO 2 4 tO 2 Thus, option (a) is correct.
to skin. People suffering from albinism
44. (b) More reactive metal than Cu can
39. (b) Whenever an acid reacts with a cannot synthesise melanin. Albinism is a
precipitate copper from copper sulphate
metal, hydrogen gas is evolved. So, when disease, in which a person has partial or
solution. The increasing order of
acetic acid reacts with a sodium metal, complete loss of pigmentation (colouring)
reactivity of given element is
hydrogen gas is produced. of the skin, eyes and hair. There is a cell
Au < Hg < Cu < Sn called the melanocyte that is responsible
− As Sn is more reactive than Cu, so it
CH3COOH + Na → CH3COONa + + H2 ↑ for giving eyes, skin and hair
would precipitate Cu from CuSO4 . pigmentation.
https://iit-jeeacademy.blogspot.com

KVPY Question Paper 2013 Stream : SA 105

In albinism, there occurs genetic 57. (b) In the digestive system, the pH of 62. (d) Let the length, breadth and
mutation in melanocytes which interfere stomach and intestine are acidic and height of cuboid be x, y and z respectively.
with their pigment. alkaline, respectively. The pH of stomach ∴Perimeter of face PQRS = 2(x + y)
51. (a) Short-sightedness (myopia) is is 1.5-2.5 (i.e. acidic) and the pH of
S R
corrected by using concave lens. These intestine is 7.4-7.6 (i.e. alkaline).
lens work by bending the light rays 58. (c) The major nitrogenous excretory y
P x Q
slightly outwards, so that they can focus product in mammals is urea. Nitrogenous
further back on the retina. wastes in the body of animals tend to z
Myopia is an eye defect in which the form toxic NH 3 , which must be excreted. D
C
eyeball grows slightly too long. This NH 3 is converted to urea in hepatocytes y
means that light does not focus on the of the body. A x B
light sensitive tissue (retina) at the back 59. (c) Thin epidermis without a cuticle Area of PQRS = xy
of the eye properly. Instead, the light rays on stem and leaves is not an adaptation
∴ 2 (x + y) + xy = 16 …(i)
focus just in front of the retina, resulting to dry habitat. This is because the cuticle
is a waxy layer on the epidermis which Similarly, for face APSD,
in distant objects appearing blurred.
prevents the entire leaf from losing water 2( y + z ) + yz = 24 …(ii)
52. (b) A person with blood group ‘O’ is a
universal donor, whereas person with from the surface. Thus, thick cuticle and for face APQB,
blood group ‘AB’ is a universal recipient. prevents water loss. 2(x + z ) + xz = 31 …(iii)
Blood group ‘O’ do not have any antigen 60. (d) Diversity of species is highest in From Eqs. (ii) and (iii), we get
on their RBC whereas blood group ‘AB’ the tropical evergreen forests primarily
(x − y ) (2 + z ) = 7 …(iv)
do not have any antibody in their blood because there are fewer ecological
obstacles for biodiversity. Like the From Eqs. (ii) and (iv), we get
plasma, but both the antigens A and B on
their RBC. climate is wet and warm, plants and 4x = 2 + 5 y …(v)
Therefore, a person with blood group ‘A’ animals have the greatest access to On solving Eqs. (i) and (v), we get
can donate blood to a person with blood consistent energy, water and carbon, etc. x = 3, y = 2, z = 5
group ‘A’ or ‘AB’ and can receive blood This reduces the selection for traits that ∴Volume of cuboid = xyz = 3 × 2 × 5 = 30
from a person with blood group ‘A’ or ‘O’. emphasise the ability to withstand
Hence, option (d) is correct.
environmental stresses such as cold and
53. (b) Mitochondria are structures drought, etc., and promotes higher rates 63. (d) Given, ABCD is a square.
within cells that convert the energy from
of speciation. x P 2x
food into a form that cells can use. D C
Although most DNA is packaged in 61. (b) We have,
chromosomes within the nucleus, a , b, c, d , e are natural number and in AP.
mitochondria also have a small amount of Let D is common difference of AP. Q
their own DNA. This genetic material is ∴Let c=C 3x 3x
known as mitochondrial DNA or
a = C − 2D
mtDNA. Thus, after the removal of
nuclei, the cell still have mtDNA. b=C − D
54. (c) The correct combination present d =C + D
A B
in DNA is guanine and cytosine. The e = C + 2D 3x
DNA consists of four types of nitrogen
a + b + c + d + e = 5C Let AB = BC = CD = AD = 3x
bases, i.e. adenine (A), thymine (T),
and b + c + d = 3C PD : PC = 1 : 2
guanine (G) and cytosine (C). Whereas
guanidine is a strong base that found in Given, a + b + c + d + e is a cube of ∴ PD = x
urine as a normal product of protein number PC = 2x
metabolism and not present in DNA. ∴ 5C = λ3 …(i) In ∆DAP and ∆QBA,
55. (d) Conjugation is the transfer of and b + c + d is a square of number ∠DAP = ∠QBA
genetic material between bacterial cells
∴ 3C = u 2 …(ii) ∠D = ∠Q = 90°
by direct cell to cell contact or by a
bridge-like connection between two cells From Eqs. (i) and (ii), we get ∴ ∆DAP ~ ∆QBA
(e.g. bacteria). Thus, conjugation is a λ3 u2 DA AP DP
= ∴ = =
process of genetic recombination not 5 3 QB BA QB
asexual reproduction.
λ3 and u 2 is a multiple of 15. 3x 10 x x
56. (a) Insects (class–Insecta or ⇒ = =
Hexapoda) are the animals constituting ∴Smallest possible value of λ = 15 and QB 3x QA
the largest biomass on the earth. In the u = 45
[Q AP = 9x2 + x2 = 10 x]
world, about 900 thousand different u 2 (45)2
∴ c= = = 675 9 3
kinds of living insects are known. This 3 3 ∴ QB = x ⇒ QA = x
represents approximately 80% of the 10 10
∴Number of digits = 3
world’s animal species.
https://iit-jeeacademy.blogspot.com

106 KVPY Question Paper 2013 Stream : SA

Area of quadrilateral BQPC = area of 3x L L L


65.(b) We have, three kind of liquids ⇒ − + x− − =0
square ABCD − (area of ∆APD + area of x, y, z and three jars J1 , J 2 , J3 contains 2 2 3 2
∆ABQ) 100 ml of liquids X , Y , Z respectively. ⇒
5x 4L
=
= (3x)2 −  × 3x × x + ×
1 1 9 3  When 10 ml of J1 transfer to J 2 2 3
x× x
2 2 10 10  ∴J1 = 90 ml of X, J 2 = 100 ml ofY and ⇒ x=
8L
27 2  123 x2 15
= 9x −  x2 +
3 10 ml of X.
2
x  =
2 20  20 When 10 ml of J 2 transfer to J3 67. (a) Case I P runs towards Q, while Q
123x2 1000 100
J2 = ofY and of X, J3 = 100 ml is stationary.
Area of quadrilateral PQBC 11 11
= 202 100 10 P Q
Area of square ABCD 9x of Z1 , ofY and of X
11 11 x
41 First ball is received at time t1 = .
= When 10 ml of J3 transfer to J1 2
60
1100 1100 110 ∴Next ball is received at time t2
64. (c) Let the side of square base of J3 = of Z1 , ofY1 , of X and
11 11 11 x−5 x 5
pyramid is x m and height of pyramid is = + 5= +
2 2 2 2
J1 = 90 + 10 ×   of X1 ,
y m. 1 100
ofY and 5
 11 121 So, ∆t = t2 − t1 = s = 2.5 s
100 2
of Z
11 Case II Q runs towards P, while P is
Similarly, we can find four operation of stationary.
y amount of X , Y , Z in J1 . P Q
We get x > z > y. x
First ball is received at time t1 = .
66. (a) For system to be in equilibrium 3
without toppling, following conditions ∴Next ball is received at time t2
must be fulfilled. x−5
x x = + 5
(i) Centre of mass C1 of sphere and upper 3
block must lie inside the edge of lower x 10
= +
block. 3 3
L–y 10
x 2 So, ∆t = t2 − t1 = s = 3.3 s
Volume of pyramid 3
C
1 1 68. (a) Energy supplied by heater = Heat
= area of base × height = x2 y y
3 3 absorbed by water + Heat absorbed by oil
When x is increased by p%, So, with water in container,
then new length= x + p% of P∆t = mw sw ∆T + mo so ∆T
100 + p  ⇒10 × 15 × 60 = 0.5 × 4200 × 3 + mo so × 3
x =   x
 100  Taking origin of axes choosen at C, we ⇒ mo so = 900 J K −1
When y is decreased by p%, have Now with oil in container,
× y = M  − y
M L P∆t = mo so ∆T + mc sc ∆T
then new height
2  2 
100 − p  ⇒ 10 × 20 × 60 = 2 × so × 2 + 900 × 2
= y − p% of y =   y y L L
 100  ⇒ + y= or y = ⇒ so =
10200
= 2.5 × 103 J K−1 kg −1
2 2 3 4
Now, volume is same.
2 (ii) Centre of mass of both of block and
1 100 + p   100 − p 
x y = 
1 2 69. (a) According to condition given in
∴ x   y sphere must lie inside the edge of table.
3 3  100   100  question, ray diagram of sphere is
2
100 + p   100 − p 
⇒ 1 =    
 100   100  r r
C1 C
⇒ (100) (100) = (10000 + 200 p + p )
2 2 L/3
C2 L/2
(100 − p ) R
⇒ p + 100 p − 100 = 0
2 2
π/4
x
⇒ p 2 + 100 p + (50)2 = (100)2 + (50)2
⇒ ( p + 50)2 = 12500
π/4
⇒ p + 50 = 12500 = 11180 . So, again taking centre of mass C2 as
⇒ p = 11180
. − 50 origin,
So, deviations are
3M  L  L L
⇒ p = 6180
. x −  + M x − −  = 0 δ1 =
π
−r
2  3  3 2
∴ 60 < p < 65 4
https://iit-jeeacademy.blogspot.com

KVPY Question Paper 2013 Stream : SA 107

δ 2 = π − 2r 73. (a) revive the individual we need to provide


π Zn + 2NaOH(aq) → Na 2ZnO4 + H2 ↑ him sugar, so that the blood sugar level
δ3 = − r
4 1 mole of H2 is produced by 1 mole of Zn, becomes normal.
Total deviation of light ray is i.e. 2 g of H2 is produced by 65.4 g of Zn 78. (c) A normal Foetal Heart Rate
3π ∴1 g of H2 is produced by =
65.4
g = 32.7 g (FHR) usually ranges from 120-160 beats
δ = δ1 + δ 2 + δ3 = − 4r
2 2 per minute. It is measurable
1 74. (a) In molecular formula of C2F4 , sonographically. Therefore in this case,
70. (d) From, qV = mv2 there are 4 F atoms. F has atomic mass of where the foetal heart rate is 80 beats
2
19, so 4 F would have atomic mass of per minute is a slower heart rate (foetal
where, V is stopping potential. 76 g. So, possible molar mass of C2F4 are bradycardia).
mv2 100 (76 + 12 + 12), 102 (76 + 13 + 13) or
V = 101 (76 + 12 + 13).
79. (b) The plants in the order (fastest to
2q slowest) in which they will dry up is
Now according to % abundance
9 × 10−31 × (4 × 106 )2 ii → i → iii.
= % of C2F4 of molar mass 100 (when both
Relative humidity is the amount of water
. × 10−19
2 × 16 the C are 12 C)
vapour present in air expressed as a
1 1
≈ 45 V = × × 100 = 0.01% percentage of the amount needed for
100 100 saturation at the same temperature. As
So, electron must move across a potential % of C2F4 of molar mass 102 (when both plants transpire, the humidity around
difference of 45 V from higher to lower the C are 13 C) saturates leaves with water vapour.
potential. 99 99
= × × 100 = 98.01% When relative humidity levels are too
71. (a) Given, 100 100 high or there is a lack of air circulation, a
Concentration of acetic acid, C = 01
. M % of C2F4 of molar mass 101 plant cannot make water evaporate by
= 100 − (98.01 + 0.01) = 198
. % transpiration or draw nutrients from the
K a of acetic acid = 10−5 soil. Therefore 95% relative humidity will
75. (b) 2,3 dimethylbut-2-ene when
According to Ostwald dilution law, reacts with bromine forms 2,3 dibromo dry up the slowest, and the 45% relative
Ka = α 2C 2,3 dimethyl butane which upon heating humidity with blowing wind will dry up
10−5 = α 2 × 01
. with alcoholic KOH produces 2,3 the fastest.
10−5 −5
dimethylbut-1,3-diene as a major
80. (d) The population growth pattern
= 10 × 10 = α 2 product. The reaction can be written as
01
. shown here is S-shaped (Sigmoidal
Br Br
α 2 = 10−4 curve). It is a population growth curve
α = 10−2 Br2/CCl4 that shows an initial rapid growth
(exponential growth) and then it slows
72. (c) Skeletal diagram of given
2, 3 dimethyl 2, 3 dibromo 2, 3 dimethyl butane down (decreases) as the carrying capacity
information can be drawn but-2-ene is reached. Carrying capacity is the
Zn O3/Zn, H2O alc. KOH
X Y 2CH3CH2CHO maximum number of individuals in a
Propionaldehyde population that the environment can
2, 3 dimethylbut-1, 3-diene
support.
On retro synthesis.
76. (c) Sister chromatids contain the
O same allele of the same gene at the same Carrying capacity
Zn, H2O
Ozonolysis
2CH3CH2CHO loci whereas non-sister chromatids
contain different alleles of the same gene
O O
in the same loci. Growth rate
Ozonide Sister chromatids are the two chromatids decreasing
Population size

Ozonolysis O3 of a replicated chromosome, which are


connected by the centromere. They are
identical to each other since they are
(Y) Growth rate
produced by DNA replication.
Br ∆ Zn dust increasing
77. (a) Insulin lowers blood sugar level
and in this case, brain is getting
inadequate sugar/glucose. Therefore, the Time
Br (X) person becomes unconscious. In order to
https://iit-jeeacademy.blogspot.com

KVPY Question Paper 2012 Stream : SA

KVPY
KISHORE VAIGYANIK PROTSAHAN YOJANA

QUESTION PAPER 2012


Stream : SA
MM 100

Instructions
There are 80 questions in this paper.
This question paper contains two parts; Part I and Part II. There are four sections; Mathematics, Physics, Chemistry
and Biology in each part.
Out of the four options given with each question, only one is correct.

PART-I (1 Mark Questions)


MATHEMATICS 5. For
22 + 42 + 62 + ... + (2n )2
to exceed 1.01, the
1. Let f (x) be a quadratic polynomial with f (2) = 10 and 12 + 32 + 52 + ... + (2n − 1)2
f (− 2) = − 2. Then, the coefficient of x in f (x) is maximum value of n is
(a) 1 (b) 2 (c) 3 (d) 4 (a) 99 (b) 100 (c) 101 (d) 150

(0.75)3 6. In ∆ABC, let AD, BE and CF be the internal angle


2. The square root of + [0.75 + (0.75)2 + 1] is bisectors with D, E and F on the sides BC , CA and AB
1 − (0.75)
respectively. Suppose AD, BE and CF concur at I and
(a) 1 (b) 2 (c) 3 (d) 4 B, D, I, F are concyclic, then ∠IFD has measure
3. The sides of a triangle are distinct positive integers (a) 15°
in an arithmetic progression. If the smallest side is (b) 30°
10, the number of such triangles is (c) 45°
(a) 8 (b) 9 (d) any value ≤ 90°
(c) 10 (d) infinitely many 7. A regular octagon is formed by cutting congruent
4. If a , b, c, d are positive real numbers such that isosceles right angled triangles from the corners of a
a a + b a + b+ c a + b+ c+ d a square. If the square has side length 1, the side
= = = , then length of the octagon is
3 4 5 6 b + 2c + 3d
2−1
is (a) (b) 2 − 1
1 2
(a) (b) 1 5−1 5−1
2 (c) (d)
(c) 2 (d) not determinable 4 3
https://iit-jeeacademy.blogspot.com

KVPY Question Paper 2012 Stream : SA 109

8. A circle is drawn in a sector of a larger circle of 1


hand, working alone, B would need 4 h more to
radius r, as shown in the figure given below. 2
complete the work than if both worked together. How
much time would they take to complete the job
working together?
(a) 4 h (b) 5 h (c) 6 h (d) 7 h
13. When a bucket is half full, the weight of the bucket
and the water is 10 kg. When the bucket is
60° two-thirds full, the total weight is 11 kg. What is the
total weight (in kg), when the bucket is completely
r
full?
The smaller circle is tangent to the two bounding 1 2
(a) 12 (b) 12 (c) 12 (d) 13
radii and the arc of the sector. The radius of the 2 3
small circle is 14. How many ordered pairs of (m, n ) integers satisfy
r r m 12
(a) (b)
2 3 = ?
12 n
2 3r r
(c) (d) (a) 30 (b) 15 (c) 12 (d) 10
5 2
15. Let S = {1, 2, 3, ..., 40) and let A be a subset of S such
9. In the figure, AHKF , FKDE and HBCK are unit
that no two elements in A have their sum divisible by
squares, AD and BF intersect in X. Then, the ratio of
5. What is the maximum number of elements
the areas of triangles AXF and ABF is
possible in A?
A F E
(a) 10 (b) 13 (c) 17 (d) 20

X
PHYSICS
H D
K
16. A clay ball of mass m and speed v strikes another
metal ball of same mass m, which is at rest. They
stick together after collision. The kinetic energy of
B C the system after collision is
(a)
1
(b)
1
(c)
1
(d)
1 (a) mv2 / 2 (b) mv2 / 4 (c) 2 mv2 (d) mv2
4 5 6 8 17. A ball falls vertically downward and bounces off a
10. Suppose Q is a point on the circle with centre P and horizontal floor.The speed of the ball just before
radius 1, as shown in the figure, R is a point outside reaching the floor (u1 ) is equal to the speed just after
the circle such that QR = 1 and ∠QRP = 2°. Let S be leaving contact with the floor (u2 ), u1 = u2. The
the point where the segment RP intersects the given corresponding magnitudes of accelerations are
circle. Then, measure of ∠RQS equals denoted respectively by a1 and a 2. The air resistance
during motion is proportional to speed and is not
negligible. If g is acceleration due to gravity, then
(a) a1 < a2 (b) a1 > a2
Q
1 1 (c) a1 = a2 ≠ g (d) a1 = a2 = g

R S P
18. Which of the following statements is true about the
2° flow of electrons in an electric circuit?
(a) Electrons always flow from lower to higher potential
(b) Electrons always flow from higher to lower potential
(c) Electrons flow from lower to higher potential, except
(a) 86° (b) 87° (c) 88° (d) 89° through power sources
(d) Electrons flow from higher to lower potential, except
11. Observe that, at any instant, the minute and hour through power sources
hands of a clock make two angles between them
whose sum is 360°. At 6:15 the difference between 19. A boat crossing a river moves with a velocity v
these two angles is relative to still water. The river is flowing with a
(a) 165° (b) 170° (c) 175° (d) 180° velocity v / 2 with respect to the bank. The angle with
respect to the flow direction with which the boat
12. Two workers A and B are engaged to do a piece of should move to minimize the drift is
work. Working alone, A takes 8 h more to complete
(a) 30° (b) 60° (c) 150° (d) 120°
the work than, if both worked together. On the other
https://iit-jeeacademy.blogspot.com

110 KVPY Question Paper 2012 Stream : SA

20. In the Arctic region, hemispherical houses called


Igloos are made of ice. It is possible to maintain a
temperature inside an Igloo as high as 20°C because
(a) ice has high thermal conductivity
(b) ice has low thermal conductivity S
(c) ice has high specific heat R
(d) ice has higher density than water (a) P < Q < R < S (b) P < Q < R = S
21. In the figure below, PQRS denotes the path followed (c)R < Q < P < S (d) P > R > Q > S
by a ray of light as it travels through three media in 25. A circular metallic ring of radius R has a small gap of
succession. The absolute refractive indices of the width d. The coefficient of thermal expansion of the
media are µ1 , µ 2 and µ3 , respectively. (The line metal is α in appropriate units. If we increase the
segment RS in the figure is parallel to PQ). temperature of the ring by an amount ∆T, then width
P of the gap
µ1
(a) will increase by an amount dα∆T
Q (b) will not change
µ2
(c) will increase by an amount (2πR − d ) α∆T
(d) will decrease by an amount dα∆T
R µ3
S 26. A girl holds a book of mass m against a vertical wall
Then, with a horizontal force F using her finger, so that the
(a) µ 1 > µ 2 > µ 3 (b) µ 1 = µ 3 < µ 2
book does not move. The frictional force on the book
by the wall is
(c µ 1 < µ 2 < µ 3 (d) µ 1 < µ 3 < µ 2
(a) F and along the finger but pointing towards the girl
22. A ray of white light is incident on a spherical water (b) µF upwards, where µ is the coefficient of static friction
drop whose centre is C as shown below. (c) mg and upwards
(d) equal and opposite to the resultant of F and mg
27. A solid cube and a solid sphere both made of same
C material are completely submerged in water but to
different depths. The sphere and the cube have same
When observed from the opposite side, the emergent surface area. The buoyant force is
light (a) greater for the cube than the sphere
(b) greater for the sphere than the cube
(a) will be white and will emerge without deviating
(c) same for the sphere and the cube
(b) will be internally reflected
(c) will split into different colours such that the angles of (d) greater for the object that is submerged deeper
238
deviation will be different for different colours 28. 92 U atom disintegrates to 214
84 Po with a half of
(d) will split into different colours such that the angles of . × 109 years by emitting six α-particles and
45
deviation will be the same for all colours
n electrons. Here, n is
23. A convex lens of focal length 15 cm is placed in front (a) 6 (b) 4 (c) 10 (d) 7
of a plane mirror at a distance 25 cm from the mirror.
Where on the optical axis and from the centre of the 29. Which statement about the Rutherford model of the
lens should a small object be placed such that the atom is not true?
final image coincides with the object? (a) There is a positively charged centre in an atom called
(a) 15 cm and on the opposite side of the mirror the nucleus
(b) 15 cm between the mirror and the lens (b) Nearly all the mass of an atom resides in the nucleus
(c) 7.5 cm and on the opposite side of the mirror (c) Size of the nucleus is comparable to the atom
(d) 7.5 cm and between the mirror and the lens (d) Electrons occupy the space surrounding the nucleus

24. Following figures show different combinations of 30. A girl brings a positively charged rod near a thin
identical bulb(s) connected to identical battery(ies). neutral stream of water from a tap. She observes that
Which option is correct regarding the total power the water stream bends towards her. Instead, if she
dissipated in the circuit? were to bring a negatively charged rod near to the
stream, it will
(a) bend in the same direction
(b) bend in the opposite direction
(c) not bend at all
(d) bend in the opposite direction above and below the rod
P
Q
https://iit-jeeacademy.blogspot.com

KVPY Question Paper 2012 Stream : SA 111

39. The major product in the following reaction is


CHEMISTRY
H3 C— C ≡≡ C— H + HBr (excess)
31. The weight of calcium oxide formed by burning 20 g
CH2 Br
of calcium in excess oxygen is
(a) 36 g (b) 56 g (c) 28 g (d) 72 g (a) (b) H3C C CH3
H3C Br
32. The major products in the reaction Br3 CCHO → NaOH
Br
are Br
H
O (c) H3C C CH2 (d) H3C CH2 CH
(a) CHBr3+ Br Br Br
H ONa
H Br 40. The major product in the following reaction at 25°C
CH 3CH 2 NH 2
(b) NaBr + C C is CH3 COOH →
H Br (a) CH3CONHCH2CH3
O (b) CH3CH == NCH2CH3
(c) NaOBr + (c) NH+3 CH2CH3 ⋅ CH3COO–
D H (d) CH3CON == CHCH3
Br Br 41. A reaction with reaction quotient QC and equilibrium
Br Br
(d) OH + ONa constant KC , will proceed in the direction of the
Br Br products when
O (a) QC = KC (b) QC < KC
(c) QC > KC (d) QC = 0
40 +
33. The number of electrons plus neutrons in 19 K is
42. Acetyl salicylic acid is a pain killer and is commonly
(a) 38 (b) 59 (c) 39 (d) 40 known as
34. Among the following, the most basic oxide is (a) paracetamol (b) aspirin
(a) Al 2O3 (b) P2O5 (c) SiO2 (d) Na 2O (c) ibuprofen (d) penicillin
35. By dissolving 0.35 mole of sodium chloride in water, 43. The molecule which does not exhibit strong hydrogen
1.30 L of salt solution is obtained. The molarity of the bonding is
resulting solution should be reported as (a) methyl amine (b) acetic acid
(a) 0.3 M (b) 0.269 M (c) diethyl ether (d) glucose
(c) 0.27 M (d) 0.2692 M 44. The following two compounds are
36. Among the quantities, density (ρ), temperature (T ), H
enthalpy (H ), heat capacity (C p ), volume (V ) and CH3
pressure ( p), a set of intensive variables are H3C H3 C H
(a) ( ρ, T , H ) (b) (H , T , V )
(c) (V , T , C p ) (d) ( ρ, T , p ) (a) geometrical isomers
(b) positional isomers
37. The value of x in KAl(SO4 )x ⋅ 12H2O is (c) functional group isomers
(a) 1 (b) 2 (c) 3 (d) 4 (d) optical isomers
38. Among the following substituted pyridines, the most 45. The graph that does not represent the behaviour of
basic compound is an ideal gas is
Me Me at constant p
at constant T
N
p (b) p
(a)
(a) (b)

N N V 1/V
at constant p
CH3 Cl

(c) pV (d) V
(d)
N N
T – 273°C T
https://iit-jeeacademy.blogspot.com

112 KVPY Question Paper 2012 Stream : SA

53. Myeloid tissue is a type of


BIOLOGY
(a) haematopoietic tissue (b) cartilage tissue
46. A smear of blood from a healthy individual is stained (c) muscular tissue (d) areolar tissue
with a nuclear stain called hematoxylin and then 54. The heart of an amphibian is usually
observed under a light microscope. Which of the
(a) two-chambered (b) three-chambered
following cell types would be highest in number?
(c) four-chambered (d) three and half-chambered
(a) Neutrophils (b) Lymphocytes
(c) Eosinophils (d) Monocytes 55. Gigantism and acromegaly are due to defects in the
function of which of the following glands?
47. Which of the following biological phenomena involves
(a) Adrenals (b) Thyroid (c) Pancreas (d) Pituitary
a bacteriophage?
−8
(a) Transformation (b) Conjugation 56. The pH of 10 M HCl solution is
(c) Translocation (d) Transduction (a) 8 (b) close to 7 (c) 1 (d) 0

48. In which compartment of a cell does the process of 57. Which one of the following organelles can synthesise
glycolysis take place? some of its own proteins?
(a) Golgi complex (b) Cytoplasm (a) Lysosome (b) Golgi apparatus
(c) Mitochondria (d) Ribosomes (c) Vacuole (d) Mitochondrion

49. Huntington’s disease is a disease of the 58. Maltose is a polymer of


(a) nervous system (b) circulatory system (a) one glucose and one fructose molecule
(c) respiratory system (d) excretory system (b) one glucose and one galactose molecule
(c) two glucose molecules
50. A cell will experience the highest level of endosmosis (d) two fructose molecules
when it is kept in
(a) distilled water (b) sugar solution 59. The roots of some higher plants get associated with a
(c) salt solution (d) protein solution fungal partner. The roots provide food to the fungus
while the fungus supplies water to the roots. The
51. When the leaf of the ‘touch-me-not’ (chui-mui, Mimosa structure so formed is known as
pudica) plant is touched, the leaf droops because (a) lichen (b) Anabaena
(a) a nerve signal passes through the plant (c) mycorrhiza (d) Rhizobium
(b) the temperature of the plant increases
(c) water is lost from the cells at the base of the leaf 60. Prehistoric forms of life are found in fossils. The
(d) the plant dies probability of finding fossils of more complex
organisms
52. If you are seeing mangroves around you, which part (a) increases from lower to upper strata
of India are you visiting? (b) decreases from lower to upper strata
(a) Western Ghats (b) Thar desert (c) remains constant in each stratum
(c) Sunderbans (d) Himalayas (d) uncertain

PART-II (2 Marks Questions)


MATHEMATICS 64. A train leaves Pune at 7:30 am and reaches Mumbai
at 11:30 am. Another train leaves Mumbai at
a 2+b
61. Let a , b, c be positive integers such that is a 9:30 am and reaches Pune at 1:00 pm. Assuming that
b 2+c the two trains travel at constant speeds, at what time
rational number, then which of the following is do the two trains cross each other?
always an integer? (a) 10:20 am
2a 2 + b2 a 2 + b2 − c2 a 2 + 2b2 a 2 + b2 + c2 (b) 11:30 am
(a) (b) (c) 2 (d)
2b2 + c2 a+ b− c b + 2c 2
a+ b− c (c) 10:26 am
(d) data not sufficient
62. The number of solutions (x, y, z) to the system of
equations x + 2 y + 4z = 9, 4 yz + 2xz + xy = 13, xyz = 3,
65. In the given figures, which has the shortest path?
such that at least two of x, y, z are integers is
(a) 3 (b) 5 (c) 6 (d) 4 (a) (b)
63. In a ∆ ABC, it is known that AB = AC. Suppose D is
the mid-point of AC and BD = BC = 2. Then, the area
of the ∆ ABC is (d)
(c)
(a) 2 (b) 2 2 (c) 7 (d) 2 7
https://iit-jeeacademy.blogspot.com

KVPY Question Paper 2012 Stream : SA 113

N
PHYSICS
66. In the circuit shown, n-identical resistors R are
connected in parallel (n > 1) and the combination is i
connected in series to another resistor R0. In the W E
adjoining circuit n resistors of resistance R are all
connected in series alongwith R0.
S
R
A clockwise current is induced in the loop when loop
R is pulled towards
n
R0 R R R (a) N (b) E (c) W (d) S
n
R0 70. 150 g of ice is mixed with 100 g of water at
R
temperature 80°C. The latent heat of ice is 80 cal/g
R and the specific heat of water is 1 cal/g°C. Assuming
E no heat loss to the environment, the amount of ice
which does not melt is
E (a) 100 g (b) 0 (c) 150 g (d) 50 g

The batteries in both circuits are identical and net


power dissipated in the n resistors in both circuits is CHEMISTRY
same. The ratio R0 /R is
71. Upon fully dissolving 2.0 g of a metal in sulphuric
(a) 1 (b) n (c) n 2 (d) 1/ n acid, 6.8 g of the metal sulphate is formed. The
67. A firecracker is thrown with velocity of 30 ms −1 in a equivalent weight of the metal is
direction which makes an angle of 75° with the (a) 13.6 g (b) 20.0 g (c) 4.0 g (d) 10.0 g
vertical axis. At some point on its trajectory, the
72. Upon mixing equal volumes of aqueous solutions of
firecracker splits into two identical pieces in such a
0.1 M HCl and 0.2 M H2SO4, the concentration of H+
way that one piece falls 27 m far from the shooting
in the resulting solution is
point. Assuming that all trajectories are contained in
(a) 0.30 mol/L (b) 0.25 mol/L (c) 0.15 mol/L (d) 0.10 mol/L
the same plane, how far will the other piece fall from
the shooting point? (Take, g = 10 ms −2 and neglect air 73. The products X and Y in the following reaction
resistance) sequence are
NO2
(a) 63 m or 144 m (b) 72 m or 99 m
(c) 28 m or 72 m (d) 63 m or 117 m
Sn/HCl (i) NaNO2/HCl
68. A block of mass m is sliding down an inclined plane X (ii) CuBr, ∆
Y
with constant speed.At a certain instant t0, its height
above the ground is h. The coefficient of kinetic NO Cl
friction between the block and the plane is µ. If the
block reaches the ground at a later instant t g , then
the energy dissipated by friction in the time interval (a) X: Y:
(t g − t0 ) is
m NH2 Br

(b) X : Y:

h
NO2 Br

θ
(c) X: Y:
(a) µmgh (b) µmgh / sin θ
Cl Cl
(c) mgh (d) µmgh / cosθ
69. A circular loop of wire is in the same plane as an NO Cl
infinitely long wire carrying a constant current i.
Four possible motions of the loop are marked by N, E, (d) X : Y:
W, and S as shown below.
Cl Cl
https://iit-jeeacademy.blogspot.com

114 KVPY Question Paper 2012 Stream : SA

74. A plot of the kinetic energy  mv2 of ejected


1
2  BIOLOGY
electrons as a function of the frequency (ν ) of incident 76. A baby is born with the normal number and
radiation for four alkali metals (M1, M 2, M3 , M 4 ) is distribution of rods, but no cones in his eyes. We
shown below. would expect that the baby would be
M1 M2 M3 M4 (a) colourblind (b) nightblind
(c) blind with both eyes (d) blind with one eye
77. In mammals, pleural membranes cover the lungs as
well as insides of the ribcage. The pleural fluid in
Kinetic energy

between the two membranes


(a) dissolves oxygen for transfer to the alveoli
(b) dissolves CO2 for transfer to the blood
(c) provides partial pressure
v
(d) reduces the friction between the ribs and the lungs
The alkali metals M1, M 2, M3 and M 4 are, 78. At which phase of the cell cycle, DNA polymerase
respectively activity is at its highest?
(a) Li, Na, K and Rb (a) Gap 1 (G1 ) (b) Mitotic (M)
(b) Rb, K, Na and Li (c) Synthetic (S) (d) Gap 2 (G 2)
(c) Na, K, Li and Rb
(d) Rb, Li, Na and K 79. Usain Bolt, an olympic runner, at the end of a
100 metre sprint, will have more of which of the
75. The number of moles of Br2 produced when two moles following in his muscles?
of potassium permanganate are treated with excess (a) ATP (b) Pyruvic acid
potassium bromide in aqueous acid medium is (c) Lactic acid (d) Carbon dioxide
(a) 1
80. Desert temperature often varies between 0-50°C. The
(b) 3
DNA polymerase isolated from a camel living in the
(c) 2
desert will be able to synthesise DNA most efficiently at
(d) 4
(a) 0°C (b) 37°C (c) 50°C (d) 25°C

PART-I
Answers
1 (c) 2 (b) 3 (b) 4 (a) 5 (d) 6 (b) 7 (b) 8 (b) 9 (b) 10 (b)
11 (a) 12 (c) 13 (d) 14 (a) 15 (c) 16 (b) 17 (a) 18 (c) 19 (d) 20 (b)
21 (b) 22 (a) 23 (a) 24 (d) 25 (a) 26 (b) 27 (b) 28 (b) 29 (c) 30 (a)
31 (c) 32 (a) 33 (c) 34 (d) 35 (c) 36 (d) 37 (b) 38 (b) 39 (b) 40 (c)
41 (b) 42 (b) 43 (c) 44 (b) 45 (b, c) 46 (a) 47 (d) 48 (b) 49 (a) 50 (a)
51 (c) 52 (c) 53 (a) 54 (b) 55 (d) 56 (b) 57 (d) 58 (c) 59 (c) 60 (a)

PART-II
61 (d) 62 (b) 63 (c) 64 (c) 65 (d) 66 (a) 67 (d) 68 (c) 69 (b) 70 (d)
71 (b) 72 (b) 73 (b) 74 (b) 75 (*) 76 (a) 77 (d) 78 (c) 79 (c) 80 (b)
* No option is correct.
https://iit-jeeacademy.blogspot.com

KVPY Question Paper 2012 Stream : SA 115

Solutions
1. (c) Let f (x) = ax2 + bx + c 5. (d) We have, ⇒ ∠FBD + ∠FID = 180°
[Q f (x) is quadratic polynomial] 22 + 42 + 62 + ... + (2n )2 [Q sum of interior opposite angle of cyclic
> 101
. quadrilateral is 180°]
f (2) = 4a + 2b + c = 10 …(i) 12 + 32 + 52 + ... + (2n − 1)2
Now, ∠FID = ∠ADC + ∠ICD
[Q f (2) = 10] Σ(2n )2 101
⇒ > [Q sum angle properties]
f (−2) = 4a − 2b + c = − 2 …(ii) Σ(2n − 1)2 100
[Q f (−2) = − 2] ⇒ ∠FID = ∠BAD + ∠ABD + ∠ICD
4Σ n 2 101 A C
On subtracting Eq. (ii) from Eq.(i), we get ⇒ > = + B+
Σ(4n 2 − 4n + 1) 100 2 2
4b = 12 ⇒ b = 3
4Σ n 2 A + C + 2B
∴Coefficient of x in f (x) = b = 3 ⇒ >
101 =
4Σn − 4 Σn + Σ1 100
2 2
2. (b) Let x = 0.75 A + B +C + B
4(n )(n + 1)(2n + 1) =
According to the question, ⇒ 2
6 101
> 180° + B
x3 4n (n + 1)(2n + 1) 4n (n + 1) =
+ (x + x2 + 1) − +n 100
2
1− x 6 2
4n (n + 1)(2n + 1) 101 ∴∠FBD + ∠FID = 180°
x3 + (1 − x)(1 + x + x2 ) ⇒ >
= 180° + B
1− x n [4(2n 2 + 3n + 1) − 12n − 12 + 6] 100 =B+ = 180°
2
4(n + 1)(2n + 1) 101
x3 + 1 − x3 1 ⇒ > ⇒ 3B = 180° ⇒ B = 60°
= = 8n 2 − 2 100
1− x 1− x ∠B 60
4(2n + 1)(n + 1) 101 ∴∠IFD = = = 30°
Now, put the value of x ⇒ > 2 2
2(2n + 1)(2n − 1) 100
1 1 7. (b) Given,
= 2n + 2 101
1 − 0.75 0.25 ⇒ > D x U 1–2x T x C
2n − 1 100
100
= =4
25 ⇒ 200n + 200 > 202n − 101 x
45° 1–2x
So, square root of the equation = 4=2 ⇒ 2n < 301
301 V S
3. (b) Given, sides of triangle are positive ⇒ n<
integer in an AP and the smallest side is 2
10. ∴Maximum value of n = 150
∴Sides of triangle are 6. (b) Given, W R
10, 10 + d , 10 + 2d , d ∈N A
We know in triangle sum of two sides is
greater than third sides.
A P Q B
∴ 10 + 10 + d > 10 + 2d …(i)
F ABCD is square of length 1 unit and a
10 + 10 + 2d > 10 + d …(ii) E
regular octagon is formed by cutting
10 + d + 10 + 2d > 10 …(iii)
congruent isosceles triangle.
From Eqs. (i), (ii) and (iii), we get
I Let DUV is isosceles right angled triangle.
d < 10
∴ ∠D = 90°
∴ d = 1, 2, 3, 4, 5, 6, 7, 8, 9
∴ ∠DVU = 45°
Hence, there are 9 triangles possible. B D C In ∆DVU,
4. (a) We have, x
cos45° =
a a+b a+ b+ c a+b+c+d 1 − 2x
= = = =k AD , BE , CF are angle bisectors of angle
3 4 5 6 1 x
A , B and C respectively. ⇒ =
On solving, we get
I is the concurrent point of angle bisector 2 1 − 2x
a = 3k, b = k, c = k, d = k and BD IF are concyclic. 1 − 2x
⇒ 2=
a 3k x
∴ = Now, BDIF is concyclic.
b + 2c + 3d k + 2k + 3k ∠B ⇒
1
2 = −2
∴ ∠IFD = ∠IBD =
3k 1 2 x
= = 1
6k 2 [Q angle on same segments are equals ] ⇒ = 2+ 2
x
https://iit-jeeacademy.blogspot.com

116 KVPY Question Paper 2012 Stream : SA

1 2 −1 In ∆AXF and ∆BAF, 12. (c) Let the time taken by A to


⇒ x= =
2 ( 2 + 1) 2 ∠F = ∠F (common) complete the job = x h and time taken by
∠X = ∠A = (90° ) B to complete the job = y h
∴ Side of regular octagon = 1 − 2x
2( 2 − 1) ∴ ∆AXF ~ ∆BAF A and B together works then they
= 1−  1 1
2 ar(∆AXF ) AF 2 1 complete the work =  +  in 1 h
∴ = = x y
= 1 − 2 ( 2 − 1) ar(BAF ) BF 2 5
Let the complete work in t h
= 1−2 + 2 10. (b) Given,
 1 1 t + 8
= 2−1 t +  =
 x y x
8. (b) Given, OA = OB = r
t 8
Q ⇒ = …(i)
B 1 y x
1
2° 1
R S P
[B would 4 h more to
2
complete the work]
R C 9
R t+
 1 1 2
Q t +  =
r B  x y y
∠QRP = 2° t 9
R ⇒ = …(ii)
A
R PQ = QR = 1 x 2y
60° ∴ ∠QPR = 2°
30° From Eqs. (i) and (ii),
O ∠RQP = 180° − 4° = 176°
r P A t 2 = 36
SP = SQ radii of circle
Also, OC is radius of sector ⇒ t =6h
∴ ∠SQP = ∠QSP
∴ OC = r 13. (d) Let the weight of bucket be x kg
180° − 2° and the weight of water completely full
Now, OC = OB + BC =
2 be y kg.
OC = OB + R [BC is radius of circle] 178°
= = 89° According to the problem,
In ∆OPB, 2 y
BP x + = 10 …(i)
sin 30° = [Q ∠BOP = 30°] ∠RQS = ∠RQP − ∠SQP 2
OB
= 176° − 89° = 87° and x+
2y
= 11 …(ii)
1 R
⇒ = 11. (a) At 6: 15, 3
2 OB
12 On solving Eqs. (i) and (ii), we get
⇒ OB = 2R
x = 7, y = 6
∴ OC = 2R + R = 3R
∴Total weight, when bucket is completely
⇒ r = 3R full is (x + y) kg i.e. 7 + 6 = 13 kg
r
⇒ R= 14. (a) We have,
3 9 3
m 12
9. (b) We have, r =
12 n
A 1 F 1 E ⇒ mn = 144
⇒ mn = 24 × 32
6 Total number of divisor of 144 is
1 X 1
the minutes hand makes an angle is α. (4 + 1)(2 + 1) = 15
1 °
∴ α = 90° + 15 ×   When m and n are positive integers.
H
1 1
D  2 If m and n are negative integers, then
K
195  °
α = 
also number of divisor is 15.

1 1  2  ∴Total ordered pairs of (m, n ) when m and
and hour hand is β. n are integers = 15 + 15 = 30

B C Given, α + β = 360° 15. (c) We have,


1
∴ β = 360° − α S = {1, 2, 3, 4,..., 40}
195  °  525  °
= 360° − 
AHKF , FKDE and HBCK is a unit square. A is subset of S whose sum of two
 = 
AD and BF intersect at X.  2   2  element of A is not divisible by 5.
In ∆ABF, AB = 2 Difference between their angles Possible set A = { 1, 2, 5, 6, 7, 11, 12, 16,
AF =1 525 195 330 17, 21, 22, 26, 27, 31, 32, 36, 37}
= − = = 165° ∴ Maximum number of elements in A
∴ FB 2 = AB 2 + AF 2 = 4 + 1 = 5 2 2 2
is 17.
https://iit-jeeacademy.blogspot.com

KVPY Question Paper 2012 Stream : SA 117

16. (b) Given situation is 19. (d) To minimize drift, let angle at 24. (d) Power consumed in each case is
which boat is directed by θ, as shown
m m Initially one of (P) V2
below. P =
u1=v u2=0 the ball is at rest 3R
v/2
Finally both balls moves + –
m m v
with same speed together vres
v θ (Q) V2 3V 2
Conservation of momentum, gives 90° P = =
R /3 R
v
mv = 2 mV ⇒ V =
2 From above velocity triangle,
v/2 1
So, kinetic energy after collision is sin θ = = 30°
2 v or θ =
(2m) V 2 = × 2m ×  
1 1 v 2
Kf = So, angle with respect to direction of flow + –
2 2  2
is 90° + θ° = 90° + 30° = 120°.
1 (R) V2
= mv2 20. (b) Ice is a bad conductor of heat, its P =
4 R
thermal conductivity is very low. So, no + –
17. (a) Air resistance is same in both exchange of heat from outside
case.When ball is moving down, air surrounding occurs in an Igloo. Thermal + –
resistance is directed away from g. . Wm −1 K −1 .
conductivity of ice is 16
+ –
21. (b) Light ray bends towards normal (S) (2V ) 2 4V 2
Air P = =
at point Q and it bends away from normal R R
resistance
at point R.
– +

v g µ1 Q
So, order of increasing power
So, acceleration of ball moving µ2 consumption is P > R > Q > S.
downwards is 25. (a) Gap or cavity also expands at
mg − kv µ3
a1 = same rate as that of metal.
R
m Hence, width of gap also increases by
Pa
a1 =  g − v
 k rallel same amount.
or s
 m  ray ∴Width of gap increases by ∆d = dα ∆T .
where, k is constant. 26. (b) f=µN
When ball is moving up, air resistance Also, the emergent ray is parallel to
and g both are directed downwards. incident ray.
v Hence, µ 1 = µ 3 < µ 2 is correct option.
F
22. (a) As, incident light is normal to the
surface, so no deviation or dispersion N
occurs.
Emergent Incident
Air g mg
beam beam
resistance
C Minimum force F must be such that
So,acceleration while moving upwards is generated friction is able to balance
k weight mg of book.
a2 = g + v 23. (a) If object is placed
m So, f = µN = µF.
Clearly, a2 > a1 . 27. (b) Given,
18. (c) The free electrons experiences Surface area of cube = Surface area of
electrostatic force in the direction sphere
opposite to the direction of electric fired O
being is of negative charge. The electric r
field always directed from higher
potential to lower potential. Therefore
electrostatic force and negative charge or at the focus of lens on opposite side of
mirror, then light rays after refraction a
electrons always flows from lower to
higher potential until the potantials from lens become parallel to the principal ⇒ 6a 2 = 4 πr 2 …(i)
axis. 4π  2π
= 
become equal. a
These parallel rays are reflected back over ⇒  =
Hence, option (c) is correct. r  6 3
same path and again converges at focus.
https://iit-jeeacademy.blogspot.com

118 KVPY Question Paper 2012 Stream : SA

Now, buoyant force is FB = V in .ρf . g 34. (d) All the central atom of the given The increasing order of basicity of
So, ratio of buoyant force on cube and oxide belong to 2nd period, as we move compounds given in options will be
sphere is from left to right in a period the basicity Me Me
(FB )cube V of oxide decreases.
= cube Cl CH3 N
(FB )sphere V sphere ∴The order of increasing basicity of oxide
3 would be
a3 2π  2 < < <
× 
3 P2O5 < SiO2 < Al 2O3 < Na 2O
= = 
4 3 4 π  3
πr Thus, the most basic oxide would be N N N N
3 Na 2O. Me Me
9 × 8 × π3 π 35. (c) Given, N
= =
16 × π 2 × 27 6 Number of moles of solute NaCl
∴ (FB )cube < (FB )sphere = 0.35 mole Thus, basicity of will be maximum.
28. (b) Decay is Volume of the solution = 1.30 L
N
− Number of moles of solute
238
92 U→ 214
84 Po + 6 2 He + ne
4
Molarity =
Conservation of mass number and atomic
Volume of solution in litres 39. (b)
0.35 Markownikoff
number gives, = = 0.269 ≈ 0.27 M H3C—C C—H + HBr addition
1.30 (Excess)
⇒ 92 = 84 + 12 − n ⇒ n = 4
36. (d) Intensive variables are those Br
29. (c) In Rutherford model, positive | HBr
charge acquires a very small place at variables which don’t depend upon the H3C—C—CH3 H3C—C==CH2
centre of atom in nucleus. quantity or size of matter. Among the | |
given quantities density, temperature Br Br
So, option (c) is incorrect. (Major)
and pressure are intensive variables
30. (a) Neutral objects are always whereas heat capacity, enthalpy and This is an electrophilic addition reaction
attracted towards both positively and which proceeds via carbonium ion
volume are an extensive variables. Thus,
negatively charged objects. So, water formation and follows Markownikoff rule.
the correct option is (d).
stream still bends in same direction.
37. (b) KAl(SO4 )x ⋅ 12H2O is an empirical 40. (c) As the reaction is occurring at

31. (c) 2Ca + O2 → 2CaO room temperature, (25°C) so it will be an
Excess formula for potash alum which is double acid base reaction.
20 1 salt. 25 ° C
Number of moles of Ca = = mole H3 C — COOH + CH3 CH2NH2 →
40 2 Potash alum is K2SO4 ⋅ Al 2 (SO4 )2 ⋅ 24H2O Acid-base
1 mole of Ca produces 1 mole of CaO ∴Empirical formula is KAl(SO4 )2 ⋅ 12H2O reaction

1 1 Thus, x = 2 . NH3 + CH2CH3 ⋅ CH3 COO−


∴ mole of Ca will produces mole
2 2 38. (b) Pyridine is basic in nature due to 41. (b) For an equilibrium reaction,
of CaO the presence of free lone pair. Its basicity A + B r C + D;
1 mole of CaO = 56 g gets affected by the presence of different QC =
[C ] [ D ]
1 1 substituents attached to it. The electron [ A ] [ B]
mole of CaO = 56 × = 28 g
2 2 withdrawing group decreases the basicity If QC = KC , the reaction is in equilibrium.
Thus, 28 g of CaO is formed by burning of pyridine, whereas electron releasing If QC > KC , the reaction proceeds in
20 g of Ca in excess oxygen. groups increases its basicity. backward reaction.
32. (a) Substituents like N(Me)2 and CH3 are If QC < KC , the reaction proceeds in
NaOH electron releasing but N(Me)2 is more forward reaction.
Br3 CCHO → CHBr3 + HC O− Na +
|| activating group than CH3 . Thus, the reaction will proceed in the
O So, H3C CH3 direction of products i.e., forward reaction
This reaction is known as bromoform when QC < KC .
N
reaction where the carbonyl carbon gets 42. (b) Acetyl salicylic which is a pain
oxidised by sodium hydroxide to sodium killer commonly known as aspirin.
salts of corresponding carboxylic acid OCOCH3
having one carbon atom less than of
N COOH
carbonyl compound and the methyl group
is converted to bromoform. basicity would be maximum while Cl
40 + is an electron withdrawing group, so
33. (c) For, 19 K
Acetyl salicylic acid
Number of electrons in K+ = 18 Cl (aspirin)
Number of neutrons in K+ = 40 − 19 = 21
43. (c) Hydrogen bonding occurs in a
⇒ Sum of electron + neutron is basicity would be least. molecule when a hydrogen atom is
= 18 + 21 = 39 N directly linked to an electronegative atom
like F, O and N.
https://iit-jeeacademy.blogspot.com

KVPY Question Paper 2012 Stream : SA 119

In case of diethyl ether (H5 C2OC2H5 ), H is 46. (a) A smear of blood from a healthy when any external pressure is applied
not directly linked to oxygen atom and individual if viewed under the microscope like shaking or touching (seismonastic
hence will not exhibit strong hydrogen will show highest number of neutrophils. movement), activates the certain
There are three types of blood cells in an contractile proteins including potassium
bonding.
individual, i.e. red blood cells ions. These chemicals make water and
44. (b) H (erythrocytes), white blood cells electrolytes flow/diffuse out of the cell,
CH3 (leukocytes) and platelets (thrombocytes). resulting in a loss of cell pressure. This
CH3 CH3 H RBCs and platelets are devoid of nucleus, causes the cell to collapse, which
(But-2-ene) (But-1-ene) thus are not stained by haematoxylin squeezes the leaves close. Stimulus, in
nuclear stain. Therefore, only WBCs are the form of touch is sometimes
Both the compounds, but-2-ene and stained. In a healthy individual, there transmitted to neighbouring leaves as
but-1-ene are positional isomers of each are five types of WBCs, i.e.neutrophils well and causing the closing of other
other because they differ in the position (accounts for 40-75% of leukocytes), neighbour leaves too.
of double bond. eosinophils (1-6%), basophils (0-1%), 52. (c) The Sundarbans delta is the
lymphocytes (20-45%) and monocytes largest mangrove forest in the world and
45. (b, c) pV = nRT (2-10%). is intersected by a complex network of
RT 47. (d) Transduction is a method of gene tidal waterways, mudflats and small
p= (for 1 mole)
V transfer in bacteria from donor to islands of salt-tolerant mangrove forests.
At constant T , p would be inversely recipient using bacteriophage. In It lies at the mouth of the Ganges.
proportional to V . transduction, at first bacteriophage A mangrove is a shrub or small tree that
infects donor bacteria and then carries grows in coastal saline or brackish water.
So, plot of p versus V would
some part of donor genome with it. When 53. (a) The process by which new blood
this bacteriophage infects new bacterial cells are formed is called haematopoiesis.
cell, it transfers that DNA into recipient The group of cells performing
p at constant T cell. haematopoiesis are called hematopoietic
48. (b) Glycolysis is the first stage of tissue. These are of two kinds, i.e.
cellular respiration that involves the myeloid tissue and lymphoid tissue.
breaking down of glucose into smaller Myeloid tissue is red bone marrow, which
V molecules to produce ATP (cellular produces erythrocytes, platelets and most
energy). Glycolysis occurs in the of the leukocytes (including the
Thus, plot (a) is correct.
cytoplasm of all living organisms because B-lymphocytes). Lymphoid tissue consists
According to ideal gas equation. of the lymphatic nodules, lymph nodes,
pV that is where glucose and other related
pV = nRT or = nR enzymes can be found in high spleen and thymus (which produces only
T T-lymphocytes).
concentrations.
As R is a gas constant, so plot between 54. (b) Vertebrate hearts can be
pV versus T would be linear and slope
49. (a) Huntington’s disease is a disease
of the nervous system. It is a fatal genetic categorised by the number of chambers
will be n. i.e. they have, like two-chambered (one
disorder that causes the progressive
breakdown of nerve cells in the brain. It atrium and one ventricle) in fishes,
deteriorates a person’s physical and three-chambered (two atria and one
mental abilities during their prime ventricle) in amphibians and reptiles;
pV working years and has no cure. and four-chambered (two atria and two
ventricles) in birds and mammals.
50. (a) A cell will experience the highest
level of endosmosis when it is kept in 55. (d) Gigantism and acromegaly are
T syndromes of excessive secretion of
distilled water. Endosmosis is the process
growth hormone from pituitary gland.
Also, from ideal gas equation p is directly by which water molecules move into the
Acromegaly is a condition that affects
1 cell. Distilled water has far less solute
proportional to . Therefore, plot adults, usually between the age of 30 and
V concentration than cell fluid. Thus, it is a
50, while gigantism affects children with
1 hypotonic solution which allows the same excess growth hormone but in a
between p versus is a straight line, i.e.
V maximum endosmosis. Other solutions way that can delay puberty as well as
like sugar solution, salt solution and causes excessive physical growth.
protein solution are hypertonic solutions
56. (b) The pH of 10−3 M HCl is close to
compared to distilled water. Cell
7, i.e., 6.98. It can be calculated as, pH
performs exosmosis in hypertonic
p = − log [H3 O+ ], we get pH equal to 8. But
solutions.
at constant p this is not correct because an acidic
51. (c) The leaves of touch-me-not plant solution cannot have pH greater than 7.
(Mimosa pudica) are very sensitive, they It may be noted that in very dilute acidic
droop on touching because of autotrophic solution, when H+ concentrations from
1/v effect. As, water within the cells of this
acid and water are comparable, the
Thus, plot (b) is incorrect. plant applies a turgour pressure, which
concentration of H+ from water cannot be
makes to leaves its stay upright. So,
neglected.
https://iit-jeeacademy.blogspot.com

120 KVPY Question Paper 2012 Stream : SA

2ab − bc + (b2 − ac) 2 ∴Solutions are  3, 2,   4, ,  .


Therefore, [H + ]total = [H + ]acid + [H + ]water 1 3 1
=
Since HCl is a strong acid and is 2b − c 2 2  2   2 2
completely ionised [H+ ]HCl = 1.0 × 10−8 Since a , b, c are positive integers and
3
Put z = , then x + 2 y = 6
The concentration of H + from ionisation a 2+b 4
is rational.
is equal to the [OH − ] from water, b 2+ c xy = 4
[H+ ]H 2O = [OH− ]H 2O = x (say) On solving, we get x = 4, y =1and x = 2,
∴ b2 − ac = 0
y=2
[H+ ]total = 1.0 × 10−8 + x ∴a , b, c are in GP.
∴Solutions are  4, 1,  and  2, 2,  .
3 3
But [H+ ] [OH− ] = 1.0 × 10−14 Let a = a , b = ar , c = ar 2, where r is also  4  4
(1.0 × 10−8 + x) (x) = 1.0 × 10−14 positive integer.
At least two of x, y, z are integer is
x2 + 10−8 x − 10−14 = 0 2a 2 + b2
 3 
(3, 1, 1),  2, 1 ,  3, 2,  ,
(a) 1
Solving of x, we get x = 9.5 × 10−8 2b2 + c2  2,   2
Therefore [H+ ] = 1.0 × 10−8 + 9.5 × 10−8 2a 2 + a 2r 2 1  4, 1, 3  ,  2, 2, 3 
= 10.5 × 10−8 = 1.05 × 10−7 = = not integer    
2a 2r 2 + a 2r 4 r2  4  4
pH = − log [H+ ] = − log (1.05 × 10−7 ) = 6.98
a 2 + b2 − c2 a 2 + a 2r 2 − a 2r 4 Hence, 5 solutions.
57. (d) Mitochondria have their own (b) =
a+ b− c a + ar − ar 2 63. (c) Given, in ∆ABC,
DNA and ribosomes. Ribosomes are the
 1 + r2 − r4  A
site of protein synthesis. Therefore,
=a  not integer
mitochondria are capable of making their  1+ r − r 
2
own protein. Mitochondria are also
capable of synthesising their own genetic a 2 + 2b2
(c) D
material. b2 + 2c2
2
58. (c) Maltose is a polymer composed of a 2 + 2a 2r 2 1
= = not integer
two glucose molecules. It is a a 2r 2 + 2 a 2r 4 r2
disaccharide formed when two units of 2
glucose are joined with α (1→ 4) bond. a 2 + b2 + c2 a 2 + a 2r 2 + a 2r 4 B C
(d) =
Maltose is also known as malt sugar and a+ b− c a + ar − ar 2 AB = AC
is found in foods in which starch is a2 1 + r 2 + r 4  D is mid-point of AC.
fermented by yeast or enzymes, such as =   AC AB
a  1+ r − r 2  ∴ AD = CD = =
in bread or brewed beverages.
2 2
59. (c) A mycorrhiza is a symbiotic = a (1 + r + r 2 ) integer
In ∆ABD,
association between roots of higher plant 62. (b) We have, AB 2 + AD 2 − 22
and a fungus. The plant makes organic x + 2 y + 4z = 9 …(i) cosA =
molecules such as sugar by 2 ⋅ AB ⋅ AD
4 yz + 2xz + xy = 13 …(ii)
photosynthesis and supplies them to the 4AD 2 + AD 2 − 4
xyz = 3 cosA =
fungus and the fungus supplies to the
⇒ x + 2 y = 9 − 4z 4AD 2
plant water and mineral nutrients, such [Q AB = 2AD ]
From Eq. (ii),
as phosphorus, taken from the soil.
2z (2 y + x) + xy = 13 5AD 2 − 4
Lichen is a symbiotic partnership of a cosA = …(i)

3
2z (9 − 4z ) + = 13 Q xy = 3  4AD 2
fungus and an alga. Rhizobium are 
z z  In ∆ABC,
nitrogen-fixing bacteria found in root
nodules of legumes. Anabaena is a genus ⇒ 8z3 − 18z 2 + 13z − 3 = 0 AB 2 + AC 2 − 22
of filamentous cyanobacteria. ⇒ (z − 1)(2z − 1)(4z − 3) = 0 cosA =
1 3 2AB ⋅ AC
60. (a) During the course of evolution ⇒ z = 1,
2, 4 4AD 2 + 4AD 2 − 4
simple organisms evolved first while cosA =
complex organisms evolved later. Put z =1, then x + 2 y = 5 and xy = 3 8AD 2
Fossilisation of complex organisms took On solving, we get x = 3, y = 1 8AD 2 − 4
later in upper strata of earth crust. 3 cosA = …(ii)
and x = 2, y = 8AD 2
61. (d) We have, a, b, c are positive 2
∴Solutions are (3, 1, 1) and  2, ,1 .
3 From Eqs. (i) and (ii), we get
a 2+ b
integers and is a rational  2  5AD 2 − 4 8AD 2 − 4
b 2+c =
1 4AD 2 8AD 2
number. Put z = , then x + 2 y = 7
2
 a 2 + b  b 2 − c  ⇒ 10AD 2 − 8 = 8AD 2 − 4
∴    xy = 6
 b 2 + c   b 2 − c On solving, we get ⇒ AD = 2
2ab − ac 2 + b2 2 − bc 3
= x = 3, y = 2 and x = 4, y = ⇒ AB = 2 2
2b2 − c2 2
https://iit-jeeacademy.blogspot.com

KVPY Question Paper 2012 Stream : SA 121

1 Path = 5 + 2 + 5 + 5 = 3 5 + 2
Area of ∆ABC = AB ⋅ AC sin A 67. (d) Given situation is
2 = 3(2.23) + 2 = 8.69 Second possible position

1
= × (2 2 )2 1 − cos2 A (d) In figure
2 30 ms–1 2m
Path of centre
75° of mass m
m 15° m
1
= × 8× 1−
9 QcosA = 3  (–27,0) (27 m, 0) (45,0) (x,0)

2 16  5  2 √13 First particle’s


position

= 7 As explosion force which splits the


√5
64. (c) Let the distance between Pune firecracker is internal to system, path of
and Mumbai be x km. centre of mass of system remains same.
Time taken by 1st train = 4 h Path = 2 + 13 + 5 Now, range of centre of mass of system is
x = 2 + 3.60 + 2.23 = 7.83 u 2 sin 2θ
∴Speed of 1st train = km/h R=
4 ∴Shortest path have option fig (d). g
7
Time taken by 2nd train = h Hence, option (d) is correct. Here, u = 30 ms−1 , θ = 15°
2 66. (a) In case I, 30 × 30 × sin (2 × 15° )
x 2x ∴ R= = 45 m
∴Speed of 2nd train = = km/h R 10
(7/2) 7
So, position (or x-coordinate) of centre of
1st train starts from 7 : 30 am and 2nd R0 n mass is at 45 m distance from origin.
train starts from 9 : 30 am. m x + m2x2
Now using, X CM = 1 1 , we get
Distance travelled by 1st train in 2 h m1 + m2
x
= km m (± 27) + mx
2 45 =
m+ m
Let they meet at time t. + – ± 27 + x
x x ⇒ 45 =
=  × t  +   t
2x
∴ 2
2 4   7  E
Total circuit resistance, Solving, we get
t =   h
14
⇒ R nR0 + R x = 90 ± 27 ⇒ x = 63 m or x = 117 m
 15  Req = R0 + =
n n So, other piece may fell at 63 m or 117 m
t =  × 60 min = 56 min
14
⇒ E nE mark.
 15  Circuit current = i1 = ⇒ i1 =
Req nR0 + R 68. (c) As block is sliding with a
∴They meet at 9: 30 + 56 = 10: 26 am constant speed, so change in kinetic
65. (d) Power dissipated in n resistors,
energy of block when it reaches bottom is
nE 2R
P1 = i12 .   =
(a) In figure R zero.
√10
 n  (nR0 + R )2 Now, by work-energy theorem,
√5
In case II, Total work done = Change in kinetic
1 R0 n energy
√8
⇒ Wfriction + Wgravitation = ∆KE
R
⇒ Wfriction = − Wgravitation
Path = 1 + 5 + 10 + 8 or Wfriction = − mgh
+ –
= 1 + 2.23 + 316
. + 2.82 = 9.21 So, energy dissipated due to friction
(b) In figure E = mgh.
Total circuit resistance, Req = R0 + nR
√10 69. (b) Magnetic field linked with loop is
Current in circuit is downwards.
2 E E
i2 = = × ×
Req R0 + nR I
1
√8
Power dissipated in n resistors, ×
× × E
nE 2R
Path = 1 + 2 + 10 + 8 P2 = (i22 ) (nR ) =
(R0 + nR )2
= 1 + 2 + 316
. + 2.82 = 8.98 × ×
As, P1 = P2
(c) In figure
nE 2 nE 2R When loop is pulled away from wire, flux
⇒ =
(nR0 + R ) 2
(R0 + nR )2 linked with loop decreases. This induces
√5 2 a current in loop which tries to oppose
⇒ nR0 + R = R0 + nR this change (in accordance with Lenz’s
√5 √5 ⇒ (n − 1)R0 = (n − 1)R law).
R
⇒ R0 = R or 0 = 1 So, current induced in loop is clockwise.
R
https://iit-jeeacademy.blogspot.com

122 KVPY Question Paper 2012 Stream : SA

70. (d) Let m gram of ice melts and this 73. (b) 2 × 5 = 2 × moles
causes final temperature of mixture at NO2 NH2 = molesBr 2 = 5 mol
0°C. Further melting of ice is then 76. (a) Absence of cone cells in eyes is
stopped. As there is no heat loss Sn/HCl (i) NaNO2/HCl
known as total colour blindness or
Heat lost by ice = Heat gained by water [Reduction] [Diazotisation]
monochromacy. This person views
⇒ mL = mw sw (∆T ) Nitrobenzene Aniline everything as if it were in a black and
X
⇒ m80 = 100 × 1 × (80 − 0) Br
+
N

NCl
white television. Monochromacy occurs
⇒ m 80 = 100 × 80 when 2 or all 3 of cone pigments are
100 × 80 (ii) CuBr, ∆
missing and colour and light vision is
⇒ m= reduced to one dimension.
80 Sandmeyer
reaction 77. (d) Pleural fluid is a serous fluid
⇒ m = 100 g
Y Diazonium produced by the serous membrane
So, 50 g of ice does not melts. Bromo salt
benzene
covering normal pleurae. The pleural
71. (b) Let the equivalent weight of fluid acts as a lubricant and allows the
metal be x g. 74. (b) According to the conservation of pleurae to slide effortlessly against each
energy, the kinetic energy of the ejected other during respiratory movements,
Number of equivalents of metal
electron is given as thus reducing the friction between the
= Number of equivalents of metal sulphate
1 1 ribs and the lungs.
Wt. of metal
=
Wt. of metal sulphate hν = hν0 + mv2 ⇒ mv2 = hν − hν0
2 2 78. (c) DNA polymerase is an enzyme
Eq. wt. of metal Eq. wt. of metal sulphate that synthesises DNA molecules from
1
2
=
6.8
= 6. 8 x = 2x + 96 or mv = hν − W0
2
deoxyribonucleotides, the building blocks
x x + 48 2
of DNA. These enzymes are essential for
where, W0 = hv0 = Work function DNA replication. S-phase or synthetic
4. 8 x = 96
(minimum energy required to eject the phase is significant due to DNA
96
x= = 20.0 g electron). synthesis. Thus, DNA polymerase
4.8
Higher is the threshold frequency (ν0 ) activity is highest at S-phase of cell cycle.
72. (b) Let the volume of HCl and H2SO4 more will be the work function. Thus, 79. (c) During vigorous muscular
be V [No. of moles = Molarity × Basicity M1 → Rb, M2 → K, M3 → Na, M4 → Li activity like running, muscles perform
× Volume] 75. (*) KMnO4 + KBr + H2SO4
anaerobic respiration after a while due to
scarcity of oxygen. During anaerobic
Moles of H+ ions in HCl
→ MnSO4 respiration in muscles, lactic acid is
= 01. × 1 × V = 0.1 V produced as a byproduct. Thus a runner
+ + K2SO4 + Br2 + H2O
Moles of H ions in H2SO4 will have lactic acid in his muscles after a
Balancing the above equation,
= 0.2 × 2 × V = 0. 4 V 100 metre sprint.
2KMnO4 + 10KBr + 8H2SO4 → 2MnSO4
Thus, resulting moles of H+ ions in 80. (b) Camel belongs to
+ 5Br2 + 6K2SO4 + 8H2O
solution = (01. + 0.4) V = 0.5 V class–Mammalia. Both birds and
No. of eq. of KMnO4 = No. of eq. of Br2 mammals are homeothermic and have a
Resulting volume of a solution = 2V
Moles 0.5 V Mole × nKMnO 4 = Mole × nBr 2 fixed 37°C body temperature. So, the
[H+ ] = = where, n is the balancing factor DNA polymerase isolated from a camel
Volume 2V
nKMnO 4 = 5 will work efficiently at temperature near
= 0.25 mol L its body temperature.
nBr 2 = 2
https://iit-jeeacademy.blogspot.com

KVPY Question Paper 2011 Stream : SA 123

KVPY
KISHORE VAIGYANIK PROTSAHAN YOJANA

QUESTION PAPER 2011


Stream : SA
MM 100

Instructions
There are 80 questions in this paper.
This question paper contains two parts; Part I and Part II. There are four sections; Mathematics, Physics, Chemistry
and Biology in each part.
Out of the four options given with each question, only one is correct.

PART-I (1 Mark Questions)


MATHEMATICS 4. Let a , b, c, d be real numbers such that|a − b| = 2,
1. Suppose a , b, c are three distinct real numbers, let |b − c| =3,|c − d| =4. Then, the sum of all possible
values of|a − d|is
(x − b)(x − c) (x − c)(x − a ) (x − a )(x − b)
P (x) = + + . (a) 9 (b) 18
(a − b)(a − c) (b − c)(b − a ) (c − a )(c − b) (c) 24 (d) 30
When simplified, P (x) becomes 5. Below are four equations in x. Assume that 0 < r < 4.
(a) 1 Which of the following equations has the largest
(b) x solution for x?
x2 + (a + b + c)(ab + bc + ca ) x x
(a) 5  1+  = 9 (b) 5  1+  = 9
(c) r r
(a − b)(b − c)(c − a )  π  17 
x
(d) 5  1+  = 9
(d) 0 1
(c) 5(1 + 2r )x = 9
 r
2. Let a , b, x, y be real numbers such that a 2 + b2 = 81,
x2 + y2 = 121 and ax + by = 99. Then, the set of all 6. Let ABC be a triangle with ∠B = 90°. Let AD be the
possible values of ay − bx is bisector of ∠A with D on BC. Suppose AC = 6 cm and
(a)  0, 
9
(b)  0, 
9
(c) {0} (d)  ,∞ 
9 the area of the ∆ ADC is 10 cm2. Then, the length of
 11  11  11 
BD in cm is equal to
1 1 1 3 3
3. If x + = a, x2 + 3 = b, then x3 + 2 is (a) (b)
x 5 10
x x 5 10
(a) a3 + a 2 − 3a − 2 − b (b) a3 − a 2 − 3a + 4 − b (c) (d)
3 3
(c) a3 − a 2 + 3a − 6 − b (d) a3 + a 2 + 3a − 16 − b
https://iit-jeeacademy.blogspot.com

124 KVPY Question Paper 2011 Stream : SA

7. A piece of paper in the shape of a sector of a circle 13. A frog is presently located at the origin (0, 0) in the
(see Fig. 1) is rolled up to form a right-circular cone XY -plane. It always jumps from a point with integer
(see Fig. 2). The value of the angle θ is coordinates to a point with integer coordinates
moving a distance of 5 units in each jump. What is
the minimum number of jumps required for the frog
to go from (0, 0) to (0, 1)?
θ 12
(a) 2 (b) 3 (c) 4 (d) 9
14. A certain 12-hour digital clock displays the hour and
the minute of a day. Due to a defect in the clock
whenever the digit 1 is supposed to be displayed it
displays 7. What fraction of the day will the clock
Fig. 1 5 show the correct time ?
Fig. 2 1 5 3 5
(a) (b) (c) (d)
10 π 9π 5π 6π 2 8 4 6
(a) (b) (c) (d)
13 13 13 13
15. There are 30 questions in a multiple-choice test. A
8. In given figure, AB = 12 cm, CD = 8 cm, BD = 20 cm, student gets 1 mark for each unattempted question,
∠ABD = ∠AEC = ∠EDC = 90°. If BE = x, then 0 mark for each wrong answer and 4 marks for each
A correct answer. A student answered x questions
correctly and scored 60. Then, the number of possible
value of x is
(a) 15 (b) 10 (c) 6 (d) 5

C PHYSICS
16. A simple pendulum oscillates freely between points A
and B.
B x E D
(a) x has two possible values whose difference is 4
(b) x has two possible values whose sum is 28
(c) x has only one value and x ≥ 12
(d) x cannot be determined with the given information C

9. Three circles each of radius 1 touch one another A B A B


externally and they lie between two parallel lines. D
The minimum possible distance between the lines is We now put a peg (nail) at the point C as shown in
(a) 2 + 3 (b) 3 + 3 above figure. As the pendulum moves from A to the
1
(c) 4 (d) 2 + right, the string will bend at C and the pendulum
3 will go to its extreme point D. Ignoring friction, the
10. The number of distinct prime divisors of the number point D
(512)3 − (253)3 − (259)3 is (a) will lie on the line AB
(a) 4 (b) 5 (c) 6 (d) 7 (b) will lie above the line AB
(c) will lie below the line AB
11. Consider an incomplete pyramid of balls on a square (d) will coincide with B
base having 18 layers, and having 13 balls on each
side of the top layer. Then, the total number N of 17. A small child tries to move a large rubber toy placed
balls in that pyramid satisfies on the ground. The toy does not move but gets
(a) 9000 < N < 10000 (b) 8000 < N < 9000 deformed under her pushing force F, which is
(c) 7000 < N < 8000 (d) 10000 < N < 12000
obliquely upward as shown in the figure.

12. A man wants to reach a certain destination. Then,


One-sixth of the total distance is muddy while half
the distance is tar road. For the remaining distance
Push F
he takes a boat. His speed of travelling in mud, in
water, on tar road is in the ratio 3 : 4 : 5. The ratio of
the durations he requires to cross the patch of mud,
stream and tar road is (a) the resultant of the pushing force F, weight of the toy,
1 4 5 normal force by the ground on the toy and the
(a) : : (b) 3 : 8 : 15 (c) 10 : 15 : 18 (d) 1 : 2 : 3
2 3 2 frictional force is zero
https://iit-jeeacademy.blogspot.com

KVPY Question Paper 2011 Stream : SA 125

(b) the normal force by the ground is equal and opposite to B


the weight of the toy
(c) the pushing force F of the child is balanced by the
equal and opposite frictional force
(d) the pushing force F of the child is balanced by the total
internal force in the toy generated due to deformation
A C
18. A juggler tosses a ball up in the air with initial speed
u. At the instant, it reaches its maximum height H,
he tosses up a second ball with the same initial The minimum value of the refractive index of the
speed. The two balls will collide at a height prism is close to
H H 3H 3 (a) 1.10 (b) 1.55 (c) 1.42 (d) 1.72
(a) (b) (c) (d) H
4 2 4 4 24. A convex lens is used to form an image of an object on
a screen. If the upper half of the lens is blackened, so
19. On a horizontal frictionless frozen lake, a girl 36 kg and
that it becomes opaque, then
a box 9 kg are connected to each other by means of a
(a) only half of the image will be visible
rope. Initially, they are 20 m apart. The girl exerts a
(b) the image position shifts towards the lens
horizontal force on the box, pulling it towards her. How
(c) the image position shifts away from the lens
far has the girl travelled when she meets the box?
(d) the brightness of the image reduces
(a) 10 m
(b) Since, there is no friction, the girl will not move 25. A cylindrical copper rod has length L and resistance
(c) 16 m R. If it is melted and formed into another rod of
(d) 4 m length 2L, then the resistance will be
(a) R (b) 2R (c) 4R (d) 8R
20. The following three objects (1) a metal tray, (2) a
block of wood and (3) a woolen cap are left in a closed 26. Two charges +Q and −2Q are located at points A and
room overnight. Next day, the temperature of each is B on a horizontal line as shown below.
recorded as T1 , T2 and T3 , respectively. The likely +Q –2Q
situation is A B
(a) T1 = T2 = T3 (b) T3 > T2 > T1 The electric field is zero at a point which is located at
(c) T3 = T2 > T1 (d) T3 > T2 = T1 a finite distance
21. We sit in the room with windows open. Then, (a) on the perpendicular bisector of AB
(b) left of A on the line
(a) air pressure on the floor of the room equals the
atmospheric pressure but the air pressure on the (c) between A and B on the line
ceiling is negligible (d) right of B on the line
(b) air pressure is nearly the same on the floor, the walls 27. A 750 W motor drives a pump which lifts 300 L of
and the ceiling water per minute to a height of 6 m. The efficiency of
(c) air pressure on the floor equals the weight of the air the motor is nearly
column inside the room (from floor to ceiling) per unit
area
(Take, acceleration due to gravity to be 10 m / s2)
(d) air pressure on the walls is zero, since the weight of air (a) 30% (b) 40% (c) 50% (d) 20%
acts downward 28. Figure below shows a portion of an electric circuit
22. A girl standing at point P on a beach wishes to reach with the currents in amperes and their directions.
a point Q in the sea as quickly as possible. She can The magnitude and direction of the current in the
run at 6 kmh −1 on the beach and swim at 4 kmh −1 in portion PQ is
the sea. She should take the path 3A
Q P
Q 1A

Sea A B C D 2A
Beach

8A
P
(a) PAQ (b) PBQ (c) PCQ (d) PDQ 4A 2A
23. Light enters an isosceles right triangular prism at
normal incidence through face AB and undergoes total (a) zero (b) 3 A from P to Q
internal reflection at face BC as shown below. (c) 4 A from Q to P (d) 6 A from Q to P
https://iit-jeeacademy.blogspot.com

126 KVPY Question Paper 2011 Stream : SA

29. A nucleus of lead Pb214


82 emits two electrons followed
When the partition is removed, the gases mix. The
by an α-particle. The resulting nucleus will have changes in enthalpy (∆H ) and entropy (∆S ) in the
(a) 82 protons and 128 neutrons
process, respectively, are
(b) 80 protons and 130 neutrons (a) zero, positive (b) zero, negative
(c) 82 protons and 130 neutrons (c) positive, zero (d) negative, zero
(d) 78 protons and 134 neutrons 36. The gas produced from thermal decomposition of
30. The number of air molecules in a (5m × 5m × 4m) (NH4 )2 Cr2O7 is
room at standard temperature and pressure is of the (a) oxygen (b) nitric oxide
order of (c) ammonia (d) nitrogen
(a) 6 × 1023 (b) 3 × 1024 (c) 3 × 1027 (d) 6 × 1030 37. The solubility curve of KNO3 in water is shown below.
250

Solubility (gram)/100 gram of water


CHEMISTRY
200
31. Two balloons A and B containing 0.2 mole and 0.1 mole
of helium at room temperature and 2.0 atm,
150
respectively, are connected. When equilibrium is
established, the final pressure of He in the system is
100
(a) 1.0 atm (b) 1.5 atm
(c) 0.5 atm (d) 2.0 atm
50
32. In the following set of aromatic compounds
NO2 0
0 10 20 30 40 50
Temperature (°C)
The amount of KNO3 that dissolves in 50 g of water at
40°C is closest to
(i) (ii) (a) 100 g (b) 150 g (c) 200 g (d) 50 g
COOCH3 OCH3 38. A compound that shows positive iodoform test is
(a) 2-pentanone (b) 3-pentanone
(c) 3-pentanol (d) 1-pentanol
39. After 2 hours the amount of a certain radioactive
(iii) (iv) substance reduces to 1/16th of the original amount
The correct order of reactivity toward Friedel-Crafts (the decay process follows first-order kinetics). The
alkylation is half-life of the radioactive substance is
(a) i >ii > iii > iv (b) ii > iv > iii > i
(a) 15 min (b) 30 min (c) 45 min (d) 60 min
(c) iv > ii > iii > i (d) iii > i > iv > ii
40. In the conversion of a zinc ore to zinc metal, the
33. The set of principal (n ), azimuthal (l) and magnetic process of roasting involves
(ml ) quantum numbers that is not allowed for the (a) ZnCO3 → ZnO (b) ZnO → ZnSO4
electron in H-atom is (c) ZnS → ZnO (d) ZnS → ZnSO4
(a) n = 3, l = 1, ml = − 1
(b) n = 3, l = 0, ml = 0 41. The number of P–H bond(s) in H3 PO2 , H3 PO3 and
(c) n = 2, l = 1, ml = 0 H3 PO4 , respectively, is
(d) n = 2, l = 2, ml = − 1 (a) 2, 0, 1 (b) 1, 1, 1 (c) 2, 0, 0 (d) 2, 1, 0
34. At 298 K, assuming ideal behaviour, the average 42. When chlorine gas is passed through an aqueous
kinetic energy of a deuterium molecule is solution of KBr, the solution turns orange brown due
(a) two times that of a hydrogen molecule to the formation of
(b) four times that of a hydrogen molecule (a) KCl (b) HCl (c) HBr (d) Br 2
(c) half of that of a hydrogen molecule 43. Among
(d) same as that of a hydrogen molecule H
35. An isolated box, equally partitioned contains two O
N
ideal gases A and B as shown
N
(i) (ii) (iii) (iv)
A B
1 atm 25°C 1 atm 25°C the compound which is not aromatic is
(a) i (b) ii (c) iii (d) iv
https://iit-jeeacademy.blogspot.com

KVPY Question Paper 2011 Stream : SA 127

44. Among the following compounds 49. The gall bladder is involved in
H3C (a) synthesising bile
CH3 (b) storing and secreting bile
H3C
CH3 (c) degrading bile
CH3 H3 C (d) producing insulin
H 3C
CH3
50. Which one of the following colours is the least useful
(i) (ii) for plant life?
(a) Red (b) Blue
H3 C (c) Green (d) Violet
CH3 CH3
51. At rest, the volume of air that moves in and out per
CH3
breath is called
H3C CH3 H3 C
(a) resting volume (b) vital capacity
CH3 (c) lung capacity (d) tidal volume
(iii) (iv) 52. How many sex chromosomes does a normal human
2,3-dimethylhexane is inherit from father?
(a) i (b) ii (c) iii (d) iv (a) 1 (b) 2
45. The major product formed in the reaction, (c) 23 (d) 46
Br 53. In the 16th century, sailors who travelled long
distances had diseases related to malnutrition,
Cl NaCN because they were not able to eat fresh vegetables
product
DMSO, heat and fruits for months at a time. Scurvy is a result of
the deficiency of
I (a) carbohydrates (b) proteins
is (c) vitamin-C (d) vitamin-D
Br CN 54. Which of the following structures is not found in
plant cells?
(i) Cl Cl (a) Vacuole (b) Nucleus
(ii)
(c) Centriole (d) Endoplasmic reticulum

CN CN 55. The cell that transfers information about pain to the


Br CN brain is called a
(a) neuron (b) blastocyst
CN (iv) Cl (c) histoblast (d) haemocyte
(iii)
56. The presence of nutrients in the food can be tested.
Benedict’s test is used to detect
I I
(a) sucrose (b) glucose
(a) i (b) ii (c) iii (d) iv (c) fatty acid (d) vitamins
57. Several minerals such as iron, iodine, calcium and
BIOLOGY phosphorus are important nutrients. Iodine is
found in
46. If parents have free earlobes and the offspring has (a) thyroxine (b) adrenaline
attached earlobes, then the parents must be (c) insulin (d) testosterone
(a) homozygous (b) heterozygous
(c) codominant (d) nullizygous
58. The principle upon which a lactometer works is
(a) viscosity (b) density
47. During meiosis, there is (c) surface tension (d) presence of protein
(a) one round of DNA replication and one division
(b) two rounds of DNA replication and one division
59. Mammalian liver cells will swell up when kept in
(c) two rounds of DNA replication and two divisions (a) hypertonic solution (b) hypotonic solution
(d) one round of DNA replication and two divisions (c) isotonic solution (d) isothermal solution

48. Blood clotting involves the conversion of 60. The form of cancer called ‘carcinoma’ is associated
(a) prothrombin to thromboplastin
with
(b) thromboplastin to prothrombin (a) lymph cells (b) mesodermal cells
(c) fibrinogen to fibrin (c) blood cells (d) epithelial cells
(d) fibrin to fibrinogen
https://iit-jeeacademy.blogspot.com

128 KVPY Question Paper 2011 Stream : SA

PART-II (2 Marks Questions)


MATHEMATICS When the spring object system is immersed in a liquid
of density ρ1 as shown in the above figure, the spring
61. Let f (x) = ax + bx + c, where a , b, c are integers,
2
stretches by an amount x1 (ρ > ρ1 ). When the
Suppose f (1) = 0, 40 < f (6) < 50, 60 < f (7) < 70 and experiment is repeated in a liquid of density (ρ2 < ρ1 ),
1000t < f (50) < 1000 (t + 1) for some integer t. Then, the spring stretches by an amount x2. Neglecting any
the value of t is buoyant force on the spring, the density of the object is
(a) 2 (b) 3 (c) 4 (d) 5 or more ρ1 x1 −ρ2x2 ρ1 x2 −ρ2x1
(a) ρ = (b) ρ =
62. The expression x1 − x2 x2 − x1
(
22 + 1 32 + 1 42 + 1 .... 2011)2 + 1 ρ1 x2 + ρ2x1 ρ1 x1 + ρ2x2
+ + + + (c) ρ = (d) ρ =
22 − 1 32 − 1 42 − 1 (2011)2 − 1 x1 + x2 x1 + x2

lies in the interval 67. A body of 0.5 kg moves along the positive X-axis
(a)  2010, 2010  (b)  2011 −
1 1 1  under the influence of a varying force F ( in newton)
, 2011 − 
 2  2011 2012  as shown below.
 1
(c)  2011,2011  (d)  2012, 2012 
1
 2  2 3
63. The diameter of one of the bases of a truncated cone
is 100 mm. If the diameter of this base is increased 2

F(N)
by 21% such that it still remains a truncated cone
with the height and the other base unchanged, the
1
volume also increases by 21%. The radius of the other
base (in mm) is
(a) 65 (b) 55 (c) 45 (d) 35 0,0 2 4 6 8 10
64. Two friends A and B are 30 km apart and they start x(m)
simultaneously on motorcycles to meet each other. The . ms−1, then
If the speed of the object at x =4 m is 316
speed of A is 3 times that of B. The distance between its speed at x = 8 m is
them decreases at the rate of 2 km per minute. Ten
(a) 3.16 ms−1 (b) 9.3 ms−1 (c) 8 ms−1 (d) 6.8 ms−1
minutes after they start, A’s vehicle breaks down and
A stops and waits for B to arrive. After how much time 68. In a thermally isolated system, two boxes filled with
(in minutes) A started riding, does B meet A? an ideal gas are connected by a valve. When the
(a) 15 (b) 20 (c) 25 (d) 30 valve is in closed position, states of the box 1 and 2
respectively, are (1 atm, V, T) and (0.5 atm, 4V, T).
65. Three taps A, B, C fill up a tank independently in 10 h, When the valve is opened, then the final pressure of
20 h, 30 h, respectively. Initially the tank is empty and the system is approximately
exactly one pair of taps is open during each hour and (a) 0.5 atm (b) 0.6 atm (c) 0.75 atm (d) 1.0 atm
every pair of taps is open at least for one hour. What is
the minimum number of hours required to fill the tank? 69. A student sees the top edge and the bottom centre C
of a pool simultaneously from an angle θ above the
(a) 8 (b) 9 (c) 10 (d) 11
horizontal as shown in the figure. The refractive
index of water which fills up to the top edge of the
4 h 7
PHYSICS pool is . If = , then cos θ is
3 x 4
66. An object with uniform density ρ is attached to a
spring that is known to stretch linearly with applied
force as shown below. θ

C
x
2 8 8 8
(a) (b) (c) (d)
7 3 45 3 53 21
https://iit-jeeacademy.blogspot.com

KVPY Question Paper 2011 Stream : SA 129

70. In the following circuit, 1Ω resistor dissipates power


P . If the resistor is replaced by 9 Ω, the power
BIOLOGY
dissipated in it is 76. You remove four fresh tobacco leaves of similar size
3Ω and age. Leave ‘leaf 1’ as it is, smear ‘leaf 2’ with
vaseline on the upper surface, ‘leaf 3’ on the lower
surface and ‘leaf 4’ on both the surfaces. Hang the
+ leaves for a few hours and you observe that ‘leaf 1’
10 V 1Ω
– wilts the most, ‘leaf 2’ has wilted, ‘leaf 3’ wilted less
than ‘leaf 2’ and ‘leaf 4’ remains fresh. Which of the
following conclusions is most logical?
P (a) Tobacco leaf has more stomata on the upper surface
(a) P (b) 3P (c) 9P (d) (b) Tobacco leaf has more stomata on the lower surface
3
(c) Stomata are equally distributed in upper and lower
surfaces
CHEMISTRY (d) No conclusion on stomatal distribution can be drawn
from this experiment
71. An aqueous buffer is prepared by adding 100 mL of
77. Vestigial organs such as the appendix exist because
0.1 mol L−1 acetic acid to 50 mL of 0.2 mol L−1 of (a) they had an important function during development
sodium acetate. If pK a of acetic acid is 4.76, the pH of which is not needed in the adult
the buffer is (b) they have a redundant role to play if an organ with
(a) 4.26 (b) 5.76 (c) 3.76 (d) 4.76 similar function fails
72. The maximum number of structural isomers possible (c) nature cannot get rid of structures that have already
for the hydrocarbon having the molecular formula formed
C4H6, is (d) they were inherited from an evolutionary ancestor in
(a) 12 (b) 3 (c) 9 (d) 5 which they were functional

73. In the following reaction sequence, X and Y , 78. Mendel showed that unit factors, now called alleles,
respectively, are exhibit a dominant/recessive relationship. In a
O monohybrid cross, the .......... trait disappears in the
X Y first filial generation.
OH (a) dominant (b) codominant
(a) H2O2 ; LiAlH4 (b) C6 H5 COOH; LiAlH4
(c) C6 H5 COOH;Zn /Hg ⋅ HCl (d) alk. KMnO4 ; LiAlH4 (c) recessive (d) semi-dominant

74. Among (i) [Co(NH3 )6 ]Cl3 , (ii) [Ni(NH3 )6 ]Cl2, 79. If a man with an X-linked dominant disease has six
(iii) [Cr(H2O)6 ]Cl3 , (iv) [Fe(H2O)6 ]Cl2 the complex sons with a woman having a normal complement of
which is diamagnetic is genes, then the sons will
(a) i (b) ii (c) iii (d) iv (a) not show any symptoms of the disease
(b) show strong symtpoms of the disease
75. At 783 K in the reaction, H2 ( g) + I2 ( g) 2HI( g), the
e (c) three will show a disease symptom, while three will not
molar concentrations (mol L−1 ) of H2 , I2 and HI at
(d) five will show a disease symptom, while one will not
some instant of time are 0.1, 0.2 and 0.4,
respectively. If the equilibrium constant is 46 at the 80. In evolutionary terms, an Indian school boy is more
same temperature, then as the reaction proceeds closely related to
(a) the amount of HI will increase (a) an Indian frog
(b) the amount of HI will decrease (b) an American snake
(c) the amount of H 2 and I2 will increase (c) a Chinese horse
(d) the amount of H 2 and I2 will not change (d) an African shark
https://iit-jeeacademy.blogspot.com

130 KVPY Question Paper 2011 Stream : SA

Answers
PART-I
1 (a) 2 (c) 3 (a) 4 (b) 5 (b) 6 (d) 7 (a) 8 (a) 9 (a) 10 (c)
11 (b) 12 (c) 13 (b) 14 (a) 15 (c) 16 (a) 17 (a) 18 (c) 19 (d) 20 (a)
21 (b) 22 (c) 23 (c) 24 (d) 25 (c) 26 (b) 27 (b) 28 (d) 29 (a) 30 (c)
31 (d) 32 (c) 33 (d) 34 (d) 35 (a) 36 (d) 37 (a) 38 (a) 39 (b) 40 (c)
41 (d) 42 (d) 43 (b) 44 (b) 45 (c) 46 (b) 47 (d) 48 (c) 49 (b) 50 (c)
51 (d) 52 (a) 53 (c) 54 (c) 55 (a) 56 (b) 57 (a) 58 (b) 59 (b) 60 (d)

PART-II
61 (c) 62 (c) 63 (b) 64 (d) 65 (a) 66 (b) 67 (d) 68 (b) 69 (c) 70 (a)
71 (d) 72 (c) 73 (b) 74 (a) 75 (a) 76 (b) 77 (d) 78 (c) 79 (a) 80 (c)

Solutions
1. (a) Given, Now, squaring both sides, we get All A , B ,C , D are in the form of (a )x = b
(x − b)(x − c) (x − c)(x − a ) 2
 x + 1  = a 2 x is largest when a is smallest.
P (x ) = +
(a − b)(a − c) (b − c)(b − a )  x ∴ In A , B , C , D
(x − a )(x − b) ⇒
1
x + 2 + 2 = a2
2 0< r < 4
+ …(i)
(c − a )(c − b) x r
1+ is smallest
P (a ) = 1 + 0 + 0 = 1 On cubing both sides, we get 17
3
 x + 1 ∴Option (b) is correct.
 = x + 3 + 3  x +
P (b) = 0 + 1 + 0 = 1 1 1
 = a …(ii)
3 3
 x x  x 6. (d) Given,
P (c) = 0 + 0 + 1 = 1
On adding Eqs. (i) and (ii), we get ABC is right angled triangle with B is 90°.
P (x) is a polynomial of degree atmost 2
 x2 + 1  +  x3 + 1  + 2 + 3  x + 1 
and also attains same value i.e. 1 for       A
 x3   x2   x
distinct values of x (i.e. a , b, c).
= a3 + a 2
∴ P (x) is an identity with only value equal  1
⇒ b +  x + 2  + 2 + 3a = a + a 2
3 3
to 1 for all R.  x  6
(x − b)(x − c) (x − c)(x − a ) 1
∴ + + ⇒ x + 2 = a3 + a 2 − 3a − b − 2
3
(a − b)(a − c) (b − c)(b − a ) x
(x − a )(x − b) 4. (b) Given,|a − b| = 2,|b − c| = 3 and 90°
=1
(c − a )(c − b) |c − d| = 4 B D C
2. (c) Given, a 2 + b2 = 81 ∴ a − b = ± 2, b − c = ± 3 and c − d = ± 4 AD is angle bisector of ∠ A.
Possible value of (a − d ) are
⇒ x2 + y2 = 121 ∴
AB BD
=
±9, ± 5, ± 3, ± 1 .
⇒ ax + by = 99 AC DC
∴ |a − d| = 9, 5, 3, 1
Now, (a 2 + b2 )(x2 + y2 ) = 81 × 121 Sum of|a − d| = 9 + 5 + 3 + 1 = 18 ⇒ AB ⋅ CD = BD ⋅ AC
a x + b y + a y + b x = 81 × 121
2 2 2 2 2 2 2 2
…(i) 5. (b) Given, 0< r < 4 Area of ∆ADC = 10
x
(ax + by) = 99 1
A = 5  1+  = 9
and r ⇒ × AB ⋅ CD = 10
⇒ (ax + by) = 99
2 2  π 2
x 1
⇒ a 2x2 + b2 y2 + 2axby = 992 …(ii) ⇒  1+ r  = 9 ⇒ × BD ⋅ AC = 10
 π 5 2
On subtracting Eq. (i) from Eq. (ii), we get
x 20
a 2 y2 + b2x2 − 2axby = 0 B =  1+  =
r 9 ⇒ BD =
 17  5 AC
⇒ (ay − bx)2 = 0 9 20
C = (1 + 2r ) =
x ⇒ BD = [Q AC = 6]
⇒ ay − bx = 0 5 6
1 1 x
10
D =  1 +
3. (a) Given, x + = a and x2 + 3 = b 1 9 ⇒ BD =
 =
x x  r  5 3
https://iit-jeeacademy.blogspot.com

KVPY Question Paper 2011 Stream : SA 131

AB
7. (a) We know, θ = 10. (c) We have, 13. (b) We have, initial position of
OB (512)3 − (253)3 − (259)3 frog = (0,0)
⇒ (512) + (−253) + (−259)
3 3 3

O (0, 6)
Now, 512 − 253 − 259 = 0
l=13
θ 12 We know that, a + b + c = 0 then 5
2nd

a + b + c = 3abc
3 3 3
(0, 1) A(4, 3)
A B ∴ (512) − (253)3 − (259)3
3
5
From first figure, = 3(512)(−253)(−259) 5
1st
AB = 2 πr = 3 ⋅ 512 ⋅ 253 ⋅ 259
O (0, 0)
OB = 13 = 3 ⋅ 29 ⋅ 11 × 23 × 7 × 37
10 π ∴ There are 6 distinct prime divisors.
∴ 2 π (5) = 13θ ⇒θ =
13 11. (b) Given, square base pyramid
8. (a) ∆ABE ~ ∆EDC After 1st jump position of frog at (4, 3).
is incomplete.
At 2nd jump position of frog at (0,6).
A
At 3rd jump position of frog at (0,1).
90°–θ
∴Minimum number of jumps required for
the frog to go from (0,0) to (0,1) and each
distance is 5 units is 3.

12 C 14. (a) Digit 1 appears in 1, 10, 11, and


12 in hour.
θ
8 ∴The clock will show the incorrect time
between 1 − 2, 10 − 11, 11 − 12, 12 − 1 day
θ 90°–θ
B and night both incorrect time
x E(20–x) D The top layer = 13 balls
(8 × 60) = 480 min
20 There are 18 layer completed.
Digit l appear in minutes 1, 10, 11, 12,
AB ED
∴ = So, total number of balls 13, 14, 15, 16, 17, 18, 19, 21, 31, 41, 51
BE CD N = 132 + 142 + 152 + 162 + ... + 302
12 20 − x = 15 min
⇒ = N = (12 + 22 + 32 + 42 + ... + 302 ) ∴ It will shows the incorrect time
x 8
− (12 + 22 + 32 ... 122 ) = 16 × 15
⇒ 96 = 20x − x2 30 × 31 × 61 12 × 13 × 25
⇒ x2 − 20x + 96 = 0 ⇒ N = − = 240 min
6 6
⇒ (x − 12)(x − 8) = 0 Total incorrect time = 240 + 480
⇒ N = 5 × 31 × 61 − 2 × 13 × 25
⇒ x = 8, 12 = 720 min
= 9455 − 650
Hence, x has two values 8 and 12 and Correct time = 24 × 60 − 720
= 8805
their difference is 4. Fraction of correction time
∴ 8000 < N < 9000 24 × 60 − 720
9. (a) Given, radius of each circle = 1 =
12. (c) Let the total distance = x 24 × 60
x 1
1 Muddy distance = =
6 2
O x x x
Water distance = − = 15. (c) Let the student answered correct
1 1 2 6 3
x =x
d Tar distance = Student answer wrong = y
1 2
1
60° D Speed travelling in mud = 3y Student unattempted = z
A
1 1 B Speed travelling by stream = 4y According to the question,
x + y + z = 30, and 4x + z = 60
1 Speed travelling in tar = 5y
x /6 x /3 x /2 x = 15, y = 15, z = 0
touch externally Ratio of time = : :
3y 4y 5y x = 14, y = 12, z = 4
∴ OA = OB = AB = 2
OD 1 1 1 x = 13, y = 9, z = 8
In ∆OAD, sin 60° = = : :
OA 18 12 10 x = 12, y = 6, z = 12
3 =
10 15 18 x = 11, y = 3, z = 16
⇒ OD = OA sin 60° = 2 × = 3 : :
2 180 180 180 x = 10, y = 0, z = 20
∴ d = 1 + OD + 1= 1 + 3 + 1 = 3 + 2 = 10 : 15 : 18 Total number of cases = 6
https://iit-jeeacademy.blogspot.com

132 KVPY Question Paper 2011 Stream : SA

16. (a) Total length of a pendulum 19. (d) As there is no external force, 23. (c) As total internal reflection occurs
remains same, so extreme point D lies on centre of mass of a system remains at at angle of incidence, i = 45°.
the line AB, as shown below. same position.
Initially,
45°
90°
Box 45°
C D
B
A

This can be proved by applying energy


conservation between extreme positions O
1
A and D (its given friction is abscent), (20,0) So, using µ = , we have
sin C
K A + U A = KB + UB = KD + UD Position of centre of mass of a system 1
taking girl at origin is µ= = 2 or µ ≈ 142
.
⇒ 0 + U A = 0 + UB = 0 + UD sin 45°
36 × 0 + 9 × 20 9 × 20
⇒ U A = UB = UD ⇒ hA = hB = hD XCM = = 24. (d) When a lens is cut into half or its
36 + 9 45
17. (a) Taking boy, toy and ground as a half part is blackened, image is formed at
composite system, we can say that there Finally, same place but its intensity is reduced.
is no external force acting on the system, 25. (c) As material of rod is not changed,
net acceleration of the system is zero. resistivity of both rods is same.
Asystem = 0 ⇒ (Fnet )system = 0 Also, volume of material is same for both
⇒ F + (mg )boy + (N )ground + (f )friction = 0 Box rods, so
18. (c) Let first ball reaches upto height A1 l1 = A2l2
O x 20 – x
H and it fells by a distance H − h, where or A1 L = A2 (2L)
it collided with second ball which rises (20,0) A
⇒ A2 = 1
upto height h. Position of centre of mass when girl and 2
box are at same position is l
Now, using R = ρ , we have
(36 × x) + (9 × x) 36x + 9x A
H–h X ′CM = =
36 + 9 45 2L  ρL 
R2 = ρ = 4 
9 × 20 36x + 9x (A1 / 2)  A1 
As, XCM = XCM ′ ⇒ =
45 45
H or R2 = 4R
⇒ 9 × 20 = 45x ⇒ x = 4 m
h So, girl travelled by 4m, when she meet 26. (b) As direction of fields of charges at
with box. points A and B,
20. (a) Most likely each of the object is in EA EA EB
thermal equilibrium with its EB + Q EB – 2Q EA
surroundings. So, T1 = T2 = T3 .
Equation of motion for first and second A B
ball, 21. (b) Gas molecules move randomly are in opposite directions to left of A or
1 and effect of gravity on them is
H − h = gt 2 …(i) insignificantly low. So, pressure exerted
right of B, so fields can be zero in these
2 regions. But in right side of B, field
1 2 by gas molecules is same everywhere.
h = ut − gt …(ii) cannot be zero as EA is very smaller than
2 22. (c) To reach point Q, using Fermat’s
EB (charge at A is smaller magnitude and
From Eqs. (i) and (ii), we have principle, girl must bend her path
its distance from B is also large).
H u 2 / 2g u towards normal as on beach velocity of
H = ut or t = = = So, field can be zero in region left of A.
u u 2g girl is more than her velocity at sea.
Substituting the value of t in Eq. (ii), we Q 27. (b) Useful power
have Work done by motor
=
u 1 u2 Time duration
h=u× − g× Sea C W
2g 2 4g 2 ⇒ Pinput ×η=
u 2 u 2g
Beach ∆t
h= − mgh
2g 8g 2 ⇒ Pinput ×η=
P t
u 2
u 4u − u
2 2 2
300 × 10 × 6
h= − = So, correct path is PCQ to reach in ⇒ η= = 0.4
2g 8g 8g shortest time. 60 × 750
2
3 u 3  u  Note Laws of refraction of light follows So, per cent efficiency is, η = 40%
or h = 3u 2 / 8 g = × = ⋅H QH =
 g 
4 g 4  2 from Fermat’s principle.
https://iit-jeeacademy.blogspot.com

KVPY Question Paper 2011 Stream : SA 133

28. (d) Using Kirchhoff’s junction rule, (i) values of l should range from 0 to n − 1 (c) OH
directions and magnitudes of currents (ii) values of m should range from −l to l  Η +/Η 2Ο
are as, CH3 CH2 C HCH2CH3 + I2 / ΝaΟΗ →
Thus, the set that is not allowed for 3-pentanol
3A
electron in H- atom is No reaction
P Q
6A 3A 1A n = 2, l = 2, m = − 1 It doesn’t give positive iodoform test.
4A The allowed set of quantum numbers for (d) CH3 CH2CH2CH2CH2OH + I2 / NaOH
2A 10 A
H - atom having n = 2 will be 1-pentanol
6A H + / H 2O
8A
l = 0 to 1 → no reaction
ml = − 1, 0, 1 It also give negative iodoform test.
4A 2A
34. (d) Average kinetic energy depends 39. (b) For first order reaction
upon the temperature and not on the 2. 303  a 
Clearly, current in the portion PQ is from type of gases involved. k= log  
t  a − x
Q to P is 6A. 3kT
For any gas, (K. E )avg = per molecule 0.693
29. (a) Decay scheme is as given below 2 also k =
214 214 The (K. E)avg of a deuterium molecule is t1/ 2
Pb A + 20 e
82 84 –1
same as that of hydrogen molecule. 0.693 2. 303  a 
210 4 ∴ = log  
X + He 35. (a) ∆H for this process = CV ∆T = 0 t1/ 2 t  a − x
82 2

Following conservation of mass number (at constant temperature) According to question,


and atomic numbers, we have ∆S for this process will be positive that is 0.693 2. 303
= log
a
Number of neutrons in X is ∆S > 0, the randomness increases the t1/ 2 2 × 60 a / 16
N = A − Z = 210 − 82 = 128 molecules of gases A and B gets
⇒ t1/ 2 = 30 min
and number of protons is 82. intermixed with each other, when the
partition is removed. 40. (c) In the process of roasting,
30. (c) From ideal gas equation, sulphide ore is converted into an oxide
Thus, the correct option is (a).
pV ore with a regular supply of air in a
pV = nRT ⇒ pV = NkBT or N = 36. (d) The thermal decomposition of
kBT furnace at a temperature below the
(NH4 )2 Cr2O7 gives chromium oxide melting point of the metal.
Substituting given values, we get (Cr2O3 ), nitrogen gas and water.
105 × 100 ∆
Thus, the conversion of sulphide ore into
⇒ N = (NH4 )2 Cr2O7 → Cr2O3 + N2 + 4H2O metal oxide is given the reaction
. × 10−23 × 273
138 ZnS → ZnO, hence option (c) is correct.
37. (a) From the graph it can be seen
⇒ N = 3 × 1027 molecules 41. (d) The structures of given
that solubility of KNO3 in water at 40°C
31. (d) Since the pressure of the helium is approximately 200 g per 100 of water. compounds can be drawn as follows :
gas in both the balloons A and B are
∴Amount of KNO3 that dissolve (or O O O
same. Therefore, the final pressure of He
solubility) in 50 g of water will
will not change. Hence, the correct option P P P
200
is (d). = × 50 = 100 g
100 H OH , HO H, HO OH
32. (c) The electron releasing groups H OH OH
attached to benzene increases the 38. (a) Iodoform test with sodium H3PO2 H3PO3 H3PO4
reactivity towards Friedel-Crafts hypoiodite is used for the detection of
Thus, the number of P– H bond(s) in
alkylation whereas the electron CH3 CO group or CH3 CH(OH) group
H3 PO2 , H3 PO3 and H3 PO4 respectively
withdrawing groups decreases the which produces CH3 CO group on
are 2, 1, 0.
reactivity. oxidation.
Among the given groups,—NO2 and Iodoform reaction with the given
42. (d) When chlorine gas is passed
COOCH3 are electron withdrawing group compounds are as follows : through an aqueous solution of KBr, the
EWG, so they will decrease the reactivity solution turns orange brown due to the
(a) CH3 —CH2— CH2— C— CH3 + I2/
where NO2 shows stronger −I effect than || NaOH evolution of bromine gas. The equation
COOCH3 . So, the reactivity of O for the above reaction can be written as
nitrobenzene towards Friedel-Craft 2-pentanone
H + / H 2O Cl 2 + 2KBr → 2KCl + Br2 ↑
alkylation will be least. OCH3 is an
electron donating group, so it will → CH3 CH2CH2COOH + CHI3 Orange brown
Butanoic acid Iodoform
increasing the reactivity. 43. (b) The conditions for a compound to
If gives positive iodoform test due to the be aromatic are
Thus, the order of reactivity towards
presence of CH3 CO group. (i) the molecule should be planar.
Friedel-Craft alkylation is
(b) CH3 —CH2— C—CH2— CH3 + I2/NaOH (ii) it should be cyclic with alternate
(iv) > (ii) > (iii) > (i) || single and double bonds.
33. (d) For any set of principal (n ), O
3-pentanone H + / H O (iii) it should follows Huckel’s rule, i.e.
azimuthal (l) and magnetic (mi ) quantum → no reaction
2

numbers, the conditions that are allowed should have (4n + 2) π electrons.
It gives negative iodoform test.
for an electron is
https://iit-jeeacademy.blogspot.com

134 KVPY Question Paper 2011 Stream : SA

π electrons present in given compounds This reaction involves SN 2 mechanism Total lung capacity refers to the total
are as follows : where CN − is substituted over Cl − as it is amount of air in the lungs after taking
H a good leaving group as compared to Br the deepest breath possible.
and I, and occurs at a primary carbon
N 52. (a) A normal human inherits only
(sp3 -hybridised).
(i) one sex chromosome (either X or Y
(4n+2)π electrons = 6π electrons 46. (b) Attached earlobes is an autosomal chromosome) from father. Sex
recessive trait. Thus, a heterozygous chromosomes, determine whether an
∴ Follows Huckel’s rule parent with free earlobes will have
individual is male or female. In human
offspring with attached earlobes.
and other mammals these are designated
(ii) (4n+2)π electrons Free earlobes Free earlobes
= 8π electrons by scientists as X and Y. In humans, the
Aa × Aa + Parents
sex chromosomes comprise one pair of the
Doesn’t follow Huckel’s rule. total of 23 pairs of chromosomes. The
other 22 pairs of chromosomes are called
AA Aa Aa aa
(iii) (4n+2)π electrons Offsprings autosomes.
Free earlobes
= 10π electrons Attached
53. (c) Scurvy is a result of the deficiency
earlobes
of vitamin-C. Vitamin-C is mainly found
Follows Huckel’s rule. 47. (d) For a complete meiotic cell
division, there takes place one round of in fruits such as oranges, grapefruit,
O lemons, strawberries and melons or it is
(4n+2)π electrons DNA replication during the S-phase and
(iv) found in vegetables such as broccoli and
= 6π electrons two divisions. This is because meiosis is a
N bell peppers. Therefore, malnutrition
process where a single cell divides twice
Follows Huckel’s rule. to produce four cells containing half the causes vitamin-C deficiency. Protein
Thus, compound (ii) is not aromatic. original amount of genetic information. deficiency malnutrition is known as
48. (c) Blood clotting involves the kwashiorkor and marasmus. Vitamin-D
44. (b) The IUPAC nomenclature of the
conversion of fibrinogen to fibrin. The deficiency causes rickets. Carbohydrates
structures given in the options are as
blood clotting mechanism takes place as deficiency causes weakness, nausea,
follows
1 follows dehydration, etc.
2
H 3C 54. (c) Centriole is not found in plant
3 Injury in blood vessels
(i) CH3 3, 4-dimethyl cells, it is found only in animal cells.
5 4 hexane Platelets clump at the wound These paired organelles are typically
6
H 3C CH3 located together near the nucleus in the
Platelets release thrombokinase centrosome, a granular mass that serves
5
6 4 as an organising centre for microtubules.
H 3C 1 2, 3-dimethyl Thrombin Prothrombin Centriole is involved in the development
(ii) 3 CH3 hexane
Thrombokinase
2 of spindle fibres in cell division.
H 3C Vitamin-K
55. (a) The brain and spinal cord are
Fibrin Fibrinogen
1
CH3 made up of many cells, including neurons
H 3C and glial cells. Neurons are cells that
2 Clot forms to prevent send and receive electro-chemical signals
(iii) 3 5 3, 4-dimethyl further blood loss for pain or pleasure to and from the brain
4 6 hexane 49. (b) The gall bladder is involved in and nervous system.
H3C CH3
storing and secreting bile. The gall ●
Blastocyst is a structure formed in the
CH3 bladder is a pear-shaped, hollow early development of mammals. It
CH3 CH3
structure located under the liver and on possesses an Inner Cell Mass (ICM)
(iv) 2, 4-dimethyl the right side of the abdomen. Its which subsequently forms the embryo.
CH3
2 4 6 hexane primary function is to store and ●
Histoblast is a cell or cell group
H 3C 1 3 5
concentrate bile, a yellow brown digestive possessing broad histogenetic capacity,
Hence, the correct option is (b). enzyme produced by the liver. i.e. capable of forming tissue.
45. (c) 50. (c) Green light is not at all useful for ●
Haemocyte is a cell of the haemolymph
Br photosynthesis. This is because, plant of various invertebrates, especially
reflects green light and due to this same arthropods.
Cl CN –
reason, plants appear green in colour. 56. (b) Benedict’s test is used to detect
DMSo, ∆ 51. (d) Tidal volume is the volume of reducing sugars such as glucose. Sucrose
Br inspired/expired air moving in and out of is a non-reducing sugar, it gives negative
I the lungs with each breath. result for Benedict’s test. Benedict’s
CN
Vital capacity is the volume that can be reagent is a complex mixture of sodium
inspired/expired after full carbonate, sodium citrate and copper (II)
expiration/inspiration. sulphate petahydrate.
I
https://iit-jeeacademy.blogspot.com

KVPY Question Paper 2011 Stream : SA 135

57. (a) Iodine is found in thyroxine. ∴ f (x) = 2x2 − 5x + 3 121 V πh  (605)2 + 60500r + (100r )2 
⇒ =
Thyroxine, also called f (50) = 2(50)2 − 5(50) + 3 100 3  (100)2


3, 5, 3′ , 5′-tetraiodothyronine or T4, is one = 5000 − 250 + 3 = 4753 121 πh
of the two major hormones secreted by Now, 1000t < f (50) < 1000(t + 1) ⇒ × (25 + 5r + r )
2
100 3
the thyroid gland (the other is ∴ 1000t < 4753 < 1000(t + 1)
triiodothyronine). Thyroxine’s principal ⇒ t < 4.753 < t + 1 πh  (605)2 + 60500r + (100r )2 
=
3 
function is to stimulate the consumption 
∴ t = 4 , t is integer. 10000 
of oxygen and thus the metabolism of all
62. (c) Let ⇒100(3025 + 605r + 121r 2 )
cells and tissues in the body. Thyroxine is
formed by the molecular addition of 2 +1 3 +1 4 +1
2 2 2
(2011) + 1 2
= 366025 + 60500r + (100r )2
S= + + + ... +
iodine to the amino acid tyrosine while 22 − 1 32 − 1 42 − 1 (2011)2 − 1 ⇒100 × 121r 2 − 10000r 2
the latter is bound to the protein r2 + 1 = 366025 − 302500
thyroglobulin. Here, Tr =
r2 −1 ⇒ 2100r 2 = 63525
58. (b) Lactometer is a device used for 63525
r2 −1 + 2 ⇒ r2 = = 30.25
testing the purity of milk. It measures ⇒ Tr = 2100
relative density of milk with respect to r2 −1
⇒ r = 30.25 = 5.5 cm = 55 mm
water, which is also called specific 2 2
gravity. If the specific gravity of a sample = 1+ 2 = 1+ 64. (d) Let the speed of B = x km/h
r −1 (r − 1)(r + 1)
of milk is within the approved ranges, the and the speed of A = 3x km/h
milk is pure. If it is not, then there is 1 1
⇒ Tr = 1 + − Distance between A and B = 30 km
some adulteration in milk. r −1 r + 1
Given, distance between them decrease
59. (b) When mammalian liver cells are 2011

kept in a hypotonic solution, endosmosis ⇒ S= ∑ Tr at 2 km per minutes.


occurs as the cell is hypertonic. Due to
r= 2 ∴ Distance decrease in one hour
2011
 1 1  = 2 × 60 = 120 km
the endosmosis, the cellular protoplasm = ∑ 1 + r − 1 − r + 1 ∴Total distance travelled by A and B in
is filled with water, it swells and the cells r= 2  
become turgid. Swelling is seen because
⇒ S = T2 + T3 + T4 + ... + T2011 one hour = (x + 3x) km = 4x km
the water flows from lower concentration 120
∴Speed = = 30 km/h
⇒ S =  1 + −  +  1 + − 
of solute to the higher concentration of 1 1 1 1
4
solute.  1 3  2 4
Hence, speed of B = 30 km/h
+  1 + −  + ... +  1 +
60. (d) Carcinoma is a category of types 1 1 1 1  Speed of A = 90 km/h
− 
of cancer that develops from epithelial  3 5  2010 2012  Distance travelled by A and B after
cells. Lymphoma is the cancer that occurs 1 1 1
⇒ S = 2010 + 1 + − − 10 min = 2 × 10 = 20 km
in lymph cells. Leukemia is blood cancer 2 2012 2011
that originates in the blood and bone So, remaining distance = (30 − 20) = 10 km
⇒ S = 2011 + − 
1 1 1 
marrow. Mesoderm is one of the germ + Time taken by B to distance travelled
2  2011 2012 
10 km =  × 60 = 20 min
layer from which skeletal muscle, bone, 10
 30 
⇒ S is lie between  2011, 2011  .
connective tissue, heart and the 1
urogenital system originate.  2 Total time taken by A = 20 + 10 = 30 min
61. (c) We have, 63. (b) Given, 65. (a) Taps A , B ,C fill up a tank
f (x) = ax2 + bx + c, a , b, c,∈ Z Diameter of base = 100 mm independently 10 h, 20 h, 30 h,
Also f (1) = 0, 40 < f (6) < 50, 60 < f (7) < 70 100 respectively.
∴Radius of base = mm = 50mm = 5 cm
∴a + b + c = 0, 40 < 36a + 6b + c < 50, 2 Given, exactly one pairs of taps is open
60 < 49a + 7b + c < 70 Let other radius of base = r during each hour and every pairs of taps
c= −a− b …(i) And height of truncated cone = h is open at least one hour.
∴ 40 < 36a + 6b − a − b < 50 Volume of initially truncated cone = V First A and B are open for one hour, then
πh B and C and then C and A
and 60 < 49a + 7b − a − b < 70 ∴ V = {(5)2 + 5r + r 2 }
∴ 
1 1  1 1  1 1
⇒ 40 < 35a + 5b < 50
3 +  +  +  +  + 
 10 20   20 30   30 10 
When radius increase by 21%
and 60 < 48a + 6b < 70 1 1 1 12 + 6 + 4
21 605 = + + =
70 ∴Radius of base = 5 + ×5=
⇒ 8 < 7a + b < 10 and 10 < 8a + b < 100 100 5 10 15 60
6 22 11
When volume increase by 21% = =
Now, a and b are integer 60 30
21 V 121 V
∴ 7a + b = 9 and 8a + b = 11 Then, V 1 = V + =
100 100 In three hours the tank will be filled
On solving these equation, we get  11  th part. Now, for minimum time,

πh  605 
2
605 
a = 2, b = − 5 Now, V 1 =    + r + r 2  30 
3  100  100 
Put the value of a , b in Eq (i), we get the rest of tank must be filled with A and
c=3 [Q r and h are same] B taps.
https://iit-jeeacademy.blogspot.com

136 KVPY Question Paper 2011 Stream : SA

∴  1 + 1  = 30 Now, by gas equation, we have These are as follows :


 10 20  20 As, n1 + n2 = n CH 2 == C == CH — CH 3 , CH 3 CH 2 C ≡≡ CH,
4V × 0.5 5V × p1
So, the rest of  1 −  th =   th part
11 19 V CH 3 — C == C — CH 3 ,
⇒ + =
 30   30  RT RT RT CH 2 == CH — CH == CH 2
of tank will taks 5 h more. ⇒ p = 0.6 atm
So, the tank will be filled in 8 h. 69. (c) Ray diagram for pool is as shown
66. (b) For equilibrium of block hung below. , , , ,
from string,
i 73. (b) In the first sequence of reaction,
Spring force + Buoyant force = Weight of
θ an alkene is getting converted into epoxy
block
group, so an oxidising agent is required
kx ρ f Vg r both H2O2 and C6 H5 COOH are oxidising
h agent but C6 H5 COOH is used as they are
m
not very sensitive to solvent polarity
while, in 2nd sequence of reaction, the
ρVg x/2
epoxy group is being reduced into an
So, we have Using n1 ⋅ sin i = n2 ⋅ sin r , we have alcoholic group, thus a reducing agent is
kx1 + ρ1Vg = ρVg 4
…(i) 1 × sin (90° − θ) = sin r …(i) required. Thus, the suitable reagents are
3
and kx2 + ρ2Vg = ρVg …(ii) O
x 4 2
Also, tan r = = = C6H5COOH
Eliminating k, we get 2h 7 × 2 7 oxidation

ρ x − ρ2x1 2 (Peroxide attack)


ρ= 1 2 ⇒ sin r = (Epoxy group)
x2 − x1 53
LiAlH4 Reduction
Substituting sin r in Eq. (i), we have
67. (d) By work-kinetic energy theorem, 4 2 8
work done is equal to change in kinetic cos θ = × = OH
3 53 3 53
energy.
x=8 70. (a) From given circuit, if i = circuit 74. (a) The magnetic character of the
So, ∫x = 4 Fdx = ∆K current, then given complexes are as follows :
V = iReq ⇒ i =
10
= 2.5 A (i) Co(NH 3 )6 ]3 +
F 4 Oxidation state of Co in [Co(NH3 )6 ]3+ is
3 So, power dissipated by circuit is +3.
25
P = i 2R = (2.5)2 × 1 = W The electronic configuration of Co3 + is
4 3d 6 4s0 .
1.5
When 1Ω resistor is replaced by a 9Ω 3d 4s 4p
resistor, then power dissipated in 9 Ω
x
4 8 10 resistor is
2
P ′ =   × (9)
10
⇒ Area under force-displacement graph  12  As NH3 is a strong field ligand, paring of
1 100 × 9 25
from x = 4 to x = 8 = m (vf2 − vi2 ) = = W electrons occur
2 12 × 12 4
1 1 1 [Co(NH3)6]3+
⇒ (3 − 15
. ) × (8 − 4) = × 3d 4s 4p
So, P ′ = P
2 2 2 ×××× ×× ×× ×× ××
× (vf2 − (3.16)2 ) 71. (d) Meq of CH3COOH = 100 × 0.1 = 10
−1 d 2sp 3 hybridised
⇒ vf = 6. 8 ms Meq of CH 3 COONa = 50 × 0.2 = 10
Thus, the complex is diamagnetic.
68. (b) Given situation is According to Henderson equation
[salt] (ii) [Ni(NH 3 )6 ]2 +
pH = pK a + log
[acid] Oxidation state of Ni in [Ni (NH3 )6 ]2+ is
p=1 atm p=0.5 atm
V 4V log [CH3 COO− ] +2.
pH = pK a +
T T log [CH3 COOH] The electronic configuration for Ni 2 + is
10 3d 8 4s0 .
Let final temperature after opening the pH = 4.76 + log
10 3d
valve is Tf , then ∆Wext = 0 and ∆Qext = 0
pH = 4.76 + log 1
So, from first law of thermodynamics,
⇒ pH = 4.76
∆U = 0 4s 4p 5d
72. (c) Nine structure isomers are
⇒ n1CV T + n2CV T = (n1 + n 2 ) CV Tf
possible for the hydrocarbon having
⇒ Tf = T molecular formula C 4 H 6 .
https://iit-jeeacademy.blogspot.com

KVPY Question Paper 2011 Stream : SA 137

Though NH 3 is a strong ligand pairing As H 2O is a weak field ligand, pairing of ears, etc. are some examples of vestigial
will not occur because, if pairing would electrons will not occur and thus the organs in human body.
occur then also 2,d-orbitals will not be complex will be paramagnetic. 78. (c) Gregor Mendel studied
available for hybridisation. 75. (a) For the reaction, inheritance of traits in pea plants. In a
3d
H 2 ( g ) + I2 ( g ) r 2HI( g ) [KC = 46] monohybrid cross, the recessive trait
[Ni(NH3)6]2+ disappears in the first filial generation.
[HI]2
QC = The traits that were visible in the
4s 4p 5d [H 2 ] [I2 ]
F1-generation are referred to as dominant
×× ×××××× ×××× 0.4 × 0.4
QC = ⇒ QC = 8 traits. This happens because recessive
. × 0.2
01
sp3d2 allele does not express itself in the
Thus, the complex is paramagnetic. As QC < KC presence of dominant allele.
(iii) [Cr(H 2O)6 ]3 + So, the reaction will proceeds in forward 79. (a) An individual gets one sex
direction. Hence, amount of HI increases. chromosome from each parent during
Oxidation state of Cr in [Cr (H2O6 )]3+ is
+ 3. 76. (b) Tobacco is a dicot plant, thus its fertilisation.
The electronic configuration of Cr 3 + is leaves have more number of stomata on + XX XY
[Ar] 3d3 4s0 . its lower surface. If you cover the leaves
of a healthy plant with vaseline, it will
[Cr(H2O)6]3+ XX XY XX XY
block its stomata and therefore it will not
Girl Boy Girl Boy
3d 4s 4p lose water through transpiration, so the
upward movement of the water in the We see from the above cross that sons get
×× ×× ×× ×××× ××
plant will stop. This will not allow the their Y-chromosome from father and
plant to wilt quickly. X-chromosome from mother.
d2sp3
As H 2O is a weak field ligand pairing of The leaf ‘2’ is smeared with vaseline on Therefore, if a man with an X-linked
electrons will not occur and the complex the upper surface, so the plant will lose dominant disease has six sons with a
is paramagnetic. water from the lower surface and in leaf woman having a normal complement of
‘3’, vaseline is smeared on the lower genes, their sons will show no symptom
(iv) [Fe(H 2O)6 ]2 +
surface therefore the water is lost from of the disease.
Oxidation state of Fe in [Fe (H2O6 )]2+ is
the upper surface. But the number of 80. (c) In evolutionary terms, an Indian
+ 2.
stomata are more on the lower surface school boy is more closely related to
The electronic configuration of Fe 2 + is therefore leaf ‘2’ will wilt more quickly Chinese horse. Human and horse
[Ar] 3d 6 4s0 . than leaf ‘3’. irrespective of their country are more
3d
77. (d) Vestigial organs are those organs generally similar in their chromosomal
[Fe(H2O6)]2+ which are no longer in use. They were management, than to rest of the species
used to play an important role in our mentioned here. This is because human
4s 4p 5d
×× ×× ×× ×× ×× ancestors, but as and when we developed and horse both belong to class
×× and evolved, some of these organs lost Mammalia. Other options like frog, snake
their functionality but managed to stay and shark belong to class– Amphibia,
sp3d2 in our body. Appendix, coccyx, external Reptilia and Chondrichthyes.
https://iit-jeeacademy.blogspot.com

138 KVPY Question Paper 2010 Stream : SA

KVPY
KISHORE VAIGYANIK PROTSAHAN YOJANA

QUESTION PAPER 2010


Stream : SA
MM 100

Instructions
There are 80 questions in this paper.
This question paper contains two parts; Part I and Part II. There are four sections; Mathematics, Physics, Chemistry
and Biology in each part.
Out of the four options given with each question, only one is correct.

PART-I (1 Mark Questions)


MATHEMATICS 5. The sides of a ∆ ABC are positive integers. The
smallest side has length 1. Which of the following
1. A student notices that the roots of the equation
statements is true?
x2 + bx + a = 0 are each 1 less than the roots of the
(a) The area of ∆ABC is always a rational number
equation x2 + ax + b = 0. Then, a + b is (b) The area of ∆ABC is always an irrational number
(a) possibly any real number (b) −2 (c) The perimeter of ∆ABC is an even integer
(c) −4 (d) −5 (d) The information provided is not sufficient to conclude
2. If x, y are real numbers such that any of the statements A , B or C above

3( x / y ) + 1 − 3( x / y ) − 1 = 24, 6. Consider a square ABCD of side 12 and let M , N be


then the value of (x + y)/(x − y) is the midpoints of AB, CD respectively. Take a point P
on MN and let AP = r , PC = s. Then, the area of the
(a) 0 (b) 1 (c) 2 (d) 3
triangle whose sides are r , s,12 is
3. The number of positive integers n in the set {1, 2, 3, rs rs
(a) 72 (b) 36 (c) (d)
………, 100} for which the number 2 4
12 + 22 + 32 + ... + n 2 7. A cow is tied to a corner (vertex) of a regular
is an integer is
1 + 2 + 3 + .... + n hexagonal fenced area of side a m by a rope of length
5a
(a) 33 (b) 34 (c) 50 (d) 100 m in a grass field. (The cow cannot graze inside
2
4. The three different face diagonals of a cuboid the fenced area). What is the maximum possible area
(rectangular parallelopiped) have lengths 39, 40, 41. of the grass field to which the cow has access to
The length of the main diagonal of the cuboid which graze?
joins a pair of opposite corners is 5 2
(a) 5 πa 2 (b) πa (c) 6 πa 2 (d) 3 πa 2
(a) 49 (b) 49 2 (c) 60 (d) 60 2 2
https://iit-jeeacademy.blogspot.com

KVPY Question Paper 2010 Stream : SA 139

8. A closed conical vessel is filled with water fully and is 13. A soldier with a machine gun, falling from an
placed with its vertex down. The water is let out at a alrplane gets detached from his parachute. He is able
constant speed. After 21 min, it was found that the to resist the downward acceleration, if he shoots 40
height of the water column is half of the original bullets a second at the speed of 500 m/s. If the weight
height. How much more time in minutes does it of a bullet is 49 g, what is the weight of the man
require to empty the vessel? with the gun ? Ignore resistance due to air and
(a) 21 (b) 14 (c) 7 (d) 3 assume the acceleration due to gravity, g = 9.8 ms−2.
9. I carried 1000 kg of watermelon in summer by train. (a) 50 kg (b) 75 kg (c) 100 kg (d) 125 kg
In the beginning the water content was 99%. By the 14. A planet of mass is moving around a star of mass M
time I reached the destination, the water content had and radius R in a circular orbit of radius r. The star
dropped to 98%. The reduction in the weight of the abruptly shrinks to half its radius without any loss of
watermelon was mass. What change will be there in the orbit of the
(a) 10 kg (b) 50 kg (c) 100 kg (d) 500 kg planet?
10. A rectangle is divided into 16 sub-rectangles as in the (a) The planet will escape from the star
figure, the number in each sub-rectangle represents (b) The radius of the orbit will increase
the area of that sub-rectangle. What is the area of (c) The radius of the orbit will decrease
the rectangle KLMN ? (d) The radius of the orbit will not change

10 4 15. Figure (i) below shows a Wheatstone’s bridge in


which P , Q, R and S are fixed resistances, G is a
galvanometer and B is a battery. For this particular
15
case, the galvanometer shows zero deflection. Now,
N M 12
only the positions of B and G are interchanged, as
25 shown in figure (ii). The new deflection of the
galvanometer
K L
(a) 20 (b) 30 (c) 40 (d) 50 Q
P Q P

PHYSICS B
G
11. A hollow pendulum bob filled with water has a small
hole at the bottom through which water escapes at a S R S R
constant rate. Which of the following statements
describes the variation of the time period T of the G
pendulum as the water flows out? B
(a) T decreases first and then increases Figure (i) Figure (ii)
(b) T increases first and then decreases
(c) T increases throughout (a) is to the left
(d) T does not change (b) is to the right
(c) is zero
12. A block of mass M rests on a rough horizontal table. (d) depends on the values of P ,Q , R and S
A steadily increasing horizontal force is applied such
that the block starts to slide on the table without 16. 12 positive charges of magnitude q are placed on a
toppling. The force is continued even after sliding has circle of radius R in a manner that they are equally
started. Assume the coefficients of static and kinetic spaced. A charge Q is placed at the centre, if one of
friction between the table and the block to be equal. the charges q is removed, then the force on Q is
The correct representation of the variation of the (a) zero
frictional force f, exerted by the table on the block qQ
(b) away from the position of the removed charge
with time t is given by 4 πε0 R 2
f f 11qQ
(c) away from the position of the removed charge
(b) 4 πε0 R 2
(a)
qQ
(d) towards the position of the removed charge
4 πε0 R 2
(0, 0) t (0, 0) t
f
17. An electric heater consists of a nichrome coil and runs
f under 220 V, consuming 1 kW power. Part of its coil
(c) (d) burned out and it was reconnected after cutting off
the burnt portion.The power it will cunsume now is
(a) more than 1 kW (b) less than 1 kW, but not zero
(0, 0) t (0, 0) t
(c) 1 kW (d) 0 kW
https://iit-jeeacademy.blogspot.com

140 KVPY Question Paper 2010 Stream : SA

18. White light is split into a spectrum by a prism and it 25. Among NH3 , BCl3 , Cl2 and N2, the compound that
is seen on a screen. If we put another identical does not satisfy the octet rule is
inverted prism behind it in contact, what will be seen (a) NH3 (b) BCl3 (c) Cl 2 (d) N2
on the screen ? 26. The gas produced on heating MnO2 with conc. HCl is
(a) Violet will appear where red was
(a) Cl 2 (b) H2 (c) O2 (d) O3
(b) The spectrum will remains same
(c) There will be no spectrum, but only the original light 27. The number of covalent bonds in C 4 H7 Br , is
with no deviation (a) 12 (b) 10 (c) 13 (d) 11
(d) There will be no spectrum, but the original light will
be laterally displaced 28. An aqueous solution of HCl has a pH of 2.0. When
water is added to increase the pH to 5.0, the
19. Two identical blocks of metal are at 20°C and 80°C, hydrogen ion concentration
respectively. The specific heat of the material of the (a) remains the same (b) decreases three-fold
two blocks increases with temperature. Which of the
(c) increases three-fold (d) decreases thousand-fold
following is true about the final temperature T f when
the two blocks are brought into contact (assuming 29. Consider two sealed jars of equal volume. One
that no heat is lost to the surroundings)? contains 2 g of hydrogen at 200 K and the other
(a) Tf will be 50°C contains 28 g of nitrogen at 400 K. The gases in the
(b) Tf will be more than 50°C two jars will have
(c) Tf will be less than 50°C (a) the same pressure
(d) Tf can be either more than or less than 50° C (b) the same average kinetic energy
depending on the precise variation of the specific heat (c) the same number of molecules
with temperature (d) the same average molecular speed
20. A new temperature scale uses X as a unit of 30. Identify the stereoisomeric pair from the following
temperature, where the numerical value of the choices.
temperature tx in this scale is related to the absolute (a) CH 3 CH 2 CH 2OH and CH 3 CH 2OCH 3
temperature T by tx = 3T + 100. If the specific heat of (b) CH 3 CH 2 CH 2 Cl and CH 3 CHClCH 3
a material using this unit is 1400 J kg −1K −1, its H
|
specific heat in the SI system of units is (c) CH3  C == C  CH3 and CH3  C  C  CH3
(a) 4200 J kg −1 K−1 | | |
(b) 1400 J kg −1 K−1 H H H
(c) 466.7 J kg −1 K−1 CH2
(d) impossible to determine from the information provided

and
CHEMISTRY (d)

21. The boiling point of 0.01 M aqueous solutions of


sucrose, NaCl and CaCl 2 would be BIOLOGY
(a) the same (b) highest for sucrose solution
31. Which of the following is a water borne disease?
(c) highest for NaCl solution (d) highest for CaCl 2 solution
(a) Tuberculosis (b) Malaria
22. The correct electronic configuration for the ground (c) Chicken pox (d) Cholera
state of silicon (atomic number = 14) is 32. In his seminal work on genetics, Gregor Mendel
(a) 1s2 2s2 2 p 6 3s2 3 p 2 (b) 1s2 2s2 2 p 6 3 p 4 described the physical traits in the pea plant as being
(c) 1s2 2s2 2 p 4 3s2 3 p 4 (d) 1s2 2s2 2 p 6 3s1 3 p5 controlled by two ‘factors’. What term is used to
23. The molar mass of CaCO3 is 100 g. The maximum define these factors today?
amount of carbon dioxide that can be liberated on (a) Chromosomes (b) Genes
heating 25 g of CaCO3 is (c) Alleles (d) Hybrids
(a) 11 g (b) 5.5 g 33. A majority of the tree species of peninsular Indian
(c) 22 g (d) 2.2 g origin fruit in the months of
(a) April-May (b) August-September
24. The atomic radii of the elements across the second
(c) December-January (d) All months of the year
period of the periodic table
(a) decrease due to increase in atomic number 34. In frogs, body proportions do not change with their
(b) decrease due to increase in effective nuclear charge growth. A frog that is twice as long as another will be
(c) decrease due to increase in atomic weights heavier by approximately
(d) increase due to increase in the effective nuclear charge (a) two-fold (b) four-fold (c) six-fold (d) eight-fold
https://iit-jeeacademy.blogspot.com

KVPY Question Paper 2010 Stream : SA 141

35. Which of the following has the widest angle of (b) bacterial metabolism inside the dung releases heat
binocular vision? (c) undigested material releases heat due to oxidation by
(a) Rat (b) Duck (c) Eagle (d) Owl air
(d) dung is dark and absorbs a lot of heat
36. The two alleles of a locus which an offspring receives
from the male and female gametes are situated on 39. Which one of these is the correct path for a reflex
(a) two different homologs of the same chromosome action?
(b) two different chromosomes (a) Receptor → Motor neuron→ Spinal cord → Sensory
(c) sex chromosomes neuron→ Effector
(d) a single chromosome (b) Effector→ Sensory neuron→ Spinal cord → Motor
neuron→ Receptor
37. Ants locate sucrose by (c) Receptor→ Sensory neuron→ Spinal cord→ Motor
(a) using a strong sense of smell neuron→ Effector
(b) using a keen sense of vision (d) Sensory neuron → Receptor→ Motor neuron→ Spinal
(c) physical contact with sucrose cord→ Effector
(d) sensing the particular wavelength of light 40. Insectivorous plants digest insects to get an essential
emitted/reflected by sucrose nutrient. Other plants generally get this nutrient
38. The interior of a cow dung pile kept for a few days is from the soil. What is this nutrient?
quite warm. This is mostly because (a) Oxygen (b) Nitrogen
(a) cellulose present in the dung is a good insulator (c) Carbon dioxide (d) Phosphates

PART-II (5 Marks Questions)


MATHEMATICS
A
1. In a ∆ ABC , D and E are points on AB, AC R RA
V
respectively such that DE is parallel to BC. Suppose RV
BE , CD intersect of O. If the areas of the triangles + –
ADE and ODE are 3 and 1 respectively. Find the
P
area of the ∆ ABC, with justification.
A
2. Leela and Madan pooled their music CD’s and sold R RA
them. They got as many rupees for each CD as the V
total number of CD’s they sold. They share the RV
money as follows: Leela first takes 10 rupees, then + –
Madan takes 10 rupees and they continue taking 10 Q
rupees alternately till Madan is left out with less In each case, the resistance is estimated by using
than 10 rupees to take. Find the amount that is left V
out for Madan at the end, with justification. Ohm’s law Rest = , where V and I are the readings
I
3. (a) Show that for every natural number n of the voltmeter and the ammeter, respectively. The
relatively prime to 10, there is another natural meter resistances RV and RA are such that
number m all of whose digits are 1’s such that RA << R << RV .The internal resistance of the battery
n divides m. may be ignored. The absolute error in the estimate of
the resistance is denoted by δR =|R − Rest|.
(b) Hence or otherwise show that every positive (a) Express δRP in terms of the given resistance values.
rational number can be expressed in the form (b) Express δRQ in terms of the given resistance values.
a (c) For what value of R will δRP ≈ δRQ ?
for some natural numbers a , b, c.
10 (10c − 1)
b
5. A point source is placed 20 cm to the left of a concave
lens of focal length 10 cm.
(a) Where is the image formed?
PHYSICS (b) Where to the right of the lens would you place a
4. Consider the two circuits P and Q shown below, concave mirror of focal length 5 cm, so that the final
image is coincident with the source?
which are used to measure the unknown resistance
(c) Where would the final image be formed, if the concave
R. mirror is replaced by a plane mirror at the same
position?
https://iit-jeeacademy.blogspot.com

142 KVPY Question Paper 2010 Stream : SA

6. A block of mass m is sliding on a fixed frictionless CO2H


concave surface of radius R. It is released from rest at CH3
point P which is at a height of H << R from the lowest NH2
point Q. III IV
The following observations were made.
(i) The compound in the bottle A did not dissolve
θ in either 1 N NaOH or 1 N HCl.
(ii) The compound in the bottle B dissolved in
R 1 N NaOH but not in 1 N HCl.
(iii) The compound in the bottle C dissolved in both
P 1 N NaOH and 1 N HCl.
H m
(iv) The compound in the bottle D did not dissolve
Q
in 1 N NaOH but dissolved in 1 N HCl.
(a) What is the potential energy as a function of θ, taking
the lowest point Q as the reference level for potential (Fill up the blanks)
energy? (a) Indicate the compounds in : bottle A = ......, bottle B =
(b) What is the kinetic energy as a function of θ? ......, bottle C = ...... and bottle D = ...... .
(c) What is the time takes for the particle to reach from (b) The compound with the highest solubility in distilled
point P to the lowest point Q? water is ...... .
(d) How much force is exerted by the block on the concave (Indicate the answers by the compound
surface at the point Q ? numbers)
9. Assume that a human body requires 2500 kcal of
energy each day for metabolic activity and sucrose is
CHEMISTRY the only source of energy, as per the equation
7. Copper in an alloy is estimated by dissolving in conc. C12H22O11 (s) + 12 O2 ( g) → 12 CO2 ( g) + 11H2O(l);
nitric acid. In this process, copper is converted to
cupric nitrate with the evolution of nitric oxide (NO). ∆H = − 56
. × 106 J.
The mixture when treated with potassium iodide (Fill up the blanks)
forms cupric iodide, which is unstable and (a) The energy requirement of the human body per day is
decomposes to cuprous iodide and iodine. ……… kJ.
The amount of copper in the alloy is estimated by (b) The mass of sucrose required to provide this energy
titrating the liberated iodine with sodium is ……… g and the volume of CO2 (at STP) produced
thiosulphate. The reactions are is ……… litres.
a Cu + b HNO3 → c Cu(NO3 )2 + d NO + e H2O
f CuI2 → g Cu 2 I2 + h I2 BIOLOGY
i Na 2 S2 O3 + j I2 → k Na 2 S4 O6 + l NaI
10. Mohini, a resident of Chandigarh went to Shimla
(Fill up the blanks) with her parents. There she found the same plant
(a) The coefficients are : a = ......, b = ......, c = ......, d = ...... that they have in their backyard, at home. However,
and e = ...... . she observed that while the plants in their backyard
(b) The coefficients are : f = ......, g = ...... and h = ...... . bore white flowers, those in Shimla had pink flowers.
(c) The coefficients are : i = ......, j = ......, k = ...... and l = She brought home some seeds of the plant from
...... . Shimla and planted them in Chandigarh. Upon
(d) If 2.54 g of I2 is evolved from a 2.0 g sample of the performing self-breeding for several generations she
alloy, what is the percentage of copper in the alloy?
(atomic weights of iodine and copper are 127 and 63.5,
found that the plant from Shimla produced only
respectively). white flowers.
8. You have been given four bottles marked A, B, C and D (a) According to you what might be the reason for
each containing one of the organic compounds given this observation, genetic or environmental factors?
below (b) Suggest a simple experiment to determine whether
this variation is genetic in nature.
NH2 CO2H (c) Suggest another experiment to check whether this
variation in flower colour is due to environmental
factors.
I II
https://iit-jeeacademy.blogspot.com

KVPY Question Paper 2010 Stream : SA 143

11. The breakdown of glucose in a cell occurs in any one After a period of growth, the following observations
of the following pathways : were made
CO2 + H2O (in the presence of O2, A. Lime water turns milky; the dye colour
e.g. in mitochondria) remains the same.
Glucose Pyruvic Ethanol + CO2 (in the absence of O2, B. The dye colour changes; lime water does not
acid e.g. yeast)
turn milky.
Lactic acid (in the absence of O2,
e.g. lactic acid bacteria) C. Lime water turns milky; the dye colour
remains the same.
Three experiments (A, B, C) have been set up. In
each experiment, a flask contains the organism (a) Question Identify which of the reactions in the
in growth medium, glucose and a brown dye pathways depicted above is taking place in each
that changes its colour to yellow when the pH experiment. Give reasons for your answer.
decreases. (b) Question Identify which of the reactions in
the pathways depicted above is expected to
occur in Red Blood Cells (RBCs).
12. A scientist has a house just beside a busy highway.
He collects leaves from some plants growing in his
garden to do radio-carbon dating (to estimate the age
Lime of the plant by estimating the amount of a
water
radioisotope of carbon in its tissues). Surprisingly the
Organism in culture radio-carbon dating shows that the plant is a few
medium
+ Glucose + Dye thousand years old.
The mouth of the flask is attached to a test tube (a) Was the result of the radio-carbon dating wrong or can
containing lime water (calcium hydroxide as shown you propose a reason for such an observation?
in the figure). In C, but not in A and B, air is (b) What simple experiment can be done to test the reason
removed from the flask before beginning the that you have proposed?
experiment.

Answers
PART-I
1 (c) 2 (d) 3 (b) 4 (a) 5 (b) 6 (b) 7 (a) 8 (d) 9 (d) 10 (d)
11 (b) 12 (a) 13 (c) 14 (d) 15 (c) 16 (d) 17 (a) 18 (d) 19 (b) 20 (c)
21 (d) 22 (a) 23 (a) 24 (b) 25 (b) 26 (a) 27 (a) 28 (d) 29 (c) 30 (c)
31 (d) 32 (c) 33 (a) 34 (d) 35 (d) 36 (a) 37 (c) 38 (b) 39 (c) 40 (b)
https://iit-jeeacademy.blogspot.com

144 KVPY Question Paper 2010 Stream : SA

Solutions
PART 1 6. (b) Given ABCD is square
41
1. (c) We have, D N C
x2 + bx + a = 0 …(i)
l
x2 + ax + b = 0 …(ii)
h 40 s
Let α, β are the roots of equation h 12
12
x2 + bx + a = 0 C P
39 O
r 6
∴ α + β = − b; αβ = a b r
Now, given roots of the equation A l B A 6 M B
x2 + ax + b = 0 is greater than 1 of the Given, l2 + h 2 = 392
roots of Eq. (i). AB = BC = 12 units
⇒ b2 + h 2 = 402 M is mid-point of AB,
∴α + 1 + β + 1 = − a and (α + 1)(β + 1) = b
⇒ l2 + b2 = 412 N is mid-point of CD,
α + β + 2 = − a and αβ + α + β + 1 = b
⇒ 2(l + b + h 2 ) = 392 + 402 + 412
2 2
P is point of MN and CP = S
⇒ − b + 2 = − a and a − b + 1 = b
⇒ l2 + b2 + h 2 = 2401 Q APB is an isosceles triangle,
⇒ b − a = 2 and 2b − a = 1
∴Length of longest diagonal ∴ AP = PB = r
On solving these equations, we get
a = − 3, b = − 1, = l2 + b2 + h 2 OP = MB = 6 units
∴ a + b = − 3 − 1=− 4 = 2401 = 49 P
2. (d) We have, 5. (b) We have,
sides of ∆ABC are positive integer and s
3( x/ y ) + 1 − 3( x/ y ) − 1 = 24 r
3x/ y 8 length of smallest side is 1.
6
⇒ 3 ⋅ 3x/ y − = 24 ⇒ .3x/ y = 24
3 3 A
B 12 C
⇒ 3x/ y = 9 ⇒ 3x/ y = 32 Area of triangle whose sides r , s and 12
x are the area of ∆PBC
⇒ =2
y C B 1 1
= × Base × Height = × BC × OP
Using componendo and dividendo, we get 2 2
x + y 2 +1 x + y 1
= × 12 × 6 = 36 sq units
= ⇒ =3
x−y 2−1 x−y B a=1 C 2
3. (b) We have, We know that the sum of two sides of 7. (a) Total area of the grass field graze
triangle is greater than third side. by the cow = 2 (area of sector APQ + area
1 + 2 + 3 + 4 + ... + n
2 2 2 2 2

∴ b + 1> c of sector BQR + area of sector CRS)


1 + 2 + 3 + 4 + ... + n
n (n + 1) (2n + 1) ⇒ c − b< 1 S
R
a a
6 1+ c> b 60° a/2
=
n (n + 1) ⇒ b −c< 1 C
120°

2 −1 < b − c < 1 a Q
2n + 1 a
= = k (let) b, c are integers. 60°
3 60° 60° 3a/2
∴ b −c = 0 ⇒ b = c
3k − 1 a+b+c
a a
∴ n= Semi-perimeter = A
2 120°
2
3k − 1 2b + 1
Now, 1 ≤ ≤ 100 = = b+
1
2 2 2 5a/2
⇒ 2 ≤ 3k − 1 ≤ 200 Area A = s(s − a )(s − b)(s − c)
⇒ 2 + 1 ≤ 3k ≤ 200 + 1
=  b + 1   b + 1 − 1  b + 1 − b  b + 1 − b
⇒ 3 ≤ 3k ≤ 201     
 2  2  2  2  P
201
⇒ 1≤ k ≤  120 3a 
2 2
b + 1  1  1  1  = 1 b2 − 1
× π   + × π ×   
3 =  5a 60
  b −     
⇒ 1 ≤ k ≤ 67  2  2  2  2 2 4  360  2 360  2 
= 2
2
Number of odd integer = 34  × π   
Since, b is integer. 60 a
 +
4. (a) Let the length, breadth and height ∴ Area of ∆ABC is always irrational  360  2 
of cuboid is l, b and h respectively. number.
https://iit-jeeacademy.blogspot.com

KVPY Question Paper 2010 Stream : SA 145

2 π  25a 2 9a 2 a 2  ∴
ac bd
× × ef =
10 12
× × 25 15. (c) B
=  + + 
3  4 8 8 bc de 4 15
P Q
⇒ af = 50
2 π  60a 2 
=   = 5 πa
2
Area of rectangle KLMN = af = 50 A C
3  8  G
11. (b) Time period of a pendulum
8. (d) Let r and h be radius and height of depends on its length as
S R
cone, respectively. T ∝ l
r where, l = length of pendulum. + D –
As water flows out of the bob of B
pendulum, centre of mass of bob go down In case (i), galvanometer shows zero
as the level of water falls. But when bob deflection.
r/2 is completely emptied, centre of mass of
h ∴ VB = VD
bob again reaches to its centre. Hence,
P Q
effective length of pendulum first ⇒ = …(i)
h/2 increases then decreases. S R
So, time period of pendulum first When battery B and galvanometer G are
increases then decreases. interchanged, position of galvanometer is
1 2 as shown below,
∴ Volume of cone = πr h 12. (a) Force of friction is self adjusting.
3 B
When applied force is increased friction,
Given, rate of outflow of water in 21 min, first increases till it reaches a maximum
h P Q Galvanometer is
then h changes to value called limiting friction. – connected across
2
If applied force is further increased, then + A and C
∴ Rate of outflow of water friction does not increases further and A C
1 πr 2 h  body begins to move.
=  πr 2h − ×  = 21 min
3 4 2 Force of friction then remains constant S R

7 πr 2h 1 πr 2h and its value is given by D


⇒ = 21 min ⇒ = 3 min
8× 3 3 8 f = µN G

∴Time required to empty vessels in 3 min. 13. (c) In given case, rate of momentum Now, ratio of resistances across
9. (d) We have, change of bullets is equal to weight of galvanometer is
soldier. S R
1000 kg of watermelon in which 99% are and
water. If M = mass of soldier and his gun and P Q
m = mass of bullet. Then, S R
∴990 kg water and 10 kg rest. As from Eq. (i), =
N
Now, x kg watermelon has Mg = × m (v − 0) P Q
∆t
98 2x Hence, galvanometer still shows zero
= x water + rest
(N / ∆t ) × mv deflection because Wheatstone’s bridge is
100 100 ⇒ M=
Weight of solid part should remain same g balanced.
2x N 16. (d) Force on charge Q is initially zero
∴ = 10 ⇒ x = 500 where, is the number of bullets fired
100 ∆t as forces of 12 charges balances each
∴Weight reduction = (1000 − 500)kg per second. other.
= 500 kg 40 × 49 × 10−3 × 500 q3 q2
⇒ M= q1
10. (d) Given, 9.8
q4
= 100 kg
10 c 4 q12
a
14. (d) When star shrinks without losing q5
b
its mass, its gravitational acceleration on
its surface increases but there is no q11
N d M 12 d 15 q6
a e change in force exerted by this star on a
b
f f 25 distant object like a planet. Force of q10
gravitational attraction of star on planet q7
K L q9
is q8
Area of rectangles are
GMm
ac = 10 F= As shown in above figure, forces of
r2 diametrically opposite charges balances
bc = 4
where, M = mass of star, m = mass of each other, hence net force on Q is zero.
bd = 12 planet and r = orbital radius of planet. When one of the charge q (let q1 ) is
de =15 As, this force remains same there is no removed, net force on Q is now the
ef = 25 change in orbital radius of planet. unbalanced force of diametrically
opposite charge.
https://iit-jeeacademy.blogspot.com

146 KVPY Question Paper 2010 Stream : SA

kqQ qQ 20. (c) Given, tx = 3T + 300 respective octets (i.e., 8 electrons in their
i.e. Force, F = =
R2 4πε0 R 2 outermost shell or 2 electrons in case of
So, change in temperatures in two
H, Li and Be to attain stable nearest
and this force vector points towards the systems are related as
noble gas configuration).
position of the removed charge. tx 2 − tx1 = (3T2 + 300) − (3T1 + 300)
The Lewis structures of given molecules
V2 = 3 (T2 − T1 )
17. (a) Power P = as V remains are as follows
R or ∆tx = 3∆T
constant in given condition, while current (a) NH3 H× N × H
Now to convert units, we use
changes as resistance is reduced. H
1 N1 u1 = N 2u2
Power consumed by heater coil P ∝ . 1400 J N2 J As the octet is complete, hence it follows
R =
∆tx kg ∆T kg octet rule.
On cutting part of coil, its resistance
1400 N 2 ××
decreases. So, power consumed by coil ⇒ = × × Cl× ×
increases and it will be more than 1 kW. 3 ∆T ∆T ××
×
××
(b) BCl3, ×× Cl× B × Cl ××
18. (d) First prism separates white light 1400 ∆T
⇒ N2 = ×× ××
into its constituent colours which are 3∆T
Octet rule is not followed in this case as
then recombined by the second inverted 1400
⇒ N2 = = 466.7 there are total 6 electrons in the
prism into white light. 3 outermost shell of B even after bonding.
Hence, in SI system, value of specific heat
al ××
ter
La iation is c = 466.7 J kg −1 K −1 . (c) Cl2 , Cl × Cl× ×
v ××
de
t 21. (d) As boiling point is a colligative Octet rule is followed in case of Cl 2 .
ligh
h ite property, it depends only on the number ×
W (d) N2 , N ×
N
R of particles present in a solution. ×

t Aqueous solution having more number of


e ligh V
As both the N have complete octet. Thus,
it
Wh particles will have highest elevation in
it follows octet rule.
boiling point.
Hence, the correct option is (b).
C12H22O11 (aq) r No ions
26. (a) On heating MnO2 with conc. HCl,
As two prisms combinedly form a glass NaCl r Na + + Cl − (Total ions = 2)
MnCl 2 and H 2O are released with the
slab, light beam is deviated on a parallel CaCl 2 r Ca 2+ + 2Cl − (Total ions = 3)
path. evolution of Cl 2 gas. The chemical
Thus, CaCl 2 will have highest boiling equation for it can be written as
19. (b) When blocks are brought into point.
contact, hotter one lost heat and colder MnO2 + 4HCl → MnCl 2 + 2H2O + Cl 2
22. (a) As silicon belongs to group 14
one gains that heat.
whose general configuration is ns2np 2.
27. (a) The structure of C4 H7 Br can be
Let T = final temperature of the blocks. Thus, the correct electronic configuration written as
Then, of Si is 1s2 2s2 2 p 6 3s2 3 p 2. H H H H
Heat lost by hotter block = Heat gained ∆ 2 7 2 10
23. (a) CaCO3 → CaO + CO2 H
1
C
3
C
5
C
9
C
12
Br
by colder block 4 6
25 1 8 11
⇒ ms1 (Ti − Tf )Hot block = ms2 (Tf − Ti )Cold block No. of moles of CaCO3 = = mole H H
100 4
Thus, total number of covalent bonds in
s1 (Tf − Ti )Cold block 1 mole of CaCO3 gives 1 mole of CO2.
or = C 4 H 7Br is 12.
s2 (Ti − Tf )Hot block 1 1
∴ mole of CaCO3 gives mole of CO2. 28. (d) As we know
Here, we are using two different specific 4 4
pH = − log [H + ]
heats s1 and s2 as it is given that specific 1 mole of CO2 = 44 g
heat of material increases with 1 44 As the initial pH of aqueous solution = 2
So, mole of CO2 = = 11 g
temperature. So, s1 > s2. 4 4 ∴ Concentration of H+ , [H + ] = 10−2
s1 Thus, 11 g of CO2 is released on heating As the final pH of aqueous solution = 5
⇒ >1
s2 25 g of CaCO3 . ∴ [H+ ] = 10−5
(Tf − Ti )Cold block 24. (b) On moving across the second
⇒ >1 [H+ ]f 10−5
(Ti − Tf )Hot block period of the periodic table, the atomic ∴ +
=
radii of the elements decrease, due to [H ]i 10−2
T − 20
⇒ >1 increase in effective nuclear charge, the = 10−3
80 − T electrons of all the shells are pulled little +
Thus, H concentration decreases by
⇒ T − 20 > 80 − T closer to nucleus thereby making each
thousand fold.
individual shell smaller and smaller.
⇒ T + T > 80 + 20
25. (b) According to octet, the atoms of 29. (c) For Ist jar
⇒ 2T > 100 2
different elements combine with each No. of moles of H 2 = =1
⇒ T > 50°C other in order to complete their 2
https://iit-jeeacademy.blogspot.com

KVPY Question Paper 2010 Stream : SA 147

∴No. of molecules of H 2 in 1 mole 34. (d) A frog that is twice as long as The sensory nerve fibres bring sensory
= 6.022 × 1023 another will be heavier by approximately impulses from the receptor organ to the
For 2nd jar eight folds. It is because their weight is central nervous system (brain and spinal
28 determined by their folds, so by applying cord). The motor nerve fibres relay the
No. of moles of N 2 = =1 unitary method it is eight folds. Frog motor impulses from the central nervous
28
shape is triangular. system to the effector organs. Reflex
∴No. of molecules of N2 in 1 mole action is a form of animal behaviour in
= 6.022 × 1023 which the stimulation of a sensory organ
Thus, the gases in two jars will have 13 13 23 23 (receptor) results in the activity of some
same number of molecules. organ without the intervention of will.
30. (c) The type of isomers in the given 40. (b) Insectivorous plants mostly
13 23 thrive in marshes and rocky outcrops or
options are as follows
=1 =8
(a) CH3 CH2CH2OH and CH3 CH2OCH3 other nitrogen poor areas where sunlight
Initial size Final size
and water are abundant. These plants
They are functional isomers. 35. (d) Owl’s eye have the widest angle have evolved the ability to trap and
(b) CH3 CH2CH2Cl and CHCHClCH3 of binocular vision among rat, duck and digest insects, which are an excellent
They are positional isomers. eagle. Binocular vision is the vision using source of nitrogen as they contain around
two eyes with overlapping fields of view, 10% nitrogen by mass. Insectivorous
(c) H3C C C CH3
allowing good perception of depth. Unlike plants are able to obtain between 10-80%
H H many birds with eyes that sit at an angle, of their total nitrogen from insects.
Cis owl’s eye face directly forward giving
them incredible binocular vision. PART 2
H
Although their large eyes cannot move or
1. Given in ∆ABC, D and E are points on
and CH3C C CH3 roll-like human eyes can, owls can move
AB , AC respectively such that DE is
their heads nearly all the way around,
parallel to BC.
H allowing them to have a 270 degree range
Trans of vision without moving their bodies. BE and CD intersect at O.
As they are cis and trans form, so they Area of ∆ADE = 3 sq units
36. (a) An allele is a varient form of
are geometrical isomers which are a type gene. Some genes have a variety of Area of ∆DOE = 1sq units
of stereoisomers. different forms, which are located at the Area of ∆BEC = Area of ∆BDC
CH2 same position on a chromosome. Humans A
are called diploid organisms because they
have two alleles at each genetic locus,
(d) and
with one allele inherited from each 3
parent. Each pair of alleles represents E
These are structural isomers. D
the genotype of a specific gene. 1
Thus, the correct option is (c).
37. (c) Ants locate sucrose by physical x O x
31. (d) Water borne diseases such as contact with sucrose. Ants can smell food
cholera is caused by drinking using their antennae, which can detect y
contaminated or dirty water. Cholera is minute odours. But sugars actually do B C
caused by the bacterium Vibrio cholerae. not have a smell. Therefore, when the [same base between same parallels]
32. (c) The term allele was formerly scout worker ants locate sugar while

used by Mendel for the factors foraging, they take a piece back home to
(BOD ) + ar (DOE ) = ar (COE ) + ar (DOE )
representing the two alternate forms of a the colony, while doing this, the ant will
character, e.g. tallness and dwarfness in leave a chemical trial which the other ⇒ ar (BOD ) = ar (COE ) = x
case of height in pea (T and t). ants can follow. ⇒ ar (BOC ) = y
33. (a) This happens because the 38. (b) The interior of a cow dung pile We know that area of two triangle having
temperature starts to increase all over kept for a few days is quite warm. This is equal altitude is the same as the ratio of
the country in March and by April. The because bacterial metabolism inside the their respective bases.
x BO y
interior parts of the peninsular record dung releases heat in the form of biogas ∴ = =
mean daily temperature of 30-35°C. (CH4 + CO2 ). This happens by anaerobic 1 OE x
Maximum temperature rises sharply digestion of cow dung by methanogenic ⇒ y = x2
exceeding 45°C by the end of May. The bacteria. Now, ∆ADE and ∆ABC are similar.
warm temperature can affect the
39. (c) The correct path for a reflex ar (ADE ) DE 2 ar (ODE )
ripening process of fruits and vegetables ∴ = =
by speeding up the production of ethylene
action is ar (ABC ) BC 2 ar (OBC )
Receptor → Sensory neuron → Spinal 3 1
gas which rushes the ripening or fruiting ⇒ =
process. cord → Motor neuron → Effector 4 + 2x + y y
https://iit-jeeacademy.blogspot.com

148 KVPY Question Paper 2010 Stream : SA

⇒ 3 y = 4 + 2x + y Consider 9m, which has only 9 as its (b) In case Q,


⇒ 2 y = 2(2 + x) digits and b divisible by t.
A
9m R
⇒ y=x+2 Let k=
t
⇒ x2 − x − 2 = 0 [Q y = x2] I
∴ 9k = 9m ⋅ 2r ⋅ 5s = (10c − 1)2r ⋅ 5s V
⇒ (x − 2)(x + 1) = 0
where, c is the number of digits in m. E
⇒ x = 2, x ≠ −1
We can find d such that + –
∴Area of ∆ABC = 4 + 2x + y
qd = 10b (10c − 1) V = I (R + RA )
= 4 + 4 + 4 = 12 sq units
R =   − RA = Rest − RA
[such that 10b is a suitable power of 2 V
2. Let the total number of CD’s sold by ⇒
if s > r and a suitable power of s if r > s] I
the Leela and Madan together = x
p pd a
Total money obtained by them ∴ = = So, error in case Q is
q qd 10b (10c − 1) ⇒ δRQ = |Rest − R| = RA
= (x × x ) = x 2
2 where, pd = a. (c) If R = RA RV then,
They divided x in such that,
x2 = 10 (an odd number) + a number less
4. (a) In case P, δRP 2
Rest R2
= = est ≈1
than 10 IR R δRQ RA RV R2
⇒ x = 10q + r [Q0 ≤ r < 10] RA
A 5. Given situation is
⇒ x2 = (10q + r )2
⇒ x2 = 100q2 + 20qr + r 2 V
IV RV
r = 10 (an odd number ) + a number less
2 I
than 10 E 20 cm
+ –
0 ≤ r < 10
∴Taking r = 0, 1, 2, 3, ..., 9, we get r = 4 or 6 Current through cell is f=–10 cm
r = 16 or 36 I = IR + IV = V / R + V / RV (a) We use lens formula to get position of
r 2 = 10 + 6 or 3(10) + 6 where, V = potential drop across image u = − 20 cm, f = − 10 cm.
Hence, the amount left for Madan at the resistance R. 1 1 1
As, − =
end is 6 rupees. 1 1   R + RV  v u f
⇒ I =V +  =V 
3. (a) We have,  R RV   RRV  1 1 1 −1 1
⇒ = + = −
A natural number ‘m’ whose all digits are 1 V v f u 10 20
⇒ RRV = (R + RV )
∴ m = 1, 111, 1111, 11111, ..., (1111 ... 1) I 1 − 2 − 1 −3
⇒ = =
A natural number ‘n’ which is relatively  V
⇒  RV −  R =
VRV v 20 20
prime to 10 when n + 1number  I I −20
⇒ v= cm
1, 111, 1111, 11111, …, (1111…1) divides VRV 3
⇒ R=
by n, then n + 1remainder obtained. I (RV − V /I ) Image is virtual and it is in front of lens
∴ Possibilities of remainder are 0, 1, 2, 3,   20
at cm.
, n − 1which are n in numbers, where V  RV  3
two remainder are same. ⇒ R= . 
I R − V  (b) Concave mirror again converge rays
Let two numbers x =11 1 1  V
I 
... and and its image formed is coincident with
V
y = 111 ... 1 having say i digits and j digits But = Rest = estimated resistance object.
respectively which leave the same I
remainder after division by n.  RV 
∴ R = Rest  
We take, i< j  RV − Rest 
y − x is divisible by n;  −1 
R 
= Rest .   1 − est  
O
But y − x = 11... 1000 ... 0 where j − i  RV  
number of 1’s and remaining zero. Since,  I
n is coprime to 10. We see that n divides  R 
= Rest  1 + est 
m = 111 ... 1 a number having 1’s as its  RV 
digits. x
Using binomial approximation and
(b) Let the positive rational number is
neglecting higher order terms. Let mirror is placed at distance x behind
p
, where q ≠ 0 Now, error in case P is the lens. Now, first image I1 of lens acts
q like object for mirror.
δRP = |Rest − R|
and q = 2r ⋅ 5s ⋅ t where t is coprime to 10 Hence, for mirror,
2
Choose any number m having only is as Rest
= u = −  + x cm.
20
its digit and is divisible by t. RV  3 
https://iit-jeeacademy.blogspot.com

KVPY Question Paper 2010 Stream : SA 149

Given, f = − 5 cm As there is lens in between, so final 2mgH


⇒ N = mg +
By lens equation, we get position of image formed is calculated as R
image I 2 from concave mirror as 1 1 1
− = ⇒ 
N = mg  1 +
2H 

1 1 1 v u f  R 
+ =
v u f 1 1 1 2 This is the force by block on the concave
= + =−
1 1 1 v −10 12 120 surface.
or =− +
v u f ∴ v = − 60 cm 7. The balanced chemical equations for
1 −1 1 So, final image is formed at a distance of the given equations can be written as
⇒ = +
v −  20 + x (− 5) 60 cm left of lens. (i) 3Cu + 8HNO3 → 3Cu(NO3 )2 + 2NO
 
 3  6. (a) Mass m is at height H from point Q, + 4H2O
5 −  + x (ii) 2CuI2 → Cu 2I2 + I2
20 where potential energy is taken zero.
1 1 1  3  (iii) 2Na 2S2O3 + I2 → Na 2S4 O6 + 2NaI
⇒ = − = A
v  20 + x 5  20 
+ x
  5
θ
Thus,
 3   3 
(a) The coefficients are : a = 3, b = 8,
1 − (3x + 5) − (3x + 5) R
= = R–h c = 3, d = 2 and e = 4
v 15  20 + x 5 (20 + 3x)
  m (b) The coefficients are : f = 2, g = 1, h = 1
 3 
B (c) The coefficients are : i = 2, j = 1, k = 1
As this image is acting like an object for h and l = 2
lens, so we have from lens equation, Q 2.54
5 (20 + 3x) (d) No. of moles of I2 = = 0.01 mol
u=− cm, f = − 10 cm From geometry of above figure, if at some 254
(3x + 5)
angle θ, height of mass m above lowest Moles of CuI2 = 2 moles of I2
and v = + 20 cm point Q is h, then from ∆ABC, = 1 mole of Cu
1 1 1 R−h = 2 × 0.01 = 0.02
⇒ + = cosθ = ⇒ h = R (1 − cosθ)
v u f R Weight = mole × atomic weight
1 (3x + 5) −1 Hence, potential energy of m as a Weight of Cu = 0.02 × 63.5 = 127
⇒ − = . g
20 5 (20 + 3x) 10 function of θ is
% of copper present in the alloy
5 (20 + 3x) − 20 (3x + 5) 1 PE = U (θ) = mgh 127
.
⇒ =− = × 100 = 63.5%
20 × 5 (20 + 3x) 10 ⇒ U (θ) = mgR (1 − cosθ) 2
100 + 15x − 60x − 100 1 (b) Kinetic energy at position θ = Loss of
⇒ =− 8. Conclusions that can be made from the
2000 + 300x 10 potential energy that occurred in given observations are as follows
reaching this position
2000 = 150 x (a) (i) As the compound in the bottle A did
2000 ⇒ Kinetic energy is K (θ) = mgH − U (θ) not dissolve in either 1 N NaOH or 1 N
=x ⇒ K (θ) = mgH − mgR (1 − cosθ)
150 HCl. This indicates that the molecule is
40 = mg (H − R (1 − cosθ)) neutral. Thus, the compound A is (III).
⇒ x= cm = 13.33 cm
5 (c) For H < < R, mass m will oscillate
CH3
So, mirror is placed at 13.33 cm behind about mean position in SHM. Time
the concave lens. period of oscillation of m is
(c) When a plane mirror is placed at same 1 R (ii) As the compound in the bottle B
T = dissolved in 1 N NaOH but not in 1 N
position, then 2π g
For plane mirror is f = ∞, HCl shows that the compound is acidic in
Time taken to travel from P to Q is nature, thus the compound in bottle B is
u = −  + x
20 one-fourth of this time period. (II).
 3  1 R
i.e. t=
= − 
20 40  8π g CO2H
+ 
 3 5
(d) From energy conservation at lowest
60
=− = − 12 cm point, if m has velocity v, then (iii) The compound in bottle C dissolved
5 1
mv2 = mgH in both 1 N NaOH and 1 N HCl indicates
Now, by mirror equation, 2 that the compound is amphoteric in
1 1 1 mv2 2mgH
+ = ⇒ mv2 = 2mgH or =
nature. Thus, the compound in bottle C is
v u f R R (IV).
1 1 CO2H
or =− But this centripetal force is resultant of
v u force of normal reaction N and weight of
−1
= −  
1 body.
⇒ NH2
v  12  mv2
⇒ N − mg = (amphoteric because of its Zwitter ion)
So, v = 12 cm R
https://iit-jeeacademy.blogspot.com

150 KVPY Question Paper 2010 Stream : SA

(iv) The compound in bottle D dissolved = 342 × 1866


. In experiment C, since the lime water
in both 1 N NaOH, but dissolve in 1 N = 638172
. g turns milky, ethanol fermentation is
HCl shows that the compound is a base, 1 mole of C12H22O11 gives 12 moles of CO2 occurring. In addition, since removal of
thus the compound is (I). air did not affect the reaction, the
∴1.866 moles of C12H22O11 gives
fermentation is anaerobic and yeast must
NH2 = 1866
. × 12 moles of CO2 be the organism in the flask.
= 22.392 moles of CO2 (b) In RBCs, lactic acid fermentation
At STP 1 mole of CO2 = 22.4 L occurs.
(b) The compound with highest solubility
in distilled water is (iv), 22.392 moles of CO2 = 22.4 × 22.392 L 12. (a) The result of radio-carbon dating
was correct.
COOH = 5015808
. L
Reason Vehicles running on the highway
10. (a) Difference in flower colour is most
beside the house emitted carbon dioxide
NH2 likely due to environmental factors.
from the combustion of petrol or diesel,
(b) Perform cross-breeding between the which are fossil fuels. The carbon in this
This is due to Zwitter ion formation plants from Chandigarh and those from
which can be shown as carbon dioxide, coming from living
Shimla to find out whether we get any material that has been converted into

COO pink flowers or flowers with any shade of petroleum millions of years ago, would
colour between pink and white in the get assimilated into the tissues of the
+
NH3 F1 -generation. plant as it uses carbon dioxide from the
(c) Grow the plants from Chandigarh in surrounding atmosphere for
9. C12H22O11 (s) + 12 O2 ( g ) Shimla and check whether they still photosynthesis. Therefore, tissues of the
→ 12CO2 ( g ) + 11H2O(l) produce white flowers or bear pink plant, when used for radio-carbon dating,
∆H = − 5.6 × 105 J flowers. would show the age of the plant to be
(a) 1 kcal = 418. kJ many thousands of years old.
11. (a) In experiment A, ethanol
∴2500 kcal = 2500 × 418. kJ = 10450 kJ fermentation occurs producing CO2, (b) A simple experiment to test the
turning lime water milky. Since acid is validity of this explanation would be to
So, the energy requirement of the human
not produced, the dye colour does not collect seeds from the plants and grow
body per day is 10450 kJ.
them in a plot of land away from the
(b) No. of moles of sucrose change.
highway or other sources of CO2 coming
10450 × 103 J In experiment B, lactic acid fermentation from the burning of fossil fuels.
= = 1866
. mol takes place, which produces acid but does
5.6 × 106 J Radio-carbon dating of plants growing
not produce CO2. Hence, dye colour from these seeds should show them as
Weight of sucrose changes to yellow but the lime water does young plants.
= molar mass × number of moles not turn milky.
https://iit-jeeacademy.blogspot.com

KVPY
KISHORE VAIGYANIK PROTSAHAN YOJANA

QUESTION PAPER 2009


Stream : SA
MM 100

Instructions
There are 80 questions in this paper.
This question paper contains two parts; Part I and Part II. There are four sections; Mathematics, Physics, Chemistry
and Biology in each part.
Out of the four options given with each question, only one is correct.

PART-I (1 Mark Questions)


MATHEMATICS 6. Let P1 , P2 , P3 , P4 , P5 be five equally spaced points on
x+5 the circumference of a circle of radius 1, centred at O.
1. The real numbers x satisfying > 1 are precisely
1−x Let R be the set of points in the plane of the circle
those which satisfy
that are closer to O than any of P1 , P2 , P3 , P4 , P5 . Then,
(a) x < 1 (b) 0 < x < 1 (c) − 5 < x < 1 (d) − 1 < x < 1
R is a
2. Let tn denote the number of integral-sided triangles (a) circular region
with distinct sides chosen from {1, 2, 3, ...., n}. Then, (b) pentagonal region
t20 − t19 equals (c) rectangular region
(a) 81 (b) 153 (c) 163 (d) 173 (d) oval region that is not circular
3. The number of pairs of reals (x, y) such that 7. A company situated at (2, 0) in the XY -plane charges
x = x2 + y2 and y = 2xy is ` 2 per km for delivery. A second company at (0, 3)
charges ` 3 per km for delivery.
(a) 4 (b) 3 (c) 2 (d) 1
The region of the plane where it is cheaper to use the
4. How many positive real numbers x satisfy the first company is
equation x3 − 3|x| + 2 = 0 ? (a) the inside of the circle (x + 5.4)2 + y2 = 18.72
(a) 1 (b) 3 (c) 4 (d) 6 (b) the outside of the circle (x + 1.6)2 + ( y − 5.4)2 = 18.72
5. Let (1 + 2x) = a 0 + a1x + a 2x + K + a 20x20.Then,
20 2 (c) the inside of the circle (x − 1.6)2 + ( y + 5.4)2 = 18.72
(d) the outside of the circle (x − 5.4)2 + ( y + 1.6)2 = 18.72
3a 0 + 2a1 + 3a 2 + 2a3 + 3a 4 + 2a5 + K + 2a19 + 3a 20
equals 8. In a right ∆ ABC, the incircle touches the hypotenuse
5 ⋅ 320 − 3 5 ⋅ 320 + 3 5 ⋅ 320 + 1 5 ⋅ 320 − 1 AC at D. If AD = 10 and DC = 3, the inradius of ABC is
(a) (b) (c) (d)
2 2 2 2 (a) 5 (b) 4 (c) 3 (d) 2
https://iit-jeeacademy.blogspot.com

152 KVPY Question Paper 2009 Stream : SA

9. The sides of a quadrilateral are all positive integers


and three of them are 5, 10, 20. How many possible
value are there for the fourth side?
(a) 29 (b) 31 (c) 32 (d) 34
10. If the volume of a sphere increases by 72.8%, then its © (d)
surface area increases by
(a) 20% (b) 44% (c) 24.3% (d) 48.6% 17. Consider two spherical planets of same average
density. Second planet is 8 times as massive as first
11. If the decimal 0. d25d25d25... is expressible in the
planet. The ratio of the acceleration due to gravity of
form n / 27, then d + n must be
the second planet to that of the first planet is
(a) 9 (b) 28 (c) 30 (d) 34
(a) 1 (b) 2 (c) 4 (d) 8
12. At what time between 10 O’clock and 11 O’clock are the
18. Two immiscible liquids A and B are kept in an
two hands of a clock symmetric with respect to the
U-tube. If the density of liquid A is smaller than the
vertical line (give the answer to the nearest second)?
density of liquid B, then the equilibrium situation is
(a) 10h 9m 13s (b) 10h 9m 14s
(c) 10h 9m 22s (d) 10h 9m 50s
13. A woman has 10 keys out of which only one opens a
(a) (b) A
lock. She tries the keys one after the another
A
(keeping aside the failed ones) till she succeeds in
opening the lock. What is the chance that it is the
seventh key that works?
7 1 3 1
(a) (b) (c) (d)
10 2 10 10
A
14. In a certain school, 74% students like cricket, 76% (c)
(d) None of these
students like football and 82% like tennis. Then, all
the three sports are liked by at least
(a) 68% (b) 32% (c) 77% (d) 36%
15. Let Sn be the sum of all integers k such that 19. In the figure given below, a ray of light travelling in
+1
2n < k < 2n , for n ≥ 1. Then, 9 divides Sn if and a medium of refractive index µ passes through two
only if different connected rectangular blocks of refractive
(a) n is odd (b) n is of the form 3k + 1 indices µ1 and µ 2 (µ 2 > µ1 ).
(c) n is even (d) n is of the form 3k + 2
θ1

PHYSICS
16. A boy standing on the footpath tosses a ball straight
up and catches it. The driver of a car passing by µ
moving with uniform velocity sees this.
µ1 µ2 θ2

The angle of incidence θ1 is increased slightly. Then,


the angle θ 2 is
(a) increases (b) decreases (c) remains same
(d) increases or decreases depending on the value of
(µ 1 / µ 2 )
The trajectory of the ball as seen by the driver will be 20. Two charges of same magnitude move in two circles
of radii R1 = R and R2 = 2R in a region of constant
uniform magnetic field B0.
The work W1 and W 2 done by the magnetic field in
the two cases respectively, are such that
(a) W1 = W2 = 0 (b) W1 = W2 ≠ 0
(a) (b) (c) W1 = W2 (d) W1 < W2
https://iit-jeeacademy.blogspot.com

KVPY Question Paper 2009 Stream : SA 153

21. Two charges + q and − q are placed at a distance b The bulb will light up, if
apart as shown in the figure given below. (a) S1 , S2 and S3 are all closed
B (b) S1 is closed but S2 and S3 are open
(c) S2 and S3 are closed but S1 is open
P A (d) S1 and S3 are closed but S2 is open

C 29. Two bulbs, one of 200 W and the other of 100 W are
connected in series with a 100 V battery which has
b/2 no internal resistance.
100V
+q –q
b
The electric field at a point P on the perpendicular
bisector as shown is
(a) along vector A (b) along vector C
(c) along vector B (d) zero
200 W 100 W
22. A block of mass M is at rest on a plane surface Then,
inclined at an angle θ to the horizontal. The (a) the current passing through the 200 W bulb is more
magnitude of force exerted by the plane on the than that through the 100 W bulb
block is (b) the power dissipation in the 200 W bulb is more than
(a) Mg cos θ (b) Mg tan θ (c) Mg sin θ (d) Mg that in the 100 W bulb
23. We are able to squeeze snow and make balls out of it (c) the voltage drop across the 200 W bulb is more than
because of that across the 100 W bulb
(a) anomalous behaviour of water (d) the power dissipation in the 100 W bulb is more than
(b) large latent heat of ice that in the 200 W bulb
(c) large specific heat of water 30. A solid cube and a solid sphere of identical material
(d) low melting point of ice and equal masses are heated to the same
24. Which of the following phenomena can be temperature and left to cool in the same
demonstrated by light, but not with sound waves in surroundings. Then,
an air column? (a) the cube will cool faster because of its sharp edges
(a) Reflection (b) Diffraction (b) the cube will cool faster because it has a larger surface
(c) Refraction (d) Polarisation area
(c) the sphere will cool faster because it is smooth
25. The temperature of a metal coin is increased by (d) the sphere will cool faster because it has a larger
100°C and its diameter increases by 0.15%. Its area surface area
increases by nearly
(a) 0.15% (b) 0.30% (c) 0.60% (d) 0.0225%
26. The note “Saa” on the Sarod and the Sitar have the CHEMISTRY
same pitch. The property of sound that is most 31. The element X which forms a stable product of the
important in distinguishing between the two
type XCl4 is
instruments is
(a) Al (b) Na (c) Ca (d) Si
(a) fundamental frequency (b) displacement amplitude
(c) intensity (d) waveform 32. A mixture of NH4Cl and NaCl can be separated by
235
27. 92 U atom disintegrates to 207
82 Pb with a half-life of
(a) filtration (b) distillation
(c) sublimation (d) decantation
10 yr. In the process, it emits 7 α particles and
9

n β− particles. Here, n is 33. The pair in which the first compound is ionic and the
(a) 7 (b) 3 (c) 4 (d) 14 second compound is covalent, is
28. Consider the following circuit given below. (a) Fe(OH)2 , CH3 OH (b) Fe(OH)2 , Cu(OH)2
S1 (c) CH3 OH, CH3 CH2OH (d) Ca(OH)2 ,Cu(OH)2
34. In the reaction, SO2 + 2H2S → 3S + 2H2O, the
substance that is oxidised is
S2 (a) SO2 (b) H2O (c) S (d) H2S
35. Sodium oxide dissolves in water to give sodium
S3 hydroxide which indicates its
(a) acidic character (b) basic character
(c) amphoteric character (d) ionic character
https://iit-jeeacademy.blogspot.com

154 KVPY Question Paper 2009 Stream : SA

36. For an ideal gas, Boyle’s law is best described by Given the solubility curves of KNO3 and KCl, which
of the following statements is not true ?
(a) (b)
(a) At room temperature, the solubility of KNO3 and KCl
p p are not equal
(b) The solubilities of both KNO3 and KCl increase with
T V temperature
(c) (d) (c) The solubility of KCl decreases with temperature
(d) The solubility of KNO3 increases much more compared
p p to that of KCl with increase in temperature

V T
BIOLOGY
37. The pH values of (i) 0.1 M HCl (aq), (ii) 0.1 M KOH,
(iii) tomato juice and (iv) pure water follow the order. 46. Which one of the following is the smallest in size?
(a) (i) < (iii) < (iv) < (ii) (b) (iii) < (i) < (iv) < (ii) (a) Bacteria (b) Mitochondrion
(c) (i) < (ii) < (iii) < (iv) (d) (iv) < (iii) < (ii) < (i) (c) Mammalian cell (d) Virus

38. When calcium carbide is added to water, the gas that 47. If birds are moved from 30°C - 10°C, their body
is evolved is temperature
(a) carbon dioxide (b) hydrogen (a) changes from 30°C - 10°C
(c) acetylene (d) methane (b) increases by 10°C
(c) does not change at all
39. The atomic radii of the alkali metals follow the order
(d) decreases by 10°C
(a) Li > Na > K > Cs (b) K > Cs > Li > Na
(c) Na > K > Cs > Li (d) Cs > K > Na > Li 48. Ascorbic acid is a/an
(a) strong inorganic acid (b) hormone
40. The number of possible structural isomers of C3 H4 is
(c) vitamin (d) enzyme
(a) 1 (b) 2 (c) 3 (d) 4
49. Bile salts
41. Among the four compounds, (i) acetone, (ii) propanol,
(a) breakdown polypeptide chains
(iii) methyl acetate and (iv) propionic acid, the two
that are isomeric are (b) emulsify fats and solubilise them
(a) methyl acetate and acetone (c) digest fats
(b) methyl acetate and propanol (d) help breakdown of polysaccharides
(c) propionic acid and methyl acetate 50. Dietary fibres are composed of
(d) propionic acid and acetone
(a) cellulose (b) proteins
42. One mole of nitrogen gas on reaction with 3.01 × 1023 (c) amylase (d) unsaturated fats
molecules of hydrogen gas produces 51. ‘On the Origin of Species, by Means of Natural
(a) one mole of ammonia Selection’ was written by
(b) 2.0 × 1023 molecules of ammonia
(a) Hugo de Vries (b) Charles Dickens
(c) 2 moles of ammonia
(c) Charles Darwin (d) Alfred Russell Wallace
(d) 3.01 × 1023 molecules of ammonia
52. Unlike humans, dogs cannot perspire to get rid of
43. Saponification is excess metabolic heat. They lose metabolic heat by
(a) hydrolysis of an ester (b) hydrolysis of an amide
(a) panting
(c) hydrolysis of an ether (d) hydrolysis of an acid chloride
(b) taking a bath
44. A concentrated solution of lead nitrate in water can (c) running in windy conditions
be stored in (d) rolling in the mud
(a) an iron vessel (b) a copper vessel
(c) a zinc vessel (d) a magnesium vessel 53. Haemodialysis is a treatment option for patients with
Solubility malfunctions of
45. (a) kidney (b) liver
g/L
250 (c) heart (d) lungs
200 54. An individual has ‘O’ blood group if his/her blood sample
KNO3
150 (a) clumps only when antiserum A is added
KCI (b) clumps only when antiserum B is added
100
(c) clumps when both antiserum A and antiserum B are
50 added
0 (d) does not clump when either antiserum A or antiserum
20 40 60 80 100 B is added
Temperature (ºC)
https://iit-jeeacademy.blogspot.com

KVPY Question Paper 2009 Stream : SA 155

55. In warmer weather, curd from milk forms faster 58. The part of the human brain that governs memory
because and intelligence is
(a) bacteria diffuse better in warmer milk (a) cerebrum
(b) the rate of bacterial multiplication increases (b) medulla
(c) lactogen is better dissolved (c) hypothalamus
(d) it is easier to separate protein from water (d) cerebellum

56. Seedlings grown in dark are 59. Saturated dietary fats increase the risk of heart
disease by
(a) similar to those grown in light
(a) widening arteries by thinning their walls
(b) taller than those grown in light
(b) narrowing veins by carbohydrate deposition
(c) shorter than those grown in light
(c) narrowing arteries by fat deposition
(d) they do not grow at all
(d) narrowing arteries by carbohydrate deposition
57. In humans, Rhesus condition can arise when 60. Rotation of crops is carried out to
(a) father is Rh + and mother is Rh − (a) increase variation in the mineral content of the soil
(b) father is Rh − and mother is Rh + (b) increase diversity of plant habitats
(c) either father or mother is Rh + (c) increase in nitrogen content of the soil
(d) either father or mother is Rh − (d) increase convenience for the farmer

PART-II (2 Marks Questions)


MATHEMATICS PHYSICS
61. Let loga b = 4, logc d = 2, where a , b, c, d are natural 66. A spring balance A reads 2 kg with a block of mass m
numbers. Given that b − d = 7, the value of c − a is suspended from it. Another balance B reads 3 kg
(a) 1 (b) − 1 when a beaker with a liquid is put on its pan. The
(c) 2 (d) − 2 two balances are now so arranged that the hanging
mass m is fully immersed inside the liquid in the
62. Let P (x) = 1 + x + x2 + x3 + x4 + x5 . What is the beaker as shown in the figure given below.
remainder when P (x12 ) is divided by P (x)?
(a) 0 (b) 6
(c) 1 + x (d) 1 + x + x2 + x3 + x4
A
63. In a ∆ ABC, the altitudes from B and C on to the
opposite sides are not shorter than their respective
opposite sides. Then, one of the angles of ABC is
(a) 30° (b) 45°
(c) 60° (d) 72°
B
64. In a ∆ ABC, AB = AC = 37. Let D be a point on BC
such that BD = 7, AD = 33. The length of CD is In this situation,
(a) 7 (b) 11 (a) the balance A will read 2 kg and B will read 5 kg
(c) 40 (d) not determinable (b) the balance A will read 2 kg and B will read 3 kg
(c) the balance A will read less than 2 kg and B will read
65. A line segment l of length a cm is rotated about a between 3 kg and 5 kg
vertical line L keeping the line l in one of the (d) the balance A will read less than 2 kg and B will read
following three positions (I) l is parallel to L and is at 3 kg
a distance of r cm from L, (II) l is perpendicular to L 67. According to the quantum theory, a photon of
and its mid-point is at a distance r cm from L, (III) l electromagnetic radiation of frequency ν has energy
and L are in the same plane and l is inclined to L at E = hν, where h is known as Planck’s constant.
an angle 30° with its mid-point at a distance r cm According to the theory of relativity, a particle of
from L. Let A1, A2 , A3 be the areas so generated. If mass m has equivalent energy E = mc2, where c is
r > (a / 2),then
speed of light. Thus, a photon can be treated as a
(a) A1 < A3 < A2 (b) A2 < A1 < A3 hν
particle having effective mass m = 2 .
(c) A1 = A3 < A2 (d) A1 = A2 = A3 c
https://iit-jeeacademy.blogspot.com

156 KVPY Question Paper 2009 Stream : SA

If a flash of light is sent horizontally in earth’s


gravitational field, then photons while travelling
CHEMISTRY
a horizontal distance d would fall through a 71. Reaction of NaCl with conc. H2SO4 liberates a gas, X
distance given by that turns moist blue litmus paper red. When gas X is
gd 2 h mcd 2 passed into a test tube containing egg shell powder
(a) (b) (c) (d) zero
2c2 mc h suspended in water another gas, Y is generated which
when passed through lime water makes it milky. The
68. A solid square plate is spun around different axes gases X and Y, respectively, are
with the same angular speed. In which of the (a) HCl and CO2 (b) Cl 2 and CO2
following choice of axis of rotation will the kinetic (c) SO2 and CO2 (d) SO2 and HCl
energy of the plate be the largest?
(a) Through the centre, normal to the plate
72. 10 mL of an aqueous solution containing 222 mg of
(b) Along one of the diagonals of the plate
calcium chloride (mol. wt. = 111) is diluted to 100 mL.
The concentration of chloride ion in the resulting
(c) Along one of the edges of the plate
solution is
(d) Through one corner normal to the plate
(a) 0.02 mol/L (b) 0.01 mol/L
69. An object is placed 0.40 m from one of the two (c) 0.04 mol/L (d) 2.0 mol/L
lenses L1 and L2 of focal lengths 0.20 m and
73. Aluminium reduces manganese dioxide to manganese at
0.10 m respectively, as depicted in the figure. The
high temperature. The amount of aluminium required to
separation between the lenses is 0.30 m. reduce one g mole of manganese dioxide is
L1 L2 (a) 1/2 g mol (b) 3/4 g mol
(c) 1 g mol (d) 4/3 g mol
74. Ethanol on reaction with alk. KMnO4 gives X which
when reacted with methanol in the presence of an acid
0.40m 0.30m gives a sweet smelling compound Y, X and Y
respectively, are
(a) acetaldehyde and acetone
The final image formed by these two lenses (b) acetic acid and methyl acetate
system is at (c) formic acid and methyl formate
(a) 0.13 m to the right of the second lens (d) ethylene and ethyl methyl ether
(b) 0.05 m to the right of the second lens 75. The pH of a 10 mL aqueous solution of HCl is 4. The
(c) 0.13 m to the left of the second lens amount of water to be added to this solution in order to
(d) infinity change its pH from 4 to 5 is
70. 5 charges each of magnitude 10−5 C and mass 1 kg (a) 30 mL (b) 60 mL (c) 90 mL (d) 120 mL
are placed (fixed) symmetrically about a movable
central charge of magnitude 5 × 10−5 C and mass
BIOLOGY
0.5 kg as shown in the figure given below. The
charge at P1 is removed. The acceleration of the 76. Proteins are synthesised on
central charge is (a) cytoskeleton (b) mitochondria
P1 (c) ribosomes (d) Golgi apparatus
77. Which of the following allows light to focus in visual
perception?
P2 P5 (a) Retina (b) Iris
O (c) Retinal pigment (d) Cornea
78. During cell division, if there is one round of chromosome
duplication followed by one round of cell division, the
number of chromosomes the daughter cells will have as
P3 P4
compared to the mother is
[Given, (a) equal (b) double (c) half (d) one fourth
1
OP1 = OP2 = OP3 = OP4 = OP5 = 1 m, = 9 × 109] 79. Similar type of vegetation can be observed, in the same
4πε 0 (a) latitude (b) longitude (c) country (d) continent
(a) 9 ms −2 upwards 80. Which of the following ecological food chains does not
(b) 9 ms −2 downwards represent an erect pyramid of numbers ?
(c) 4.5 ms −2 upwards (a) Grass–Rodent–Snake (b) Tree–Bird–Avian parasite
(d) 4.5 ms −2 downwards (c) Grass–Deer–Tiger (d) Insect–Chicken–Human
https://iit-jeeacademy.blogspot.com

KVPY Question Paper 2009 Stream : SA

Answers
PART-I
1 (d) 2 (a) 3 (a) 4 (a) 5 (c) 6 (b) 7 (b) 8 (d) 9 (a) 10 (b)
11 (d) 12 (b) 13 (d) 14 (b) 15 (c) 16 (b) 17 (b) 18 (c) 19 (b) 20 (a)
21 (a) 22 (a) 23 (*) 24 (d) 25 (b) 26 (d) 27 (c) 28 (c) 29 (d) 30 (b)
31 (d) 32 (c) 33 (a) 34 (d) 35 (b) 36 (c) 37 (a) 38 (c) 39 (d) 40 (b)
41 (c) 42 (b) 43 (a) 44 (b) 45 (c) 46 (d) 47 (c) 48 (c) 49 (b) 50 (a)
51 (c) 52 (a) 53 (a) 54 (d) 55 (b) 56 (b) 57 (c) 58 (a) 59 (c) 60 (a)

PART-II
61 (a) 62 (b) 63 (b) 64 (c) 65 (d) 66 (c) 67 (a) 68 (d) 69 (d) 70 (c)
71 (a) 72 (c) 73 (d) 74 (b) 75 (c) 76 (c) 77 (d) 78 (a) 79 (a) 80 (b)
* No option is correct.

Solutions
1. (d) We have, So, the number of triangle with largest 4. (a) We have,
x+ 5 side 20 and middle side.
x3 − 3|x| + 2 = 0
>1
1− x a = 11, then other sides are 21 − 11, 11 − 1
Case I x > 0
i.e. 10, 10 (11, 10, 10) 1 triangle. Similarly
x + 5 > 0, 1 − x > 0 a = 12, (smallest sides are (9, 10, 11) = 3 ∴ x3 − 3x + 2 = 0
∴ x> − 5 …(i) triangle a = 13, smallest sides are ⇒ (x − 1) (x − 1) (x + 2) = 0
x<1 …(ii) (8, 9, 10, 11, 12) = 5 triangle ⇒ x = 1, − 2
x+ 5 ∴Total number of triangles on Since, x> 0
Again, >1 ∴ x≠−2
1− x 1 + 3 + 5 + 7 + ... + 17 = 81
x=1
⇒ x + 5 > 1− x 3. (a) We have, Case II x < 0
⇒ x + 5 > (1 − x)2 x = x2 + y2 …(i) ∴ x3 + 3x + 2 = 0
[squaring both sides] and y = 2xy …(ii) Graph of x + 3x + 2
3

⇒ x + 5 > 1 + x2 − 2x Q y − 2xy = 0 Y
⇒ x − 3x − 4 < 0
2
⇒ y (1 − 2x) = 0
⇒ (x − 4) (x + 1) < 0 ⇒ y = 0, x =
1
⇒ x∈ (− 1, 4) …(iii) 2
From Eqs. (i), (ii) and (iii), we get Put y = 0 in Eq. (i), we get X′ X
–1 O
x ∈ (− 1, 1) x = x2 + 0
∴ − 1< x < 1 ⇒ x − x2 = 0
2. (a) tn denotes the number of integral ⇒ x (1 − x) = 0
Y′
sided triangle with distincts sides from ⇒ x = 0, x = 1
{1, 2, 3, ..., n}. t19 is the number of 1 Clearly, from graph.
Put x = in Eq. (i), we get
triangle formed by the sides from 2 It has one solution lie between (− 1, 0).
{1, 2, 3, ..., 19} and t20 is the number of 1  1
2
∴ Positive value of x = 1
triangle formed by the distinct sides from =   + y2
2  2 Hence, only one solutions.
{1, 2, 3, ..., 20}.
1 1
Any triangle counted in t19 is also ⇒ y2 = − 5. (c) We have,
2 4
counted in t20 , but t20 − t19 is the number 1 1 (1 + 2x)20 = a0 + a1 x + a2x2 + ... + a20 x20
of triangle counted in t20 but not in t19 . ⇒ y = ⇒ y=±
2
4 2 Put x = 1,
A triangle is counted in t20 but no t19 if
∴ Value of (x, y) are (0, 0) (0, 1) 320 = a0 + a1 + a2 + ... + a20 …(i)
and only if its largest side is 20.
Put x = − 1,
The middle side of is a and the smallest  1 , 1   1 − 1 .
    1 = a0 − a1 + a2 − a3 + ... + a20 …(ii)
side can be 21 − a to a − 1.  2 2   2, 2 
https://iit-jeeacademy.blogspot.com

158 KVPY Question Paper 2009 Stream : SA

On adding Eqs. (i) and (ii), we get 8. (d) We have, Volume of sphere after increase is V ′ and
320 + 1 ABC is a right angled triangle. AC is radius is r′.
= a0 + a2 + a4 + K + a20 4
2 hypotenuse of ∆ ABC. The incircle touch ∴ V ′ = πr ′3
the hypotenuse at D. 3
On subtracting Eq. (ii) from Eq. (i), we get
A ⇒ V ′ = (V + 72.8% of V )
320 − 1
= V  1 +
= a1 + a3 + a5 + K + a19 728  1728 v
2  =
10  1000  1000
Now, we have 10 1728  4 3 
⇒ V′=  πr 
3a0 + 2a1 + 3a2 + 2a3 + K + 2a19 + 3a20 D 1000  3 
= 3 (a0 + a2 + a4 + K + a20 ) r ∴
1728 4 3 4 3
. πr = πr ′
3 O F
+ 2 (a1 + a3 + ... + a19 ) 1000 3 3
r r r ′3 r′
 320 + 1  320 − 1 ⇒ = 1.728 ⇒ = 1.2
= 3   + 2   3
 2   2  C 3 E r B r r
∴Increase in surface area of sphere
5⋅ 3 20
+ 1 Given, AD = 10
= r′
2
2 CD = 3 =   = 1.44 = 144%
r
OE = OF = r (radius of circle)
6. (b) P1 , P2 , P3 , P4 , P5 be five equally ∴Surface area increase = (144 − 100)%
OE = BE = BF = r
spaced points on the circumference of = 44%
circle of radius 1. AD = AF = 10
11. (d) Let x = 0. d 25d 25d 25 …(i)
P3 P2 [AD and AF are tangents on a circle
from external points are equal] 1000x = d 25. d 25d 25 …(ii)
Similarly, CD = CE = 3 On subtracting Eq. (i) from Eq. (ii), we get
Q3 Q2 d 25
In ∆ABC, AC 2 = AB 2 + BC 2 999x = d 25 ⇒ x =
999
(13)2 = (10 + r )2 + (3 + r )2
Q4 O R Q1 But given, x=
n
P4
Q5
P1 ⇒ 169 = 100 + 20r + r 2 + 9 + 6r + r 2
27
⇒ 2r 2 + 26r − 60 = 0 n d 25 d 25
∴ = ⇒n =
⇒ r 2 + 13r − 30 = 0 27 999 37
P5
⇒ (r + 15) (r − 2) = 0 d ∈ {1, 2, 3, 4, 5, 6, 7, 8, 9}
Let R which is near to point O. ⇒ r = 2, r ≠ − 15 n is integer.
∴ OR is lie between the pentagonal region ∴d25 is a multiple of 37.
9. (a) We have three sides of
Q1 , Q2 , Q3 , Q4 , Q5 . quadrilateral are 5, 10, 20. When, put d = 9
7. (b) Let P (x, y) be any point lie in Let the fourth sides of quadrilateral = x Then, n is multiple of 37.
We know that in quadrilateral. Sum of 925
XY -plane. ∴ n= = 25
three side is greater than fourth sides 37
P (x, y) ∴ 5 + 10 + 20 > x …(i) ∴ n + d = 25 + 9 = 34
B (0, 3)
5 + 10 + x > 20 …(ii) 12. (b) Exactly at 10 O’clock the hour
5 + 20 + x > 10 …(iii) hand has travelled 300° from 12 O’clock.
P (2, 0) 10 + 20 + x > 5 …(iv) One hour = 60 minute.
From Eq. (i) x < 35 One minute hand moves 1° and hour
30  °  1  °
From Eq. (ii) x > 5 clock hand move   = 
From Eq. (iii) x > − 15  360   12 

According to problem, 2PA < 3PB From Eq. (iv) x > − 25 Assuming we have made it to 10 O’clock
Now, x is a positive integer. and now the hour and the minute hand
4PA < 9PB
2 2
∴ From Eqs. (i), (ii), (iii) and (iv), start moving spontaneously.
⇒ 4 [(x − 2)2 + ( y − 0)2 ] < 9 12
5 < x < 35 11
[(x − 0)2 + ( y − 3)2 ] 1
∴ Value of x is 6, 7, 8, 9, 10, 11, 12, 13, 14,
⇒ 4 (x − 4x + 4 + y ) < 9
2 2
15, 16, 17, 18, 19, 20, 21, 22, 23, 24, 25, 10 2
(x2 + y2 − 6x + 9) 26, 27, 28, 29, 30, 31, 32, 33, 34
⇒ 5x2 + 5 y2 − 54 y + 16x + 65 > 0 9 3
∴There are 29 possible values of x.
⇒ x + y − 10.8 y + 3.2x + 13 > 0
2 2
10. (b) Let initial volume of sphere is V 8 4
⇒ (x + 1.6)2 + ( y − 5.4)2 > 18.72 and radius is r.
Hence, the region is outside the 4 3 7 5
∴ V = πr 6
(x + 16
. )2 + ( y − 5 ⋅ 4)2 = 18.72 3
https://iit-jeeacademy.blogspot.com

KVPY Question Paper 2009 Stream : SA 159

If the hands of the watch are symmetric (2n − 1)(2n ) ⋅ 3 length column of B must appear above
Sn =
with vertical line. 2 this line in other limb of U-tube. It is as
Supposing this happens when x minutes But Sn = 9m, m∈ I shown below.
have passed x minutes = (6x)° have been (2n − 1)2n ⋅ 3
covered our hour hand would cover. ∴ = 9m
2
 1 ° 1 °
(6x)  =  x ⇒ (2n − 1)2n − 1 = 3m A
 
 12   2  B
⇒ 2n (2n − 1) = 6m
x °
∴ Our hand has covered  300 +  It is possible when, n is even.
 2
On subtracting this from 360° to find the 16. (b) Velocity of a ball is measured
angle from 12 O’clock anti-clockwise, relative to a fixed object or frame. If
frame of reference is moving, then object This is shown in option (c).
we get
will have a velocity opposite to that of
x ° x ° 19. (b) Given situation is
360° −  300 +  =  60 −  frame of reference. In given case, car is
 2  2 frame of reference. Due to motion of car,
So, they are symmetric. the ball has two velocities θ1
x A
∴ 60 − = 6x (i) vertical velocity
2 (ii) horizontal velocity opposite to r1
120  ° r2 B
⇒ x =   = 9 min 13.8 s motion of car. r3
 13  Because of there two velocities, path of
r4 C
∴ Time = 10 h 9 m 14 s ball will be parabolic in car frame. θ2
13. (d) Woman has 10 keys out of which
only one opens a lock.
Trajectory of v2
The first keys works with probability
1 Car When θ1 is increased at point A following
ball is a
10 frame Snell’s law (i ∝ r ), then r1 increases.
parabola
The conditional probability that the second from front v1=–vcar When r1 increases, r2 decreases at point B.
1 to back
keys works given that first failed = . Decrease of r2 causes a decrease of r3 as
9 (i ∝ r).
∴ Required probability (seventh key So, trajectory of ball will be as that of
Also r3 = r4 , so when r3 decreases, r4 also
works) option (c).
decreases. This causes a decrease in
= P (I fails) ⋅ P (2nd/I fails) ⋅ P (III /2 fails) 17. (b) Given, mass of second planet value of θ2.
P (7th/6th fails) = 8 × mass of first planet So, a increase of θ1 causes a decrease in θ2.
=
9 8 7 6 5 4 1 1
× × × × × × = ⇒ M2 = 8 M1 … (i)
10 9 8 7 6 5 4 10 4 3 4 3 20. (a) Force on a charged particle
⇒ πR2 × ρ = 8 × πR1 × ρ moving in region of magnetic field is
14. (b) Given, 74% students like cricket 3 3
F = q(v × B)
76% students like football ∴Density of both planets is same.
Clearly, F is perpendicular to both v and B.
82% students like tennis ⇒ R32 = 8R13
As, magnetic force is perpendicular to
∴ 26% student not like cricket or R2 = 2R1 … (ii)
velocity of moving charged particle. So,
24% student not like football So, ratio of acceleration due to gravity of work done in time ∆t is
the second planet to that of the first
18% student not like tennis W = (F ⋅ v) ∆t = 0
planet is
Student all the three sports like at least Hence, W1 = W2 = 0
 GM2 
= 100% − (sport not likes)  
 2  2 21. (a) Given situation is
= 100% − (26 + 24 + 18)% g2  R2   M2   R1 
= =  × 
g1  GM1   M1   R2  E1 sin θ E1
= 100% − 68% = 32%
 
 R2 
15. (c) We have, 2n < k < 2n + 1 , k ∈ N  1  θ E1 cos θ + E2 cos θ
θ Resultant
Number of integer between 2n and 2n + 1
2
8M1  R1  2
n+1 = ×  = E2
is i.e k = 2 − 2 −1
n
M1  2R1  1 E2 sin θ
First term = 2n + 1 So, g2 = 2 g1 . θ θ
n+1
Last term = 2 −1 q q
18. (c) As density of B is more than that
2n + 1 − 2n − 1 n
∴ Sn = [2 + 1 + 2n + 1 − 1] of A, a small volume of B weighs equals to As charges are of equal magnitude,
2 a large volume of A. direction of resultant field is along the
2n + 1 − 2n − 1 n Hence, if we draw a horizontal line from angle bisector of the angle formed by field
Sn = (2 )(1 + 2)
2 bottom of column of A, then a lesser vectors.
https://iit-jeeacademy.blogspot.com

160 KVPY Question Paper 2009 Stream : SA

22. (a) Weight of mass M can be resolved amplitude rises or falls in a given time So, power dissipation is more in the 100 W
into components parallel and duration. bulb. This makes option (d) correct.
perpendicular to inclined plane as This is shown by waveform of sound
shown below.
30. (b) Area of cube is more than that of
wave. Two waves can have same a sphere for same mass and density.
N=M
amplitude and frequency but can have
Cube also have sharp edges that radiates
different waveforms. As shown here.
more effectively than a flat surface.
A So, rate of cooling for cube is much rapid
Mg sin θ θ
Mg cos θ 1 than sphere. Effect of sharp edges is
θ Mg 2 prominent only at very high
t temperatures. So, option (b) is correct.
Force of block on incline is Mg cosθ, so
force exerted by block on mass M is 31. (d) As element X is forming a stable
Mg cosθ directed perpendicularly up the product of the type XCl 4 , so X must be
plane. tetravalent. Among the given elements Al
is trivalent, Na is monovalent, Ca is
23. (No option is matching) 27. (c) Given, 7 α particles are emitted divalent and Si is tetravalent. Thus,
235
When we squeeze the snow due to from U.
92 option (d) is correct.
increased pressure, melting point of ice
⇒ 235
92 U → 207
78 A + 7(42 He) 32. (c) A mixture of NH4 Cl and NaCl can
(snow) is lowered and it melts. As
be separated by the process of
pressure is removed, water formed is Now, nβ − particles are emitted to produce sublimation. In this process, solid directly
again frozen because melting point is 207
82 Pb. changes to gaseous state without passing
again raised. This process is called
⇒ 207
→ 207
+ n (0−1 β ) into liquid state. NH4 Cl sublimes to
regelation of ice. 78 A 82 Pb
gaseous NH3 and HCl upon heating
None of the option given is correct. Conservation of atomic number, gives whereas NaCl does not sublime, the
24. (d) Polarisation is a process of 78 = 82 − n ⇒ n = 82 − 78 = 4 reaction can be written as
alignment of electric vectors (or plane of So, 4 β − particles are emitted in given NH4 Cl(s) → NH3 ( g ) + HCl( g )
oscillation of particles) in a particular nuclear reaction. 33. (a) An ionic compound is formed
direction when an electromagnetic wave
28. (c) In given circuit, when S1 and S3 when metal and a non-metal reacts
(or a mechanical transverse wave) passes
whereas covalent compound is formed
through a narrow slit. Sound waves in air both are closed, then source is short
when 2 non-metals react with each other.
are longitudinal waves, so they cannot be circuited. Closing either S1 or S3 does not
In the given options, Fe(OH)2, Ca(OH)2
polarised. Polarisation is shown only by completes any of path. Hence, they
and Cu(OH)2 are ionic while CH3 OH,
transverse waves. produce no change in circuit.
CH3 CH2OH are covalent.
25. (b) Percentage increase in area S1
Thus, the correct pair in which first
Final area − Initial area
= × 100 compound is ionic and the second compound
Initial area is covalent is given in option (a).
π (r22 − r12 ) 34. (d) In the given reaction,
= × 100 S2
πr12 Reduction
(r1 + ∆r )2 − r12 S3
= × 100 SO2 + 2H2S → 3S + 2H2O
r12
Oxidation
where, ∆r = increase in radius.
 r 2 + 2r1 ∆r + ∆r 2 − r12  On closing S2 alone the circuit containing
Here, H2S is getting oxidised to H2O.
=  1  × 100

 r12  bulb is complete. Hence, bulb will light 35. (b) Na 2O + H2O → 2NaOH
up when S2 is closed alongwith S3 but S1 Sodium Oxide Sodium
2r1 ∆r + ∆r 2 hydroxide
= × 100 is open. Option (c) is correct.
r12 As Na 2O is a metal oxide which on
29. (d) Resistance of bulb is inversely dissolving with water gives NaOH which
As ∆r is small, we can neglect ∆r 2. proportional to its rated power. is metal hydroxide, indicates the basic
2r ∆r character of Na 2O.
≈ 1 2 × 100 2
V rated 1
r1 ⇒ R= ⇒R ∝ 36. (c) According to Boyle’s law, the
Prated Prated
 ∆r  pressure of a given mass of an ideal gas is
= 2 × 100 As bulbs are in series, so same current inversely proportional to its volume at a
 r1  flows through them at all instances. constant temperature.
= 2 × 015
. % = 0.30% Power dissipation in a series combination 1
∴ p∝
26. (d) Property of sound that makes is V
difference between two sounds having P = I 2R ⇒ P ∝ R Both option c and b is showing
same frequency and amplitude is called 1 relationship between p and V but the
or P∝ correct option is (c), as it is showing
timber or quality of sound. This is how Prated
https://iit-jeeacademy.blogspot.com

KVPY Question Paper 2009 Stream : SA 161

inverse relationship and option (b) is have different functional groups, so they much more compared to that of KCl
showing a linear relationship. are functional isomers. with increase in temperature. Thus,
42. (b) N2 + 3H2 2NH3 the statement (a) is correct.
p - 46. (d) Virus is the smallest in size
6.022 × 10 molecules of H2 = 1mole
23
among bacteria, mammalian cell and
∴3.011 × 1023 molecules of H2 = 0.5 mole.
mitochondrion. Virus ranges in size from
3 moles of H2 reacts with 2 moles of NH3 . about 20-400 nm in diameter. Bacterial
2
So, 1 mole of H2 reacts = moles of NH3 . cells range in size from 0.2-10 µm.
3 Mammalian cells are between 10-100 µm
2 in diameter. Mitochondria are commonly
∴0.5 mole of H2 react with × 0.5 between 0.75-3 µm in diameter.
V 3
1
37. (a) pH stands for potenz (power) of = mole 47. (c) Birds and mammals are
3 endothermic animals, i.e. their core body
hydrogen. The pH values for acidic
1 mole of NH3 contains 6.022 × 1023 temperature is kept nearly constant
solution ranges from 0 to 7, for pure through thermal homeostasis. Therefore,
molecules.
water its value is 7 and for basic solution, 1 if birds are moved from 30°-10°C their
it ranges from 7-14. So, mole of NH3 will have
3 body temperature does not change at all.
As HCl is a strong acid, the pH value will 1
= 6.022 × 1023 × 48. (c) Ascorbic acid is also known as
be least. On the other hand, KOH is a 3 vitamin-C, a vitamin found in citrus
strong base, its pH value will be = 2.0 × 1023 molecules. fruits and is an essential nutrient
maximum, tomato juice contains citric
43. (a) Saponification is the process of involved in the repair of connective tissue
acid which is a weak acid, so its pH value
hydrolysis of glyceryl ester of stearic acid and the enzymatic production of certain
will be more than HCl, but will be less
with sodium hydroxide, which produces neurotransmitters. In the body, it acts as
than H2O and KOH.
glycerol and soap. The reaction can be an antioxidant, helping to protect cells
Thus, the correct order of pH values will be from the damage caused by free radicals.
written as
(i) < (iii) < (iv) < (ii) O
49. (b) Bile salts consist of sodium
38. (c) When calcium carbide is added to CH2—O—C—C17H35 bicarbonate, sodium glycocholate and
water, acetylene gas is evolved. The sodium taurocholate. These are the
O
reaction can be written as primary components of bile produced in
CaC2 + 2H2O → Ca(OH)2 + CH≡≡ CH CH—O—C—C17H35 ∆ the liver. Their main function is
+3NaOH
(Calcium (Acetylene) emulsification of fat (i.e. breakdown of
carbide)
3C17H35COO– Na+ large fat molecules into small droplets)
39. (d) On moving down the group, as CH2—O—C—C17H35 (soap) and their solubilisation.
the atomic number of alkali metal +
increases, their atomic radii also O CH2OH 50. (a) Dietary fibres are composed of
increases because here shielding effect non-starch polysaccharides such as
predominates over nuclear charge. CHOH cellulose, dextrins, inulin, lignin, chitins,
Thus, the correct order of atomic radii of pectins, beta-glucans, waxes and
CH2OH oligosaccharides. Dietary fibres, also
alkali metal is Glycerol
known as roughage is the portion of plant
Cs > K > Na > Li
44. (b) A concentrated solution of lead derived food that cannot be completely
40. (b) Two structural isomers of C3 H4 nitrate in water can be stored in copper broken down by digestive enzymes.
are possible, which are propyne and vessel as copper can not react with lead Rest others, i.e. amylase, proteins and
cyclopropane. nitrate solution because it is less reactive unsaturated fats are although basic
than lead. Whereas, all other given components of diet but are not considered
CH3 C ≡≡ CH metals are more reactive than lead and as dietary fibres.
(Propyne) can easily react (displace) it from its
(Cyclopropene) 51. (c) ‘On the Origin of Species, by
solution.
41. (c) The molecular formulas for the Means of Natural Selection’ or ‘The
45. (c) The solubility of KCl increases Preservation of Favoured Races in the
given compounds are as follows with increase in temperature. Thus, the
acetone – C3 H6 O propanol – C3 H8 O Struggle for Life’, published on
statement (c) is incorrect.
24 November 1859 is a work of scientific
methyl acetate – C3 H6 O2 (a) From the graph, it is clear that at literature by Charles Darwin which is
propionic acid – C3 H6 O2. room temperature the solubility of considered to be the foundation of
As molecular formula for both propionic KNO3 and KCl are not equal. Thus, evolutionary biology.
acid (CH3 CH2 COH) and methyl acetate the statement (a) is correct.
 (b) The solubility of both KNO3 and KCl 52. (a) Panting refers to breathing
O increases with temperature. Thus, quickly and loudly through mouth. It
statement (b) is correct. helps dogs to get rid of excess metabolic
functional (CH3  COCH3 ) are same but heat when they are hot or engaged in
 (d) From the graph, it can be seen that
vigorous exercise.
O the solubility of KNO3 increases
https://iit-jeeacademy.blogspot.com

162 KVPY Question Paper 2009 Stream : SA

This happens because panting helps dogs 58. (a) The part of the human brain that 63. (b) ABC is a triangle in which BE
to circulate the necessary air through governs memory and intelligence is and CF are altitude.
their bodies to cool down. cerebrum. It is the largest part of the A
human brain associated with most
53. (a) Haemodialysis is a process of
critical and intelligent brain function
purifying the blood of a person whose
such as thoughts and actions.
kidneys are not working normally. It
F E
helps to filter waste, removes extra fluid 59. (c) Saturated dietary fats increase
and balances electrolytes (sodium, the risk of heart disease by narrowing
potassium, bicarbonate, chloride, arteries by fat deposition. Saturated fat
calcium, magnesium and phosphate). raises the level of cholesterol in your
During haemodialysis, blood is removed blood. High levels of Low Density Lipid
from the body and filtered through a (LDL) cholesterol in blood are B C
man-made membrane called a dialyser, responsible for their deposition in blood Given, BE ≥ AC
vessel, thus narrowing the lumens and
or artificial kidney and then the filtered CF ≥ AB
blood is returned to the body. increasing the risk of heart diseases and
BE
stroke. In ∆ABE, sin A =
54. (d) An individual has ‘O’ blood group AB
if his/her blood sample does not clump
60. (a) Rotation of crops is carried out to AB sin A = BE
increase variation in the mineral content
when either antiserum ‘A’ or antiserum AB sin A ≥ AC [Q BE ≥ AC] …(i)
of the soil. Prolonged planting of the
‘B’ is added. Antiserum is a blood serum Similarly in ∆ACF,
same crop type leads to the depletion of
containing antibodies against specific CF
specific nutrients in the soil. sin A =
antigens. Clumping or Agglutination is AC
the process that occurs if an antigen is 61. (a) We have, log a b = 4,
⇒ AC sin A = CF
mixed with its corresponding antibody. log c d = 2, a , b, c, d ∈ N
A person with ‘O’ blood group neither ⇒ AC sin A ≥ AB [QCF ≥ AB] …(ii)
⇒ b = a 4 , d = c2
have ‘A’ nor ‘B’ antigens on his/her red From Eqs. (i) and (ii), we get
⇒ b − d = 7 = a 4 − c2
blood cells, but both ‘a’ and ‘b’ antibodies (AB + AC ) sin A ≥ (AB + AC ) ⇒ sin A ≥ 1
in his/her plasma. Thus, they are also ⇒ 7 = (a 2 + c) (a 2 − c) ⇒ sin A = 1 [0 ≤ sin A ≤ 1]
called as universal donor. ⇒ 7 × 1 = (a 2 + c) (a 2 − c) ⇒ A = 90° [Q sin 90° = 1]
55. (b) The rate of bacterial ∴ a 2 + c = 7 and a 2 − c = 1 Now, from Eqs. (i) and (ii), we get
multiplication increases in warmer On solving, we get a = 2 and c = 3 AB ≥ AC
weather thereby forming curd from milk ∴ c− a= 3− 2=1 and AC ≥ AB
faster. This happens because
62. (b) We have, ∴ AB = AC
Lactobacillus (i.e. bacteria that turns
milk into curd) is more active in summers P (x) = 1 + x + x2 + x3 + x4 + x5 Hence, angles are 45°, 45°, 90°.
and develops more, which inturn 1 − x6 64. (c) Given, AB = AC = 37
P (x ) =
accelerates the fermentation process. 1− x AD = 33
56. (b) Seedlings grown in dark are  a ( − rn) BD = 7
Q a + ar + ar + K + ar =
2 n

taller than those grown in light as they  1− r  A
develop long hypocotyls (embryonic shoot)
It has 5 roots let α1 , α 2 , α3 , α 4 , α5 they
and their cotyledons remain closed
are 6th roots of unity
around the epicotyl in an apical hook.
This process is referred to as etiolation. Now,
37 37
P (x12 ) = 1 + x12 + x24 + x36 + x 48 + x 60
57. (c) In humans, Rhesus condition can 33
arise when either father or mother is ∴ P (x12 ) = P (x) ⋅ Q (x) + R (x)
Rh positive. Rhesus (Rh) factor is an Here, R (x) is a polynomial of maximum
inherited protein found on the surface of degree 4.
red blood cells. If an individual’s blood Put x = α1 , α 2 , α3 , α 4 , α5 ; we get B 7 D E C
has the protein, he/she is Rh positive and R (α1 ) = 6 = R (α 2 ) = R (α3 ) In ∆ABE,
if his/her blood lacks the protein, he/she is
= R (α 4 ) = R (α5 ) AB 2 = AE 2 + BE 2 …(i)
Rh negative. A baby may have the blood In ∆ADE,
type and Rh-factor of either parent or a ∴ R (x) − 6 = 0 has 6 roots, which
contradicts that R (x) is maximum of AD 2 = AE 2 + DE 2 …(ii)
combination of both parents. Rh factor
follows a common pattern of law of degree 4. ⇒ AB − AD 2 = BE 2 − DE 2
2

dominance because Rh-positive gene is a ∴So it is an identity. ⇒ AB 2 − AD 2 = (BE + DE ) (BE − DE )


dominant gene. Q R (x ) = 6
https://iit-jeeacademy.blogspot.com

KVPY Question Paper 2009 Stream : SA 163

⇒ AB 2 − AD 2 = (CE + DE )(BD ) 1 B /C
=
[Q BE = CE ] 2 a /2
9
⇒ AB 2 − AD 2 = CD ⋅ BD ⇒ BC =
4
AB 2 − AD 2
⇒ CD = A3 = πa  r − + r + 
a a
BD ∴
 4 4
372 − 332 ⇒ A3 = 2 πra
⇒ CD = [given]
7 ∴ A1 = A2 = A3 By lens equation,
(37 + 33) (37 − 33) 1 1 1
⇒ CD = 66. (c) When block is dipped in water, it − =
7 v u f
70 × 4 displaces some water which exerts
⇒ CD = = 40 buoyant force on block. As a result, or
1 1 1
= + =
1
+
1
7 v f u 0.2 (−0.4)
reading on scale A will be lower than 2 kg.
65. (d) Case I Area is generated by line 1 1
Due to reaction of block (which is equal to ⇒ =
segment l is curved surface area of buoyant force), beaker of water is pushed. v 0.4
cylinder = 2πra So, reading of scale B will be more ⇒ v = 0.4 m
∴ A1 = 2 πra than 3 kg. Now, this image acts like a virtual object
67. (a) In time t, a particle of mass m for second lens.
falls by a distance For second lens, u = + 0.1m, f = − 0.1m
1 2
h= gt
2
a
Now, distance covered horizontally = d
and speed in horizontal direction = c.
r d
Time to travel distance, d = t =
L c Final
d image u=+0.1 m
Case II Area is formed in circle. at ∞
123

h
By lens equation,
So, photons falls through a distance, 1 1 1 1 1
= + = +
2
gd 2 u − 01
gt = × g ×   =
a/2 1 2 1 d v f . 01.
r h=
2 2  c 2c2 1
⇒ =0 ⇒ v=∞
v
68. (d) Kinetic energy of a rotating body
is Hence, final image is formed at infinity.
L 1 2 Alternate Method For lens L1 ,
K = Iω u = − (2f ) so image is formed at 2f
Area of circular region 2
distance. Now, image distance for L2 is
 2
a 
2 0.1 m and focal length of L2 is also 0.1 m.
A2 = π   r +  −  r −  
a ω
    Hence, object for L2 is at focus, so its
 2 2 
image is formed at infinity.
= 2πra
70. (c) Forces on charge at point O,
Case III Area generated to form a initially balances each other as it is given
frustrum. that acceleration occurs when charge at
point P1 is removed. This means resultant
r–a/4 where, I = moment of inertia of a body.
of force due to charges at points P2 , P3 , P4
For square plate, moment of inertia is and P5 is equal and opposite to force due
largest along an axis perpendicular to to at point P1 .
a/2
plane of plate and through its one of Hence, acceleration of charge at point O
a/4 corner. As mass distribution is now
r is directed along OP1 .
B C farthest from axis of rotation. F
30°

So, kinetic energy of plate is largest in Acceleration =


a/2 m
option (d).
 Kq1 q2 
 2 
69. (d) Image of first lens acts like object  r  9 × 109 × 10−5 × 5 × 10−5
A = =
for second lens. m (1)2 × 1
In ∆ABC, Now, for first lens,
= 4.5 ms −2
BC u = − 0.40 m, f = + 0.20 m
sin 30° =
AC
https://iit-jeeacademy.blogspot.com

164 KVPY Question Paper 2009 Stream : SA

4 Ribosomes can be found floating within


71. (a) When concentrated sulphuric ∴ 1 mole of MnO2 reacts = moles of Al
acid reacts with NaCl, then sodium 3 the cytoplasm or attached to the
bisulphate and HCl gas (X) is formed. ⇒ Amount of Al required to reduce 1 g endoplasmic reticulum.
The HCl gas released is acidic in nature 4
mole of MnO2 = g mol. 77. (d) The cornea acts as the eye’s outer
and turns blue litmus red. 3
most lens. It functions like as window
2NaCl + H2SO4 → Na 2SO4 + 2HCl 74. (b) Ethanol on reaction with alkaline that controls and focusses the entry of
(X ) KMnO4 gives acetic acid (X), which when light into the eye (visual perception). The
When HCl gas (X) is passed into a testtube reacts with methanol in the presence of cornea contributes between 65-75
containing egg shell powder which an acid gives methyl acetate (Y), which is per cent of the eye’s total focussing
contains calcium carbonate suspended in a sweet smelling compound. power. When light strikes the cornea, it
water another gas CO2 (Y) is released, Alkaline bends or refracts the incoming light on to
CH3 CH2OH → CH3 COOH(X )
which turns lime water milky. Ethanol
KMnO4
Acetic Acid
the lens. The cornea covers the pupil (the
HCl + CaCO3 → CaCO2 + CO2 + H2O H+
opening at the centre of the eye), iris (the
CH3 COOH + CH3 OH  → coloured part of the eye), and anterior
(X ) Egg shell (Y ) Methanol
powder chamber (the fluid filled inside of the
CH3 COOCH3 (Y ) + H2O eye).
CO2 + Ca(OH)2 → CaCO3 + H2O
Methyl acetate
Milky
(Sweet smell) 78. (a) One round of chromosome
Thus, the gases X and Y respectively are Thus, X and Y respectively are acetic acid duplication followed by one round of cell
HCl and CO2. and methyl acetate. division leads to equal number of
72. (c) Initial concentration of CaCl 2 in chromosomes in the daughter cells as
75. (c) Given,
compared to the mother cell.
222 × 10−3 pH of 10 mL of HCl solution = 4
solution = = 0.2 M
111 × 10 × 10−3 ∴Concentration of H+ ions = 10−4 M 79. (a) Latitude and temperature are
related to each other in a way that, as we
On dilution, After dilution,
approaches the equator, the temperature
M1V1 = M2V 2 pH of HCl becomes = 5 gets warmer and as we approaches the
0.2 × 10 = M2 × 100 ∴Concentration of H+ ions = 10−5 M poles, it gets cooler. Since, same
As we know, vegetation grows in the same climatic
M2 = 0.02 M zone, therefore similar type of vegetation
M1V1 = M2V 2
∴ On dilution, the final concentration of can be observed in the same latitude.
CaCl 2 will become 0.02 M. = 10−4 × 10 = 10−5 × V 2
V 2 = 100 mL 80. (b) Tree–Bird–Avian parasite does
CaCl 2 → Ca 2+ + 2Cl − not represent an erect pyramid of
0.02M 0.02M 2 × 0.02 So, 90 mL of water should be added for number. Instead, it is an inverted
∴ The concentration of Cl − ion in the the pH change from 4 to 5. pyramid of number. All ecological
resulting solution = 0.04 mol/L. 76. (c) Ribosomes are the site where RNA pyramids of number are erect except in
is translated into protein. This process is parasitic food chain, where one primary
73. (d) 4Al + 3MnO2 → 3Mn + 2Al 2O3 producer supports numerous parasites
called protein synthesis. Protein is needed
3 moles of MnO2 reacts with 4 moles for many cell functions such as repairing which further support more
of Al. damage or directing chemical processes. hyperparasites.
https://iit-jeeacademy.blogspot.com
https://iit-jeeacademy.blogspot.com
https://iit-jeeacademy.blogspot.com

KVPY Practice Set 1 Stream : SA 167

KVPY
KISHORE VAIGYANIK PROTSAHAN YOJANA

PRACTICE SET 1
Stream : SA
MM 100

Instructions
There are 80 questions in this paper.
This question paper contains two parts; Part I and Part II. There are four sections; Mathematics, Physics, Chemistry
and Biology in each part.
Out of the four options given with each question, only one is correct.

PART-I (1 Mark Questions)


MATHEMATICS 4. When 10 is subtracted from each of the given
16 observation, the mean is reduced to 60%. If 5 is
1. If x = 3 + 1, then value of x4 + 4
is added to all the given observation, the mean will be
x
(a) 25 (b) 30 (c) 60 (d) 65
(a) 54 (b) 55
(c) 58 (d) 56 5. In the given figure, AB is the diameter of the circle
centered at O. If ∠COA = 60° , AB = 2r , AC = d and
2. Three friends Ajay, Vijay and Sanjay move along a
circular path of length 1.2 km with speeds of 6 km/h, CD = l, then l is equal to
8 km/h and 9 km/h respectively. Ajay and Vijay move B D
in the same direction but Sanjay move in opposite
direction, if they all start at the same time and from
same place. How many time will Ajay and Sanjay O
meets anywhere on the path by the time Ajay and 60° C
Vijay for the first time anywhere on the path?
(a) 6 times (b) 7 times A
(c)8 times (d) 9 times
d 3d
(a) d 3 (b) (c) 3d (d)
3. A cone is within the cylinder and cylinder is within a 3 2
cube touch by all vertical faces with same bases and
height, then the ratio of their volume will be 6. The sum of all integers x for which x4 + x3 + x2 + x + 1
(a) 14 : 11 : 13 (b) 42 : 33 : 11 is a perfect square
(c) 56 : 36 : 22 (d) None of these (a) 2 (b) 4 (c) 3 (d) 6
https://iit-jeeacademy.blogspot.com

168 KVPY Practice Set 1 Stream : SA

7. The sum of all 3-digit numbers which are equal to 11


times the sum of squares of their digits is
PHYSICS
(a) 1212 (b) 1353 (c) 1452 (d) 1364 16. A solid cylinder of mass m and length
8. If 2f (xy) = { f (x)} + { f ( y)} for all x, y ∈ R, and f (1) = 2,
y x l is placed vertically on the ground.
l
then the value f (5) − f (3) is equal to If Y = Young’s modulus of cylinder’s
(a) 12 (b) 24 (c) 36 (d) 48 material, then strain energy stored is
9. A job has to completed by 12 boys in 15 days. If three m2 g 2l m2 g 2l m2 g 2l m2 g 2l
(a) (b) (c) (d)
boys are absent from the first day, then by what 3AY 6AY 2AY AY
percentage should the remaining boys increase their 17. A metallic wire is loaded at ends with two masses is
rate of working to complete the job placed over a slab of ice.
1 1
(a) 33 % (b) 22 %
3 2
2 1
(c) 40 % (d) 30 %
3 3
10. The number of positive integer x which satisfies the m m
x  x 
condition = , where x is greatest integer
99  101  This wire passes through slab without splitting it
into two pieces. This is due to
functions (a) depression of melting point
(a) 2499 (b) 2500 (b) elevation of melting point
(c) 2501 (d) None of these (c) high conductivity of metal wire
11. In a regular heptagon ABCDEFG the side of (d) high specific heat of ice
1 1 18. Two identical boxes one of them is filled with
heptagon is 1, then diagonals + is equal to
AC AD nitrogen and other is filled with helium are put on a
1 2 1 fast moving train.
(a) (b) (c) 1 (d)
2 3 4 If train is suddenly stopped, then what will be the
ratio of rise of temperature of two boxes nearly?
12. In the given figure, the length of AB is
(a) 2 : 1 (b) 1 : 4 (c) 4 : 7 (d) 1 : 1
A B
19. A gas expands from state a to state b as shown below.
3 cm

p
6 cm a
4 cm

(a) 3 (b) 2 6 (c) 3 6 (d) 4 b


V
13. There are 20 units cubes all of whose faces are white,
and 44 units cubes all of whose faces are red. They Temperature of gas during the above expansion
are put together to form a bigger cube (4 × 4 × 4). process
What is the minimum number of white visible on this (a) decreases continuously (b) increases continuously
larger cube? (c) decreases then increases (d) increases then decreases
(a) 20 (b) 14 (c) 12 (d) 8
20. An elastic ball of mass m is suspended with an ideal
14. A larger tanker can be filled by two pipes A and B in thread. Another ball of same mass hits ball with
60 min and 40 min respectively. How many minutes velocity v0 as shown below.
will take to fill the empty tanker if only B is used in
the first half of the time and A and B are both used in
the second half of the time? m
(a) 15 (b) 20 (c) 27.5 (d) 30
37° v
15. If α and β are acute angles such that 0

3 1
cos2 α + cos2 β = and sin α − sin β = , then α + β m
2 4
equals Impulsive tension in the string due to collision is
π π π π 5 18 4
(a) (b) (c) (d) (a) mv0 (b) mv0 (c) mv0 (d) mv0
6 4 3 2 7 17 9
https://iit-jeeacademy.blogspot.com

KVPY Practice Set 1 Stream : SA 169

21. A stationary radioactive nucleus decays as: 27. A body falls through a viscous fluid starting from rest
X → 4
+Y
2He
towards ground. Then, after a long time, which of the
If speed of α-particle is v, then speed of daughter most likely to be correct?
nucleus Y will be (a) No energy is dissipated by body
4v 2v (b) Rate of potential energy dissipation is constant
(a) (b)
A−4 A−4 (c) Rate of kinetic energy dissipation is constant
4v 2v (d) Whole of the energy of body can be dissipated before
(c) (d) reaching ground
A+ 4 A+ 4
28. Identical blocks of wood are piled over each other as
22. 30 g ice at 0°C is mixed with 25 g of steam at 100°C. shown below.
The resulting mixture is
(Latent heat of fusion = 80 cal/g, latent heat of D
C
vapourisation = 540 cal/g and specific heat of B
water = 1 cal) A
(a) water and ice at 0°C l
(b) water at 100°C If length of each block is l, then maximum possible
(c) water and steam at 100°C projection for topmost block is
(d) water, ice and steam at 50°C l l l l
(a) (b) (c) (d)
23. An object falling freely from rest covers a distance s 2 3 4 6
in 5th second. Then, distance travelled by object in 29. For a two particle systems, kinetic energy K and
7th second is potential energy U varies with separation r of
13
(a) s (b)
9
s (c)
9
s (d)
11
s
particles as
9 13 11 9
24. A pendulum bob is given a push when it is suspended
freely. K
E
Energy
A B C
U
l

m
u The system is a bound system for
If bob successfully completes the vertical circle, then (a) r = rA (b) r = rB
least ratio of kinetic energies at bottom and top of the (c) r = rC (d) all points A, B and C
circle is 30. A point source of light S, placed at a distance L in
(a) 2 : 1 (b) 5 : 1 (c) 7 : 1 (d) 1 : 2 front of the centre of a plane mirror of width 1 m,
25. A wood block is floating in benzene at 0°C. hangs vertically on a wall.
A man walks in front of mirror along a line parallel.

L
d = 1m

S
It is given,
Density of wood = 880 kg m −3 2L
−3
Density of benzene = 900 kg m To the mirror at a distance 2L from mirror as shown
−4 −1 in the above figure. Distance upto which source is
Cubical expansion coefficient of wood = 15
. × 10 K
visible to man is
Cubical expansion coefficient of benzene (a) 1 m (b) 2 m (c) 3 m (d) 4 m
. × 10−3 K −1
= 12
Minimum temperature at which wooden block just
sink in is
CHEMISTRY
(a) 22°C (b) 10°C (c) 12°C (d) 15°C 31. The correct order of the lattice energies of the
26. Mass of the largest stone that can be moved by following ionic compounds is
flowing water stream depends on density of water, (a) NaCl > MgBr2 > CaO > Al 2O3
acceleration due to gravity and velocity of flow. Then, (b) NaCl > CaO > MgBr2 > Al 2O3
mass m is proportional to (c) Al 2O3 > MgBr2 > CaO > NaCl
(d) Al 2O3 > CaO > MgBr2 > NaCl
(a) v2 (b) v4 (c) v6 (d) v−1
https://iit-jeeacademy.blogspot.com

170 KVPY Practice Set 1 Stream : SA

32. Average volume available to a molecule in a sample 43. Clemmensen reduction of ketone is carried out in the
of ideal gas at STP is presence of which of the following reagents?
(a) 3.72 × 10−20 cm3 (b) 2. 69 × 1019 cm3 (a) Zn-Hg with HCl (b) LiAlH4
(c) 22400 cm3 (d) 22400 × 6.02 × 1023 cm3 (c) H2 and Pt as a catalyst (d) Glycol with KOH
33. Among the quantities, boiling point (I), entropy (II), 44. Standard electrode potential of three metals X, Y and
pH (III) and emf of a cell (IV), intensive properties Z are − 1 . 2 V, + 05
. V and − 30
. V, respectively.
are The reducing power of these metals will be
(a) Both I and II (b) I, II and III (a) Y > X > Z (b) Z > X > Y
(c) I, III and IV (d) All of these (c) X > Y > Z (d) Y > Z > X
34. The number of radial nodes of 3s and 2p -orbitals are 45. When white phosphorus is heated with caustic soda,
respectively the compounds formed are
(a) 2, 0 (b) 0, 2 (c) 1, 2 (d) 2, 1 (a) PH3 + NaH2PO3 (b) PH3 + NaH2PO2
(c) PH3 + Na 2HPO3 (d) PH3 + NaH2PO4
35. A sulphur containing species that cannot be a
reducing agent is
(a) SO2 (b) SO32− (c) H2SO4 (d) S2− BIOLOGY
36. The energy (in J) corresponding to light of 46. During urine formation, the filtration of blood at the
wavelength 45 nm, is closest to glomerulus is
(h = 6.63 × 10−34 Js, speed of light = 3 × 108 ms−1 ) (a) an active process
(a) 6.63 × 108 (b) 6.67 × 1011 (b) an osmotic process
−15
(c) 4.42 × 10 (d) 4.42 × 10−18 (c) a pressure dependent physical process
(d) a non-energy-mediated transport process
37. The reaction N2 + 3H2 → 2NH3 is used to produce
ammonia. When 450 g of hydrogen was reacted with 47. Grave’s disease is associated with
nitrogen, 1575 g of ammonia were produced. The (a) insufficiency of thyroid hormones
percentage yield of reaction is closest to (b) excess of thyroid hormones
(a) 61.8 (b) 72.4 (c) 51.8 (d) 89.1 (c) insufficiency of corticosteroids
(d) excess of growth hormones
38. The number of isomers for the compound with the
molecular formula C2BrClFI is 48. ‘Imperfect fungi’ is a group represented by fungal
(a) 3 (b) 4 (c) 5 (d) 6 species which have
39. Time required to deposit one millimole of aluminium (a) simple mycelia
metal by the passage of 9.65 A through molten (b) no known mechanism of sexual reproduction
electrolyte containing aluminium ion is (c) unknown phylogenetic relationship
(a) 30 s (b) 10 s (c) 30,000 s (d) 10,000 s (d) lost its survival mechanism against harsh environment

40. The IUPAC name of the following compound is 49. Which of the following is not a characteristic of
phylum Chordata?
(a) Pharyngeal slits (b) Amniotic egg
(c) Post-anal tail (d) Notochord
(a) 4, 4, 3-trimethyl hex-1-yne 50. The energy rich fuel molecules produced in the TCA
(b) 4, 4, 3-trimethyl hex-1-ene cycle are
(c) 3, 4, 4-trimethyl hex-1-yne
(a) 2 GTP, 2 NADH and 1 FADH2
(d) 3, 4, 4-trimethyl hex-1-ene
(b) 1 GTP, 2 NADH and 2 FADH2
41. Which of the following compounds will not undergo (c) 1 GTP, 3 NADH and 1 FADH2
aldol condensation? (d) 2 GTP and 3 NADH
(a) Methanal (b) 2-methyl pentanal 51. In Drosophila melanogaster males, homologous
(c) Cyclohexanone (d) 1-phenyl propanone chromosomes pair and segregate during meiosis but
42. Which is the most basic compound among the given crossing over does not occur. At which stage of
options? meiosis does segregation of 2 alleles of a gene take
place in their individuals?
NO2
(a) Zygotene (b) Diakinesis
(c) Anaphase-I (d) Anaphase-II
N
| 52. Excess oxygen consumed after a vigorous exercise is
H N (a) to pump out lactic acid from muscle
I II III (b) to increase the concentration of lactic acid in muscle
(c) to reduce dissolved carbon dioxide in blood
(a) I (b) II (c) III (d) Both I and III
(d) to make ATP for gluconeogenesis
https://iit-jeeacademy.blogspot.com

KVPY Practice Set 1 Stream : SA 171

53. Mark the correct relationship (d) Evolution need not always lead to a better phenotype
(a) ψw = ψ p − (ψπ + ψm ) (b) ψw = ψ p + ψs + ψm 57. Which one of the following compounds is generally
(c) ψw = ψ p + ψπ − ψm (d) None of these translocated in the phloem?
54. Hydrogen bonds occur between which of the following (a) Sucrose (b) D-glucose
constituents of DNA? (c) D-mannose (d) D-fructose
(a) Sugar and base (b) Phosphate and base 58. Which organelles would be more prominent in a
(c) Complementary bases (d) Phosphate and sugar secretory cell than in a non-secretory cell?
55. Which one of the following neurotransmitters is (a) Golgi bodies (b) Lysosomes
secreted by the pre-ganglionic neurons of (c) Mitochondria (d) Pinocytic vesicles
sympathetic nervous system? 59. In the conversion of RuBP to GP (PGA)
(a) Epinephrine (b) Acetylcholine (a) a molecule of carbon dioxide is accepted
(c) Dopamine (d) Nor-epinephrine (b) a stable six-carbon molecule is produced
56. Which of the following statements about evolution is (c) ATP is generated
(d) hydrogen is combined with oxygen to form water
incorrect?
(a) Evolution is the product of natural selection 60. Adventitious roots develop in
(b) Evolution is goal-oriented (a) creepers (b) trailers
(c) Prokaryotes evolve faster than eukaryotes (c) twinners (d) All of these

PART-II (2 Marks Questions)


MATHEMATICS PHYSICS
61. Let P (a , b) be a variable point satisfying 66. A thin rod of length f / 3 is placed along the principal
4 ≤ a + b ≤ 9 and b − 4ab + a ≤ 0. Let R be the
2 2 2 2 axis of a concave mirror of focal length f such that its
complete equation represented in XY-plane in which image which is real and elongated just touches the
P can lie, the area of region R is rod. Linear magnification obtained is
2π 4π 5π 3
(a) (b) π (c) (d) (a) 1 (b)
3 3 3 2
1 5
(c) (d)
62. Let S1 (n ) be the sum of first n terms of arithmetic 2 2
progression 8, 12, 16, ... and let S2 (n ) be the sum of 67. Cube of side a is located in three dimensional
the first n terms of arithmetic progression 17, 19, 21, cartesian space as shown in the figure given below.
... if for some value of n , S1 (n ) = S2 (n ), then this
z
common sum is
(a) 216 (b) 260
(c) 200 (d) None of these B C
63. On a card, the following three statements are found
A
1. on this card exactly one statement is false D
P O
2. on this card exactly two statement are false F
y
3. on this card exactly three statement are false Q
The number of false statement on the card is exactly G E
(a) 0 (b) 1 (c) 2 (d) 3 x
64. ∆ABC is right angled at A. The circle with centre A Unit vector in the direction PQ (from centre of face
and radius AB cuts BC and AC internally at D and E ABOG to centre of face OGEF) is
respectively. If BD = 20 and DC = 16, then the length (a) − 2$j − $
2k
AC equals
(a) 6 21 (b) 6 26 (c) 30 (d) 32 (b) − 2$j + 2 k$
1 $ 1 $
65. The coefficient of x30 in the expansion of (c) j− k
2 2
(1 + 2x + 3x2 + K + 20x19 + 21x20 )2 is 1 1 $
(d)− $j + k
(a) 2706 (b) 2450 (c) 1481 (d) 256 2 2
https://iit-jeeacademy.blogspot.com

172 KVPY Practice Set 1 Stream : SA

Na/ ether
68. In given circuit, cells have zero internal resistances. 75. C5 H12 + Cl2  
Light
→ C5 H11Cl   → C
Power dissipated in resistors R1 = 20 Ω and R2 = 20 Ω A B
is Only one structure is possible for B. Identify A, B and
C in the reaction
(a) CH3 C(CH3 )2 CH3 , CH3 C(CH3 )2 CH2Cl,
R1 20 Ω R2 20 Ω CH3 C(CH3 )2 CH2CH2C(CH3 )2 CH3
+
(b) CH3 CHCH2CH3 , CH3 C(Cl)(CH3 )CH2CH3 CH3 ,
– + – 10 V
CH2C(CH3 )2 C(CH3 )2 CH2CH3
10 V (c) Both (a) and (b)
(a) P1 = 5 W , P2 = 0 W (b) P1 = 10 W , P2 = 5 W (d) None of the above
(c) P1 = 5 W , P2 = 5 W (d) P1 = 5 W , P2 = 10 W
69. A piece of wood of mass 0.03 kg is dropped from the BIOLOGY
top of a 100 m high building. At same instant, a
bullet of mass 0.02 kg is fired from the ground, with a 76. Which one of the following statements regarding
velocity of 100 ms −1along same vertical line. Bullet plant growth hormones is correct?
gets embeded in block of wood. Height to which (a) Gibberellins do not play any role in flowering
combined system rises above the top of the building (b) Auxin and cytokinin inhibit cell division
is (Take, g = 10 ms−2) (c) ABA inhibits root growth and promotes shoot growth
(a) 10 m (b) 40 m (c) 30 m (d) 20 m at low water potential
(d) ABA promotes leaf senescence independent of ethylene
70. If force F, length L and time T are chosen as
fundamental quantities, then mass is 77. A woman with one gene for haemophilia and a gene
−1
(a) [FL T ] −2 −1
(b) [FL T ] 2 for colourblindness on one of the X-chromosomes
(c) [FL T −2] (d) [FL0 T −2] marries a normal man. How will the progeny be?
(a) All sons and daughters haemophilic and colourblind
(b) 50% haemophilic colourblind sons and 50% colourblind
CHEMISTRY carrier daughters
(c) 50% haemophilic daughters and 50% colourblind
71. 10 dm3 of an ideal monoatomic gas at 27°C and daughters
. × 105 N - m−2 pressure heated at constant
101 (d) Haemophilic and colourblind daughters
pressure to 127°C. Thus, entropy change in JK −1 is
78. Which one of the following relationships is true in
(a) 2.422 (b) 5.98
(c) − 2 .422 (d) − 5.981
water at 25°C?
(a) [OH− ] = [H2O− ] (b) [H+ ] = [H2O]
72. What is the mass of precipitate formed when 50 mL −14
of 16.9% solution of AgNO3 is mixed with 50 mL of (c) Kw > 1 × 10 (d) [H+ ] = [OH− ]
5.8% NaCl solution ? 79. Identify the correct match between the animal
(Molar mass of Ag = 1078. , N = 14, O = 16, Na = 23 and (flatworm, earthworm, roundworm) and its body
Cl = 35.5) cavity type (acoelomate, coelomate,
(a) 28 g (b) 3.5 g (c) 7 g (d) 14 g pseudocoelomate).
73. Which of the following diatomic molecules would be (a) Roundworm–Pseudocoelomate;
Earthworm–Acoelomate; Flatworm–Coelomate
stabilised by removal of an electron?
(b) Roundworm–Acoelomate; Earthworm–Coelomate;
(a) C2 (b) CN
Flatworm–Acoelomate
(c) N2 (d) O2
(c) Roundworm–Pseudocoelomate;
74. When calcium carbide is hydrolysed, compound X is Earthworm–Coelomate; Flatworm–Acoelomate
formed as a major product, which then reacts with (d) Roundworm–Coelomate;
dilute sulphuric acid in the presence of mercuric Earthworm–Pseudocoelomate; Flatworm–Acoelomate
sulphate, a compound Y is formed. 80. A sequence of amino acids may end in either an
Compounds X and Y are amino group (—NH2 ) or a carboxyl group (—COOH).
(a) C2H2 and CH3 CHO What is the theoretical number of chemically
(b) CH4 and HCOOH different dipeptides that may be assembled from
(c) C2H4 and CH3 COOH 20 different amino acids?
(d) C2H2 and CH3 COOH (a) 40 (b) 80 (c) 160 (d) 400
https://iit-jeeacademy.blogspot.com

KVPY Practice Set 1 Stream : SA 173

Answers
PART-I
1 (d) 2 (b) 3 (b) 4 (b) 5 (a) 6 (a) 7 (b) 8 (b) 9 (a) 10 (a)
11 (c) 12 (c) 13 (c) 14 (d) 15 (c) 16 (b) 17 (a) 18 (a) 19 (d) 20 (c)
21 (a) 22 (c) 23 (a) 24 (b) 25 (a) 26 (c) 27 (b) 28 (d) 29 (d) 30 (c)
31 (d) 32 (a) 33 (c) 34 (a) 35 (c) 36 (c) 37 (a) 38 (d) 39 (a) 40 (c)
41 (a) 42 (b) 43 (a) 44 (b) 45 (b) 46 (c) 47 (b) 48 (b) 49 (b) 50 (c)
51 (c) 52 (d) 53 (b) 54 (c) 55 (b) 56 (b) 57 (a) 58 (a) 59 (a) 60 (a)

PART-II
61 (d) 62 (b) 63 (c) 64 (b) 65 (a) 66 (b) 67 (b) 68 (a) 69 (b) 70 (b)
71 (a) 72 (c) 73 (d) 74 (a) 75 (a) 76 (d) 77 (b) 78 (d) 79 (c) 80 (d)

Solutions
1. (d) We have, x = 3 + 1 πx3 πx3 DB = DC = l
∴V1 : V 2 : V3 = x3 : :
Both sides squarring, we get 4 12 [Q two tangents are equal
π π from external points]
⇒ x2 = 3 + 2 3 + 1 = 4 + 2 3 = 1: :
4 12 1
4 4 4 ⇒ ∠BOD = ∠COD = ∠BOC
⇒ = = 22 22 2
x 2
4 + 2 3 2( 2 + 3) = 1: :
28 84 ⇒ ∠BOC = 180° − ∠COA
= 2(2 − 3) = 4 − 2 3
2
= 42 : 33 : 11 = 180° − 60° = 120°
⇒ x4 + 4 =  x2 + 2  − 8
16 4 1 1
4. (b) Let the observation are ⇒ COD = ∠BOC = × 120° = 60°
x  x  2 2
x1 , x2 , x3 , K , xn
= (4 + 2 3 + 4 − 2 3 ) 2 − 8 n
In ∆OCD, tan ∠COA =
CD
= (8)2 − 8 = 64 − 8 = 56 ∑ xi OC
i =1
2. (b) Time taken by Ajay and Vijay to ∴ x= ⇒ tan 60° =
l
n d
meet first time anywhere on the path
When 10 is subtracted from each ⇒ l=d 3
Distance 12
.
= = = 0.6 h observation, then mean = x − 10
Relative speed 8.6 6. (a) Let y2 = x4 + x3 + x2 + x + 1
∴ x − 10 = 60% of x
2
x2
Consider  x2 +  = x4 + x3 +
Time taken by Ajay and Sanjay to meet 60 x
⇒ x − 10 = x
anywhere 100  2  4
Distance 1.2
= = = 0.8 h
= x + x + x + x + 1 −  x + x + 1

3 3 2
Relative speed 9 + 6 ⇒ x − x = 10 ⇒ x = 25 4 3 2
5 4 
The number of times Ajay and Sanjay When 5 is added to each observation, 1
= y2 − (3x2 + 4x + 4)
meets anywhere on the path by the time then new mean is 25 + 5 = 30 4
Ajay and Vijay meets each other for the
5. (a) Given, ∠COA = 60° As discriminant of 3x2 + 4x + 4 is
36 1
first time = = 7 , i.e. 7 times. AO = OB = OC = r negative.
4.8 2 ∴ 3x3 + 4x + 4 > 0
In ∆AOC, AO = CO = r and ∠ COA = 60°
3. (b) Let the side of cube = x 2
Thus,  x2 +  < y2
x
x B D
∴ Radius of cylinder = Radius of cone =  2
2
Height of cylinder = Height of cone = x r
| y |>  x 2 + 
x
l ⇒
Volume of cube (V1 ) = x3 O 60°  2
Volume of cylinder 60° x  1
r d C But x + =  x +  x is non-negative,
2
2  2
2
πx
3
(V 2 ) = π   ⋅ x =
x
A
 2 4 ∀x ∈ I
Volume of cone ∴∆AOC is an equilateral triangle.  x2 + x  = x2 + x < | y|
1  x
2
πx 3  2 2
(V3 ) = π   ⋅x = ∴ AC = d = r
3  2 12
https://iit-jeeacademy.blogspot.com

174 KVPY Practice Set 1 Stream : SA

x 5π
If x is even, then| y|≥ x2 + +1 8. (b) We have, ∠ABC =
2 2f (xy) = {f (x)} + {f ( y)}
y x 7
5 2 BC = AC
⇒ y2 ≥ x4 + x3 + x2 + x + 1 + x Putting y = 1
4 ∴ ∠BAC = ∠BCA
5 2 ∴ 2f (x) = f (x) + {f (1)}x
⇒ y2 ≥ y2 + 5π  π
=  π −
x 1
4 f (x) = 2x  =
2 7 7
which is not possible. ∴ f (5) = 25 = 32 and f (3) = 23 = 8
4π 3π
f (5) − f (3) = 32 − 8 = 24 ∴ ∠GAC + ∠GAD′ = + = π
If x ≠ 0, then x = 0 is the only solution 7 7
when x is even. 9. (a) M1 = 12 boys, D1 = 15 days, Hence, CAD′ are collinear.
x 1
If x is odd, then x2 + + is an integer. R1 = Rate of working π
2 x ∠GCA = ∠GD ′ A =
M2 = 9 boys, D2 = 15 days, R2 = Rate of 7
 x 1
So,| y|≥  x +  +
2
working ⇒ ∠CAB = ∠ACB
 2 2
∴ M1 D1 R1 = M2D2R2 ∴ ∆GCD′ ~ ∆BAC
In this case,
⇒ 12 × 15 × R1 = 9 × 15 × R2 GC CD ′ GD ′
 x2 x 3  = =
y2 ≥ x4 + x3 + x2 + x + 1 +  − −  R1 9 3 BA AC BC
 4 2 4 ⇒ = =
R2 12 4 AC CD ′ CA + AD ′
x 2
x 3 ⇒ = =
i.e. y2 ≥ y2 +  − −  R2 4 BA GC AD
 4 2 4 ⇒ =
R1 3 [QGC = GD = AD ]
1
= y2 + (x2 − 2x − 3) (R − R1 ) AC AC + AD
4 Percentage increase = 2 × 100 ⇒ =
1 R1 AB AD
and hence (x2 − 2x − 3) ≤ 0 1 AC + AD 1 1
4 R  ⇒ = = +
x2 − 2x − 3 ≤ 0 x ∈ [−1, 3] =  2 − 1 × 100 AB AC ⋅ AD AC AD
 R1 
∴ There are exactly 3 integer 0, −1and 3 1 1 1
⇒ + = =1 [Q AB = 1]
=  − 1 × 100
4 AC AD AB
for which the expression is perfect square
3 
and sum = 0 − 1 + 3 = 2 12. (c) We have, radius of circle are 3, 2
1
7. (b) Let three-digits number are = × 100 and 1.5, respectively.
3
100a + 10b + c 1 A N B
= 33 %
Given, 100a + 10b + c = 11(a 2 + b2 + c2 ) ...(i) 3 3
M T
(99a + 11b) + (a − b + c) = 11(a 2 + b2 + c2 ) 10. (a) We know, O 3 2
1
99a + 11b is divisible by 11.  x   x  R S
∴a + b + c is must divisible by 11.  K − 1 =  K + 1 when K ≥ 2 2 4
   
Hence, so, a − b + c P Q
K2 − 4
−8 ≤ a − b + c ≤ 18 Then, x = when K is even and In ∆ORS,
4
We conclude a − b + c is either 0 or 11. OR = OP − RP
K2 − 5
Now, putting b = a + c in Eq. (i), we get is when K is odd.
⇒ OR = 3 − 2 = 1[Q RP = SQ = 2]
4
100a + 10(a + c) + c = 11 [a 2 + (a + c)2 + c2 ] ⇒ OS = 3 + 2 = 5
 x   x 
⇒ 2a 2 + (2c − 10) a + 2c2 − c = 0 ∴  100 − 1 =  100 +
   1 ∴ RS 2 = PQ 2 = OS 2 − OR 2
∴The first two terms of this expression are
= (5)2 − (1)2 = 25 − 1 = 24
even third term should be even as well K = 100 which is even.
1002 − 4 ⇒ PQ = 24 = 2 6
⇒ c is even ∴Positive integers of x =
In ∆SMT ,
D = (2c − 10)2 − 4 × 2(2c2 − c) 4
= (25)2 − 1 = 2500 − 1 SM = QM − SQ
= 4(−3c2 − 8c + 25) is a square of c = 0
= 2499 = 4.5 − 2 = 2.5
When c = 0
[QQM = AP − BT ]
∴ 2a 2 − 10a = 0 ⇒ a = 5, a ≠ 0 11. (c) Reflect the heptagon with AG as
b = a + c⇒b = 5 + 0 = 5 an axis to obtain another heptagon ST = 2 + 15
. = 3.5
∴Number are 550 AB ′C ′ D ′ E ′ F ′G ′ MT 2 = ST 2 − SM 2
Now, when b = a + c − 11 F F = (3.5)2 − (2.5)2
E E′
∴2a 2 + (2c − 32a ) + 2c2 − 23c + 131 = 0 G = 12.25 − 6.25 = 6
D = 4(−3c2 + 14c − 16) is square c = 3 MT = 6
Q 2a 2 − 26a + 80 = 0, a = 5, a ≠ 8 D D′
∴ AB = AN + NB
∴ b= a+ c= 5+ 3= 8 A = PQ + MT
∴ Number are 803 C C′ =2 6+ 6=3 6
B B′
∴ Sum = 550 + 803 = 1353
https://iit-jeeacademy.blogspot.com

KVPY Practice Set 1 Stream : SA 175

13. (c) Given, 4 × 4 × 4 cubes is made 64 1


⇒ cos (α + β ) = and cos (α − β ) = 1 21. (a) By momentum conservation,
faces 1 × 1 × 1 cubes. 2 4v = (A − 4)v′
π
Total cubes = 64, White = 20, Red = 44 ∴ α+β= ⇒ v′ =
4v
3 A−4
To find minimum number of visible white 2
1  mgx 
box counting total visible faces of unit 16. (b) dU =   ⋅ A ⋅ dx 22. (c) Heat required to melt ice
2  9A 
cube. = Q1 = mL = 30 × 80 = 240 cal
Total number of faces of small cube on 1 m2 g 2A l
U = ∫0
2 Heat taken by water formed to reach at
x dx
bigger cube except boundary cubes 2 l2A 2Y 100°C, Q2 = ms∆T = 3000 cal
= 4 × 6 = 24 m2 g 2l Heat given by steam on condensation
= .
Counting boundary cubes 6AY = Q3 = mL = 25 × 540 = 13500 cal
= 16 + 8 + 8 = 32 17. (a) Ice melts at lower temperature As heat taken by ice is less than heat
∴Total visible faces = 56 due to increase in pressure. As wire given by steam on condensation. So,
But we have 44 red cube. passes, the water formed is again freezes resulting mixture is at 100°C.
and hence wire passes without cutting
∴Minimum number of white faces cubes Steam condensed
ice. Maximum heat absorbed by ice
which are visible = 56 − 44 = 12 =
18. (a) Conservation of energy gives, Latent heat of vapourization
14. (d) Let x minute will be taken. In
1
one minute A can fill the
1
part of mu 2 = ∆U =
5400
= 10 g
60 2 50
1
tanker and in one minute B can fill the ⇒ mu 2 = nCV ∆T So, resulting mixture contains
1 2 (30 + 10 = 40 g) of water and
part of tanker.
40 mu 2 nMu 2 Mu 2 (25 − 10 = 15 g) of steam at 100°C.
⇒ ∆T = = =
Both can fill in t minute 2nCV 2nCV 2CV
t t 23. (a) For a freely falling body,
+ =1 3 distances travelled in each successive
60 40 ∆TN 2 (MN 2 . CV He ) 14 2 R
∴ = = × second increases in the ratio of successive
⇒ t = 24 min ∆THe (MHe ⋅ CV N ) 4 5
R odd integers.
2
1 2
Both can fill in one minute part of i.e. s1 :s2: s3 : s4 : s5 : s6 : s7 : : 1 : 3 : 5 : 7 : 9 : 11 : 13
24 21 2
= ≈ s5 9 13
tanker 10 1 ⇒ = ⇒ s7 = s5
s7 13 9
1 =     + ⋅  
x 1 x 1 19. (d) We plot process along with
 2   40  2  24  isotherms. 24. (b) For successful rotation,
x 1 1 vbottom = 5 gl
⇒  +  =1 p
2  40 24  T1 vtop = gl
x  3 + 5 120 × 2 T2 These values are minimum possible
⇒   = 1⇒ x = T3
2  120  8 values.
⇒ x = 30 min T4 So, ratio of kinetic energies,
1 2
15. (c) We have, mvbottom
K bottom 2
3 = = 5 :1
⇒ cos2 α + cos2 β = V K top 1 2
mvtop
2 2
1 + cos 2α 1 + cos 2β 3
⇒ + = Clearly, temperature initially increases 25. (a) Wooden block sinks, when
2 2 2 then decreases. Density of wood = Density of benzene
⇒ cos 2α + cos 2β = 1 20. (c) Linear momentum is conserved in ρw ρb
⇒ =
⇒ 2 cos (α + β ) cos (α − β ) = 1 the horizontal direction. 1 + rw ∆T 1 + rb ∆T
1 ⇒ mv0 sin 37° + 0 = mV − mv sin 37°
⇒ cos (α + β ) cos (α − β ) = ...(i) ρb − ρw
2 ⇒ ∆T = = 21.7°C
Along common normal, ρw rw − ρb rb
1
and sin α sin β = e(v0 − 0) = V sin 37° + v
4 So, at 22°C wood block sinks in benzene.
27 11
1 ⇒ V = v0 and v = v0 26. (c) m = k ρa gb vc
⇒ 2 sin α sin β = 34 34
2
So, impulsive tension ⇒ [M] = [Ma L−3 a + b + c T −2b − c ]
1
⇒ cos (α − β ) − cos (α + β ) = ...(ii) = mv0 cos 37° + mv cos 37° ⇒ a = 1, b = − 3, c = 6
2
∴ m ∝ v6
= mv0   + mv0 
4 11 4 
From Eqs. (i) and (ii), we get × 
 5  34 5  27. (b) After a long time, terminal
cos (α + β ) cos (α − β ) − cos (α − β )
velocity is attained to kinetic energy is
= mv0  
18
+ cos (α + β ) = 0 constant and only potential energy is
 17 
dissipated at a constant rate.
https://iit-jeeacademy.blogspot.com

176 KVPY Practice Set 1 Stream : SA

28. (d) 2wx = w − x ⇒ x =


l l 35. (c) If the specie is a reducing agent, 39. (a) 1 mole of Al requires
2  6 it means it can be oxidised easily thus it = 3 × 96500 C
should have an oxidation number less
D 10−3 moles of Al requires
than the maximum values of oxidation
C
number. Oxidation number of S in the = 3 × 96500 × 10−3 C
B
A given species are = 3 × 96.5 C [1C = As]
x
(i) SO2 = 3 × 9.65 As
x + 2(−2) = 0 3 × 96.5
w 2w Time (s) = As = 30 s
l
— –x x=+ 4 9.65 A
2
(ii) SO32− 40. (c) The IUPAC name of the following
29. (d) As total energy of system (U + K )
x + 3(−2) = − 2 compound is
is negative for all values of r, system is a
bound system. x− 6= − 2 5 3
2 1
30. (c) From ray diagram, x=+ 4 6 4
(iii) H 2 SO4
A 2(1) + x + 4(−2) = 0 3, 4, 4-trimethylhex-1-yne .
2+ x− 8= 0 41. (a) Aldehydes or ketones having
atleast one α −H atom undergo aldol
x=+ 6
condensation. The structures of given
d S (iv) S2− compounds are as follows:
x=−2
(a) HCH (c) H3C—CH—CH
As the maximum value of oxidation
number of S is − 2. Thus, H 2SO4 can not O CH3 O
B act as a reducing agent. Methanol 2-methyl pentanal
36. (d) The wavelength of light is related (No α-H atom) (1-α-H atom)
Distance, AB = d + d + d = 3d hc
to its energy by the equation E = O
∴ Distance required = 3 m. λ Ph
31. (d) Lattice energy is defined as the Given, λ = 45 nm
amount of energy required to completely = 45 × 10−9 m (b) (d) CH2—C—CH3
seperate one mole of a solid ionic [Q 1 nm = 10−9 m]
compound into gaseous constituent ions. Cyclohexanone O
6.63 × 10−34 Js × 3 × 108 ms−1 (4-α-H atoms)
It is directly proportional to the charge of Hence, E = 1-phenyl propanone
the ions. Thus, greater the ionic charge, 45 × 10−9 m (5-α-H atoms)

larger is the lattice energy. = 4.42 × 10−18 J Thus, among the given compounds,
Hence, correct order is 37. (a) N2 + 3H2 → 2NH3 methanal has no α-H atom.
Al 2O3 > CaO > MgBr2 > NaCl 6 g of hydrogen produces 34 g of NH3 Hence, it will not give aldol condensation.
32. (a) One mole of ideal gas at STP ∴450 g of hydrogen produces 42. (b) Compound II is most basic among
(22.4 L) contains 6.022 × 1023 atoms 34 the given compounds. This is because the
= × 450 = 2550 g of NH3 lone pair present on nitrogen in pyridine
i.e., 6.022 × 1023 atoms are present in 6
22400 mL. does not take part in delocalisation and
Actual ammonia produced in the solution
hence they are available for donation.
∴ Average volume per molecule = 1575 g Whereas, in compound I and III the lone
22400
= cm3 ∴ % yield =
1575
× 100 pair on N atom takes part in resonance
6.022 × 1023 2550 and will not be available for donation, so
= 3.72 × 10−20 cm3 = 61.76 % ≈ 61.8% their basicity will be less.
33. (c) Intensive properties are those 38. (d) The possible isomer of C2BrClFI 43. (a) In Clemmensen reduction,
properties which do not depend upon the are as follows ketones are reduced to alkanes with the
quantity or size of matter. Among the help of Zn-Hg in the presence of HCl.
Br Cl Br F
given quantities, boiling point (I), pH (III) For example,
(i) C==C (ii) C==C
and emf (IV) are intensive properties I F Cl I
whereas entropy (II) is an extensive
H3C Zn-Hg H3C
C==O CH2
property. Br Cl Br F H3C HCl
H3C
34. (a) Number of radial nodes (iii) C==C (iv) C==C
= n − l−1 F I I Cl 44. (b) Higher the reduction potential of
For 3 s-orbital a metal, lesser will its reducing power.
n = 3, l = 0 Br I Br I As the reduction potentials of a metal is
(v) C==C (vi) C==C
∴ Number of radial nodes = 3 − 0 − 1= 2 decreasing in the order Y > X > Z, thus
Cl F F Cl the reducing power will decrease in the
For 2p-orbital,n = 2, l = 1 Therefore, the above compound has
order Z > X > Y.
Number of radial nodes = 2 − 1 − 1= 0 6 isomers.
https://iit-jeeacademy.blogspot.com

KVPY Practice Set 1 Stream : SA 177

45. (b) When white phosphorus is heated 53. (b) Water potential (ψw ) is actually Y
with caustic soda , then sodium determined by taking into account factors
hypophosphite with phosphine is formed. like osmotic (or solute) potential (ψs ) ,
4P + 3NaOH + 3H2O → pressure potential (ψ p ) and matrix or
White Caustic capillary potential (ψm ). The formula for 60°
phosphorus soda X′ X
NaH2PO2 + PH3 calculating water potential is O 15° (2,0) (3,0)
Sodium Phosphine ψw = ψs + ψ p + ψm
hypophosphite
54. (c) Hydrogen bonding in DNA occurs
46. (c) The filtration of blood at the
between complementary bases in order to Y′
glomerulus is a pressure dependent
keep the two strands of DNA helix b
physical process known as renal Let = tan θ
together. These bonds occur as 2 a
ultrafiltration. The force of hydrostatic 2
hydrogen bonds between adenine and  b  − 4b + 1 ≤ 0
pressure in the glomerulus (the force of  
thymine and 3 hydrogen bonds between  a a
pressure exerted from the pressure of the
blood vessel itself) is the driving force cytosine and guanine. ⇒ tan 2 θ − 4 tan θ + 1 ≤ 0
that pushes filtrate out of the capillaries 55. (b) Both sympathetic and ⇒ tan θ ∈ [2 − 3 , 2 + 3 ]
and into the slits in the nephron. parasympathetic preganglionic neurons ⇒ θ ∈ [15° , 75° ]
47. (b) Grave’s disease is an immune are cholinergic meaning they release π
Area of region = (32 − 22 )
system disorder that results in the Acetylcholine (Ach) at the synapse in the 3
overproduction of thyroid hormones ganglion. Ach basic functions involve the π 5π
= (9 − 4) =
(hyperthyroidism). Its symptoms include control of skeletal muscles via activation 3 3
anxiety, irritability, tremor, heat of the motor neurons as well as
sensitivity, enlargement of thyroid gland, 62. (b) Given,
stimulating the muscles of the body.
change in menstrual cycle, etc. S1 (n ) = 8 + 12 + 16 + K + n terms
56. (b) Evolution is not goal oriented. n n
48. (b) The ‘imperfect fungi’ belongs to Evolution simply depends on the S1 (n ) = [16 + (n − 1) 4] = (12 + 4n )
class Deuteromycetes. They are called as 2 2
environment, which the organisms live
imperfect because sexual reproduction is and S2 (n ) = 17 + 19 + 21 + K+ n terms
and try to survive. The environment is fit
absent in these forms. They reproduce n n
for strongest individual who can survive S2 (n ) = [34 + (n − 1) 2] = (32 + 2n )
only by asexual spores called conidia. 2 2
and reproduce. Evolution uses the theory
49. (b) Presence of amniotic egg is not a of natural selection where there is S1 (n ) = S2 (n )
characteristic of phylum Chordata. The n n
variation. We have variations of traits, Q (12 + 4n ) = (32 + 2n ) ⇒ n = 10
four features shared by all chordates are heredity and different reproductive 2 2
presence of a single notochord, a dorsal strategies as a result of natural selection. Q S1 (10) = 5(16 + 36) = 260 = S2 (10)
hollow nerve cord, pharyngeal slits and a
57. (a) The sucrose is actively Q Common sum = 260
post-anal tail. Amniotic eggs are present
in reptiles, birds and mammals only. transported against its concentration 63. (c) If any statement is true, then
gradient into the phloem cells using the remaining 2 are false.
50. (c) TCA cycle or Citric acid cycle is a
electrochemical potential of the proton 64. (b) Given, ∆ABC is right angled at A.
series of reactions that produces one GTP
gradient. This is coupled to the uptake of A is centre of circle and AB is radius of
or ATP as well as three NADH molecules
and one FADH2 molecule in each turn, sucrose with a carrier protein called the circle.
which will be used in further steps of sucrose-H+ symporter. BD = 20
cellular respiration to produce ATP for 58. (a) A secretory cell would need CD = 16
the cell. secretory enzymes and glycoproteins
C
51. (c) During anaphase-I, no crossing required in secretions, which are
over leads to segregation of alleles. produced in Golgi bodies. 16
Anaphase-I begins when the two 59. (a) Riboluse 1,5-Biphosphate (RuBP) E
D
chromosomes of each bivalent (tetrad) is the first acceptor of CO2 in the r 20
separate and start moving toward formation of two molecules of
opposite poles of the cell as a result of the 3-Phosphoglyceraldehyde (PGA) during A B
r
action of the spindle, but their the Calvin cycle of photosynthesis. r
centromeres are still attached.
60. (a) Horizontal stem of creepers often
52. (d) After vigorous exercise, excess develop adventitious roots from the
F
oxygen is required by the body to make In ∆ABC, AC 2 + r 2 = BC 2
nodes. Adventitious roots are the roots
ATP for gluconeogenesis, to metabolise
which arise from an organ other than a AC 2 + r 2 = (36)2 ...(i)
lactic acid, to replenish phosphocreatine
root. They generally develop from stem CB and CF are secant of circle.
and glycogen and to pay back any oxygen
that has been borrowed from nodes, internodes, leaves, etc. Q CE − CF = CD × CB
haemoglobin. 61. (d) Given, 4 ≤ a 2 + b2 ≤ 9 ⇒ (AC − r ) (AC + r ) = 16 × 36
and b2 − 4ab + a 2 ≤ 0 AC 2 − r 2 = 16 × 36 ...(ii)
https://iit-jeeacademy.blogspot.com

178 KVPY Practice Set 1 Stream : SA

From Eqs. (i) and (ii), we get Time at which bullet and block collide is 72. (c) 16.9% solution of AgNO3 means
d 100
2AC 2 = 36 (36 + 16) = 36 × 52 t= = = 1s 16.9 g AgNO3 is present in 100 mL
v 100 − 0
⇒ AC = 6 26 solution.
65. (a) We have, Speed of wood just before collision ∴ 8.45 g AgNO3 will be present in 50 mL
= gt = 100 ms−1 solution.
(1 + 2x + 3x + K + 21x )
2 20 2
Speed of bullet at t = 1s is Similarly, 5.8 g NaCl is present in
= (1 + 2x + 3x2 + K + 21x20 )
= v − gt = 100 − 10 × 1 = 90 ms−1 . 100 mL solution.
(21x20 + 20x19 + K + 3x 2 + 2x + 1)
Let v′ = velocity of bullet + block system ∴2.9 g NaCl is present in 50 mL solution.
Coefficient of x30 is
after collision. Then, by momentum
11 × 21 + 12 × 20 + ... + 21 × 11 AgNO 3 + NaCl → AgCl + NaNO 3
conservation, we have
Initial 8.45 2.9 0 0
= 2(11 × 21 + 12 × 20 + 13 × 19 + 14 × 18 − (0.03) × 10 + (0.02) × 90 = (0.05) v′ moles 169.8 58.5
+ 15 × 17) + 16 × 16 ⇒ v′ = 30 ms−1 = 0.049 = 0.049
= 2(231 + 240 + 247 + 252 + 255) + 256 Now, maximum height reached After
= 2(1225) + 256 = 2450 + 256 = 2706 v2 reaction 0 0 0.049 0.049
= − distance through
66. (b) Given situation is 2g Mass of compound = moles × molar mass
which block fells in 1 s
 30 × 30   1 ∴ Mass of AgCl precipitated
2
=  −  × 10 × 1  = 45 − 5 = 0.049 × 143.5 = 7.03 g
B′ A′ A B  2 × 10   2 
73. (d) The species formed after removal
C = 40 m of an electron from the given diatomic
F
70. (b) M = kF a LbT c molecules are as follows
where, k is a constant. (a) C2 → C2+ + e−
By mirror formula, [M] = [MLT −2 ]a Lb T c (b) CN → CN+ + e−
1

1 1
= − ⇒ vB = −
5f Equating dimensions, we have (c) N2 → N+2 + e−
5
vB f f 2 a=1 …(i) (d) O2 → O+2 + e−
3
b+ a = 2 …(ii) The stability of diatomic molecule can be
Hence, image length is l′ = f  − 2 =
5 f
2  2 −2a + c = 0 …(iii) determined by calculating its bond order.
Putting value of a from Eq. (i) in Eq. (ii), More is the bond order, more is the
l′ f / 2 3
So, magnification is m = = = we get stability of a molecule.
l f /3 2 1
b+ a = 0 B.O = (Nb − N a )
67. (b) Coordinates of points P and Q are 2
b+ 1= 0
∴ The bond orders of diatomic molecules
P :  , 0,  and Q :  , , 0
a a a a
b = −1 …(iv)
2 2   2 2  with their ionic species are given below.
Again putting value of a from Eq. (i) in
So, unit vector along PQ is B.O
Eq. (iii), we get
a a$
− $j + k −2a + c = 0
(i) C2 2.0
^ PQ 2 2 C+2
PQ = = −2 × 1 + c = 0
(ii) 1.5
|PQ| a2 a2 (iii) CN 2.5
+ −2 + c = 0
4 4 (iv) CN+ 2.0
$ $ c=2
= 2 (− j + k ) (v) N2 3
⇒ a = 1, b = −1 and c = 2
10 (vi) N2+ 2.5
68. (a) By KVL, I1 = So, [M] = [FL−1 T 2 ]
20 (vii) O2 2
100 71. (a) From ideal gas equation, (viii) O 2+ 2.5
⇒ P1 = I12R1 = × 20 = 5 W
400 pV = nRT
As the bond order increases from 2 to 2.5
I 2 = 0 ⇒ P2 = 0 pV 1. 01 × 105 N - m −2 × 10 × 10−3 m3
and n= = when an e− is removed from O2 molecule
RT 8. 314 J mol −1 K −1 × 300 K (O+2 ), so it become stabilised.
69. (b)
= 0.405 mol [1N - m =1 J] 74. (a) Calcium carbide on hydrolysis
u=0 5
For monoatomic gases, C p = R = 2 .5R gives acetylene as a major product which
2 then reacts with dil H2SO4 in the
T1 = 300 K and T2 = 400 K presence of HgSO4 to give acetaldehyde.
R = 8. 314 J mol −1 K −1 CaC2 + 2H2O → C2H2 + Ca(OH)2
100 m T Calcium (X)
∆ S = 2. 303 nC p log 2 carbide Acetylene
T1
CH CH3  C  H
400 dil H 2 SO4 / HgSO4
   
= 2.303 × 0.405 × 2. 5 × 8.314 log →
v=100 ms–1 300 CH O
∆S = 2. 422 JK −1
(Y )
Acetaldehyde
https://iit-jeeacademy.blogspot.com

KVPY Practice Set 1 Stream : SA 179

75. (a) As only one structure of B with 76. (d) ABA or Abscisic Acid promotes water is neither acidic nor basic, but
molecular formula C5 H11 Cl is possible, leaf senescence independent of ethylene. neutral. Therefore, water at 25°C will
thus the structure of B would be Other statements can be corrected as have H+ = OH− .
CH3 Gibberellins play the most important role 79. (c) Acoelomate Animals like
 in flowering. It is seen that treatment of sponges, coelenterates and flatworms are
CH3  C  CH2 Cl. gibberellin on biennials or long day without a coelom or any other internal
 plants, stem elongation occurs before cavity except the digestive tract.
CH3 flower primordia are formed. Auxin and Pseudocoelomate Roundworms have a
CH3 cytokinin promote cell division. body cavity derived directly from the
 It is seen that ABA and ethylene can blastocoel of the embryo. It is called
CH3  C  CH3 + Cl 2 ←
Light
 control and induce root and shoot growth
 pseudocoel because like true coelom, it is
under water stress or low water not lined by peritoneum but is bounded
CH3
( A) potential. with ectoderm on the outer side and
CH3 77. (b) The cross for the given question endoderm on the inner side.
 would be Coelomate Animals with a tube within a
CH3  C  CH2 Cl XhcX × X Y tube body plan have a fluid-filled body
 cavity between the body wall and the
CH3
(B ) digestive tract as like in earthworms. It
XhcX XhcY XX XY is derived from embryonic mesoderm and
Wurtz
reaction  Na / ether Colourblind Colourblind Normal Normal is lined by peritoneum.
↓ haemophilic haemophilic girl boy
CH3 CH3 carrier girl boy 80. (d) A dipeptide is made up of 2
  ∴ The progeny be 50% haemophilic amino acids which may be same or
CH3  C  CH2  CH2  C  CH3 colourblind sons and 50% colourblind different. The total possible number of
  carrier daughters. different dipeptides that may be
CH3 CH3 assembled from 20 different amino acids
78. (d) In pure water, the concentration
(C) will thus be
of hydrogen ions and concentration of
hydroxyl ions are equal. Consequently 20n = 202 = 20 × 20 = 400
https://iit-jeeacademy.blogspot.com

180 KVPY Practice Set 2 Stream : SA

KVPY
KISHORE VAIGYANIK PROTSAHAN YOJANA

PRACTICE SET 2
Stream : SA
MM 100

Instructions
There are 80 questions in this paper.
This question paper contains two parts; Part I and Part II. There are four sections; Mathematics, Physics, Chemistry
and Biology in each part.
Out of the four options given with each question, only one is correct.

PART-I (1 Mark Questions)


MATHEMATICS 4. Difference between the corresponding roots
99 x2 + ax + b = 0 and x2 + bx + a = 0 is same and a ≠ b,
1. The remainder when 5 is divided by 13
then
(a) 6 (b) 8
(c) 9 (d) 10 (a) a + b + 4 = 0 (b) a + b − 4 = 0
(c) a − b − 4 = 0 (d) a − b + 4 = 0
2. A polynomial p(x) when divided by x2 − 3x + 2 leaves
5. Let P be a point in the interior of the rectangle
remainder 2x − 3, then
ABCD, which of the following sets of numbers can
(a) p(x) must have a root between 0 and 3 form the areas of the four triangles PAB, PBC, PCD,
(b) p (x) cannot have a root between 0 and 3 PDA in same order
(c) p (x) must have a real root but may or may not be (a) 10, 9, 12, 5 (b) 21, 15, 6, 12
between 0 and 3 (c) 10, 9, 8, 6 (d) 12, 8, 7, 5
(d) p (x) need not have a real root
6. Let x1 , x2 , K , xn be n observation such that
3. A shopkeeper increases the price of a commodity by n n
x% some time later, he reduces the new price by y% ∑ xi2 = 400 and ∑ xi = 80. Then, a possible value of
i =1 i =1
and notices that the price is now the same as it was
1 1 n among the following is
originally. The value of − is
y x (a) 15 (b) 18 (c) 9 (d) 12

1 7. The set S = {1, 2, 3, K , 12} is to be partitioned into


(a) − (b) 0
100 three sets A, B, C of equal size. Thus, A ∪ B ∪ C = S,
1 A ∩ B = B ∩ C = A ∩ C = φ. The number of ways of
(c) (d) None of these partition S is
100
12! 12! 12! 12!
(a) (b) (c) (d)
3! (4!)3 3! (3!)4 (4!)3 (3!)4
https://iit-jeeacademy.blogspot.com

KVPY Practice Set 2 Stream : SA 181

8. You have a measuring cup with capacity 25 ml and


another with capacity 110 ml, the cups have no
PHYSICS
markings showing intermediate volumes. Using large 16. A box is falling freely inside the box, a particle is
container and as much tap water as you wish. What projected with some velocity v with respect to the box
is the smallest amount of water you can measure at angle θ.
accurately?
(a) 1 ml (b) 5 ml
(c) 10 ml (d) 25 ml
v
9. Let A, B, C, D be collinear points in that order.
Suppose AB : CD = 3 : 2 and BC : AD = 1 : 5. Then,
AC : BD is θ
(a) 1 : 1 (b) 11 : 10 For an observer sitting in the box, path of particle is
(c) 16 : 1 (d) 17 : 13
(a) (b)
10. Let ABC be triangle with AB = AC = 6. If the
circumradius of the triangle is 5, then BC equals
25 (c)
(a) (b) 9 (d)
3
48
(c) (d) 10
5 17. Potential energy of a system as a function of a
11. Two cars start together in the same direction from parameter x is U (x) = (x + 1) (x + 2).
the same place. The first goes with a speed of Then, consider following statements:
10 km/h. The second goes at a speed of 8 km/h in the −3
1 I. Point x = , corresponds to an equilibrium
first hour and increase the speed by km each 2
2 position.
succeeding hour.
II. Points x = − 1 and x = − 2, corresponds to
After how many hours will the second car overtake equilibrium position of system.
the first, if both go non-stop. −3
(a) 9 h (b) 5 h III. At point x = , system is in stable equilibrium.
2
(c) 7 h (d) 8 h
−3
IV. At point x = , system is in unstable
12. If x, y are natural numbers such that x2 + 2013 = y2, 2
then the minimum value of xy is equilibrium.
(a) 645 (b) 658 (a) All statements are correct
(c) 668 (d) 671 (b) Statements I and IV are correct
13. A cube has each edge 2 cm and a cuboid is 1 cm long, (c) Statements I and III are correct
2 cm wide and 3 cm high. The paint in a certain (d) Only statement II is correct
container is sufficient to paint an area equal to 18. On a temperature scale X, water boils at − 60° X and
54 cm 2. Which one of the following is true? freezes at − 180° X. What would be a room
(a) Both cube and cuboid are painted temperature of 25° C on X-scale?
(b) Only cube can be painted (a) − 18° X (b) − 38° X (c) − 150° X (d) − 130° X
(c) Only cuboid can be painted
19. In given nuclear reaction,
(d) Neither cube nor cuboid can be painted 9
4 Be +α→ 12
6C + X, particle X is
14. How many positive real number x are there such that 0
(a) 42 He (b) −1 e (c) 11 H (d) 10 n
(x)x x
= (x x )x ?
20. Two blocks of masses 0.2 kg and 0.5 kg are placed
(a) 1 (b) 2 22 m apart on a rough flat horizontal surface (µ = 05
. ).
(c) 4 (d) Infinite At time t = 0, blocks are pushed towards each other
15. Let 0 < a < b < c be three distinct digits. The sum of with equal forces of 3 N on each of the block.
all 3-digit number formed by using all the 3-digit Blocks collide with each other in time duration
number once each is 1554. The value of b is (a) 1 s (b) 2 s
(a) 1 (b) 2 (c) 3 (d) 4 (c) 3 s (d) 2 s
https://iit-jeeacademy.blogspot.com

182 KVPY Practice Set 2 Stream : SA

21. A table-tennis ball is floating in air by a jet of water (a) 1 (b) 0.5
emerging from a nozzle. (c) 0.25 (d) 1.25
26. If a low pressure centre is developed in atmosphere
(very common in India in summer), the wind will flow
radially towards centre. The whirlpool of wind
formed will rotate in India as
h
(a) clockwise only
Ground (b) anti-clockwise only
level (c) clockwise or anti-clockwise
(d) whirlpools are not formed in India
27. Which of these paths correctly describes motion of
moon around observed from a space station?
If mass of ball is m and water stream rises to height h
above ground, then water flow rate is
m g m h
(a) (b) Moon
Sun Moon
2 h 2 g (a) Sun (b)

2h
(c) m 2 gh (d) m
g
Moon
22. A system under goes three processes listed in table (c)
below. All quantities are (in kJ). (d)

Process ∆Q ∆W ∆U Sun
Sun
Process 1 → 2 a 100 100
Process 2 → 3 b − 50 c
Process 3 → 1 100 d − 200 28. Correct variation of velocity of a table-tennis ball
dropped from top of a 14-story building is
Then, value of c is v
(a) v (b)
(a) 200 kJ (b) 50 kJ
(c) 100 kJ (d) 0 kJ
23. Force necessary to accelerate a mass of 1 kg at
t
10 ms−2 vertically upwards is (Take, g = 10 ms−2) t

(a) 1 N (b) 2 N (c) 10 N (d) 20 N (c) v v


24. In Rutherford’s scattering experiment, choose the (d)
correct statements are given below.
I. Only α-particles are scattered backwards but t
t
not protons.
II. α-particles cannot be effectively scattered by 29. A vessel with water is placed on a weighing pan, it
electrons because α-particles are positively reads 600 g. Now a hollow ball of mass 40 g and
charged. volume 50 cm3 is kept immersed in water by tying it
III. Radius nucleus of target is between to bottom with a thread of negligible mass.
. × 10−15 m to 48
96 . × 10−15 m.
IV. α-particles with energy greater than a certain
critical value are not scattered back.
(a) Only statement II is correct 500 g
(b) Only statement III is correct
(c) Statements I, III and IV are correct
(d) All statements are correct Reading of pan is now
(a) 690 g
25. The ratio of the height above the surface of earth to
(b) 550 g
the depth below the surface of earth for gravitational (c) 650 g
acceleration to be same (assuming small height) is (d) 610 g
https://iit-jeeacademy.blogspot.com

KVPY Practice Set 2 Stream : SA 183

30. Plane face of a plano-convex lens is silvered. Given, (b) 3, 5-dimethyl-1-cyclohexene


radius of convex face is 12 cm and refractive index of (c) 1, 5-dimethyl-5-cyclohexene
medium is 3 / 2 . (d) 1, 3-dimethyl-5-cyclohexene
36. Which of the following isomerisms is shown by
pentan-2-one and 3-methylbutanone?
(a) Stereoisomerism (b) Position isomerism
(c) Functional isomerism (d) Chain isomerism
37. What is the maximum number of orbitals that can be
Power of resulting system is identified with the following quantum numbers?
25 25
(a) D (b) − D n = 3, l = 1and ml = 0
3 3 (a) 1 (b) 2
− 25 25 (c) 3 (d) 4
(c) m (d) m
3 3
38. The major product of the following reaction is
(i) O
CH 3CH== CHCH2CH3  
3
→
( ii ) hydrolysis, Zn
CHEMISTRY (a) CH3 CHO + CH3 CH2CHO
31. By heating 10 g CaCO3 , 5.6 g CaO is formed. The (b) CH3 COOH + CH3 COCH3
weight of CO 2 obtained in this reaction is closest to (c) CH3 COOH + CH3 CH2COOH
(a) 5.6 g (b) 2.4 g (d) CH3 COOH + CO2
(c) 4.4 g (d) 3.6 g
39. Among the following, which is an incorrect statement.
32. Which of the following plot obeys the Raoult’s law at (a) PH5 and BiCl5 do not exist
all concentration? (b) pπ − dπ bonds are present in SO2
(c) SeF4 and CH4 have same shape
Vapour pressure

Vapour pressure

(d) I3+ has bent structure

(a) (b) 40. Predict the effect of increased pressure on the


following reaction equilibrium,
2SO2 ( g ) + O2 ( g ) - 2SO3 ( g )

Mole fraction of solvent Mole fraction of solvent (a) equilibrium shift to the right
(b) equilibrium shift to the left
Vapour pressure

Vapour pressure

(c) no effect on equilibrium


(d) reaction stops
(c) (d) 41. The solubility product of BaCl2 is 4 × 10−9. Its
solubility in mol L−1 is
(a) 4 × 10−3 (b) 4 × 10−9
Mole fraction of solvent Mole fraction of solvent (c) 1 × 10−3 (d) 1 × 10−9

33. The correct order of increasing ionic character is 42. Chlorobenzene on treatment with sodium in dry
ether gives diphenyl. The name of the reaction is
(a) BeCl 2 < MgCl 2 < CaCl 2 < BaCl 2
(a) Fittig reaction
(b) BeCl 2 < MgCl 2 < BaCl 2 < CaCl 2
(b) Wurtz-fittig reaction
(c) BeCl 2 < BaCl 2 < MgCl 2 < CaCl 2
(c) Sandmeyer reaction
(d) BaCl 2 < CaCl 2 < MgCl 2 < BeCl 2
(d) Gattermann reaction
34. In the reaction,
43. A sample of unknown gas is placed in a 2.5 L bulb at
3Br2 + 6CO32− + 3H2O → 5Br− + BrO3− + 6HCO3−
a pressure of 360 torr and at a temperature of 22.5°C
(a) Bromine is oxidised and the carbonate radical is reduced
and is found to weight 1.6616 g. The molecular
(b) Bromine is reduced and the carbonate radical is oxidised
weight of the gas is closest to
(c) Bromine is neither reduced nor oxidised
(a) 80 g (b) 55 g
(d) Bromine is both reduced and oxidised
(c) 34 g (d) 55 g
35. IUPAC name of the following compound
44. Consider the isoelectronic ions
K + ,S2− ,Cl− and Ca 2+
is The radii of these ionic species follow the order
CH3 CH3 (a) Ca 2+ > K+ > Cl − > S2− (b) Cl − > S2− > K+ > Ca 2+
2− −
(c) S > Cl > K > Ca
+ 2+
(d) K+ > Ca 2+ > S2− > Cl −
(a) 3, 5-dimethyl cyclohexene
https://iit-jeeacademy.blogspot.com

184 KVPY Practice Set 2 Stream : SA

45. The reaction of toluene with Cl2 in the presence of x x


(a) (b) (c) x (d) 2x
FeCl3 gives predominantly 4 2
(a) benzoyl chloride (b) benzyl chloride 53. Kreb’s cycle was discovered by Krebs in pigeon
(c) o and p-chlorotoluene (d) m-chlorotoluene muscles in 1940. Which step is called gateway
step/link reaction/transition reaction in respiration?
(a) Glycolysis
BIOLOGY (b) Formation of acetyl Co-A
(c) Citric acid formation
46. Most common type of phospholipids in the cell
(d) ETS terminal oxidation
membrane of nerve cell is
(a) phosphatidylcholine (b) phosphatidylinositol 54. Which homeostatic function of the liver is controlled
(c) phosphatidylserine (d) sphingomyelin and monitored in the pancreas?
(a) Deamination of amino acids
47. Graft rejection does not involve
(b) Release of glucose
(a) erythrocytes
(c) Release of iron
(b) T-cells
(d) Release of toxins
(c) macrophages
(d) polymorphonuclear leukocytes 55. During generation of an action potential,
depolarisation is due to
48. Horse-shoe crabs belong to the group
(a) K+ efflux (b) Na + efflux
(a) Onychophora (b) Chelicerata
(c) Na + influx (d) K+ influx
(c) Uniramia (d) Crustacea
56. If liver from body is removed then which component
49. The first living being on the earth were anaerobic
of blood increases?
because
(a) Ammonia (b) Protein
(a) there was no oxygen in air
(c) Urea (d) Uric acid
(b) oxygen damages proteins
(c) oxygen interferes with action of ribozymes 57. In his classical experiments on pea plants, Mendel
(d) they evolved in deep sea did not use
(a) seed shape (b) flower position
50. The presence of Salmonella in tap water is indicative
(c) seed colour (d) pod length
of contamination with
(a) industrial effluents (b) human excreta 58. In phylum, which group contains the greatest
(c) agricultural waste (d) None of these number of species?
(a) Class (b) Family (c) Genus (d) Order
51. The secondary order of protein structure is
(a) the sequence of amino acids in the polypeptide chain 59. Cell division is initiated by
(b) the formation of peptide bonds between amino acids (a) centrosome (b) centromere
(c) the coiling of the polypeptide chain (c) centriole (d) None of these
(d) the folding of the coiled polypeptide chain 60. The part of human hindbrain that is responsible for
hand-eye coordination is
52. The amount of DNA in a mammalian cell in early
(a) cerebellum (b) pons Varolii
prophase-I is x. What is the amount of DNA in the
(c) medulla oblongata (d) thalamus
same cell in anaphase-I of mitosis?

PART-II (2 Marks Questions)


MATHEMATICS 63. If there are three different kinds of mangoes for sale
in a market. Then, number of ways of purchase of
61. A circle is inscribed in an equilateral triangle with
side length of 6 units. Another circle is drawn inside 25 mangoes are
the triangle (but outside the first circle), tangent to (a) 2925 (b) 325
the first circle and two of the sides of the triangle. (c) 351 (d) 2600
The radius of the smaller circle is 64. Four natural number m, n if
1 2 1
(a) (b) (c) (d) 1 (1 − y)m (1 + y)n = 1 + a1 y + a 2 y2 + K and a1 = a 2 = 10,
3 3 2
then (m, n ) is
62. If x2 y3 = 6. Then, the minimum value of 3x + 4 y for
(a) (20, 45) (b) (35, 20)
positive values of x and y is
(c) (45, 35) (d) (35, 45)
(a) 6 (b) 8 (c) 10 (d) 12
https://iit-jeeacademy.blogspot.com

KVPY Practice Set 2 Stream : SA 185

65. In a ∆ABC, with ∠A = 90°, the bisector of the angle When a very distant object is viewed by this lens
B and C meet at P. The distance from P to the system, choose the correct option.
hypotenuse is 4 2. The distance AP is (a) Final image is formed at mid-point of lens separation
(a) 8 (b) 4 (c) 8 2 (d) 4 2 (b) Final image is virtual
(c) Diverging lens increases the magnification five times
PHYSICS (d) Final image is inverted and diminished
66. A planet contains a single type of gas in its 70. Current is flowing through a uniform thick rod of
atmosphere having molecular mass of 138. × 10−28 kg. cross-sectional area A, under an applied potential
Distribution of speeds in atmosphere is given below. difference V across its ends.
Speed (ms−1) Percentage of molecules A vd
100 10
200 30
500 20
+ –
800 20
1000 20 V

Escape speed for the planet is 900 ms . Assuming −1 Let electrons flow through the thick rod with velocity
stable atmospheric conditions, the possible estimated vd$ . A hole is drilled in the rod and its central portion
1 A
reduction in temperature of the planet in few years of area is removed.
2
will be (Use, temperature,
2 Let electrons flow through the hollow rod with
mvrms
T= . × 10−23 )
and K B = 138 velocity vd$ . Then, ratio vd$ / vd$ will be
3 KB 2 2 1

1
(a) 100 K (b) 200 K (c) 70 K (d) 20 K (a) 2 (b) (c) 1 (d) 4
2
67. A right angle ruler used generally in tailoring or
drafting hangs from rest from a peg P as shown
below.
CHEMISTRY
P 71. A bomb calorimeter has a heat capacity of 783 J°C −1
θ and contains 254 g of water, which has a specific heat
of 4.184 g −1 °C −1. Heat absorbed/evolved by a
reaction when the temperature changes from 23.73°C
2L

L
. °C is closest to
to 2601
(a) 1.78 kJ absorbed (b) 2.42 kJ absorbed
(c) 1.78 kJ evolved (d) 4.21 kJ absorbed
72. Isostructural species are those species which have
One arm is L cm long and other arm is 2 L cm long.
the same shape and hybridisation. Among the given
Value of angle θ is such that
1 1 species, identify the isostructural pairs.
(a) tanθ = (b) sinθ = (a) NF3 and BF3 (b) BF4− and NH+4
4 4
1 1 (c) BCl3 and BrCl3 (d) NH3 and NO3−
(c) cos θ = (d) sec θ =
4 4 73. Which among the following will form geometrical
68. 1 kg of steam at 100°C and 101 kPa occupies 1.68 m3 isomers?
space. What per cent of heat of vaporisation of water
is used for expansion of water into steam? (a) (b) (c) (d)
(a) Nearly 7% (b) Nearly 17%
(c) Nearly 70% (d) Nearly 12% 74. Consider the following reaction,
69. A telephoto lens system consists of a converging lens CH
( f = + 60
. cm) placed 4 cm in front of a diverging lens 
(CH 3CH 2O− Na + )
CH 3CH 2OH
B ← CH3 C  Br  → A
(f = − 2.5 cm). (Major) (CH 3CH 2OH)
 (Major)

CH
4.0 A and B respectively are
cm (a) (CH3 )3 COCH2CH3 in both cases
(b) (CH3 )2 C == CH2 in both cases
(c) (CH3 )3 COCH2CH3 and (CH3 )2 C== CH2
f=6 cm f=–2.5 cm (d) (CH3 )2 C == CH2 and (CH3 )3 COCH2CH3
https://iit-jeeacademy.blogspot.com

186 KVPY Practice Set 2 Stream : SA

75. Which of the following species contains equal number 77. Which one of the following genotypes cannot occur
of σ and π-bonds? amongst the offspring from a mating between a person
(a) HCO3− (b) XeO4 of blood group A and a person of blood group B?
(c) (CN)2 (d) CH2 (CN)2 (a) AA (b) AB (c) AO (d) BO
78. The weed killer DCMU blocks the flow of electrons
BIOLOGY from the electron transport chains in
photophosphorylation. Why does this kill the plant?
76. A piece of mammalian tissue was homogenised and
(a) Active transport of mineral ions is prevented
subjected to differential centrifugation. The diagrams
(b) ATP and reduced NADP are not produced
below indicate the relative activity certain
(c) Photoactivation of the chlorophyll cannot occur
biochemical processes in these fractions. Which of the
(d) Photolysis of water does not occur
following fractions indicates the maximum hydrolytic
enzyme activity? 79. Which one of the following is the correct description
of a certain part of a normal human skeleton?
Activity Activity
(a) Parietal bone and the temporal bone of the skull are
(a) Nuclei (b) Nuclei joined by fibrous joint
(b) First vertebra is axis which articulates with the
Mitochondria Mitochondria
occipital condyles
Lysosomes Lysosomes (c) The 9th and 10th pairs of ribs are called the floating ribs
(d) Glenoid cavity is a depression to which the thigh bone
Ribosomes Ribosomes articulates
80. Which of the following statements correctly describes
Activity Activity a codon?
(a) A length of DNA which codes for a particular protein
(c) Nuclei (d) Nuclei
(b) A part of the transfer RNA molecule to which a specific
Mitochondria Mitochondria amino acid is attached
(c) A part of the transfer RNA molecule which recognises
Lysosomes Lysosomes the triplet code on the messenger RNA
(d) A part of the messenger RNA molecule that has a
Ribosomes Ribosomes sequence of bases coding for an amino acid

Answers
PART-I
1 (b) 2 (a) 3 (c) 4 (a) 5 (b) 6 (b) 7 (c) 8 (b) 9 (d) 10 (c)
11 (a) 12 (b) 13 (a) 14 (b) 15 (b) 16 (b) 17 (c) 18 (c) 19 (d) 20 (d)
21 (a) 22 (c) 23 (d) 24 (c) 25 (b) 26 (b) 27 (d) 28 (d) 29 (c) 30 (b)
31 (c) 32 (c) 33 (a) 34 (d) 35 (a) 36 (d) 37 (a) 38 (a) 39 (c) 40 (a)
41 (c) 42 (a) 43 (c) 44 (c) 45 (c) 46 (a) 47 (a) 48 (b) 49 (a) 50 (b)
51 (c) 52 (c) 53 (b) 54 (b) 55 (c) 56 (a) 57 (d) 58 (a) 59 (a) 60 (a)

PART-II
61 (a) 62 (c) 63 (c) 64 (d) 65 (a) 66 (c) 67 (a) 68 (a) 69 (c) 70 (c)
71 (d) 72 (b) 73 (a) 74 (c) 75 (b) 76 (c) 77 (a) 78 (b) 79 (a) 80 (c)
https://iit-jeeacademy.blogspot.com

KVPY Practice Set 2 Stream : SA 187

Solutions
1. (b) 599 = 598 ⋅ 5 = (52 )49 ⋅ 5 ⇒α 2 + β 2 − 2αβ = γ 2 + δ 2 − 2γδ 9. (d) Given, ABCD is collinear.
= (25)49 ⋅ 5 = 5(26 − 1)49 ⇒ (α + β )2 − 4αβ = (γ + δ)2 − 4γδ
A B C D
= 5(26k − 1)[Q (a − b)n = nk − (b)n] ⇒ a 2 − 4b = b2 − 4a
∴AB + BC + CD = AD
= 5 × 26k − 5 = 5 × 26k − 13 + 8 ⇒ (a 2 − b2 ) + 4(a − b) = 0 AB 3
=
= 13(10k − 1) + 8 ⇒ (a − b) (a + b + 4) = 0 CD 2
∴When 599 is divided by 13 the ⇒ a+ b+ 4= 0 [Q a ≠ b] BC 1
and =
remainder is 8. 5. (b) P be an interior point of AD 5
2. (a) Let AB 3
rectangle ABCD. ⇒ =
p (x) = q(x) (x2 − 3x + 2) + (2x − 3) CD 2
D C
AC − BC 3
p (x) = q(x) (x − 1) (x − 2) + (2x − 3) ⇒ =
BD − BC 2
p(1) = 0 + (2 − 3) = − 1
p(2) = 0 + (4 − 3) = 1 ⇒ 2AC − 2BC = 3BD − 3BC
P ⇒ BC = 3BD − 2AC ...(i)
∴ p(1) < 0 and p(2) > 0
BC 1
∴ p (x) has one root lie between 1 and 2. and =
A B AD 5
∴ p (x) must have a root lie between 0
∴ Area of ∆APB + area of ∆PCD 5BC = AD
and 3.
= Area of ∆PBC + Area of ∆PAB ⇒ 5BC = AB + BC + CD
3. (c) Let the original price of commodity
=P ∴ In option (b), 21 + 6 = 15 + 12 ⇒ 4BC = AB + CD
Price of commodity when price x% increase ∴ Option (b) is correct. ⇒ 4BC = AC − BC + BD − BC
P (100 + x) 6. (b) We have, ⇒ 6BC = AC + BD ...(ii)
= P + x% of P =
100 n n From Eqs. (i) and (ii), we get
Price of commodity when priceY % ∑ xi2 = 400 and ∑ xi = 80 6(3BD − 2AC ) = AC + BD
i =1 i =1
decrease from the increase of x% ⇒ 18BD − 12AC = AC + BD
100 + x  P (100 + x)
= P 
Y We know, ⇒ 13AC = 17BD
 − ×
 100  100 100 x12 + x22 + x32 + K + xn2 AC 17
⇒ ⇒ =
P (100 + x)  100 − y  n BD 13
=  
 100  x + x2 + x3 + K + xn 
2
≥  1
100 10. (c) We have,

Given, the reduces price is equal to  n  In ∆ABC,
original price. Σxi2 (Σxi )2 AB = AC = 6
⇒ ≥
100 + x   100 − y 
∴ P     =P n n2 Circumradius (R ) = 5
 100   100 
400 (80)2 6400 A
⇒ ≥ ⇒ n≥ = 16
 100 + x   100 − y  = 1 n n2 400
   
 100   100  5
∴ n ≥ 16 6 6
x y xy
⇒ 1+ − − =1 7. (c) The set S is divided into three
100 100 (100)2 5 5
equal size.
x y xy B C
⇒ − = ∴ Each set has 4 elements. a
100 100 (100)2 We know,
∴ Total number of ways in partition is
xy abc
⇒ x− y= R=
100
12
C4 × C4 × C4 .
8 4
4∆
1 1 1 12! 8! 4! 12! (a ) (6) (6)
Divide by xy, we get − = × × = 5=
y x 100 4! 8! 4! 4! 0! 4! (4!)3  12 + a   12 + a − 6
   
4. (a) Let α , β are roots of equation 8. (b) Put x time of water of 110 ml to  2   2 
4
x2 + ax + b = 0 container and take y time of water of  12 + a − 6  12 + a − a 
   
25 ml from container.  2   2 
∴ α + β = − a , αβ = b
Then, container contains 110x − 25 y 36a
and γ , δ are roots of equation ⇒ 5=
= 5(22x − 5 y) (12 + a ) (a ) (a ) (12 − a )
x2 + bx + a = 0 4
∴Container contains multiple of 5. 16
∴ γ + δ = − b, γδ = a
∴Smallest amount of water be measure ⇒ 5=
36
Given, |α − β | = | γ − δ |
accurately 5 ml. 144 − a 2
(α − β )2 = (γ − δ)2
https://iit-jeeacademy.blogspot.com

188 KVPY Practice Set 2 Stream : SA

− F + µm2 g
x  x −  = 0
2 3
⇒144 − a 2 =  
36 a2 = = − 1ms−2
 5  2 m2
3 9
2
⇒ x = , x ≠ 0⇒x = Now, from equation of motion, we have
a 2 = 144 −  
36
⇒ 2 4 1
 5 s = ut + at 2
9
9  144 × 16 ∴ Hence, two solution x = 1, 2
= 144  1 −  = 4 1
 25  25 22 = 0 + (10 − (− 1)) t 2
15. (b) Given, 0 < a < b < c 2
144 × 16 48
⇒ a2 = = Here, a, b, c are distinct. ⇒ t = 2s
25 5
∴Three digits number formed by using a, 21. (a) Force on ball, F = v ∆m
11. (a) Let the second car overtakes in b, c where digits are not repeated is 3! = 6 ∆t
t hours. ∆m
Sum of all the three digits number are where, = flow rate of water.
∴Distance covered by first = Distance ∆t
2! (a + b + c) (102 + 10 + 1) ∆m
covered by second car
= 2(a + b + c) (100 + 10 + 1) ⇒ v = mg
 1  ∆t
10t = 8 +  8 +  +  8 + 2   
1
 2   2  = 222(a + b + c) ∆m mg m g
⇒ = =
 t − 1  Given, 222(a + b + c) = 1554 ∆t
+  8 +  + K +  8 + 
3 2 gh 2 h
 1554
 2   2  a+ b+ c= =7 22. (c) For a cyclic process,
222
1 1→ 2→ 3→ 1
⇒ 10t = 8t + (1 + 2 + 3 + K + t − 1) The possible digits whose sum seven are
2 Σ∆U = 0
1, 2, 4
1 (t ) (t − 1)
⇒ 10t = 8t + ∴ b= 2 ⇒100 + c + (− 200) = 0 or c = 100 kJ
2 2
⇒ t 2 − t = 8t ⇒ t − 1 = 8 ⇒ t = 9 16. (b) With respect to observer, there is 23. (d) From free body diagram, F
no acceleration in the vertical velocity F − mg = ma
∴The second car overtake the first car in
component. So, path of particle is a ⇒ F = m( g + a ) a
9 h.
straight line as in option (b).
12. (b) We have, = 1 (10 + 10)
17. (c) F = − dU = − (2x + 3) = 20 N
⇒ x2 + 2013 = y2 dx mg
⇒ y2 − x2 = 2013 For stable equilibrium, 24. (c) Electrons are not effective in
3
⇒ ( y + x) ( y − x) = 3 × 11 × 61 F = 0⇒x = − scattering α-particles because they are
2 about 7000 times lighter than α-particles.
xy is minimum when d 2U
=2
gh = g  1 −
y − x = 33 : y + x = 61 Also, 2h 
dx2 25. (b) 
 R
∴ x = 14, y = 47 So, there is a minima ofU.
gd = g  1 − 
∴Minimum value of xy = 14 × 47 = 658 d
i.e. system is in stable equilibrium. and
 R
13. (a) We have, 18. (c) By principle of thermometry, ⇒ gh = gd ⇒ 2h = d
Edge of cube = 2 cm X − (− 180° X) 25 − 0
= h 1
∴ Total surface area of cube − 60° X − (− 180° X) 100 ⇒ = = 0.5
d 2
= 6 (side )2 = 6(2)2 = 24 cm 2 X + 180° X 1
⇒ = 26. (b) India is in northern hemisphere
Length, breadth and height of cuboid are 120° X 4
1, 2 and 3 respectively 120° X and due to rotation of earth, radially
⇒ X + 180° X = = 30° X rushing wind will tend to rotate in
Total surface area of cuboid 4
anti-clockwise sense.
= 2(lb + bh + hl) X = −180° X + 30° X ⇒ X = − 150° X
27. (d) Imagine earth rotating around
= 2(2 + 6 + 3) 19. (d) Following conservation of mass sun and moon around earth.
= 22 cm 2 number and atomic number, we have
Path of
4 Be + 2 He → 6 C + 0 n
9 4 12 1
Total surface area of both cube and
moon
cuboid is 24 + 22 = 46 So, particle 1
0n is a neutron.
which is less than 54 cm 2. Sun E
20. (d)
∴Both cube and cuboid can be painted.
m=0.2 m=0.5
14. (b) Given, (x)x x
= (x x ) x Path of
3 3x A B earth
x
= (x ) 2 = x2
22 m 28. (d) Velocity increases with time and
Case I When base x = 1 then reaches terminal velocity. Velocity
Acceleration of blocks are
Case II When base x ≠ 1 F − f1 F − µm1 g remains constant after reaching terminal
3x a1 = = = 10 ms−2 speed.
Then, x3 / 2 = m1 m1
2
https://iit-jeeacademy.blogspot.com

KVPY Practice Set 2 Stream : SA 189

29. (c) As density of ball The IUPAC name of the above given direction where there are less number of
 = 40 = 0.8 g cm −3  is less than water, it compound is 3, 5-dimethyl cyclohexene. moles (according to Le-chatelier
 
 50  36. (d) As pentan-2-one principle).
tends to float. (CH3 COCH2CH2CH3 ) and 3-methyl 2SO2 ( g ) + O2 ( g ) - 2SO3 ( g )
When ball is kept immersed, downthrust butanone (CH3 COCH(CH3 )CH3 ) have Hence, the reaction will more towards
= weight of 50 cm3 of water = 50 g similar molecular formula, but different right.
carbon skeletons. Thus, they are chain
So, scale reading = 600 + 50 = 650 g 41. (c) Given,
isomers and will exhibit chain isomerism.
30. (b) Pcombination = 2(Plens ) + Pmirror solubility product of BaCl 2 , Ksp = 4 × 10−9
37. (a) The given value of n = 3 suggests
2 × 100 Let the solubility of BaCl 2 be S.
= + 0 that the shell is 3. For n = 1, l has 3
flens (in cm) values, i.e. + 1, 0 and − 1hence there occur BaCl 2 - Ba 2+ + 2Cl −
200 3 orbitals in p-subshell namely px , py and S S 2S
= pz . Thus, the given values for n = 3, l = 1
− 12 / (15
. − 1) Ksp = [Ba 2+ ] [Cl − ]2 = (S) (2S)2 = 4S 3
and ml = 0 suggests that the orbital is 1 1/3
=
100
=−
25
D 3py . Hence, the maximum number of  K  3  4 × 10−9 
− 12 3 S =  sp  =  
orbitals that can be identified with given  4   4 
31. (c) CaCO3 → CaO + CO2 quantum number is only 1.
= 1 × 10−3 mol L−1
Molar mass of CaCO3 38. (a) O3
CH3CH==CHCH2CH3 42. (a) Chlorobenzene on treatment with
= 40 + 12 + 16 × 3 = 100 g sodium in dry ether gives diphenyl. The
O
Molar mass of CaO = 40 + 16 = 56 g reaction is known as Fittig reaction.
Molar mass of CO2 = 12 + 16 × 2 = 44 g CH3CH CHCH2CH3
100 g of CaCO3 produces 44 g of CO2 —Cl + 2Na + Cl—
44 O O
∴ 10 g of CaCO3 produces = × 10 = 4.4 g ozonide
100
32. (c) According to Raoult’s law the H3O+/ZnO Dry ether
+ NaCl
vapour pressure of volatile component is ∆
directly proportional to its mole fraction. CH3CHO + OHCCH2CH3
Diphenyl
If the solution obeys Raoult’s law at all This reaction is known as ozonolysis
concentration its vapour pressure would reaction in which the addition of ozone
43. (c) According to ideal gas equation,
vary linearly from zero to the vapour molecule to alkene gives ozonide and pV = nRT
pressure of pure solvent. Thus, the then cleavage of ozonide by Zn—H2O to Given,
correct plot will be (c). smaller molecules occurs. 360
Pressure = 360 torr = atm
33. (a) The ionic character is decided by 39. (c) PH5 does not exist due to very 760
Fajan’s rule. According to this rule, larger less electronegativity difference between Volume = 2 .5
is size of cation, smaller the size of the P and H. Hydrogen is slightly more Temperature = 22 .5 + 273 = 295.5 K
anion and lesser is the charge on the electronegative than phosphorus, thus
cation or high, thus more will be the ionic Weight of a gas = 16616
.
could not hold significantly the sharing
character. As the anion and charge of n pV
electrons. On the other hand, BiCl5 does ∴ n=
the cation in all the given compounds are RT
not exist due to inert pair effect. This is
same. So, the ionic character is only W W pV WRT
dependent on the size of cation. As the because on moving down the group, +5 Also n = ⇒ = =M =
oxidation state becomes less stable while M M RT pV
size of cation increases in the order
Be2+ < Mg 2+ < Ca 2+ < Ba 2+ +3 oxidation state become more stable Substituting the values,
due to inert pair effect. 1.6616 × 0.082 × 295.5
∴ The ionic character will also increase in M= = 34.26 g
the same manner, i.e. In SO2, pπ-dπ and pπ - pπ both types of 360
× 2.5
bonds are present. 760
BeCl 2 < MgCl 2 < CaCl 2 < BaCl 2
0 SeF4 has sp3 d-hybridisation whereas CH4 ≈ 34 g
34. (d) 3Br2 + 6CO32− + 3H2O → 5Br− has sp3 -hybridisation. Thus, they both 44. (c) Isoelectronic species are those
+5 have different geometry. species which have same number of
+ B rO3− + 6HCO3−
I3+ has a bent shape due to the presence of electrons.

In the reaction, Br2 is reduced to Br 2 lone pairs on central I atom. As all the given elements are
(oxidation number decreases from zero to
40. (a) Any change in the concentration, isoelectronic with each other. Thus, the
− 1) and Br2 is oxidised to BrO3− (oxidation
pressure and temperature of the reaction radii/size of isoelectronic species is
number increases from zero to +5 ).
results in change in the direction of inversely proportional to the atomic
35. (a) equilibrium. This change in the direction 1
1 number, i.e. size ∝
2 of equlibrium is governed by Z
6
Le-Chatelier’s principle. Thus, the correct order is
5 3
H3 C CH3 On increasing pressure, volume S2− > Cl − > K+ > Ca 2+
4 decreases. The reaction will move in the
https://iit-jeeacademy.blogspot.com

190 KVPY Practice Set 2 Stream : SA

45. (c) The reaction of toluene with 51. (c) The structures adopted by 59. (a) Centrosomes are made up of a
Cl 2 in the presence of FeCl3 gives polypeptides can be divided into four pair of centrioles and other proteins. The
predominantly o and p chlorotoluene. levels of organisation, i.e., the primary, centrosomes are important for cell division
This reaction follows electrophilic secondary, tertiary and quaternary and produce microtubules that separate
substitution mechanism and Cl + act as as structures. The secondary structure DNA into two new identical cells.
electrophile. pertains to the coiling of the polypeptide
chains into regular structure such as 60. (a) The cerebellum is the part of
CH3 α-helices and β-pleated sheets. hindbrain responsible for hand-eye
CH3 52. (c) At prophase-I, DNA replication coordination. It is responsible for
Cl2, FeCl3 has already occurred and the original maintaining equilibrium, transfer of
Cl+ amount of DNA has been doubled to x. information, fine adjustments to motor
(Chlorination)
Cl At anaphase-I, the amount of DNA in the actions, coordinating eye movements, etc.
Toluene p-chlorotoluene cell remains the same because no Coordination and body balance, posture
cytokinesis has occurred yet to separate during walking, riding, standing,
CH3 the cytoplasm. swimming, running are all maintained by
+ 53. (b) If O2 is not available, pyruvic acid the cerebellum.
Cl undergoes anaerobic respiration / 61. (a) Given, ABC is an equilateral
o-chlorotoluene fermentation, but under aerobic triangle.
condition, the pyruvic acid enters into
46. (a) Phosphatidylcholine is a class of mitochondria and converted to acetyl A
phospholipids that are a major Co-A. Acetyl Co-A functions as substrate
component of biological membranes entrant for Krebs’ cycle. So, it is a
(i.e. nerve cell membrane). It functions in connecting link between glycolysis and
the production of brain chemical called Kreb’s cycle.
acetylcholine used for nerve impulse 54. (b) Glucose is stored in the liver as O
transmission at the synapse. glycogen. Glycogen can be converted to O′ P
47. (a) Graft rejection does not involve free glucose by the process of r
erythrocytes. Transplant or graft glycogenolysis, which involves the B D′ D C
rejection occurs when transplanted tissue activation of a phosphorylase enzyme by
AB = BC = AC = 6
is rejected by the recipient’s immune the hormone glucagon. Glucagon is made
1
system, which destroys the transplanted by the pancreas and is released when the BD = BC = 3
blood sugar levels fall. There release of 2
tissue. Rejection is an adaptive immune
response via cellular immunity (mediated glucose is a homeostatic function of liver AD = AB sin B
by killer T-cells), macrophages and that is controlled and monitored in the AD = 6 × sin 60°
polymorphonuclear leukocytes (i.e., pancreas.
3
55. (c) As the membrane potential is AD = 6 × =3 3
neutrophils, eosinophils and basophils). 2
increased, sodium ions channels open, 1 1
48. (b) Horse-shoe crabs are marine and OD = AD = × 3 3 = 3
allowing the influx of Na + ions into the
brackish water arthropods. They 3 3
cell. The inward flow of sodium ions
resemble crustaceans but belong to increases the concentration of positively In ∆O ′ PO, ∠OO ′ P = 30°
separate subphylum of the arthropods, charged cations in the cell and causes OP = OD − PD = 3 − r
i.e. Chelicerata. The entire body of the depolarisation, where the potential of the
horse-shoe crab is protected by a hard OO ′ = 3 + r
cell is higher than the cell’s resting
carapace. OP 3+ r
potential. ∴ sin 30° = =
49. (a) The first living being on the earth OO ′ 3−r
56. (a) Ammonia is toxic waste product
were anaerobic or heterotrophic bacteria which is converted into urea in the liver. 1 3+ r
⇒ =
because the primordial atmosphere was This urea then enters the excretory 2 3−r
virtually oxygen-free. These organisms system to get eliminated from the body.
must have degraded simple compounds High levels of ammonia in blood is an ⇒ 3 − r = 2 3 + 2r
1
present in the primordial oceans. They indication of liver damage. ⇒ r=
may have had RNA genomes and used 57. (d) Mendel did not choose pod 3
RNA as biological catalysts. length. The seven contrasting traits he 62. (c) We have, x2 y3 = 6
50. (b) Salmonella bacterium causes took were 3x 3x 4 y 4 y 4 y
+ + + + 1/5
3 ≥  9x × 64 y 
● Plant height ● Flower position 2 3
salmonellosis infection. The bacteria 2 2 3 3
spread through human or animal faeces. ● Pod colour ● Pod shape  4 27 
5 
Thus, the presence of Salmonella in tap ● Flower colour ● Seed shape

● Seed colour
[∴ AM ≥ GM ]
water is due to contamination through 1/5
3x + 4 y ≥ 5  × 6
16
human excreta. Salmonella outbreaks 58. (a) The levels of classification from ⇒ [x2 y3 = 6]
are commonly associated with eggs, meat the broadest to the narrowest, i.e. in term  3 
and poultry, but these bacteria can also of having highest members to the lowest ⇒ 3x + 4 y ≥ 10
contaminate other foods such as fruits members are kingdom, phylum, class, ∴ Minimum value of 3x + 4 y is 10.
and vegetables. order, family, genus and species.
https://iit-jeeacademy.blogspot.com

KVPY Practice Set 2 Stream : SA 191

63. (c) We have three different kinds of 66. (c) Present temperature, 69. (c)
mangoes and we can select 25 mangoes in . × 10 −28 × (10 × (100) 2 + 30 × (200) 2
138
all. Hence, we select 0 or 1 or 2 or 3 ... + 20 × (500) 2 + 20 × ( 800) 2 + 20 × (1000) 2 ) Rays
mangoes from each kind of mangoes. T1 = from B′ B′′
. × 10 −23
3 × 138 object
Let x1 , x2 and x3 be different kinds of A′
10 −5 × 10 4 (10 + 30 × 4 + 20 × 25 + 20
mangoes.
× 64 + 20 × 100)
∴ x1 + x2 + x3 = 25 ⇒ x1 , x2 , x3 ≥ 0 =
3 A′′
∴Total number of selection = 25 + 3 − 1C3 − 1
10 −1 × (10 + 120 + 500 + 1280 + 2000)
= 27C2 = For convex lens, image is at focal distance
3 6 cm from the lens.
27 × 26
= = 351 =
3910
= 130.33 ≈ 130 K For concave lens,
1× 2 30
u = + 2 cm, f = − 2.5 cm
64. (d) We have, (1 − y)m (1 + y)n In few years, all the molecules with speed
1 1 1 1 1
> 900 ms −1 will left the atmosphere. ⇒ = + = +
m(m − 1) 2 u −2.5 2
=  1 − my + y + K So, then temperature will be
v f
 2  ⇒ v = + 10 cm
− . × 10−28 × (10 × (100)2 + 30 × (200)2
138
 1 + ny + n ( n 1)
y2 + K Linear magnification, produced by

 2  +20 × (500)2 + 20 × (800)2 ) v 10
T2 = diverging lens is m = = =5
⇒ (1 − y)m (1 + y)n = 1 + (n − m) y . × 10−23
3 × 138 u 2
n (n − 1) m(m − 1) 10−5 × 104 × (10 + 30 × 4 So, diverging lens increases the
+  + − nm y2 + K magnification five times.
 2 2  + 20 × 25 + 20 × 64)
=
Here, a1 = n − m = 10 ...(i) 3 70. (c) I = nevd A
n (n − 1) m(m − 1) 10−1 × (10 + 120 + 500 + 1280) ⇒ vd =
I
=
V /R
a2 = + − nm = 10 =
2 2 3 neA neA
1910 V
n 2 − n + m2 − m − 2mn = = 63.66 K ⇒ vd =
⇒ a2 = = 10 30 ρlne
2
Reduction in temperature is nearly, So, drift speed is independent of area of
⇒ (n − m)2 − (n + m) = 20 ...(ii) ∆T = T1 − T2 = 130 − 63.66 conductor.
From Eqs. (i) and (ii), we get = 66.34 K ≈ 70 K Hence, the ratio of vd$ /vd$ is 1.
n + m = 80
2 1
...(iii) 71. (d) Given, specific heat of water
67. (a) Let x = mass per unit length of
From Eqs. (i) and (ii), we get
ruler. = 4.184 g −1 ° C−1
n = 45, m = 35 Heat capacity of calorimeter = 783 J°C −1
Forces on ruler are as shown below.
∴ (m, n ) = (35, 45) Mass of water = 254 g
P
65. (a) We have, ∆T = 26.01 − 23.73 = 2 .28°C
2
L/

ABC is a right angle triangle. L qbomb = C∆T = 783 × 2 .28 J = 1785.24 J


B θ θ qwater = m × specific heat × ∆T
= 254 × 4184
. × 2 .28
–θ

= 2423.04 J
90

xL
Q
4√2 Heat absorbed = 1785 .24 J + 2423.04 J
2xL = 4208.28 J ≈ 4.21 kJ
P N
4√ 2 72. (b) The hybridisation of any molecule
4√ 2 Equating moments about P,
can be calculated using formula
C L
M A (x ⋅ L) sin (90 − θ) = L (2xL) sin θ 1
2 X = (valence electrons number of atoms
∠A = 90°, the angle bisector of B and C 2
sin θ 1
meet at P ⇒ = tan θ = monoatomic ± anion/cation)
The distance from P to hypotenuse is 4 2 cosθ 4
The shape and hybridisation of given
∴ PQ = 4 2 68. (a) 1 kg of water expands from molecules are as follows
Here, PQ is the radius of incircle of ∆ABC. 1000 cm3 to 1.68 m3 .
Molecule Shape Hybridisation
∴PQ = PM = PN = radii of incircle of ∴ ∆V = 168
. − 0.001 ≈ 168
. m3
NF3 Pyramidal sp 3
∆ABC So, work done in expanding against
∴ AP 2 = PM 2 + AM 2 BF3 Triangular sp 2
pressure is
planar
AP 2 = (4 2 )2 + (4 2 )2 ∆W = p∆V = 101 × 10 3 × 168
. = 169 kJ
BF4− Tetrahedral sp 3
[Q AM = PN ] ∆W 169
Now, = = 0.0748 NH4+ Tetrahedral sp 3
AP 2 = 32 + 32 mLV 1 × 2260
AP = 64 = 8
https://iit-jeeacademy.blogspot.com

192 KVPY Practice Set 2 Stream : SA

CH
Molecule Shape Hybridisation IAIO × IBIO
2
 CH 3CH 2OH
BCl 3 Triangular sp H3 C — C  Br → (i)
planar  SN 1

BrCl 3 T-shaped sp 3d CH3 CH3 IAIO IAIO IBIO IOIO


3 
NH3 Pyramidal sp CH3  COCH2CH3 IA I O × I BI B
NO −3 Triangular sp 2 
CH3 (ii)
planar
( Major)
As BF4− and NH+4 have same shape and ( A) I AI B IA I B I BI O IB I O
hybridisation. Thus, they are As in the given compound is 3°alkyl
isostructural pair. halide, so it can either go SN 1or E1 IA I A × IB I B
73. (a) Geometrical isomers are those reaction, which is dependent on the (iii)
isomers, which have same molecular nutrophile. As CH3 CH2O– Na + is a strong
formula, but different spatial Nu s, so it will prefer to undergo E1 IA I B IA I B IAIB IA I B
arrangement of atoms about the double reaction whereas CH3 CH2OH is a weak
bond. In geometrical isomers, both the Nu s which undergoes SN 2. IA I A × IB I O
carbon atoms of a double bond should 75. (b) Number of π and σ bands in (iv)
contain different substituents. given species are as follows :
xH Hx IA I B IA I O IAIB IA I O
No. of σ and
Structure
(a) π bonds Thus, in all four possibilities for
y H3C CH3 y (a) O σ bonds = 4 F1 -generation can be IA IB , IA IO, IBIO , IOIO
σ π π bonds = 1 but never IA IA or AA.
As both the substituents are different σ C σ
hence, they will show geometrical –O O—H
σ 78. (b) Only the electron transport
isomerism. system produces ATP and reduced NADP
(b) O σ bonds = 4
y H 3C in the plant. Without these compounds,
CH3 x σ π π bonds = 4
σ Xe σ the Calvin cycle cannot proceed and
(b)
O πσ π π O carbon-fixation cannot occur and their is
y CH3 CH3 x no respiratory substrate available for
O
It does not show geometrical isomerism. respiration.
(c) 2π σ 2π σ bonds = 3
Hx N ≡≡ C  C ≡≡ N 79. (a) Immovable / fixed / fibrous joints
σ σ π bonds = 4
(c) x are present between the skull bones. So,
(d) H σ bonds = 6
y H Hx between parietal bone and the temporal
2π σ σ 2π
σ π bonds = 4
N ≡≡ C  C  C ≡≡ N bone of the skull are joined by fibrous
It does not show geometrical isomerism. σ σ σ
 joint. Other statements can be corrected
Hx H as First cervical vertebra is atlas not
(d) axis. The 11th and 12th pairs of ribs are
76. (c) The graph (c) is correct as, it
Hx called floating ribs. Glenoid cavity is
shows the highest biochemical activity in
located at the end of scapula close to
It does not show geometrical isomerism. the lysosomes. These are membrane
coracoid process.
bound organelles which contain enzymes
CH3
that degrade polymers into their 80. (c) The triplet codon is made up of
 CH 3CH 2O− Na +
74. (c) CH3  C == CH2 ← monomeric subunits, i.e. hydrolytic 3 nucleotide bases and is located at the
E1 enzymes. centre of the middle loop of tRNA
77. (a) There are four possible mating molecule and base pairs with the
crosses which can occur with persons of complementary bases on an mRNA
blood group A and blood group B. molecule during protein synthesis.
https://iit-jeeacademy.blogspot.com

KVPY Practice Set 3 Stream : SA 193

KVPY
KISHORE VAIGYANIK PROTSAHAN YOJANA

PRACTICE SET 3
Stream : SA
MM 100

Instructions
There are 80 questions in this paper.
This question paper contains two parts; Part I and Part II. There are four sections; Mathematics, Physics, Chemistry
and Biology in each part.
Out of the four options given with each question, only one is correct.

PART-I (1 Mark Questions)


MATHEMATICS (a) is at least 30
(b) is at least 20
1. Let E (n ) denote the sum of the even digits of n. For (c) is exactly 25
example E(1243) = 2 + 4 = 6, then the value of (d) cannot be determined by the data
E (1) + E (2) + E (3) + K + E (100) is equal to
(a) 200 (b) 300 (c) 400 (d) 500
6. Let a, b and c such that a + b + c = 0 and is defined as
a2 b2 c2
2. The greatest possible perimeter of right angle P= + + , then the value of P
triangle with integer side length if one of the sides 2a + bc
2
2b + ac
2
2c + ab
2

has length 12 is is equal to


(a) 80 (b) 84 (c) 72 (d) 82 1
(a) 1 (b)
2
3. In a party, each man danced with exactly four women 1
and each woman danced with exactly three men. (c) (d) 2
4
Nine men attended the party, then number of woman
attended the party is 7. In a rectangle ABCD, AB = 8 and BC = 20, let P be a
(a) 12 (b) 9 (c) 6 (d) 8 point on AD such that ∠BPC = 90°. If r1 , r2 and r3 are
radii of the incircles of ∆ APB, BPC and CPD
4. If 3 + 2 = 985 and 3 − 2 = 473, then the value of xy
x y x y
respectively, then the value of r1 + r2 + r3 is equal to
is
(a) 6 (b) 8 (c) 10 (d) 12
(a) 36 (b) 72 (c) 48 (d) 54
5. A certain school has 300 students. Every student 8. If sin θ + cos θ = 3, then the value of tan θ + cot θ is
reads 5 newspapers and every newspaper is read by (a) 1 (b) 2
60 students. Then, the number of newspaper (c) 2 (d) None of these
https://iit-jeeacademy.blogspot.com

194 KVPY Practice Set 3 Stream : SA

9. Numbers 1, 2, 3, ... 100 are written down each of the The correct option is
cards A, B and C. One number is selected at random (a) HI > HII > HIII
from each of the cards. The probability that the (b) HI < HII < HIII
numbers so selected can be the measures (in cm) of (c) HI = HII < HIII
three sides of the right angled triangles no two of (d) data insufficient to conclude
which are similar is
4 3 3! 17. Potential energy of a spring when stretched through
(a) (b) (c) (d) None of these a distance x is 10 J. Ratio of work done for every ad-
1003 503 1003
ditional distance is
10. In a triangle with integer side length, one side is (a) 1 : 1 : 1 : .... (b) 1 : 2 : 4 : .....
three times as long as a second side and the length of (c) 1 : 3 : 5 : .... (d) 1 : 4 : 9 : 16 : ...
the third side is 17. What is the greatest possible
perimeter of the triangle? 18. At what temperature, the celsius and farenheit scale
give the same temperature value?
(a) 46 (b) 47 (c) 48 (d) 49
(a) 0°C (b) − 10°F
11. One morning, each member of Kanchan’s family (c) − 40°C (d) − 20°C
drank 8 ounce mixture of coffee and milk. The
amount of coffee and milk varied from cup to cup, but 19. In given nuclear transformation,

α
1 92 U
238
 → B Th A β→ D Pa
C

E
→ 92U
234
were never zero. Kanchan drank th of the total
7 A, B, C , D and E are
2
amount of milk and th of the total amount of (a) A = 234, B = 90, C = 234, D = 91, E = β
17
(b)A = 234, B = 90, C = 238, D = 94, E = α
coffee, then the number of people are there in
(c) A = 238, B = 93, C = 234, D = 91, E = β
Kanchan’s family are
(d)A = 234, B = 90, C = 234, D = 93, E = α
(a) 8 (b) 9 (c) 7 (d) 17
20. A particle is subjected to two simple harmonic
12. Let ABC be a triangle with ∠ABC = 90°. Let P and Q
motions
are mid-point of legs AB and BC, respectively.  π 

Suppose that AQ = 19 and PC = 22, then length of AC (x = 2 sin ωt) cm and x = 2 sin ω t +   cm
  3 
is equal to
(a) 24 (b) 25 (c) 26 (d) 30 time t is in seconds. Maximum speed of the particle,
rad
13. Number of natural numbers n between 1 and 2019 if ω = 1 is
8n s
(both inclusive) is an integer is cm cm
9999 − n (a) 6 (b) 3
s2 s2
(a) 0 (b) 1 (c) 2 (d) 3 cm 1 cm
(c) 2 (d)
14. A ray of light originating at the vertex A of a square s2 2 s2
ABCD passes through the vertex B after getting
21. A man crosses a 320 m wide river perpendicular to
reflected by BC, CD and DA in that order. If θ is the
the current in 4 min. If in still water, he can swim
angle of the initial position of the ray with AB, then 5
sin θ equals with a speed times that of the current, then the
2 3 3 4 3
(a) (b) (c) (d) speed of the current (in m/min) is
13 13 5 5
(a) 30 (b) 40 (c) 50 (d) 60
15. Let m be the number of ways in which two couples
can be seated on 4 chairs in a row, so that no wife is 22. Consider a 16 cm × 8 cm uniform rectangular sheet
next to her husband and n be the number of ways in with its sides parallel to axes and its centre at origin.
which they can be seated in 4 chairs in a circle. In the
other case rotation are considered different
configurations. Then,
(a) m = n (b) m = 2n (c) m = 4n (d) m = 8n (0,0)

PHYSICS
16. A sample of pure ice is taken and following are If exactly one quarter of this sheet is removed,
recorded. coordinates of centre of mass of remaining sheet are
I. Heat to melt the ice. (a) (4 / 3, 2 / 3) (b) (0, 2 / 3)
II. Heat to warm ice cold water to 100°C. −4 −2
(c) (4 / 3, 0) (d)  , 
III. Heat to vaporize the water at 100°C.  3 3
https://iit-jeeacademy.blogspot.com

KVPY Practice Set 3 Stream : SA 195

23. For four processes A, B, C and D, loge p versus loge V U


F
F
graph are given below. (a) (b)
ln p
B
C U
D r0
r0 r
r
A

(d)
(c)

ln V
F
Isothermal process is F
U r0
(a) A (b) B (c) C (d) D r0
r
24. In given set up, U
focal length of mirror = 20 cm,
focal length of lens = 15 cm
29. A metal sphere is held suspended along a wall as
and separation of mirror and lens = 40 cm. shown below.
A point source S of light ‘S’ is placed on principal axis
at distance d from lens.
If the final beam comes out parallel to the principal
axis, then value of d is 12 cm
f=20 cm
f=15 cm
r=5 cm
S

If string can break at a pull of 15 N, then maximum


density of material of sphere can be
(a) 44 kg m −3 (b) 64 kg m −3 (c) 54 kg m −3 (d) 74 kg m −3
d 40 cm
(a) 4 cm (b) 8 cm (c) 12 cm (d) 16 cm 30. An object at infinity forms an image of size 2 cm by a
convex lens of focal length 30 cm.
25. A gas satisfies the relation pV = K , where
5/ 3

p = pressure, V = volume and K = constant. The Now, a concave lens of focal length 20 cm is placed
dimensions of constant K are between the convex lens and image at a distance of
26 cm from convex lens. Image size now will be
(a) [ML4 T −2 ] (b) [ML2 T −2 ] (c) [M0 L0 T 0 ] (d) [MLT −2 ]
(a) 1.25 cm (b) 2.5 cm (c) 1.05 cm (d) 2 cm
26. When temperature of a semiconductor is raised, then
choose the correct option.
(a) None of electron jump to higher energy level
CHEMISTRY
(b) All electrons likely to jump at higher energy levels 31. If 500 mL of a 5M solution is diluted to 1500 mL.
(c) Electrons whose energies are close to fermi energy are What will be the molarity of the solution obtained?
likely to jump to higher energy levels (a) 1.5 M (b) 1.66 M (c) 0.017 M (d) 1.59 M
(d) Electrons having lesser energy than fermi energy are
likely to jump to higher energy level 32. A plot of volume (V ) versus temperature (T ) for a gas
at constant pressure is a straight line passing
27. An incense stick is lighted in a closed room in which
through the origin.
there is no flow of air. Then, choose the correct option
given below.
(a) Flow of smoke is initially turbulent, then laminar p1
(b) Flow of smoke is initially laminar, then turbulent p2
Volume (mL)

(c) Flow of smoke is turbulent only


p3
(d) Flow of smoke is laminar only
28. Which of these graphs correctly shows potential p4
energy and force between two atoms in a diatomic
molecule? (U = potential energy, F = force, r = distance) Temperature (K)
https://iit-jeeacademy.blogspot.com

196 KVPY Practice Set 3 Stream : SA

The plot of different values of pressure are shown in 44. The correct IUPAC name of the following compound
figure. Which of the following orders of pressure is
correct?
(a) p1 > p2 > p3 > p4 (b) p1 = p2 = p3 = p4 is
(c) p1 < p2 < p3 < p4 (d) p1 < p2 = p3 < p4
33. A substance X gives brick red flame and breaks down
on heating to give oxygen and a brown gas. The (a) 3-(1-ethyl propyl) hex-1-ene
substance X is (b) 4-ethyl-3-propyl hex-1-ene
(a) magnesium nitrate (b) calcium nitrate (c) 3-ethyl-4-ethenyl heptane
(c) barium nitrate (d) strontium nitrate (d) 3-ethyl-4-propyl hex-5-ene
34. The major product obtained when 1 butyne reacts 45. The period number in the long form of the periodic
with excess HBr is table is equal to
(a) 2, 2-dibromobutane (a) magnetic quantum number of any element of the
(b) 2-bromobutane period
(c) 1, 1, 2, 2-tetrabromobutane (b) atomic number of any element of the period
(d) 1, 2-dibromobutene (c) maximum principal quantum number of any element
of the period
35. Which of the following sulphides when heated (d) maximum azimuthal number of any element of the period
strongly in air gives the corresponding metal?
(a) Cu 2S (b) CuS (c) Fe2S3 (d) HgS
BIOLOGY
36. Sulphur in + 3 oxidation state is present in
(a) dithionous acid (b) sulphurous acid 46. Which of the following biomolecules is common to
(c) dithionic acid (d) pyrosulphuric acid respiration-mediated breakdown of fats,
carbohydrates and proteins?
37. The correct order of boiling points of given
(a) Glucose-6-phosphate
compounds is n-butylamine (I), diethyl amine (II),
(b) Fructose-1,6-bisphosphate
N, N-dimethylethylamine (III).
(c) Pyruvic acid
(a) III < II < I (b) I < II < III
(d) Acetyl Co-A
(c) III < I < II (d) II < I < III
. × 10−34 Js. The
38. The value of Planck’s constant is 663 47. You are given a tissue with its potential for
−1 differentiation in an artificial culture. Which of the
velocity of light is 3 × 10 ms . Which value is closest
8
following pairs of hormones would you add to the
to the wavelength in nanometer of a quantum of light
medium to secure shoots as well as roots?
with frequency of 8 × 10−15 s−1?
(a) IAA and gibberellin
(a) 2 × 10−25 (b) 3 × 107
(c) 4 × 101 (d) 5 × 10−18 (b) Auxin and cytokinin
(c) Auxin and abscisic acid
39. The correct order of electron affinities of N, O, S and (d) Gibberellin and abscisic acid
Cl is
(a) N < O < S < Cl (b) O < N < Cl < S
48. The partial pressure of oxygen in the alveoli of the
(c) O ≈ Cl < N ≈ S (d) O < S < Cl < N
lungs is
(a) equal to that in the blood
40. The number of lone pairs on central metal atom Xe in (b) more than that in the blood
XeF2 , XeF4 and XeF6, respectively are (c) less than that in the blood
(a) 2, 3, 1 (b) 1, 2, 3 (c) 4, 1, 2 (d) 3, 2, 1 (d) less than that of carbon dioxide
41. When aqueous solution of benzene diazonium 49. When cell has stalled DNA replication fork, which
chloride is boiled, the product obtained is checkpoint should be predominantly activated?
(a) C6 H5 CH2OH (b) C6 H6 + N2 (a) G1 / S
(c) C6 H5 COOH (d) C6 H5 OH (b) G2 / M
42. The set of quantum number for 19th electron of (c) M
chromium (Z = 24) is (d) Both G2 / M and M
1 1
(a) 4, 0, 0, + (b) 4, − 1, 1, − 1, + 50. Which of the following is the least likely to be
2 2 involved in stabilising the three-dimensional folding
1 1
(c) 3, 2, 2, + (d) 3, 2, − 2, + of most proteins?
2 2 (a) Hydrogen bonds
43. Among the given carbocations, the most stable (b) Electrostatic interaction
carbocation is (c) Hydrophobic interaction
(a) methyl (b) allyl (c) benzyl (d) vinyl (d) Ester bonds
https://iit-jeeacademy.blogspot.com

KVPY Practice Set 3 Stream : SA 197

51. Name a peptide hormone which acts mainly on (c) release bicarbonate ions by the liver
hepatocytes, adipocytes and enhances cellular (d) reduce the rate of heartbeat
glucose uptake and utilisation. 56. Lack of relaxation between successive stimuli in
(a) Insulin (b) Glucagon sustained muscle contraction is known as
(c) Secretin (d) Gastrin
(a) fatigue (b) tetanus
52. If a colourblind man marries with a woman who is (c) tonus (d) spasm
homozygous for normal colour vision, the probability
57. Which of the following guards the opening of
of their son being colourblind is
hepatopancreatic duct into the duodenum?
(a) 0 (b) 0.5
(c) 0.75 (d) 1 (a) Ileocaecal valve (b) Pyloric sphincter
(c) Sphincter of Oddi (d) Semilunar valve
53. One of the major components of cell wall of most
fungi is 58. Which one of the following cell organelles is enclosed
(a) peptidoglycan (b) cellulose by a single membrane?
(c) hemicellulose (d) chitin (a) Chloroplasts (b) Lysosomes
(c) Nuclei (d) Mitochondria
54. A tall true breeding garden pea plant is crossed with
a dwarf true breeding garden pea plant. When the 59. Which of the following features is not present in the
F1-plants were selfed, the resulting genotypes were in phylum Arthropoda?
the ratio of (a) Metameric segmentation
(a) 1 : 2 : 1 :: Tall heterozygous : Tall homozygous : Dwarf (b) Parapodia
(b) 3 : 1 :: Tall : Dwarf (c) Jointed appendages
(c) 3 : 1 :: Dwarf : Tall (d) Chitinous exoskeleton
(d) 1 : 2 : 1 :: Tall homozygous : Tall heterozygous : Dwarf
60. Water soluble pigments found in plant cell vacuoles
55. Reduction in pH of blood will are
(a) reduce the blood supply to the brain (a) chlorophylls (b) carotenoids
(b) decrease the affinity of haemoglobin with oxygen (c) anthocyanins (d) xanthophylls

PART-II (2 Marks Questions)


MATHEMATICS 65. In a ∆ ABC, let I denotes the incenter. Let the line AI,
BI and CI intersects the incircle at P, Q and R,
61. Let S be a set of real numbers with mean m. If the
means of set S ∪ {15} and S ∪ {15, 1} are m + 2 and respectively. If ∠BAC = 40°, then the value of
m + 1 respectively. Then, number of elements S has ∠ QPR in degree is
(a) 4 (b) 5 (c) 6 (d) 7 (a) 50° (b) 65°
(c) 55° (d) 60°
62. For natural numbers x and y, let (x, y) denote the
greatest common divisor of x and y. The pairs of
natural number x and y with x ≤ y satisfy the PHYSICS
equation xy = x + y + (x, y) is
 c + t3 
(a) 2 (b) 3 (c) 4 (d) 5 66. Power is given by P = a + bt 2 +   , where t is
 d 
63. In a ∆ ABC, X and Y are points on the segment AB
and AC respectively, such that AX : XB = 1 : 2 and time. Then,
AY : YC = 2 : 1. If the area of ∆ AXY is 10, then the I. [a ] = [ML2T−3 ]
area of ∆ ABC is
II. [b] = [ML2T−5 ]
(a) 30 (b) 45 (c) 60 (d) 27
64. Let x1 , x2 , x3 , K , x2019 be the real numbers different III. [c] = [T3 ]
from 1, such that x1 + x2 + x3 + K + x2019 = 1 and IV. [d ] = [ML− 2T−6 ]
x1 x2 x2019 Which of the above statements are correct?
+ +K+ = 1, then the value of
1 − x1 1 − x2 1 − x2019 (a) Statements I and IV are correct
x12 x22 2
x2019 (b) Statements I and III are correct
+ +K+ is equal to
1 − x1 1 − x2 1 − x2019 (c) Statements I, II and III are correct
(a) 0 (b) 1 (c) 2019 (d) None of these (d) All statements are correct
https://iit-jeeacademy.blogspot.com

198 KVPY Practice Set 3 Stream : SA

67. A rolling sphere collides with a cube of equal mass. − 2x


U =
Surface is frictionless. Radius of sphere is 1 cm and x +4
2

its initial angular speed is 1 radian per second. Then, angular frequency of small oscillations of the
ω body about the position of stable equilibrium is
2 cm (a) 5 rad s−1 (b) 10 rad s−1
r=1 cm
(c) π rad s−1 (d) 25 rad s−1
If side of cube is 2 cm and collision is elastic, then
after collision,
(a) ωsphere = 0 and vcube = 0.01 ms−1
CHEMISTRY
rad
(b) ωsphere = 1 and vcube = 0.01 ms−1 71. The heat of formation of C12H22O11 (s), CO2 ( g) and
s H2O(l) are −530, − 943
. and −683
. kcal/mol,
(c) ωsphere = 0 and vcube = 0 respectively. The amount of C12H22O11 required to
rad
(d) ωsphere = 1 and vcube = 1ms−1 supply 2700 kcal of energy is
s (a) 382.70 g (b) 832.74 g
68. A current of 1 mA enters the network of resistors as (c) 463.9 g (d) 682.6 g
shown below. 72. The following alcohol after treatment with acid gives
2kΩ compound A. Ozonolysis of A gives nonan -2, 8 dione.
The compound A is
CHOHMe
H+
A 1kΩ 1kΩ 1kΩ B A
Me
2kΩ
Now, consider the following statements: (a) (b)
2
I. Current through 2 kΩ resistor is mA.
5
1 (c) (d)
II. Current through lower 2 kΩ resistor is mA.
5
1
III. Current through middle 1 kΩ resistor is mA. 73. An electric current is passed through silver nitrate
5 solution using silver electrodes. 10.79 g of silver was
3 found to be deposited on the cathode. If the same
IV. Current through middle 1 kΩ resistor is mA.
5 amount of electricity is passed through copper
Which of the above statements are correct? sulphate solution using copper electrodes, the weight
(a) Only statement I is correct of copper deposited on the cathode is
(b) Statements II and III are correct (a) 1.6 g (b) 2.3 g
(c) Statements I and III are correct (c) 3.2 g (d) 6.4 g
(d) Statements III and IV are correct 74. Suppose 10−17 J of light energy is needed by the
69. A steel ball travels through a hollow U-tube with interior of human eye to see on object. Calculate the
separation in limbs 1 m, as shown below. number of photons of green light (λ = 550 nm) needed
to generate this minimum amount of energy.
(a) 26 (b) 27 (c) 28 (d) 29
1m
75. The Gibbs’ free energy change, ∆G°, for the following
reaction is 63.3kJ
v
Radius of U-tube is slightly greater than ball such Ag2CO3 (s) º 2Ag+ (aq) + CO32− (aq)
that ball travels through it and leaves from other end The Ksp of Ag2CO3 (s) in water at 25°C is closest to
with same uniform speed of 3 ms−1. (a) 3.2 × 10−26 (b) 8 × 10−12
Mass of steel ball is 0.02 kg. What is the approximate (c) 2.9 × 10−3 (d) 7.9 × 10−2
reading on spring balance attached to the tube?
(a) 0.1 N (b) 0.2 N (c) 0.5 N (d) 0 N BIOLOGY
70. A body of mass m (= 5 g), is moving in one dimension 76. Impulses travel very rapidly along nerves to the leg
under influence of a conservative force. Potential muscles of a mammal. Which fact accounts for the
energy of the body is given by speed at which they travel?
https://iit-jeeacademy.blogspot.com

KVPY Practice Set 3 Stream : SA 199

(a) A nerve impulse is an all-or-nothing phenomenon

Rate of reaction

Rate of reaction
(b) The nerves contain myelinated fibres
(c) There is high concentration of Na + ions inside the
axons (c) (d)
(d) There is a potential difference across the axon
membranes
Substrate Substrate
77. In a certain plant, yellow fruit colour (Y) is dominant concentration concentration
to green (y) and round shape (R) is dominant to oval 79. Assume that the average amino acid residue have a
(r). The two genes involved are located on different
molecular weight of 110. The DNA strand coding for
chromosomes. Which of the above will result when
plant YyRr is self-pollinated? a polypeptide chain of molecular weight 20,000 has a
(a) 9 : 3 : 3 : 1 ratio of phenotypes only
length of
(b) 9 : 3 : 3 : 1 ratio of genotypes only (a) 182 nucleotides
(c) 1 : 1 : 1 : 1 ratio of phenotypes only (b) 252 nucleotides
(d) 1 : 1 : 1 : 1 ratio of phenotypes and genotypes (c) 540 nucleotides
(d) 760 nucleotides
78. Which graph shows the expected relationship
between enzyme activity and substrate 80. Which of the following is a correct statement?
concentration? (a) Salvinia, Ginkgo and Pinus all are gymnosperms
(b) Sequoia is one of the tallest trees
Rate of reaction
Rate of reaction

(c) The leaves of gymnosperms are not well-adapted to


extremes of climate
(a) (b)
(d) Gymnosperms are both homosporous and
heterosporous

Substrate Substrate
concentration concentration

Answers
PART-I
1 (c) 2 (b) 3 (a) 4 (c) 5 (c) 6 (a) 7 (b) 8 (d) 9 (d) 10 (d)
11 (a) 12 (c) 13 (b) 14 (a) 15 (c) 16 (b) 17 (c) 18 (c) 19 (a) 20 (a)
21 (d) 22 (d) 23 (c) 24 (c) 25 (a) 26 (c) 27 (b) 28 (d) 29 (a) 30 (b)
31 (b) 32 (c) 33 (b) 34 (a) 35 (d) 36 (a) 37 (a) 38 (c) 39 (a) 40 (d)
41 (d) 42 (a) 43 (c) 44 (b) 45 (c) 46 (d) 47 (b) 48 (b) 49 (a) 50 (d)
51 (a) 52 (a) 53 (d) 54 (d) 55 (b) 56 (b) 57 (c) 58 (b) 59 (b) 60 (c)

PART-II
61 (a) 62 (b) 63 (b) 64 (a) 65 (c) 66 (c) 67 (b) 68 (b) 69 (b) 70 (a)
71 (d) 72 (d) 73 (c) 74 (c) 75 (b) 76 (b) 77 (a) 78 (b) 79 (c) 80 (b)
https://iit-jeeacademy.blogspot.com

200 KVPY Practice Set 3 Stream : SA

Solutions
1. (c) E (1) + E (2) + E (3) + K + E (100) 5. (c) We have, r3 = 6 − 2 5
= Sum of all even digits from 1 to 100 Total number of students = 300 ∴r1 + r2 + r3 = 12 − 4 5 + 6 5 − 10 + 6 − 2 5
= Sum of all even digits in One student read = 5 newspapers =8
[01 + 02 + 03 + K + 98 + 99 + 100] Number of newspaper read by 300 8. (d) Given, sin θ + cos θ = 3
students = 5 × 300 = 1500
= 0 × 20 + 2 × 20 + 4 × 20 + 6 × 20 + 8 × 20 Squaring both sides,
Number of different newspaper
[Q there are 2 × 100 = 200 digits and sin 2 θ + cos2 θ + 2 sin θ cos θ = 3
200 1500
each digit appears = 20 times] = = 25 ⇒ 1 + 2 sin θ cos θ = 3
10 60
⇒ sin 2 θ = 2
= (2 + 4 + 6 + 8) × 20 = 20 × 20 = 400 6. (a) Given,a + b + c = 0
∴Maximum value of sin θ = 1
2. (b) Let the other sides of right angle ∴ a3 + b3 + c3 = 3abc Hence, sin θ + cosθ = 3 not possible.
triangle be x and y. a2 b2 c2
Now, P = + + 9. (d) Given,
P 2a 2 + bc 2b2 + ac 2c2 + ab
Number 1, 2, 3, ... 100 are written on card
Let a = 1, b = − 1, c = 0 A , B and C.
x y 1 1
∴ P= + + 0 ∴ Total number of outcomes
2+ 0 2+ 0 = 100 × 100 × 100 = 1003
1 1
R 12 Q ⇒ P = + =1 Numbers are selected such that formal
2 2 three sides of right angle triangle and
∴ x = 12 + y2
2 2
7. (b) Given, ABCD is a rectangle. triangle are not similar.
⇒ x − y2 = 144
2
AB = 8, BC = 20 For a right angle triangle
⇒ (x + y) (x − y) = 72 × 2 Let AP = x (2n + 1)2 + (2n 2 + 2n )2 = (2n 2 + 2n + 1)2 ,
x and y are integer. ∴ PD = 12 − x n ∈N
∴Maximum value of x + y = 72 for n = 1, 2, 3, 4, 5, 6. If n > 6 then length
4 P 16
∴Maximum perimeter of triangle A D of longest side is greater than 100.
= 12 + x + y C1 C3 ∴ n (E ) = 6 × 3!
= 12 + 72 = 84 r1 r3 6 × 3!
8 P (∈) =
3. (a) There are 9 men and each man r2 r2 (100)3
C2 6× 6 9
danced with 4 women. = 3 =
⇒ Number of dancing pairs = 9 × 4 = 36 2 × (50)3 2(50)3
B 12 C
Now, let number of women = x 10. (d) Given,
In ∆APB and ∆DPC,
∴Each woman danced with 3 men. One side of triangle is three times the
∆PBA ~ ∆CPD
∴Number of dancing pairs = 3x AB AP
second side and third side is 17.
∴ 3x = 36, x = 12 ∴ = Let the sides of triangle are x, 3x, 17.
PD CD
Hence, 12 women attended the party. 8 x We know, in triangle sum of two sides is
⇒ = greater than third side.
4. (c) Given, 12 − x 8
⇒ x + 3x > 17
3x + 2y = 985 ...(i) ⇒ x2 − 20x + 64 = 0
17
3x − 2y = 473 ...(ii) ⇒ (x − 16) (x − 4) = 0 ⇒ x>
4
On adding Eqs. (i) and (ii), we get x = 4, 16
2 ⋅ 3x = 1458 and x + 17 > 3x
In ∆APB,
17
3x = 729 1 ∴ x<
× AB × AP 2
3x = 36 ∆ 2
r1 = =
1 17 17
⇒ x=6 s (AB + PB + BP ) ∴ < x<
2 4 2
On subtracting Eq. (ii) from Eq. (i), we get
8× 4 4.25 < x < 8.5
2 ⋅ 2y = 512 r1 =
8+ 4+ 4 5
2y = 256 Since, sides of triangle are integer.
[Q BP 2 = AP 2 + BA 2
2y = 28 ∴ x = 5, 6, 7, 8
BP = 16 + 64 = 4 5]
⇒ y=8 ∴Maximum value of x = 8
r1 = 12 − 4 5
∴ xy = 6 × 8 Similarly, ∴Sides of triangle are 8, 24, 17
= 48 r2 = 6 5 − 10 ∴Perimeter = 8 + 24 + 17 = 49
https://iit-jeeacademy.blogspot.com

KVPY Practice Set 3 Stream : SA 201

4 4 15. (c) m = Number of ways in which two


11. (a) Let the total milk is 7M ounce ⇒ AC 2 = (361 + 484) = × 845
and total coffee is 17C ounce. 5 5 couples can be seated in 4 chairs in a row
The ratio of total milk and coffee drank ⇒ AC 2 = 4 × 169 such that no wife is next to husband.
by Kanchan’s must be integer. ⇒ AC = 4 × 169 = 26 H1
7M + 17C 8n
∴ be an integer (which is the 13. (b) Let =λ
M + 2C 9999 − n
Total number of people in Kanchan’s H2 W2
∴ 8n = 9999λ − λn
family)
9999λ
7M + 17C 3C ⇒ n=
= 7+ 8+ λ W1
M + 2C M + 2C
3C 3 ⇒ n ∈[1, 2019] ∴ m = 2! × 2! × 2! = 8 ways H1 × W1 X
∴ 0< < 9999λ
M + 2C 2 ∴ 1≤ ≤ 2019 n = Number of ways in circular
8+ λ permutation = 2! = 2
3C
=1 9999λ ∴ m = 4n
M + 2C ⇒ ≥1
3C 3 8+ λ 16. (b) Let m kg of ice is taken.
[Q is integer less than ]
M + 2C 2 ⇒ 9999λ ≥ 8 + λ Then, HI = mLf = 334 m J
8 9999λ and HII = mc∆T = m (419. ) (100)
C= M ⇒ λ≥ and ≤ 2019
9998 8+ λ = 419 mJ
∴Total number of people in Kanchan’s
7M + 17M ⇒ 9999λ ≤ 16132 + 2019λ HIII = mLV = m (2260) = 2260 m J
family =8 16132 ∴ HI < HII < HIII .
M + 2M ⇒ λ≤
7990 17. (c) For additional extension x, work
12. (c) Given, 8 16132
∴ ≤ λ≤ done is
ABC is a right angled triangle 9998 7990 1 1
W = k (2x)2 − kx2
∠ABC = 90° ∴ λ = 1, 2, λ is an integer. 2 2
P and Q are mid-points of sides AB and For λ = 2, n is not an integer. 1 1 1
W = k (4x2 ) − kx2 = k (3x2 )
BC respectively. Hence, only one value is possible. 2 2 2
C 14. (a) Let side of square ABCD ∴Ratio is 1 : 3 : 5 : ....
AB = x 18. (c) Let x = temperature value.
F Then,
D C
θ C− 0 F − 32
22

Q =
100 − 0 212 − 32
19
θ θ
G θ E when C = F = x,
x x − 32
A P B ⇒ = ⇒ x = − 40
1 100 180
∴ BQ = BC = CQ θ
2 A B ∴ − 40°C = − 40°F
1
AP = AB = BP In ∆ABE, 19. (a) We have,

BE α β
2 tan θ = 92 U
238
 → 90 Th
234
→ 91 Pa
234

In ∆PBC, AB β
→ 92 U 234
PC 2 = BC 2 + PB 2 BE = x tan θ
20. (a) Amplitude of resultant motion is
1
PC 2 = BC 2 + AB 2 ...(i) CE = BC − BE = x(1 − tan θ)
A= A12 + A22 + 2A1 A2 cosφ
4 In ∆CEF,
In ∆AQB, tan θ =
CE
⇒ CF = x(cot θ − 1) = 2+ 2+ 2× 2× 2×
1
= 6
1 CF 2
AQ 2 = AB 2 + BC 2 ...(ii)
4 Now in ∆GHF, So, maximum acceleration = ω2A
On adding Eqs. (i) and (ii), we get FH CE cm
tanθ = = = 12 × 6= 6
5
PC 2 + AQ 2 = (AB 2 + BC 2 ) GH DF s2
4 CE x(1 − tan θ) 21. (d) vr2 = vm2 − v2
= =
5AC 2 DC − CF x(2 − cot θ) 320
⇒ PC 2 + AQ 2 = v= = 80 m/min
4 1 − tan θ 4
tan θ =

4
AC = (PC 2 + AQ 2 )
2 2 − cot θ vr
5 2
4 ⇒ 2 tan θ − 1 = 1 − tan θ ⇒ tanθ =
⇒ AC = (192 + 222 )
2 3 v
5 2 vm
∴ sinθ =
[Q PC = 19, AQ = 22] 13
https://iit-jeeacademy.blogspot.com

202 KVPY Practice Set 3 Stream : SA

5
vm = vr 35. (d) HgS, when heated strongly in air
3 gives mercury and sulphur dioxide.
2
θ
vr2 =  vr  − (80)2
5 HgS + O2   → Hg + SO2
Roasting
3 
The process is known as roasting where
16 2
vr = (80)2 the sulphide ore is directly heated in
9 presence of air (O2 ) to get the respective
80 × 3 metal.
vr = = 60 m/min
4 36. (a) The oxidation state of S in the
14
.
22. (d) X = Mx − mx′ = 4M (0) − M (4) Density of sphere = −4
given options are as follows
M−m 4M − M 4 π × (5) × (10 )
2
(i) Dithionous acid
=−
4 = 44 kg m −3 O O
3 30. (b)  
My − my′ 4M (0) − M (2) 2 HO  S  S  OH
Y = = =− I2
M−m 4M − M 3 2(x) + 2(−2) + 2(− 1) = 0
I1
23. (c) For isothermal process, 2x − 4 − 2 = 0
pV = constant 2x = 6
26 cm 4 cm
x=+ 3
⇒ p = K ⇒ log p = − log V + log K 1 1 1
V For concave lens using, − = (ii) Sulphurous acid
v u f
This is a straight line with negative O
1 1 1
slope. we have − = ⇒ v = 5 cm
v 4 −20 S
24. (c) As emergent beam is parallel, so v HO OH
Magnification of concave lens = = 1.25
mirror must forms image at focus of lens. u 1(x) + 1(− 2) + 2(− 1) = 0
As size of I1 is 2 cm. x=+ 4
∴ Size of I 2 = 2 × 125
. = 2.5 cm (iii) Dithionic acid
S 31. (b) Given that, O O
 
M1 = 5 M, V1 = 500 mL, HO  S  S  OH
15 V 2 = 1500 mL  
cm O O
For dilution, M1V1 = M2V 2
25 2(x) + 2(− 1) + 4(− 2) = 0
cm 5 × 500 = M × 1500
Clearly, d = 12 cm. 5 2x −2 − 8 = 8
M = = 1.66 M
−2 3 2x = 10
25. (a) [K ] = [ p] [V 5 /3 ] = [MLT2 ] ⋅ [L3 ]5 /3 32. (c) According to Boyle’s law, at x=+ 5
[L ]
constant temperature, the volume of a (iv) Pyrosulphuric acid
= [ML4 T −2] given mass of a gas is inversely O O
1
26. (c) Electrons which have maximum proportional to its pressure, i.e. p ∝  
energies are near to fermi energy level. V HO  S  O  S  OH
These electrons can jump to higher As, V1 > V 2 > V3 > V 4  
energy levels. ∴ p1 < p2 < p3 < p4 O O
33. (b) Calcium nitrate gives brick red 2(x) + 5(− 2) + 2(− 1) + = 0
27. (b) As smoke rises up in air, speed of
flame which breaks down on heating to 2x − 10 − 2 = 0
smoke increases and flow becomes
give oxygen and NO2 which is a brown 2x = + 12
turbulent. x=+ 6

gas 2Ca(NO3 )  → 2CaO + O2 + 4NO2
28. (d) As, F = − dU , option (d) is Thus, the correct option is (a).
dr 34. (a) When but-1-yne reacts with
correct. excess HBr, the major product obtained is 37. (a) Boiling point of a compound is
2,2 dibromobutane. This reaction follows dependent of on H-bonding present in it.
29. (a) T cosθ = mg and T sinθ = N Intermolecular H-bonding is more in
Markownikoff’s rule
w 13 primary than in secondary amines as
⇒ T = = w CH3 CH2C≡≡ CH + HBr →
cosθ 12 Butyne
there are two H-atoms available for
At T = 15 N and w = mg , H-bonding. Tertiary amines do not have
CH3 CH2 C == CH2 →
HBr
intermolecular H-bonding due to the
13 
15 = mg [Q g = 10 ms −2] absence of H-atom. Therefore, the order
12 Br
Br of boiling points of the given amines is as
15 × 12 = 13 × 10m
12 × 15  follows
⇒ m= ≈ 14
. kg CH3 CH2  C  CH3 nC4 H9 NH2 > (C2H5 )2 NH > C2H5 N(CH3 )2
13 × 10 
n-butylamine diethylamine N,N dimethylethylamine
Br (1°) (2°) (3°)
2, 2 dibromobutane
https://iit-jeeacademy.blogspot.com

KVPY Practice Set 3 Stream : SA 203

38. (c) Given velocity of light 43. (c) Among the given carbocations 48. (b) The partial pressure of oxygen
= 3 × 108 ms−1 benzyl and allyl carbocations are more ( pO2 ) in alveoli of lungs is 104 mm Hg,
stable than methyl and vinyl carbocations which is more than that of blood in the
Frequency of light = 8 × 1015 s−1
because they have delocalised electrons. blood capillaries of lung alveoli (40 mm
C 3 × 108 ms−1 An allylic carbocation has two resonance
As we know, λ = = Hg). This difference allows passive
ν 8 × 1015 s−1 structures whereas in benzylic diffusion of O2 from air filled in the lungs
carbocation has five resonance to the blood vessels of lung alveoli.
= 3.75 × 10−8 m
structures. Thus, benzyl carbocation is 49. (a) Stalled fork activates checkpoint
1 m = 109 nm the most stable one.
signaling and pauses replication. Since
∴3.75 × 10 m = 3.75 × 10−8 × 109 nm
−8
+ + G1 / S checkpoint checks DNA damage, cell
= 3.75 × 101 nm ≈ 4 × 101 nm RCH==CHCH2 RCH—CH==CH2 size prior to S-phase (i.e. DNA replication
(Allylic carbocation) phase) this checkpoint would be activated
39. (a) Electron affinities of II period
element are less negative as compared to + by stalled DNA replication fork.
corresponding III period element. This is +
—CHR ==CHR
50. (d) Ester bonds are the least likely to
because of small size of II period be involved in stabilising the 3-D folding of
elements. Also, nitrogen has the most proteins. A long protein chain gets
least electron affinity due to stable folded upon itself like a hollow woolen ball,
half-filled configuration. Thus, the giving rise to a tertiary (3D) structure. This
==CHR + ==CHR
correct order of electron affinity is structure is stabilised by several types of
N < O < S < Cl. +
bonds, i.e. hydrogen bonds, ionic bonds, van
40. (d) Xe atom has 8 electrons in its der Waal’s interactions, covalent bonds and
outermost shell. In case of XeF2, out of + hydrophobic bonds.
these 8 electrons, 2 are used for bond ==CHR
Ester bond is formed between sugar and
formation, while 3 pairs remains phosphate in a nucleotide and is not
non-bonded, i.e. it has 3 lone pairs. (Benzyl carbocation)
involved in stability of a polypeptide
In case XeF4 , 4 electrons of Xe are used 44. (b) chain. Thus, option (d) is correct.
for bonding. Thus it has 2 lone pairs. 2 1 51. (a) Insulin is the peptide hormone
In case of XeF6 , 6 electrons are involved CH CH2 which enhances the uptake of glucose
for bond formation, thus, it has only 1 5
3 molecules by liver cells (hepatocytes) and
CH2 CH 4 H2C 6
lone pair. CH3 CH2 CH CH3 fat cells (adipocytes) for its cellular
utilisation. Such an activity of insulin
F CH2
F F brings down the level of glucose in the
F F H3C
Xe Xe Xe F blood.
F F F F Thus, the correct IUPAC name of the 52. (a) The cross for the question is
F F given compound is 4-ethyl-3-propyl XX × XC Y
XeF2 XeF4 XeF6 hex-1-ene.
(3 lone pairs) (2 lone pairs) (1 lone pair)
45. (c) Since, each period starts with the
41. (d) When aqueous solution of filling of electrons in a new principal XXC XY XC X XY
benzene diazonium chloride is boiled, it quantum number, therefore the period Carrier Normal Carrier Normal
gives phenol. number in the long form of the periodic daughter son daughter son
+ table refers to the maximum principal
N≡≡NCl – OH Since the male offsprings get
quantum number of any element in the X-chromosome from their mother who is
period. Thus, period number = maximum normal homozygous, thus, none of the son
∆ n of any element. would be colourblind.
(where, n = principal quantum number) 53. (d) Cell wall of the most fungi is
Benzene Phenol 46. (d) Carbohydrates, fats and proteins made up of chitin. Chemically it is
diazonium
all can be used as a substrate in cellular N-acetyl glucosamine. It is found in the
chloride
respiration. All of them first get exoskeleton of insects.
42. (a) The electronic configuration of converted to acetyl Co-A to enter Kreb’s 54. (d)
chromium (Z = 24) is cycle of aerobic cellular respiration. Thus, Parents TT × tt
it is the common factor of respiration
1s2 2s2 2 p 6 3s2 3 p 6 4s1 3d5 (Tall) (Dwarf)
entering Kreb’s cycle after breakdown of
for 19th electron the orbital is 4s F1-generation Tt (Heterozygous tall
carbohydrates, fats and proteins. On selfing)
Thus, n=4 47. (b) When a tissue with a potential of T t
l = 0 to 3 differentiation is grown in an artificial TT Tt
m = − 3 to 3 T (Tall) (Tall)
medium containing auxin and cytokinin
1 F2-generation
s=+ in a specific ratio, it starts Tt tt
2 differentiating. Thus, root and shoot t (Tall) (dwarf)
Thus, among the given options, only (a) differentiation occurs. Auxin initiates
corresponds to the above given values. Phenotypic ratio 3 : 1 [Tall : Dwarf]
root formation while cytokinin starts
So, option (a) is correct. Genotypic ratio 1 : 2 : 1
shoot formation.
https://iit-jeeacademy.blogspot.com

204 KVPY Practice Set 3 Stream : SA

55. (b) Reduction of pH of blood, i.e. (x − 1) ( y − 1) = (x, y) + 1 = − (x1 + x2 + K + x2019 )


increase in acidity favours the dissociation Put x = 2, y = 3  x x2 x2019 
of oxyhaemoglobin thereby giving up more +  1 + + K+ 
(2 − 1) (3 − 1) = GCD of (2, 3) + 1  1 − x1 1 − x2 1 − x2019 
O2. When this phenomenon occurs due to
2 = 1+ 1 = −1+ 1= 0
increase in CO2 concentration, then it is
called Bohr effect. ∴ (2, 3)
65. (c) Given,
x = 3, y = 3
56. (b) Sustained muscle contraction due In ∆ABC, I is incentre of ∆ABC.
to repeated stimulus is known as tetanus. (3 − 1) (3 − 1) = 4 = GCD of (3, 3) + 1 A
This results due to muscle fatigue. x = 2, y = 4
57. (c) Sphincter of Oddi guards the also satisfies P

°
40
opening of hepatopancreatic duct into the When x > 3 not satisfies the equation.
duodenum. Hepatopancreatic duct brings ∴ Only 3 pairs (2, 3), (3, 3) and( 2, 4)
secretion of liver as well as pancreas to I
satisfy the equation.
the duodenum. Q R
63. (b) In ∆ABC, X and Y are points on
58. (b) Lysosomes are hydrolytic AB and AC respectively. B C
enzymes containing cell organelles which AX 1 AY 2
are bounded by a single membrane. = and = AI, BI and CI intersect the circle at P, Q,
XB 2 YC 1 R respectively.
Other organelles like chloroplast,
Area of ∆AXY 1
mitochondria and nuclei have double = In ∆IBC, ∠BIC + ∠IBC + ∠ICB = 180°
membrane system. Area of ∆BXY 2 ∠B ∠C
∠ BIC + + = 180°
59. (b) Parapodia are present in aquatic ⇒ Area of ∆BXY = 20 2 2
∠B + ∠C 
animals, i.e. annelids like Nereis which [Q area of ∆AXY = 10] ∠BIC = 180° −  
help them in swimming. Other three  2 
⇒ Area of ∆ABY = Area of ∆AXY
− ∠
= 180° −  
features, i.e. metameric segmentation, 180 A
+ Area of ∆BXY 
jointed appendages and chitinous  2 
exoskeleton are present in phylum = 10 + 20 = 30 ∠A
= 90° + = 90 + 20 = 110
Arthropoda. Out of these, metameric 2
segmentation is visible as tagmetisation. A
1 [Q ∠A = 40° ]
60. (c) Anthocyanins are water soluble 2 ∠BIC = ∠QIR = 40°
X
vacuolar pigments that may appear red, 1
Y ∠QPR = QIR
purple or blue depending on pH. It is 2 2
1
impermeable to cell membranes of plants
[Q angle in a segment is half of angle
and can leak out only when membrane is B C
on a centre segment of circle]
damaged or dead. Area of ∆ABY 2 1
⇒ = ∠QPR = × 110° = 55°
61. (a) Let the set S has n elements. Area of ∆CBY 1 2
∴Mean of S and {15} 30 66. (c) By homogenity principle,
S + 15 ∴ Area of ∆CBY = = 15
m+ 2= 2 [a ] = dimensions of power
n+1 W
∴ Area of ∆ABC = Area of ∆ABY = [ML2 T −3 ]
⇒ (n + 1) (m + 2) = S + 15 T
+ Area of ∆CBY
Q m = S  [bt 2 ] = dimensions of power
(n + 1) (m + 2) = nm + 15 = 30 + 15 = 45
 n  [ML2T −3 ]
⇒ [b] = = [ML2 T −5 ]
⇒ m + 2n = 13 ...(i) 64. (a) We have, [T 2 ]
Also, mean of S and {15, 1} is x1 + x2 + x3 + K + x2019 = 1 [c] = dimensions of t3 = [T3 ]
S + 15 + 1 and 1 +
x x2
+ K+
x2019
=1
−1 3
[d t ] = dimensions of power
m + 1=
n+ 2 1 − x1 1 − x2 1 − x2019 [T3 ]
⇒ [d ] = = [M−1 L−2 T −6 ]
⇒ (n + 2) (m + 1) = S + 16 x12 x22 x32 2
x2019 [ML2T −3 ]
⇒ + + + K+
= nm + 2m + n + 2 = nm + 16 1 − x1 1 − x2 1 − x3 1 − x2019 So, statement IV is incorrect.
⇒ n + 2m = 14 ...(ii) x2 − x1 + x1 x2 − x2 + x2 67. (b) Before collision, velocity of
= 1 + 2 +K
From Eqs. (i) and (ii), we get 1 − x1 1 − x2 translation of sphere
n=4 x2 − x2019 + x2019 = vcube = rω = 1 cms−1 = 0.01 ms−1
+ 2019
∴S has 4 elements. 1 − x2019 As collision is elastic, translational
62. (b) We have, (x, y) = GCD of x and y. x1 (x1 − 1) x1 x2 (x2 − 1) kinetic energy of sphere is transferred to
= + + + x2 the cube but its rotational kinetic energy
Given, xy = x + y + (x, y) 1 − x1 1 − x1 1 − x2
remains constant.
xy − x − y = (x, y) x2019 x2019 − 1) x2019 ∴ After collision, vsphere = 0,
+ K+ +
xy − x − y + 1 = (x, y) + 1 1 − x2019 1 − x2019 vcube = 0.01 ms−1 and ωsphere = 1rad s−1 .
https://iit-jeeacademy.blogspot.com

KVPY Practice Set 3 Stream : SA 205

68. (b) By KVL, 71. (d) 75. (b) ∆G° is related to K sp by the
C12H22O11 (s) + 12O2 ( g ) → 12CO2 ( g ) equation
a 2kΩ
+ 11H2O(l) ∆G° = − 2.303 RT log K sp
1kΩ b D a+b ∆H C° = [12∆ f H ° (CO2 ) + 11∆ f H ° (H2O)] ∆G° = + 63.3 kJ = 63.3 × 103 J
A
1–a C 1kΩ 1kΩ B − [∆ f H ° (C12H22O11 )] 63.3 × 103 = − 2.303 × 8.314 × 298 × log Ksp
= [12(−94.3) + 11 (− 68.3)] − [− 530] log K sp = − 11.09
1–a–b 2kΩ = − 1352 .9 J kcal mol −1 K sp = 8.0 × 10−12
we have, V AD = 2a = 1 − a + b Thus, number of moles of C12H22O11 76. (b) The nerves are myelinated with
Also, VCD = 2 (1 − a − b) = b + a + b required for 2700 kcal of energy unmyelinated segments called nodes of
2700 Ranvier. The high phospholipid content
∴ a=
2 = ≈ 2 mol = 682.6 g
1352.9 of the myelin sheath offers electrical
5
insulation, thus saltatory conduction
1 72. (d) The alcohol on treatment with
and b= occurs as impulse jumps from one node to
5 acid gives an alkene (A) which on the next. This form of conduction
Hence, current through 2 kΩ resistor ozonolysis will give nonan-2, 8 dione. The facilitates a very rapid transmission of
2 reaction can be shown as impulses.
= a = mA.
5 CHOHMe + 77. (a) The cross for the question is
H
and current through middle 1 kΩ resistor YyRr × YyRr
Me
1
= b = mA. A
Gamets
5
O3
YR yR Yr yr YR yR Yr yr
69. (b) Change in momentum of steel
ball = − 2mv 3 1 YR yR Yr yr
2
2mv 2mv 4
Force on U-tube = = O
∆t  πd  5 YR YYRR YyRR YYRr YyRr
 
 2v  6 7
8 9

Nonan-2, 8 dione
4 mv2 4 × 0.02 × 9 yR YyRR yyRR YyRr yyRr
= = 73. (c) Number of equivalents of silver
πd . ×1
314
formed = number of equivalents of copper Yr YYRr YyRr YYrr Yyrr
≈ 0.23 N formed.
= 0.2 N In AgNO3 , Ag is in + 1oxidation state. yr YyRr yyRr Yyrr yyrr

70. (a) As, U = 2−2x In CuSO4 , Cu is in + 2 oxidation state. The given Punnett square shows
x + 4 108
∴ Equivalent weight of Ag = = 108 9 : 3 : 3 : 1 ratio of the phenotypes only.
dU −2 (− 2x) (− 1) (2x) 1
⇒ = 2 +
dx x + 4 (x2 + 4)2 Equivalent weight of Cu 78. (b) As the substrate concentration
63.6 increases, the rate of reaction increases
−2 4x 2
= = 31.8
= + 2 until a maximum, when saturation of all
x2 + 4 (x2 + 4)2 the enzymes active sites occurs. When this
Weight of silver Eq. wt of silver
dU 2 4x2 = happens, the limiting factor is enzyme
= 0 when 2 = 2 Weight of copper Eq. wt of copper concentration. Thus, graph (b) is correct.
dx x + 4 (x + 4)2 10.79 108
= 79. (c) Average amino acid residues
⇒ 2(x2 + 4) = 4x2 wCu 31.8 molecular weight = 110
⇒ 2x2 = 8 10.79 × 31.8 Polypeptide chain of molecular weight
wCu = 20,000
⇒ x=± 2 108 20,000 = = 182 amino acids
= 3.2 g 110
As U is minimum at x = + 2 .
− dU hc A triplet of bases in the DNA molecule
∴ F= =0 74. (c) Energy of one photon = codes for one amino acid in a polypeptide
dx λ
chain.
Now, restoring force constant. 6.626 × 10−34 Js × 3 × 108 ms−1
= To translate 182 amino acids, there must
d 2U 1 550 × 10−9 m be a minimum of 182 × 3 = 546
k= = units
dx 2
8 = 3.61 × 10−19 J nucleotides.
at x = 2
∴ Number of photons 80. (b) Sequoia is one of the tallest tree
k 1  1  energy required species, known as red wood tree. It is a
∴ ω= = =  
−3  = gymnospermic plant.
m 8m  8 × 5 × 10  energy of one photon
Salvinia is an angiosperm, but Ginkgo
100 10−17 and Pinus are gymnosperms.
= =
4 3.61 × 10−19 Gymnosperms are well-adapted to
extremes of climate and are
= 5 rad s−1 = 27.67 ≈ 28 heterosporous.
https://iit-jeeacademy.blogspot.com

206 KVPY Practice Set 4 Stream : SA

KVPY
KISHORE VAIGYANIK PROTSAHAN YOJANA

PRACTICE SET 4
Stream : SA
MM 100

Instructions
There are 80 questions in this paper.
This question paper contains two parts; Part I and Part II. There are four sections; Mathematics, Physics, Chemistry
and Biology in each part.
Out of the four options given with each question, only one is correct.

PART-I (1 Mark Questions)


MATHEMATICS 5. If a , b, c ≥ 4 are integers, not all equal and
1. A natural number K is such that K 2 < 2019 < (K + 1)2. 4abc = (a + 3) (b + 3) (c + 3), then (a + b + c) is equal to
(a) 14 (b) 15 (c) 16 (d) 18
Then, the largest prime factor of K is
(a) 11 (b) 13 (c) 7 (d) 5 6. In a ∆ABC, right angled at A, the altitude through A
and the internal bisector of ∠A have lengths 3 and 4
2. If real number a, b, c, d, e satisfy
respectively. Then, the length of median through A is
a +1=b+2=c+3=d +4=e+5
(a) 20 (b) 24 (c) 15 (d) 10
= a + b + c + d + e + 3, then the value of
a 2 + b2 + c2 + d 2 + e2 is equal to 7. A rectangular floor that is 10 feet wide and 17 feet
long is tiled with 170 one-foot square tiles. A bug
(a) 8 (b) 9 (c) 10 (d) 11
walks from one corner to the opposite corner in a
3. Let a semi-circle with centre O and diameter AB. Let straight line including the first and the last tile, how
P and Q be points on the semi-circle and R be a point many tiles does the bug visit?
on AB extended such that OA = QR < PR if
(a) 17 (b) 25 (c) 26 (d) 27
∠POA = 102°, then ∠PRA is equal to
(a) 51° (b) 34° 8. Ashwani computes the mean µ, the median M and
(c) 25.5° (d) None of these the modes of the 365 values that are the dates of
2019. Thus his data consists of 12 1s, 12 2s ..., 12 28s,
4. If x = cos 1° cos 2° cos 3° K cos 89° and
11 29s, 11 30s and 7 31s. Let d be the median of
y = cos 2° cos 6° cos 10° K cos 86°, then the integer modes. Which of the following is correct?
2  y
nearest to log 2   is (a) µ < d < M (b) M < d < µ
7  x (c) M = d = µ (d) d < µ < M
(a) 16 (b) 17 (c) 18 (d) 19
https://iit-jeeacademy.blogspot.com

KVPY Practice Set 4 Stream : SA 207

9. A sequence of numbers is defined recursively by v


3 an − 2 ⋅ an − 1 (ms–1)
a1 = 1, a 2 = and a n = for all n ≥ 3.
7 2a n − 2 − a n − 1 15
p
Then, a 2019 can be written as , where p and q are
q
t(s)
relatively prime number, then the value of p + q is 3 4
equal to Now, consider the following statements:
(a) 6057 (b) 8087 (c) 8078 (d) 4039 I. Force acting on particle is 50 N.
10. A child builds towers using identically shaped cube of II. Force stops at t = 3 s.
different color. Then, number of different tower with
III. Force stops at t = 4 s.
a height 8 cubes can the child build with 2 red cubes,
3 blue cubes and 4 green cubes (one cube is left out) IV. Particle receives an impulse at t = 4 s.
is Which of the above statements are correct?
(a) 24 (b) 288 (c) 312 (d) 1260 (a) Statements II and IV are correct
11. The least possible value of (b) Statements I, II and IV are correct
(x + 1) (x + 2) (x + 3) (x + 4) + 2019 is (where, x is real) (c) Statements I, III and IV are correct
(a) 2017 (b) 2018 (c) 2019 (d) 2020 (d) Statements III and IV are correct

12. Two circles of radius 5 are externally tangent to each 17. Correct graph of experimental values of specific heat
other and are internally tangent to a circle of radius of a constant volume of hydrogen gas is
13 at points A and B, as shown in the figure. The (a) CV (b) CV
m
distance AB can be written in the form , when m
n 3
and n are relatively prime. —R 5/2 R
2
3/2 R
T(K) T(K)
80 3000 80 600

(c) CV (d) CV

A B 7/2 R 7/2 R
Then, m + n is 5/2 R
(a) 21 (b) 29 (c) 69 (d) 58 3/2 R 3/2 R
T(K) T(K)
13. A person X is running around a circular track 80 3000 80 600 3000
completing one round in 40 s. Another person Y
running in opposite direction meets X every 15 s. The 18. Ratio of nuclear density of nuclei 142
53 I and 56 Ba is
139

time, expressed in seconds, taken to Y to complete (a) 142 : 139 (b) 53 : 56


one round is (c) 139 : 142 (d) None of these
(a) 12.5 (b) 24 (c) 25 (d) 55 19. A particle starts from origin, it accelerates first
14. Consider all 6-digit numbers of the form abccba, t0 second and then deaccelerates at same rate till 2 t0
where b is odd. Then, number of all such 6-digit second along the positive x-direction. Variation of
numbers that are divisible by 7 is displacement x with time t for the particle is given by
(a) 70 (b) 80 (c) 75 (d) 85 (a) x (b) x
15. Let ABCD be trapezium in which AB is parallel to
CD and AD is perpendicular to AB. Suppose ABCD
has incircle which touches AB at Q and CD at P.
Given that PC = 36 and QB = 49, then length PQ is t t
t0 2t0 t0 2t0
(a) 85 (b) 84 (c) 76 (d) 80
(d) x
(c) x

PHYSICS
16. Velocity-time graph of a particle of mass 10 kg
pushed along a frictionless surface by an external
force is as shown below. t t0 2t0
t
t0 2t0
https://iit-jeeacademy.blogspot.com

208 KVPY Practice Set 4 Stream : SA

20. A mass m initially at rest is pulled with a force F. 15Ω


A
If force is proportional to instantaneous time t, then
kinetic energy of the particle is proportional to R1 2Ω 3Ω
(a) t 2 (b) t −2 (c) t 4 (d) t 0
B
21. Density of ice is x (g/cc) and that of water is g (g/cc).
(a) 20Ω (b) 10Ω
Change in volume in cc when m grams of ice
(c) 5Ω (d) 25Ω
completely melts is
(a) m ( y − x) (b) ( y − x) / m 26. Fermi energy level for an electron is
 1 1 (a) a possible energy value that an electron can have in
(c) mxy ( y − x) (d) m  − 
 y x free state
(b) an unfilled energy level that can be occupied by two
22. How much work is required in units of electron-volt electrons of opposite spins
to carry an electron from the positive terminal of a (c) lowest energy value possible for a bound electron
12 V battery to the negative terminal in external (d) highest occupied energy level at absolute zero kelvin
circuit? upto which every possible energy levels are filled
(a) 12 eV (b) − 12 eV 27. Solar cookers are not very popular because
(c) 6 eV (d) − 6 eV (a) they are bulky
23. Following graph shows atmospheric pressure, gauge (b) they are not put into kitchen
pressure and absolute pressure. (c) they cook food in large time
p (d) sun changes its position rapidly
28. A cubical block of side 1 m and mass 10 kg is placed
p2 on a rough surface. Block can be toppled by applying
p3 a force horizontally at its upper edge. Minimum
p1 value of F is

F
Then, choose the correct option.
(a) p1 = gauge pressure, p2 = atmospheric pressure, (a) 100 N (b) 200 N
p3 = absolute pressure (c) 50 N (d) 25 N
(b) p1 = atmospheric pressure, p2 = gauge pressure,
p3 = absolute pressure 29. Potential energy between two molecules as a function
(c) p1 = absolute pressure, p2 = atmospheric pressure, of their separation is as shown below.
p3 = gauge pressure U
(d) p1 = gauge pressure, p2 = absolute pressure,
p3 = atmospheric pressure
24. A man can walks on hard ground with a speed of
0.6 Å 1.8 Å
5 ms−1 and on sandy ground with 3 ms−1. x
0.4 Å 1.2 Å
Let he is standing on border of sandy and hard
ground and wishes to reach the tree situated on the
sandy ground as shown below.
Force between particles is zero at
O
D Tree (a) x = 0.4 Å (b) x = 0.6 Å
(c) x = 12
. Å (d) x = 18
. Å
Man 120 m 30. Using following figures,
E
A B
100 m C
He can reach tree in least time when he walks on µ1 µ1 µ2 µ µ2
µ
hard ground upto point E and then he walks straight
towards tree along EO. Distance AE is
(a) 10 m (b) 20 m
(c) 30 m (d) 50 m Relation between refractive indices µ1 and µ 2 is
25. Equivalent resistance between A and B is 6 Ω. Value (a) µ 1 < µ 2 (b) µ 1 µ 2 = µ 2
of resistance R1 is (c) µ 1 > µ 2 (d) µ 1 = µ 2
https://iit-jeeacademy.blogspot.com

KVPY Practice Set 4 Stream : SA 209

Cu 2+ / Cu (s), E° = + 034.
CHEMISTRY
I2 (s) / I− , E° = + 054
.
31. How many moles of magnesium phosphate (a) Cu will reduce Br − (b) Cu will reduce Ag
Mg 3 (PO 4 )2 will contain 0.25 mole of oxygen atom? (c) Cu will reduce I− (d) Cu will reduce Br2
(a) 0.02 (b) 3125
. × 10−2
39. The carboxylic acid which reduces Tollen’s reagent is
. × 10−2
(c) 125 (d) 2.5 × 10−2
(a) acetic acid (b) oxalic acid
32. At what temperature will the r.m.s velocity of SO 2 be (c) formic acid (d) lactic acid
the same as that of O 2 at 303 K ? 40. The correct order of C—O bond length among
(a) 350 K (b) 505 K CO, CO32− , CO 2 is
(c) 606 K (d) 100 K
(a) CO2 < CO32− < CO (b) CO < CO32− < CO2
33. When acidified sodium extract of organic compound (c) CO32− < CO2 < CO (d) CO < CO2 < CO32−
is treated with acetic acid and lead acetate, a black
41. When 22.4 L of H2 ( g) is mixed with 11.2 L of Cl 2 ( g)
precipitate is obtained. This suggests that the
organic compound contains each at STP, the moles of HCl( g) formed is equal to
(a) chlorine (b) phosphorus (a) 1 mole of HCl ( g ) (b) 2 moles of HCl ( g )
(c) sulphur (d) nitrogen (c) 0.5 mole of HCl ( g ) (d) 1.5 moles of HCl ( g )

34. Acetone is treated with excess of ethanol in the 42. A 1 L flask contains 32g of O 2 gas at 27°C. What
presence of hydrochloric acid. The product obtained is mass of O 2 must be released to reduce the pressure in
O the flask to 12.315 atm?
 (a) 8 g (b) 16 g (c) 24 g (d) 0 g
(a) CH3 CH2CH2 C CH3
O
43. What is the orbital angular momentum of an electron
 in f-orbital ?
(b) CH3 CH2CH2 C CH2CH2CH3 15
. h 6h 3h 3h
(a) (b) (c) (d)
π π π π
OH
(c) (CH3)2C 44. Aluminium trifluoride is treated with anhydrous HF
OC2H5 and then with NaF. When gaseous BF3 is passed
through the solution obtained, a precipitate X is
OC2H5 formed. The formed pricipitate X is
(d) (CH ) C (a) Na3[AlF6 ] (b) Na[BF4 ] (c) AlF3 (d) H3 [AlF6 ]
3 2
OC2H5 45. Which one of the following will be aromatic?
+
35. The electronegativity of the following elements
(a) (b)
increases in the order
(a) C < N < Si < P (b) N < Si < C < P
(c) Si < P < C < N (d) P < Si < N < C
36. The de-Broglie wavelength associated with particle of (c) (d) ρ
−6 −1 σ
mass of 10 kg moving with a velocity of 10 ms is
(a) 6.63 × 10−7 m (b) 6.63 × 10−16 m
(c) 6.63 × 10−21 m (d) 6.63 × 10−29 m BIOLOGY
37. The product formed when 1-bromo-3- 46. Which cells of ‘crypts of Lieberkuhn’ secrete
chlorocyclobutane reacts with two equivalents of antibacterial lysozyme?
metallic sodium in ether, is (a) Argentaffin cells (b) Paneth cells
Cl (c) Zymogen cells (d) Kupffer cells

(a) (b) 47. Which among the following are the smallest living
cells, known without a definite cell wall, pathogenic
Br to plants as well as animals and can survive without
oxygen?
(c) (d) (a) Bacillus (b) Pseudomonas
(c) Mycoplasma (d) Nostoc
38. E° values of some redox couples are given below. On
the basis of these values choose the correct option. 48. The cell organelle responsible for extracting energy
from carbohydrates to form ATP is
Br2 / Br− , E ° = + 190
.
(a) lysosome (b) ribosome
Ag / Ag (s), E° = + 080
+
.
(c) chloroplast (d) mitochondrion
https://iit-jeeacademy.blogspot.com

210 KVPY Practice Set 4 Stream : SA

49. DNA fragments are 55. Fruit and leaf drop at early stages can be prevented
(a) positively charged by the application of
(b) negatively charged (a) cytokinins
(c) neutral (b) ethylene
(d) either positively or negatively charged depending on (c) auxins
their size (d) gibberellic acid
50. An important characteristic that hemichordates 56. Which one of the following options best represents
share with chordates is enzyme composition of pancreatic juice?
(a) absence of notochord (a) Amylase, peptidase, trypsinogen, rennin
(b) ventral tubular nerve cord (b) Amylase, pepsin, trypsinogen, maltase
(c) pharynx with gill slits (c) Peptidase, amylase, pepsin, rennin
(d) pharynx without gill slits (d) Lipase, amylase, trypsinogen, procarboxypeptidase
51. Lungs are made up of air-filled sacs, the alveoli. They 57. In the fruit fly, Drosophila melanogaster, the diploid
do not collapse even after forceful expiration, because number of chromosomes is 8. In the absence of
of crossing over or mutation, how many genetically
(a) Residual Volume (RV) unique kinds of gamete might be formed by one
(b) Inspiratory Reserve Volume (IRV) individual?
(c) Tidal Volume (TV) (a) 4 (b) 8
(d) Expiratory Reserve Volume (ERV) (c) 16 (d) 32
52. Viroids differ from viruses in having 58. Biochemical analysis of a sample of DNA shows that
(a) DNA molecules with protein coat cytosine forms 40% of the nitrogenous bases. Which
(b) DNA molecules without protein coat percentage of the bases is adenine?
(c) RNA molecules with protein coat (a) 10% (b) 20%
(d) RNA molecules without protein coat (c) 40% (d) 60%
53. Plants, which produce characteristic pneumatophores 59. The first stable product of fixation of atmospheric
and show vivipary belong to nitrogen in leguminous plant is
(a) mesophytes (b) halophytes (a) NO−2 (b) ammonia
(c) psammophytes (d) hydrophytes (c) NO3− (d) glutamate
54. Spliceosomes are not found in cells of 60. Treponema pallidum pathogen is a cause of
(a) plants (b) fungi (a) leprosy (b) plague
(c) animals (d) bacteria (c) syphilis (d) pertussis

PART-II (2 Marks Questions)


MATHEMATICS respectively. Let AI1 and AI 2 meet BC in E and F
respectively. If ∠BI1E = 60°, then ∠ CI 2F is (in
61. Integer a, b, c satisfy a + b − c = 1 and degree)
a 2 + b2 − c2 + 1 = 0, then the sum of all possible values (a) 30° (b) 45° (c) 75° (d) 60°
of a 2 + b2 + c2 is equal to 2
64. Let P (x) = a 0 + a1x + a 2x + K + a n x be a polynomial
n

(a) 17 (b) 18 (c) 20 (d) 24 in which a i is a non-negative integer for each


62. There are several tea cups in the kitchen, some with i ∈(0, 1, 2, 3, K , n ). If P(1) = 4 and P(5) = 136, then P(3)
handles and others without handles. The number of is
ways of selecting two cups without a handle and (a) 25 (b) 30 (c) 32 (d) 34
three with a handle is exactly 1200. Then, the 65. In a quadrilateral ABCD, it is given that
maximum possible numbers of cups in the kitchen is AB = AD = 13, BC = CD = 20, BD = 24. If r is the
equal to radius of the circle inscribed in the quadrilateral,
(a) 25 (b) 27 (c) 28 (d) 29 then the integer closest to r is
63. Let D be an interior point of the side BC of a ∆ABC. (a) 6 (b) 8 (c) 9 (d) 10
Let I1 and I 2 be the incenters of ∆ABD and ∆ACD
https://iit-jeeacademy.blogspot.com

KVPY Practice Set 4 Stream : SA 211

69. Velocity-time graph of an object moving along a


PHYSICS straight line is as shown below.
66. Two earthworms climb over a rough thin wall of an v
earthen pot 10 cm high placed in a lawn. (ms–1)
8
Earthworm

Wall 9
t(s)
3 5 7

One of the worm is 20 cm long and other is only


–8
10 cm long and mass of both earthworms is 20 g.
Ratio of work done by worms when they crosses half
of their length across top of the wall is If x = displacement (in m) and a = acceleration (in ms−2).
(a) 1 : 1 (b) 2 : 3 Then, correct graph is
(c) 2 : 1 (d) 1 : 2
(a) x(m) (b) x(m)
67. A jar of height 20 cm is filled with water (nw = 4 / 3).
At centre of jar on the bottom surface, a red dot is 28
made.
t(s)
t(s) 9
9

Disc a
a
(c) (ms–2) (d) (ms–2)
2.67
Water

Dot t(s) 5 9
9 t(s)
3
Minimum radius of an opaque plastic disc that
makes the dot invisible from top is
(a) 20 cm (b) 23 cm
70. A vessel has a hole of radius r = 1 cm. Vessel is
(c) 12 cm (d) 2 cm
initially full of water and hole is sealed by a ball of
68. Consider arrangements A and B for making a torch: mass m = π g. Depth of water is now slowly reduced
using a syphon and when it reaches a certain value
h0, the ball rises out of the hole.
+

– +

+ – +

(B)

Value of h0 is
(A)
(Radius of ball is slightly larger than hole but for
Now, consider the following statements. calculation both can be taken same, g = 10 ms−2)
I. Torch A is brighter. (a) 65 cm (b) 72 cm (c) 84 cm (d) 110 cm
II. Torch B is brighter.
III. Torch A lasts longer. CHEMISTRY
IV. Torch B lasts longer.
71. A solid compound X on heating gives CO 2 gas and a
Which of the above statements are correct?
residue. The residue mixed with water form Y. On
(a) Statements I and III are correct
passing an excess of CO 2 through Y in water, a clear
(b) Statements II and IV are correct
solution Z is formed. On boiling Z, compound X is
(c) Statements I and IV are correct
reformed. The compound X is
(d) Statements II and III are correct
(a) Ca(HCO3 )2 (b) CaCO3 (c) Na2CO3 (d) K 2CO3
https://iit-jeeacademy.blogspot.com

212 KVPY Practice Set 4 Stream : SA

72. Standard entropies of X 2 , Y 2 and XY3 are 60, 40 and 77. Haemophilia is caused by a sex-linked, recessive
−1 −1
50 JK mol , respectively. For the reaction, allele. A couple have a haemophilic son, a normal son
and a haemophilic daughter. What are the most
1 3
X + Y XY3 ; ∆H = − 30 kJ
- likely genotypes of the parents?
2 2 2 2
Mother Father
To be at equilibrium, the temperature should be
(a) XH X XH Y
(a) 750 K (b) 1000 K (c) 1250 K (d) 500 K
H 2 /Ni O / H O/ Zn (b) XH Y XH Y
73. A (C 4 H6 )  
→ B (C 4 H8 )  
3 2
→ CH3CHO H H
1 mol (c) X X XH Y
Identify A and B in the above reaction. (d) X YH H
XH Y

(a) and 78. Which of the following statements is correct in


relation to the endocrine system?
(b) and
(a) Adenohypophysis is under direct neural regulation of
(c) CH 3 CH 2 C ≡≡ CH and CH 3 CH == CHCH 3 the hypothalamus
(d) CH 2 ==CHCH == CH 2 and CH3 CH == CHCH3 (b) Organs in the body like gastro-intestinal tract, heart,
kidney and liver do not produce any hormones
74. In the Kjeldahl’s method for the estimation of (c) Non-nutrient chemicals produced by the body in trace
nitrogen present in a soil sample, ammonia evolved amount that act as intercellular messenger are known
from 0.75 g of sample neutralised 10 mL of 1 M as hormones
H2SO 4. The percentage of nitrogen in the soil is (d) Releasing and inhibitory hormones are produced by
(a) 37.33 (b) 45.33 (c) 35.33 (d) 45.33 the pituitary gland
75. A carbon compound contains 12.8% of carbon, 2.1% of 79. If the nucleus of a human motor neuron contains
hydrogen and 85.1% of bromine. The molecular 6.8 picograms (pg) of DNA, what mass of DNA is the
weight of the compound is 187.9. The molecular nucleus of an actively dividing human skin cell likely
formula of the compound is to contain at the end of interphase?
[Atomic weight of H = 1008
. , C = 12.0 and Br = 799
.] (a) 3.4 pg (b) 6.8 pg
(a) CH3 Br (b) CH 2 Br2 (c) C2 H 4 Br2 (d) C2 H 3 Br3 (c) 13.6 pg (d) 20.4 pg
80. The diagram represents a reaction with and without
an enzyme. What is the activation energy of the
BIOLOGY enzyme catalysed reaction?
76. The complete oxidation of one mole of glucose yields
2880 kJ of energy. The addition of one phosphate A D
molecule to ADP requires 30.6 kJ of energy per mole.
Energy

In aerobic respiration, 38 molecules of ATP are Reactants B

formed as a result of the breakdown of each glucose Product C


molecule. Which figure best represents the efficiency
Reaction
of aerobic respiration in trapping the energy released
by the glucose molecule? (a) A (b) B
(a) 23% (b) 36% (c) 40% (d) 45% (c) C (d) D

Answers
PART-I
1 (a) 2 (c) 3 (d) 4 (d) 5 (c) 6 (b) 7 (c) 8 (d) 9 (c) 10 (d)
11 (b) 12 (c) 13 (b) 14 (a) 15 (b) 16 (b) 17 (d) 18 (d) 19 (c) 20 (c)
21 (d) 22 (a) 23 (b) 24 (a) 25 (b) 26 (d) 27 (d) 28 (c) 29 (a) 30 (a)
31 (b) 32 (c) 33 (c) 34 (d) 35 (c) 36 (d) 37 (d) 38 (d) 39 (c) 40 (d)
41 (a) 42 (b) 43 (d) 44 (b) 45 (d) 46 (b) 47 (c) 48 (d) 49 (b) 50 (c)
51 (a) 52 (d) 53 (b) 54 (d) 55 (c) 56 (d) 57 (c) 58 (a) 59 (b) 60 (a)

PART-II
61 (b) 62 (d) 63 (a) 64 (d) 65 (b) 66 (b) 67 (b) 68 (c) 69 (d) 70 (a)
71 (b) 72 (a) 73 (d) 74 (a) 75 (c) 76 (c) 77 (a) 78 (c) 79 (c) 80 (b)
https://iit-jeeacademy.blogspot.com

KVPY Practice Set 4 Stream : SA 213

Solutions
22 2
1. (a) We have, 1  1 + 3  ≥ 16
= ( Π sin 4r )2  
K 2 < 2019 < (K + 1)2 266 2 r =1  b 7
∴ 2
K < 2019 1 22 3
= Π sin 4r ⇒ b≤ <6
266 2 r =1 4
⇒ K< 2019 −1
1 22 7
⇒ K < 44.93 = Π sin (92 − 4r )
66 49
∴ K = 44 2 2 r =1 ⇒ b = 4 or 5 for b = 4, c = ∈I
22 5
1
⇒ K = 4 × 11 = Π cos (4r − 2) For b = 5, c = 7
66
2 2 r =1
The largest prime factor of K is 11. ⇒ a + b + c = 4 + 5 + 7 = 16
1
2. (c) We have, ⇒ x= y
266 2 6. (b) Given,
a + 1= b + 2 = c + 3 = d + 4 = e + 5 1 133 ABC is a right angled triangle
y 66 +
= a+ b+ c+ d + e+ 3 ⇒ =2 = 2 2 2 ∠A = 90°
x
∴ b = a − 1, c = a − 2, d = a − 3, e = a − 4
2 y 2 133 C
⇒ a+ b+ c+ d + e= a− 2 ∴ log 2 = × = 19
7 x 7 2 M
a = 2, b = 1, c = 0, d = − 1, e = − 2 5. (c) We have, N a
∴a 2 + b2 + c2 + d 2 + e2 4abc = (a + 3) (b + 3) (c + 3) b 4
L
= (2)2 + (1)2 + (0)2 + (−1)2 + (−2)2 a , b, c ≥ 4, a , b, c are integers. 3
= 4 + 1 + 0 + 1 + 4 = 10 1 + 3  1 + 3 1 + 3 = 4 45°
     
3. (d) Given, AB is diameter of  a  b  c A c B
semi-circle. ⇒ 4≤ a ≤ b≤ c Altitude AL = 3
1 1 1
Q′ P ⇒ ≥ ≥ AN = 4 (AN is angle bisector of ∠A)
a b c
Q 3 3 3 AM is median of ∆ABC
⇒ 1+ ≥ 1+ ≥ 1+ 1 1
a b c Area of ∆ABC = bc = AL a
102° 2 2
3
∴  1 + 3  ≥ 4 ⇒1 + 3 ≥ (4)1/3
R′ r A r O B   ⇒ bc = 3a
 a a
Area of ∆ABC = Area of ∆ABN + Area of
Since, R can be any point. Let A and R′ 3 4−1
⇒ a ≤ 1/3 = ∆ANC
and hence, its corresponding point Q lie 4 − 1 41/3 − 1 1 1
on the arc AQ′. ⇒ bc = AB ⋅ AN sin 45°
= 423/
+ 41/3 + 1 2 2
Hence, ∠PRA cannot be determined. 1
⇒ a≤ 3+ 2+ 1 + AC ⋅ AN sin 45°
4. (d) We have, 2
∴ a < 6 ⇒ a = 4 or 5
x = cos 1° cos 2° cos 3° K cos 89° 1 1 1
3 3 5 ⇒ bc = × 4 × (b + c)
and y = cos 2° cos 6° cos 10° K cos 86° For a = 5,  1 +   1 +  = 2 2 2
 b  c 2
89
2 ⇒ bc = 2 2 (b + c)
Let x = π cos r ° 1 + 3 ≥ 5
r =1 

 ⇒ b2c2 = 8(b2 + c2 + 2bc)
b 2
89
3 ⇒ 9a 2 = 8(a 2 + 6a )
⇒x = Π cos r ° cos (89 + 1 − r )° ⇒ b≤
r =1
 5
1/ 2 ⇒ 9a − 8a 2 = 48a
2

89   −1
1  2 ⇒ a = 48
= Π sin 2r °
289 r = 1  5  BC a
= 2 + 1 < 2(2 + 1) = 6 Q AM = MC = MB = =
1 44
 2  2 2
= ( Π sin 2r ° )2 sin 90°
289 r =1 48
⇒ b < 6 ⇒ b ≤ 5, b = 5 (b ≥ a ) Q AM = = 24
44 3 25 2
1
= Π sin 2r ° ⇒ 1+ =
244 2 r =1 c 16 7. (c) The number of tiles the bug visits
16 is equal to 1 plus the number of times it
1 44 ⇒ c= ∉integer crosses a horizontal or vertical line. As it
= 44
Π sin 2r ° sin 2 (44 + 1 − r ) 3
2 2 r =1 must cross 16 horizontal lines and
⇒ a≠5
1 44 9 vertical lines. It must be that bug visits
3  3 16
= 66
Π sin 4r For a = 4,  1 +   1 +  =
 a total of 16 + 9 + 1 = 26 squares.
2 2 r =1  b  c 7
https://iit-jeeacademy.blogspot.com

214 KVPY Practice Set 4 Stream : SA

8. (d) Mean (µ ) of 365 values of the takes 11. (b) We have, So, (a , = {(9, 2) (8, 1) (7, 0) (2, 9) (1, 8)
2019 are (x + 1) (x + 2) (x + 3) (x + 4) + 2019 (9, 9) (8, 8) (7, 7) (6, 6) (5, 5) (4, 4) (3, 3)
12(1 + 2 + 3 + K + 28) (2, 2) (1, 1)}
= (x + 1) (x + 4) (x + 2) (x + 3) + 2019
+ 11 × 29 + 11 × 30 + 7 × 31 Number of pairs of (a , b) = 14
µ= = (x2 + 5x + 4) (x2 + 5x + 6) + 2019
365 Also b can be 5
= (x2 + 5x)2 + 10(x2 + 5x) + 24 + 2019
12 × 28 × 29 Q Total number of 6-digit number
+ 319 + 330 + 217 = (x2 + 5x)2 + 10(x2 + 5x) + 25
µ= 2 = 15.7 = 14 × 5 = 70
− 25 + 24 + 2019
365 15. (b) Given, PC = 36
366 th = (x2 + 5x + 5)2 + 2018
Median = observation BQ = 49
2 ∴Minimum value of x2 + 5x + 5 is 0
= 183 th observation ∴Minimum value of (x + 1) (x + 2) (x + 3) C 36 P x D
= 16 (x + 4) + 2019 is 2018. 36
x x
d (Median of modes) 12. (c) Given, PA = 13 = PB S
x
R
Mode of data = 1, 2, 3, K , 28 QA = RB = 5 49
 28  th +  28 + 1 th x x
   
   2 
Median of data = 2 B 13 T 36 Q x A
2
14 + 15 P ∴ BT = BQ − TQ
= = 14.5 = 49 − 36 [QTQ = PC ]
2
Q d<µ< M Q R BT = 13
In ∆BTC, BC 2 = TC 2 + BT 2
9. (c) We have, A B
3 ⇒ (85)2 = PQ 2 + (13)2 [QTC = PQ ]
a1 = 1, a2 = Q PQ = PA − QA
7 ⇒ PQ 2 = (85)2 − (13)2
= 13 − 5 = 8
an − 2 ⋅ an − 1 ⇒ PQ 2 = (85 + 13) (85 − 13)
an = PR = PB − RB = 8
2an − 2 − an − 1 ⇒ PQ 2 = 98 × 72
QR = 2QA = 10
3 ⇒ PQ = 49 × 2 × 36 × 2
1⋅ ∆PQR ~ ∆PAB
a1 ⋅ a2 7 = 3 ⇒ PQ = 7 × 6 × 2 = 84
a3 = = PQ QR
=
2a1 − a2 2 − 3 11 16. (b) F = ∆p = 10 × 15 = 50 N
Q
PA AB
7 10 × 13 65 ∆t 3
a2 − a3 ⇒ AB = = As velocity is constant after t = 3 s, hence
a4 = 8 4
2a2 − a3 force stops.
Q m = 65, n = 4
3 3 17. (d) At low temperature, H2 molecule
− Hence, m + n = 65 + 4 = 69
3
= 7 11 = has only translational degrees of freedom
 3 3 15 13. (b) Distance travelled by X in 40 s.
2  − ∴ f =3
 7  11 2π
θ= × 15 ...(i) Above 600 K molecule be given to vibrate
3 3 3 3 3 40
Q Sequence are 1, , , , , K , and at above 3000 K molecule dissociates.
7 11 15 19 4n − 1
18. (d) Nuclear density is a constant
3 (ρ = 2.38 × 1017 kg m−3 ). It is independent
Q an =
4n − 1 θ
of nuclear size and number of nucleons.
3 3 So, ratio is 1:1.
⇒ a2019 = =
4(2019) − 1 8075 19. (c) x0 = 1 at02
2
Q p = 3, q = 8075 Distance travelled by Y in n second 1
∴ p + q = 8078 and x = x0 + at0 (t − t0 ) − a (t − t0 )2

Q θ = 2 π −  × 15 ...(ii) 2
10. (d) We have,  n  So, correct graph is (c).
Total number of cubes = 9 From Eqs. (i) and (ii), we get 20. (c)∴ F = kt
2 Red cubes, 3 Blue cubes, 4 Green cubes 2π 2π
× 15 = 2 π −  × 15 ⇒
v
m∫ dv = k ∫ t dt
t
Number of ways making tower of height 40  n  0 0
8 cubes ∴ n = 24 ⇒ mv = kt 2
9
C2 7C3 4C4 14. (a) We have, 6-digit numbers k
⇒ v = t2
9! 7! 9! abccba , b is odd. m
⇒ × × 1=
2! 7! 3! 4! 2! × 3! × 4! If abc − cba is divisible by 7 Kinetic energy is
9 × 8 × 7 × 6 × 5 × 4! 1
= = 1260 ⇒ abc − cba = 99(a − c) = 7M K = mv2 ⇒ K ∝ t 4
2 × 3 × 2 × 4! 2
https://iit-jeeacademy.blogspot.com

KVPY Practice Set 4 Stream : SA 215

21. (d) Mass = Density ∴ 0.25 moles are present in Mg3 (PO4 )2 37. (d) The product formed when
Volume 1 1-bromo-3-chlorocyclobutane reacts with
= × 0.25 = 3125
. × 10−2 mol .
As mass remains same, 8 two equivalents of metallic sodium in
V ice × ρice = V water × ρ water 3RT ether is bicyclo [1.1.0]. This reaction is
32. (c) From kinetic gas, vrms = known as Wurtz reaction.
m M
⇒ V water =
y 3RTSO2 3RTSO2 Cl
vrms of SO2 = =
⇒ ∆V = V water − V ice MSO2 64 + 2Na
Ether

m m  1 1 bicylo [1.1.0]
= − = m −  3RTO2 3RT × 303
y x  y x vrms of O2 = = Br + 2NaCl + NaBr
M O2 32
22. (a) W = q∆V = e [(V − ) − (V + )] 38. (d) Given E° values are
According to question, vrms (SO2 ) = vrms (O2 )
= − e (0 − 12) = + 12 eV Br2 / Br = + 190 . V
3RTSO2 3RT × 303 Ag / Ag + = − 0.80 V
An electron will move from negative to =
positive terminal by itself. So, positive 64 32 Cu 2+ /Cu (s) = + 0.34 V
work is required to carry electron in 303 × 64 I− /I2 (s) = − 0.54
reverse direction. TSO2 = °
32 If the Ecell values of the redox reaction is
23. (b) pabsolute = patmospheric + pgauge TSO2 = 606 K positive, then only copper can reduce that
element. (the reaction will be feasible)
Also, pgauge > patmospheric . 33. (c) The formation of black precipitate
indicates the presence of sulphur in an For the reaction,
24. (a) Let AE = x, then EC = 100 − x.
organic compound. The sodium extract of Cu → Cu 2+ + 2e− E ° = − 0.34V
So, ED = (100 − x)2 + (120)2 organic compound contains Na 2S which Br2 + 2e− → 2Br− ; E ° = + 109 . V
forms a black precipitate of PbS when Cu + Br2 → CuBr2 ; E ° = + 0.75 V
x (100 − x)2 + 1202
∴Time taken = + treated with lead acetate. °
5 3 Since, Ecell of this reaction is positive,
Na 2S + (CH3 COO)2Pb → PbS ↓ therefore Cu can reduce Br2. While in
This is minimum when t = 10 m. Sodium Lead Lead sulphide
°
15 × R1 extract acetate (black ppt.) other reaction Evalue will be negative and
25. (b) RAB = + 2 CH3 COONa hence Cu cannot reduce other elements.
15 + R1
34. (d) When carbonyl compounds are 39. (c) Tollen’s reagent is ammoniacal
15R1
⇒ 6= treated with excess of alcohol, first silver nitrate. It is reduced to silver by
15 + R1 compounds having  C  H group.
hemiacetals are formed and then acetals
6 × 15 are formed, i.e. 
⇒ = R1 ⇒ R1 = 10 Ω O
9 CH3—C—CH3 + C2H5OH O
 
26. (d) Fermi energy level is last filled Ethanol   
energy level at zero kelvin. O Formic acid  H C OH  has  C  H group.
Acetone   
OH  O 
27. (d) As sun changes its position (CH3)2C  
rapidly, so reflector of solar cooker is to be OC2H5 Thus, it reduces Tollen’s reagent.
1 Hemiketal
adjusted nearly in every hour duration. [2Ag(NH 3 )2 ]+ + HCOOH → 2Ag ↓ + CO2
2 C2H5OH(Excess) Silver
28. (c) + 2NH +4
OC2H5
(CH3)2C 40. (d) Greater is the s-character, shorter
F
mg O OC2H5 is the bond length. The C-atom in CO32− is
Acetal sp 2-hybridised as shown.
Rotational equilibrium about O gives, 35. (c) On moving across a period from O– O
l left to right in periodic table O==C O– C
F × l = mg –
2 electronegativity increases. This is O O–
⇒ F=
mg
= 50 N
because across the period the size of an O–
atom decreases. While on moving down O C
2
the group, electronegativity decreases. O
29. (a) Force is zero when potential Thus, the correct order of The C-atom in CO2 is sp-hybridised with
energy is minimum. electronegativity is bond distance of carbon oxygen is 122 pm.
30. (a) From first ray diagram, Si < P < C < N O == C == O ←→ O+ ≡≡ C  O−
µ = µ1 (1.8) (2.1) (2.5) (3.0)
←→ O−  C ≡≡ O+
From second ray diagram, 36. (d) According to de-Broglie relation, The C-atom in sp-hybridised with C—O
µ < µ 2 ⇒µ 1 < µ 2 λ=
h bond distance 110 pm.
31. (b) Mg3 (PO4 )2 —→ 3Mg + 2P + 8O mv
C≡≡O
6.63 × 10−34 Thus, the correct order of bond length is
As 8 moles of O-atoms are present in = = 6.63 × 10−29 m
1 mole of Mg3 (PO4 )2. 10−6 × 10 CO < CO2 < CO32−
https://iit-jeeacademy.blogspot.com

216 KVPY Practice Set 4 Stream : SA

41. (a) H2 ( g ) + Cl 2 ( g ) → 2HCl( g ) As compound given in option (d) follows These plants are adapted to grow in
22.4 L at STP is occupied by 1 mole of Cl 2 Huckel’s rule. Thus, it is aromatic in highly saline areas such as mangroves.
nature. Pneumatophores help these plants in
∴ 11.2 L will be occupied by Cl 2
1 × 112
. 46. (b) The mucosa present in between respiration as they do not get sufficient
= = 0.5 mol oxygen from the soil. On the other hand
22.4 the bases of villi of small intestine (crypts
of Lieberkuhn) contain Paneth cells, vivipary aids in perennation.
As per equation,
which secrete antibacterial lysozyme. 54. (d) Spliceosome is a large molecular
1 mole of Cl 2 produces 2 mole of HCl
Kupffer cells are phagocyte cells of liver. complex found in nucleus of eukaryotic
∴ 0.5 mole of Cl 2 produces = 2 × 0.5 = 10
. Zymogen cells produce enzyme. cells of plants, animals and fungi, etc. It
mole of HCl. Argentaffin cells produce hormones. is assembled from snRNAs and protein
42. (b) From ideal gas equation, 47. (c) Mycoplasma is triple layered complexes that play an important role in
pV = nRT smallest living cells. It does not have splicing of introns. Spliceosome is absent
w in the cells of bacteria.
Also, n= definite cell wall. It is an anaerobic
M
w organism. It causes disease in plants 55. (c) Auxin delays abscission of leaves
∴ pV = RT (little leaf of brinjal) as well as in and fruits at early stages. Whenever leaf
M
animals (pleuromorphic pneumonia in or fruit fall occurs, the organ concerned
pVM
w= man). stops producing auxin. However, it
RT promotes abscission of older, mature
Substituting the values, we get
48. (d) Mitochondria is referred as
powerhouse of the cell. It contains the leaves and fruits.
12.315 × 1 × 32
= 16 g enzymes for cellular respiration. It 56. (d) Pancreas consists of exocrine and
0.0821 × 300
oxidises carbohydrate to produce ATP endocrine parts. Exocrine part secretes
∴O2 to be released = 32 − 16 = 16 g molecules in the process of aerobic alkaline pancreatic juice. This juice
43. (d) Orbital angular momentum respiration. contains trypsinogen, chymotrypsinogen,
h 49. (b) DNA fragments are negatively procarboxypeptidase, lipase, amylase and
= l(l + 1) elastase.
2π charged molecules. The reason why DNA
is negatively charged is the phosphate 57. (c) As the diploid number is 8, there
For f -orbital, l = 3
(PO2−
4 ) group that constitutes every would be 4 pairs of homologous
∴Orbital angular momentum for f -orbital
nucleotide. chromosome pairing. This gives rise to a
h 3h
= 3(3 + 1) = During the formation of phosphodiester combination of 42 = 16 kinds of gametes.
2π π
bond, nucleotides retain one of the two 58. (a) According to Chargaff’s rule,
44. (b) Anhydrous HF is a covalent negative charge, while the other is lost to A + T = G + C and A = T and G = C.
compound and is strongly H-bonded. form ester bond to new pentose.
Therefore, it does not give F – ions and ∴ Guanine + Cytosine
hence AlF3 does not dissolve in HF. NaF 50. (c) The important characteristic that 40% 40%
is an ionic compound. It contains F – ions hemichordates share with chordates is Adenine + Thymine
which combine with electron deficient pharynx with gill slits. These slits are
10% 10%
AlF3 to form the soluble complex. narrow openings in the pharynx. The
3NaF + AlF3 → Na3 [AlF6 ] position of these pharyngeal gill slits is 59. (b) N2 → N2H2 → N2H4 → 2NH3
Nitrogen Dimide Hydrazine Ammonia
Boron due to its small size and higher lateral in chordates, while dorsal in
electronegativity has greater tendency to hemichordates. The fixation of atmospheric nitrogen to
form complexes than Al. Hence, 51. (a) In lungs, even after the most ammonia is given as
forceful expiration, some of the volume of Mo Fe
precipitation of AlF3 takes place when BF3 N2 + 8e− + 8H+ + 16ATP →
is passed through Na3 [AlF6 ] solution. air remains. This volume is termed Nitrogenase

Residual Volume (RV). Due to this, 2NH3 + H2 + 16ADP + 16Pi


Na3 [AlF6 ] + 3BF3 → 3Na[BF4 ] ↓
(X) lungs do not collapse even after the most The process in which atmospheric
+ AlF3 (s) forceful expiration. RV is about nitrogen gets converted into inorganic
45. (d) The compound which follows 1100-1200 mL. nitrogenous (nitrate, ammonia)
Huckel’s rule [(4n + 2)π] will be aromatic 52. (d) Viroids differ from viruses in compounds through microorganisms is
in nature. having RNA molecules without protein called biological nitrogen-fixation.
+
coat. Viruses on the other hand possess 60. (a) Treponema pallidum pathogen is
DNA or RNA with a protein coat as their a cause of syphilis. It is a sexually
4π electrons genetic material. Viruses can infect a transmitted infection that causes infected
4π electrons
Does not follow Does not follow wide range of organisms including plants, sores, blisters or ulcers on your genitals,
Huckel's rule Huckel's rule animals or bacteria, while viroids infect anus (bottom) or mouth.
only plants.
61. (b) Given, a + b − c = 1 ...(i)
ρ 53. (b) Plants that produce 2 2 2
pneumatophores i.e. negatively geotropic and a + b − c + 1 = 0 ...(ii)
σ
8π electrons 6π electrons roots and show vivipary i.e. germination From Eqs. (i) and (ii), we get
Does not follow Follow
of seeds inside the fruits are halophytes. a 2 + b2 − (a + b − 1)2 + 1 = 0
Huckel's rule Huckel's rule
https://iit-jeeacademy.blogspot.com

KVPY Practice Set 4 Stream : SA 217

⇒ a 2 + b2 − a 2 − b2 − 1 − 2ab + 2a 64. (d) We have, 66. (b) 10 cm 20 cm


worm worm
+ 2b + 1 = 0 P (x) = a0 + a1 x + a2x2 + K + anxn
⇒ (a + b) − ab = 0 P (5) = a0 + 5a1 + 25a2 + 125a3 + 2.5 cm
5 cm
⇒ (a − 1) (b − 1) = 1 K + (5)n an
⇒ a − 1 = 1and b − 1 = 1 ⇒136 = a0 + 5a1 + 25a2 + 125a3 + 7.5 cm
5 cm
⇒ a = b = 2⇒c = 3 K + (5)n an
⇒ a − 1 = − 1and b − 1 = − 1 ai ≥ 1for i ≥ 4 , then RHS > 136 For 10 cm worm centre of mass is raised
⇒ a = b = 0 and c = − 1 ∴a4 = a5 = a 6 K = an = 0 upto height of 7.5 cm, while for 20 cm
∴ a 2 + b2 + c2 = (2)2 + (2)2 + (3)2 worm height of centre of mass is 5 cm
∴ a0 + 5a1 + 25a2 + 125a3 = 136 ...(i)
from ground.
= 4 + 4 + 9 = 17 ∴a3 can be 0 or 1 only
So, ratio of work done by 20 cm worm to
a 2 + b2 + c2 = (0)2 + (0)2 + (−1) = 1 Now, P (1) = a0 + a1 + a2 + a3 = 4 ...(ii) that of 10 cm worm is
∴Sum of all possible value is 18. If a3 = 0, then W1 mgh1 5 50
= = = = 2: 3
62. (d) Let the number of cups with a0 + 5a1 + 25a2 ≤ 4 + 20 + 100 W2 mgh2 7.5 75
handle be m and number of cups without = 124 < 136
handle be n. 67. (b) Let d = diameter of disc. Spot is
If a3 = 1
∴ m
C2 nC3 = 1200 invisible, if incident rays from dot
⇒ a0 + 5a1 + 25a2 = 11 [from Eq. (i)] d
Let C3 is divisible by 1200.
n reaching top surface at are at the
⇒ a2 = 0 2
∴n ≤ 20 when n = 21, then 21 C3 > 1200 ⇒ a0 + 5a1 = 11 critical angle.
n (n − 1) (n − 2) 1
∴ nC3 = is divisible by Also from Eq. (ii), a0 + a1 = 3 Then by sin ic =
3 µ
1200 ⇒ a1 = 2, a0 = 1
d /2
⇒ n ≠ p , p + 1, p + 2, where p prime ≥ 7 Hence, P (x) = 1 + 2x + x3 and = tan i
h
⇒ n ≠ 7, 8, 9, 10, 11, 12, 13, 14, 15, 17, 18, ∴ P (3) = 1 + 6 + 27 = 34
2h 2 × 20
⇒ d= = ≈ 46 cm
19, 20 65. (b) Given, in quadrilateral ABCD 2 16
µ −1 −1
∴Possible value of n = 3, 4, 5, 6 AB = AD = 13 9
When n = 4, then m = 25 ⇒ m + n = 29 BC = CD = 20 So, minimum radius of disc = 23 cm.
and n = 10, then m = 5 ⇒ m + n = 15 BD = 24
68. (c) In case A, voltage across bulb is
and n = 5, then m = 16 ⇒ m + n = 21 D 20 C higher, so lamb will burn brighter.
∴ Maximum value of (m + n ) = 29 In case B, voltage is same as either of
63. (a) Let ∠CI2F = θ 24 battery but each battery supplies only
13 20 half of current, hence the batteries will
∠BAE = x = ∠EAD
lasts twice as long.
and ∠DAF = y = ∠FAC
13
69. (d) From v-t graph,
∠A = 2x + 2 y A B
∠A For 0 < t < 3 s,
Area of ∆ABD
⇒ x+ y= 8 ms−1
2 = 25(25 − 13) (25 − 13) (25 − 24) a= = 2.67 ms−2
3s
A = 25 × 12 × 12 × 1
For 3s < t < 5s, a = 0
= 60
For 5s < t < 9s,
x xyy Area of ∆BCD
−16 ms−1
= 32(32 − 20) (32 − 20) (32 − 24) a= = − 4 ms−2
4s
I1 I2 = 32 × 12 × 12 × 8
θ 70. (a) Forces on the ball are
°

= 192
60

B/2 C/2
(i) weight of ball = mg
B C ∴Area of quadrilateral ABCD (ii) weight of fluid column above ball
E F
∠B = Area of ∆ABD + Area of ∆BCD = πr 2ρgh
∠AEF = ∠EBI1 + ∠BI1 E = + 60°
= 60 + 192 2
2 (iii) Buoyant force =  πr3  ρg
∠C = 252 3 
and ∠AFE = +θ
2 Radius of incircle 2 3
when mg = πr ρg − πr 2ρgh, ball will
In ∆AEF, Area of quadrilateral 3
=
∠B ∠C ∠A Semiperimeter of quadrilateral tend to rise corresponding height h0 of
+ 60° + θ + + = 180°
2 2 2 252 water in vessel is given by
= = 7.63 2
⇒ θ = 180° − 60° − 90° ⇒θ = 30 33 mg = πr 2ρg − πr 2ρgh0
3
∴ ∠CI 2F = 30° ∴ Nearest integer of r is 8.
https://iit-jeeacademy.blogspot.com

218 KVPY Practice Set 4 Stream : SA

Substituting given values, we get Thus, the correct option is (d). 76. (c) The total amount of energy used
2
2 01
.  in forming 38 ATP is
π × 10−3 × 10 = × π ×  CH2 == CH  CH == CH2   →
H 2 / Ni
 × 1000 × 10 1, 4-addition
3  100  ( A) 38 × 30.6 kJ = 1162.8 kJ
CH3 CH == CH CH3
Thus efficiency of aerobic respiration is
2 (B)
1  1162.8
− π   × 1000 × 10 × h0 × 100% = 40%
 100  O3/H2O/Zn 2800
1 2 2CH3CHO 77. (a) Since there is both a haemophilic
⇒ = − h0
100 3 and normal son, the mother must have a
⇒ h0 = (0.66 − 0.01) m = 65 cm 74. (a)In Kjeldahl’s method, percentage heterozygous genotype.
of N is given by Since there is a haemophilic daughter,
71. (b) Compound X is CaCO3 the X-chromosome from the father must
1.4 × normality of acid × volume of acid

CaCO3  → CaO + CO2 ↑ = have the recessive allele.
X Residue weight of compound
Also, 1 M H2SO4 = 2 N H2SO4 XhX XhY
CaO + H2O → Ca(OH)2
Residue Y [M = N × Basicity/Acidity]
Ca(OH)2 + CO2 + H2O → Ca(HCO3 )2 1.4 × 2 × 10
Y Excess ∴% of N = = 37.33 % XhXh XhY XhX XY

Ca(HCO3 )2  → CaCO3 + H2O + CO2 ↑ 0.75 Haemophilic Carrier Normal
X daughter daughter son
75. (c)
72. (a) For the reaction, Haemophilic
1 3 Element % of Atomic No. of Simple son
X 2 + Y2 - XY3 (∆H = − 30 kJ)
2 2 element weight moles ratio 78. (c) Endocrine cells are present in
1 3
∆S° = ∆S(°XY3 ) −  SX° 2 + SY°2  C 12.8 12 12.8/12 1.06 / different parts of the gastro-intestinal
 2 2  = 1.06 1.06 = 1 tract, e.g., gastrin, secretin, GIP. Atrial
 1 3  wall of our heart secretes a peptide
= 50 − × 60 + × 40 H 2.10 1 210
. / 1 2.10 /
 2 2  hormone called ANF (Atrial Natriuretic
= 2.10 1.06
Factor). Releasing and inhibitory
= 50 − [30 + 60] =2
hormones are released by hypothalamus.
= 50 − 90 = − 40 JK−1 mol −1 Br 85.1 80 851
. / 80 1.06 / Adenohypophysis is not under direct
Also, ∆G ° = ∆H ° − T∆S° = 1.06 1.06 = 1 control of hypothalamus.
At equilibrium, ∆G° = 0 79. (c) The amount of DNA in the 2
Hence, the empirical formula becomes
∆ H = T∆ S somatic cells, the motor neuron and the
CH2Br
∆H ° skin cell should be the same.
T = Empirical weight of CH2Br
∆S ° After interphase where DNA has already
− 30 × 103 J mol −1 = 12 + 2 + 80 = 94 replicated, the amount of DNA is
= = 750 K As we know that,
− 40 × JK−1 mol −1 doubled, i.e. 6.8 × 2 pg = 13.6 pg.
molecular wt.
73. (d) (A) (C4 H6 )  
2 H / Ni
→ (C4 H8 ) n= 80. (b) The reaction that is
1 mole empirical wt. enzyme-catalysed has lower activation
O3/ H 2O/ Zn
 → CH3 CHO 187.9 energy and is represented by the dotted
= =2
By considering the molecular formula 94 line. The energy input required to raise
C4 H6 . We can conclude that, it is an Thus molecular formula the energy of the reactants to a certain
alkene, CH2 == CH  CH == CH2 which on = n × empirical formula level before the reaction is triggered is
reduction with H2 gives CH3 CH == CH = 2 × (CH2Br) called the activation energy. This is
CH3 . Also ozonolysis ofCH3 CH == CHCH3 represented by the increase in energy of
= C2H4 Br2
will only give 2 moles of acetaldehyde. the reactants to the top of the ‘hill’, B.
https://iit-jeeacademy.blogspot.com

KVPY Practice Set 5 Stream : SA 219

KVPY
KISHORE VAIGYANIK PROTSAHAN YOJANA

PRACTICE SET 5
Stream : SA
MM 100

Instructions
There are 80 questions in this paper.
This question paper contains two parts; Part I and Part II. There are four sections; Mathematics, Physics, Chemistry
and Biology in each part.
Out of the four options given with each question, only one is correct.

PART-I (1 Mark Questions)


MATHEMATICS 5. The parabola y = ax2 − 2 and y = 4 − bx2 intersect the
coordinate axes in exactly four points and these four
1. How many positive integers less than 1000 are points are the vertices of area 12, then a + b is equal to
6 times the sum of their digits?
(a) 1/2 (b) 1 (c) 3/2 (d) 2
(a) 0 (b) 1 (c) 2 (d) 3
6. Let AB be a chord of circle with centre O. Let C be a
2. Divya inscribed a circle inside a regular pentagon, point on the circle such that ∠ABC = 30° and O lies
circumscribed a circle around the pentagon, and inside the ∆ABC. Let D be a point on AB such that
calculated the area of region between the two circles. ∠DCO = ∠OCB = 20°, then the measure of ∠CDO in
Mansi did the same with a regular heptagon. The degree is
areas of two regions A and B respectively. Each
(a) 110° (b) 70° (c) 80° (d) 20°
polygon had a side length of 2. Which of the following
is true? 7. Let a and b be natural numbers such that
(a) 7A = 5B (b) 5A = 7B 2a − b, a − 2b and a + b are all distinct squares, the
(c) A = B (d) 25A = 49B least possible value of b is
(a) 21 (b) 22 (c) 24 (d) 25
3. A box contains a collection of triangular and square
tiles. There are 25 tiles in the box containing 84 edge 8. The wealth of a person A equals the sum of that of B
total. The number of square tiles in the box are and C. If he distributes half of his wealth between B
(a) 5 (b) 7 (c) 9 (d) 11 and C in the ratio 2 : 1, then the wealth of B equals
the sum of that A and C. Then, the fraction of wealth
4. Define a function on the positive integers recursively that A should distribute between B and C in the ratio
by f (1) = 2, f (n ) = f (n − 1) + 2 if n is even, and 1 : 2, so that the wealth of C equals the sum of that of
f (n ) = f (n − 2) + 2 if n is odd and greater than one. A and B is
Then, f (2019) is equal to 1 2 3
(a) 2019 (b) 2020 (c) 2021 (d) 2018 (a) (b) (c) (d) 1
2 3 4
https://iit-jeeacademy.blogspot.com

220 KVPY Practice Set 5 Stream : SA

9. For some positive integer K, the repeating base-K (c) melting point of solid
7 (d) triple point of phase equilibrium
representation of the (base-ten) fraction is
51 17. In the cyclic process, process A → B is isothermal.
0. 23K = 0232323
. K K , then the value of K is
p
(a) 13 (b) 14 (c) 15 (d) 16 A
10. The number 1, 2, 3, ..., 9 are randomly placed into the
9 square of a 3 × 3 grid. Each square gets one number
and each of the numbers is used once. What is the C B
probability that the sum of the numbers in each row V
and each column is odd?
1 1 5 2
Correct V - T graph for the cycle is
(a) (b) (c) (d) (b)
21 14 63 21 (a) V V
B A
11. A quadrilateral is inscribed in a circle of radius
200 2. Three sides of this quadrilateral have length
200, then length of the fourth side is
C A C B
(a) 200 (b) 200 2 (c) 400 (d) 500
T T
12. Let T be the smallest positive integer which, when
divided by 11, 13, 15 leaves remainder in the sets V
(c) V (d)
{ 7, 8, 9}, {(1,2,3}, {4, 5, 6} respectively. The sum of B B
squares of the digit of T is
(a) 50 (b) 81 (c) 89 (d) 90
13. If roots of equation x − bx + c = 0 be two consecutive
2 A C C A
T T
integers, then b − 4c equals
2

(a) −2 (b) 3 (c) 2 (d) 1 18. Consider the following nuclei:


3 14 238
14. How many different words can be formed by jumbling 2 He , 7 N , 92 U
3 13 235
the letters in the word MISSISSIPPI in which no two 1 H , 6Cl , 92 U
‘S’ are together Choose the correct statements given below.
(a) 8 ⋅6 C4 ⋅7 C4 (b) 6 ⋅ 7 8 C4
I. 2 He3 and 1 H3 are isotopes.
(c) 7 ⋅6 C4 ⋅8 C4 (d) 6 ⋅ 8 ⋅7 C4
235 238
II. 92 U and 92 U are isobars.
15. A man standing on a railway platform noticed that a
train took 21 s to cross the platform which is 88 m 13 14
III. 6Cl and 7 N are isotones.
long and that it took 9 s to pass him. Assuming that (a) All statements are correct
the train was moving with uniform speed. What is (b) Both statements I and II are correct
the length of the train in meters? (c) Both statements II and III are correct
(a) 55 (b) 60 (c) 66 (d) 72 (d) Only statement III is correct
19. Consider two identical copper spheres A and B. One
PHYSICS is placed over a thermally insulating plate,while the
16. p - T curve representing phase equilibrium is given by; other hangs from an insulating thread.
p

A
B
P1

T Equal amounts of heat are given to the two spheres


and temperatures are recorded, then
The point P1 is (a) TA = TB (b) TB < TA
(a) boiling point of liquid (c) TB > TA (d) cannot be concluded
(b) condensation point of vapour
https://iit-jeeacademy.blogspot.com

KVPY Practice Set 5 Stream : SA 221

20. A boy throws a stone to hit a pole at some distance. 25. Given, A = Boltzmann constant, B = Planck’s constant
Kinetic energy K of stone varies with horizontal and C = speed of light.
displacement x as shown in figures given below. Then, quantity with dimensions of A4B−3C −2 is
(a) K (b) K
(a) universal gas constant
(b) specific heat capacity
(c) Stefan’s constant
(d) heat energy
x
x 26. Considering air resistance, if t1 = time for a thrown
K K ball in upward journey and t2 = time taken for
(c) (d) downward journey, then
(a) t1 = t2 (b) t1 > t2
(c) t2 > t1 (d) 3 t2 = 2 t1
x x 27. A car accelerates from rest at a constant rate α for
sometime after which it deaccelerates at a constant
21. Ratio of time periods of small oscillations of the rate β to come to rest.
insulated spring and mass system (shown) before and If total time is t, then maximum speed of car is
after charging the mass is
 αβ  α + β  α2 + β2   α2 − β2 
m m (a)   t (b)   t (c)   t (d)   t
α + β  αβ   αβ   αβ 
(a) equal to one (b) greater than one
(c) less than one (d) greater than or equal to one 28. A lawn roller of mass 10 kg, radius 1 m is pulled
horizontally by a handle attached to axle of the
22. The lights on a car are inadvertently left on. They roller.
dissipate 95 W.
Fully charged 12 V car battery is rated 150 Ah. Time
F
after which the car lights go OFF due to battery run
down is 0.4 m
(a) 12 h (b) 24 h (c) 18 h (d) 36 h Necessary minimum pull to raise roller above a step
23. In the arrangement shown below. of 0.4 m is
(a) 128 N (b) 134 N
(c) 213 N (d) 112 N
29. Geodesic is a
(a) straight line
A m (b) curve
2m B (c) circle
(d) may be a straight line or curve
C 30. In given combination of lenses, a parallel beam is
made incident from left as shown below.
Accelerations of masses A and B just after cutting the
string C are f= 1 m f=0.25 m
g
(a) 0, g (b) g , g (c) , g (d) 2g , g
2
24. For streamlined flow of water, consider the following
statements. 0.75 m
Emerging light rays are as shown by
I. Two streamlines does not cross each other. (c)
(a) (b) (d)
II. Streamlines are straight.
III. Streamlined flow is more likely for fluids with
low density and high viscosity.
IV. Streamlined flow is more likely for liquids with
high density and low viscosity.
CHEMISTRY
Which of the above statements are correct? 31. Haemoglobin contains 0.33% of iron by weight. The
(a) Statements I and III are correct molecular weight of haemoglobin is approximately
(b) Statements II, III and IV are correct 67200. The number of iron atoms (at. wt. of Fe is 56)
(c) Statements III and IV are correct present in one molecule of haemoglobin is closest to
(d) Statements I, III and IV are correct (a) 1 (b) 6 (c) 4 (d) 2
https://iit-jeeacademy.blogspot.com

222 KVPY Practice Set 5 Stream : SA

32. Which of the following statements is incorrect? 42. Sodium peroxide which is a yellow solid, when
(a) Angular quantum number signifies the shape of the exposed to air becomes white due to the formation of
orbital (a) H 2 O2 (b) Na 2O
(b) Energies of stationary states in hydrogen like atoms is (c) Na 2O and O3 (d) NaOH and Na 2CO3
inversely proportional to the square of the principle
quantum number 43. The products formed when the following compound is
(c) Total number of nodes for 3s-orbital is three treated with Br2 in the presence of FeBr3 are
(d) The radius of first orbit of He+ is half that of the first CH3
orbit of hydrogen atom
33. The solubility of saturated solution of calcium
fluoride is 2 × 10−4 mol L−1. Its solubility product is
closest to CH3
(a) 12 × 10−2 M 3 (b) 14 × 10−4 M 3
CH3 CH3
(c) 22 × 10−11 M 3 (d) 32 × 10−12 M 3
Br
34. The brown ring complex compound is formulated as
[Fe(H2O)5 (NO )] SO 4 . The oxidation state of iron is (a) and
(a) 1 (b) 2 (c) 3 (d) 0 CH3 CH3
35. Which one of the following has the maximum dipole Br
moment?
(a) CO2 (b) CH 4 (c) NH 3 (d) NF3 CH3 CH3
36. Which of the following is a chiral? Br Br
(a) 1,1-dibromo-1-chloropropane (b) and
(b) 1, 1 - dibromo -3-chloropropane
(c) 1, 3-dibromo-1-chloropropane CH3 CH3
(d) 1, 3-dibromo-2-chloropropane CH3 CH3
37. The correct order of acidic character of the following Br
compounds is
(c) and
I. phenol, II. o-cresol
III. p-nitrophenol IV. p-chlorophenol CH3 CH3
(a) I > II > III > IV (b) III > IV > I > II Br
(c) IV > III > II > I (d) III > II > IV > I
38. Which of the following is the correct order of size of CH3 CH3
the given species?
(a) I > I− > I+ (b) I+ > I− > I
(d) and
(c) I > I+ > I− (d) I− > I > I+
CH3 CH3
39. Which of the following is the correct representation of Br
Gay-Lussac’s law? Br

44. How many enantiomeric pairs are obtained by


monochlorination of 2, 3-dimethyl butane ?
(a) p (b) p
(a) 4 (b) 2 (c) 3 (d) 1

1/V V
45. If bond enthalpies of Cl—Cl bond, H—H bond and
H—Cl bond are 243, 435 and 431 kJ mol −1
respectively, then calculate the ∆ f H ° in kJ mol −1
(c) p (d) p of HCl.
(a) − 184 (b) − 92 (c) 170 (d) − 88
1/T T

40. A compound that gives a positive iodoform test is BIOLOGY


(a) pentanol (b) pentan-3-one
(c) pentan-2-one (d) pentanal 46. Which of the following components provides sticky
character to the bacterial cell?
41. Which of the following compounds of xenon has (a) Cell wall (b) Nuclear membrane
pyramidal geometry? (c) Plasma membrane (d) Glycocalyx
(a) XeOF4 (b) XeF2 (c) XeO3 (d) XeF4
https://iit-jeeacademy.blogspot.com

KVPY Practice Set 5 Stream : SA 223

47. Life cycle of Ectocarpus and Fucus respectively are of animal cells. If APC is defective in a human cell,
(a) haplontic, diplontic which of the following is expected to occur?
(b) diplontic, haplodiplontic (a) Chromosomes will not condense
(c) haplodiplontic, diplontic (b) Chromosomes will be fragmented
(d) haplodiplontic, haplontic (c) Chromosomes will not segregate
48. Which of the following are not polymeric? (d) Recombination of chromosome arms will occur
(a) Nucleic acid (b) Proteins 55. Zygotic meiosis is a characteristic of
(c) Polysaccharides (d) Lipids (a) Marchantia (b) Fucus
49. In case of poriferans, the spongocoel is lined with (c) Funaria (d) Chlamydomonas
flagellated cells called 56. Which one of the following generally acts as an
(a) ostia (b) oscula antagonist to gibberellins?
(c) choanocytes (d) mesenchymal cells (a) Zeatin (b) Ethylene
50. A decrease in blood pressure/volume will not cause (c) ABA (d) IAA
the release of 57. The ornithine cycle removes two waste products from
(a) renin (b) atrial natriuretic factor the blood in liver. These products are
(c) aldosterone (d) ADH (a) CO2 and urea (b) ammonia and urea
51. The vascular cambium normally gives rise to (c) CO2 and ammonia (d) ammonia and uric acid
(a) phelloderm (b) primary phloem 58. Phellogen and phellem respectively denote
(c) secondary xylem (d) periderm
(a) cork and cork cambium
52. Which of the following options best represents (b) cork cambium and cork
enzyme composition of pancreatic juice? (c) secondary cortex and cork
(a) Amylase, peptidase, trypsinogen, renin (d) cork and secondary cortex
(b) Amylase, pepsin, trypsinogen, maltase
(c) Peptidase, amylase, pepsin, renin
59. Which one of the following enzymes shows the
greatest substrate specificity?
(d) Lipase, amylase, trypsinogen, procarboxypeptidase
(a) Lipase (b) Nuclease
53. Which of the following are found in extreme saline (c) Pepsin (d) Sucrose
conditions?
(a) Archaebacteria (b) Eubacteria
60. Albinism in humans is controlled by a recessive
allele. How many copies of this allele will be found at
(c) Cyanobacteria (d) Mycobacteria
one of the poles of a cell at telophase-I of meiosis in
54. Anaphase Promoting Complex (APC) is a protein an albino person?
degradation machinery necessary for proper mitosis (a) 23 (b) 4 (c) 2 (d) 1

PART-II (2 Marks Questions)


MATHEMATICS 64. If ∆ABC is a right angle triangle with ∠ACB as its
right angle. The measure of ∠ABC = 60° and AB = 10.
61. A positive integer K is said to be good if there exists a Let P be randomly chosen inside ABC, and extend BP
partition of {1, 2, 3, K , 20} in to disjoint proper subsets to meet AC at D. Then, the probability that BD > 5 2
such that the sum of the numbers in each subset of is
the partition is K. Then good number are there
1 2− 2 3− 3 5− 5
(a) 5 (b) 6 (c) 7 (d) 4 (a) (b) (c) (d)
2 2 3 5
62. Let C1 and C 2 be circles defined by (x − 10)2 + y2 = 36 65. Let P (x) = (x − 1) (x − 2) (x − 3). For how many
and (x + 15)2 + y2 = 81 respectively. The length of the polynomials Q(x) does there exist a polynomial R(x) of
shortest line segment PQ that is tangent C1 at P and degree 3 such that P (Q(x)) = P (x) ⋅ R(x)?
to C 2 at Q is (a) 22 (b) 24 (c) 27 (d) 32
(a) 15 (b) 18 (c) 20 (d) 24
63. Suppose that a and b are digits, not both nine and PHYSICS
not both zero, and the repeating decimal 0. ab is
expressed as a fraction in lowest terms. Then, the 66. Two stones are thrown up simultaneously from the
different denominators are possible, are edge of a cliff 200 m high with initial speeds 15 ms −1
(a) 3 (b) 4 (c) 6 (d) 5 and 30 ms−1.
https://iit-jeeacademy.blogspot.com

224 KVPY Practice Set 5 Stream : SA

Correct graph of time variation of the relative 70. A calorimeter contains some ice and 10 kg water.
position of the second stone with respect to first is This calorimeter is heated over a slow burner which
(b) provides heat at a constant rate.
(a)
Temperature of calorimeter and its contents varies

(x2–x1) m
(x2–x1) m

with time as shown below.


T(°C)

t(s) t(s) 10
8 10 8 10
(c) 0
(d)
(x2–x1) m

(x2–x1) m

t(min)
–5 50 60

Amount of ice initially present is nearly


t(s) (a) 17 kg (b) 14 kg (c) 10 kg (d) 5 kg
8 10 t(s)
8 10
67. In an experiment of finding focal length of a concave CHEMISTRY
mirror by u-v method, a student prepares following
graph of u versus v graph. 71. The heat liberated from the combustion of 0.5 g of
y
carbon raised the temperature of 2000 g of water
from 24°C to 26°C. The heat of combustion of
(–22.5,0) carbon (per mole) is
u(cm) (–45,0) (–30,0) (a) −4 kcal (b) −8 kcal
x (c) −62 kcal (d) −96 kcal
(0, –22.25) 72. An organic compound of molecular formula C4H6.
(0, –30) A forms precipitates with ammoniacal silver nitrate
and ammoniacal cuprous chloride. A is an isomer B,
(0, –45) one mole of which reacts with one mole of Br2to form
1, 4-dibromobut-2-ene. A and B are
(a) CH 3 CH 2 C ≡ CH and CH 2 == CHCH == CH 2
(b) CH 3 C ≡≡ CCH 3 and CH 3 CH == C == CH 2

v(cm) CH2 CH2 CH


(c) C CH2 and
Focal length of mirror is nearly CH2 CH2 CH
(a) − 45 cm (b) − 30 cm (c) − 22.25 cm(d) − 15 cm
68. In given circuit, bulb that glows with maximum CH
(d) CH3C CCH3 and CH2 CH2
intensity is
2Ω 4Ω CH

3Ω . × 1021
73. A gas bulb of 1 mL capacity contains 20
5Ω . × 103
molecules of nitrogen exerting a pressure of 757
6Ω Nm −2. The root mean square speed of the gas
molecules is
(a) 274 ms−1 (b) 494 ms−1
+ – (c) 690 ms−1 (d) 988 ms−1
20 V
74. Which of the following statements is not correct from
(a) 4Ω bulb (b) 2Ω bulb
the view point of molecular orbital theory?
(c) 3Ω bulb (d) 6Ω bulb
(a) Be2 is not a stable molecule
69. For a linear oscillator, potential energy as a function (b) He2 is not stable, but He+2 is expected to exist
of its displacement x is (c) Bond strength of N 2 is maximum amongst the
kx2 homonuclear diatomic molecule belonging to the
U (x) = second period
2
where, k = spring constant = 05 . Nm−1. (d) The order of energies of molecular orbitals in N 2
molecule is
If total energy of the particle is 1 J, then maximum σ 2s < σ * 2s < σ 2 pz ( π 2 px ≈ π 2 py ) < ( π * 2 px ≈ π * 2 py )
amplitude of oscillation of particle is < σ * 2pz
(a) 1 m (b) 2 m (c) 3 m (d) 1.5 m
https://iit-jeeacademy.blogspot.com

KVPY Practice Set 5 Stream : SA 225

75. The atomicity of a molecule, M, if 10 g of it combine The most likely explanation of the fact that the graph
with 0.8 g of oxygen to form an oxide is closest to levels off at 180 Jm −2s−1 is that the system is
[specific heat of molecule, M is 0.033 cal deg −1 g −1 and (a) light limited and carbon dioxide saturated
molecular mass of molecule is 199.87 g] (b) light limited and the temperature is below optimum
(a) 1 (b) 2 (c) 3 (d) 8 (c) light saturated and carbon dioxide is unlimited
(d) light saturated and the temperature is above optimum
BIOLOGY 78. Certain drug acts at synapses and affects the action
of neurotransmitter substances. The table shows the
76. The following statements describe the structure of effects of four different drugs.
certain protein molecule.
Drug Effect
(I) The molecule consists of two polypeptide chains
which are folded around each other. I. Inhibits the enzyme cholinesterase.
(II) In each chain the amino acids are held in a II. Prevents the release of acetylcholine.
helix by hydrogen bonds. III. Competes with acetylcholine at receptor sites.
Which orders of structure are described by these IV. Inhibits the enzyme which destroys
statements? nor-adrenaline.
Statement (I) Statement (II) Which two drugs would prevent a skeletal
(a) Primary Tertiary muscle from responding to an electrical
(b) Secondary Tertiary stimulus in the presynaptic neuron?
(c) Tertiary Secondary (a) I and II (b) I and IV
(d) Quaternary Secondary (c) II and III (d) II and IV
77. The graph shows the relationship between oxygen 79. The diagram shows some chromosomes at late
production in photosynthesis and light intensity prophase of mitosis.
for a unicellular green organism in 0.02% sodium
hydrogencarbonate solution
Oxygen production/mm3h–1
7

5
How many chromosomes would be present in one
4 nucleus at telophase-II of meiosis?
3 (a) 6 (b) 12
(c) 18 (d) 24
2

1
80. Pyrimidine bases contain four carbon atoms and
purine bases contain 5.
0 How many carbon atoms are there in a nucleotide
20 40 60 80 100 120 140 160 180 200
–1 Light intensity/Jm–2S–1
containing cytosine?
–2 (a) 8 (b) 9 (c) 10 (d) 11

Answers
PART-I
1 (b) 2 (c) 3 (c) 4 (b) 5 (c) 6 (c) 7 (a) 8 (d) 9 (d) 10 (b)
11 (d) 12 (b) 13 (d) 14 (c) 15 (c) 16 (d) 17 (a) 18 (d) 19 (c) 20 (c)
21 (a) 22 (c) 23 (d) 24 (a) 25 (c) 26 (c) 27 (a) 28 (b) 29 (d) 30 (b)
31 (c) 32 (c) 33 (d) 34 (a) 35 (c) 36 (c) 37 (b) 38 (d) 39 (d) 40 (c)
41 (c) 42 (d) 43 (c) 44 (d) 45 (b) 46 (d) 47 (c) 48 (d) 49 (c) 50 (d)
51 (c) 52 (d) 53 (a) 54 (c) 55 (d) 56 (c) 57 (b) 58 (b) 59 (d) 60 (c)

PART-II
61 (b) 62 (c) 63 (d) 64 (c) 65 (a) 66 (d) 67 (d) 68 (a) 69 (b) 70 (a)
71 (d) 72 (a) 73 (b) 74 (d) 75 (a) 76 (d) 77 (d) 78 (c) 79 (a) 80 (b)
https://iit-jeeacademy.blogspot.com

226 KVPY Practice Set 5 Stream : SA

Solutions
1. (b) Number less than 1000 can write 3. (c) Let the number of triangular tiles C
abc = 100a + 10b + c =x
where a , b, c ∈ {0, 1, 2, 3, ..., 9} and and the number of square tiles = y

20°
a + b + c> 0

20°
A triangle has three edges and square
The sum of digits of this number is has four edges.
(a + b + c). ∴ x + y = 25 ...(i) O 30°
Given, 100a + 10b + c = 6(a + b + c) and 3x + 4 y = 84 ...(ii) B
A D
∴ 94a + 4b − 5c = 0 On solving Eqs. (i) and (ii), we get
Clearly, a > 0. No solution x = 16, y = 9
∴ a = 0 then 4b = 5c Hence, number of square tiles in box is 9. Now, OA = OC
This is possible only ∆OAC is an equilateral
4. (b) We have, f (1) = 2
b = 5 and c = 4 ⇒ ∠CAO = ∠ACO = 60°
∴Number is 54. and f (n ) = f (n − 2) + 2, n is odd
⇒ ∠ACD = 60° − 20° = 40°
∴ f (3) = f (1) + 2 = 2 + 2 = 4
Hence, only one number i.e., 54. OC = OB
2. (c) In ∆OPB, f (5) = f (3) + 2 = 4 + 2 = 6
∴ ∠OBC = ∠OCB = 20°
π r π ∴ Similarly, f (2019) = 2020 ⇒ ∠OBA = 10° = ∠OAB
cos = ⇒ r = R cos
5 R 5 5. (c) We have, ⇒ ∠DAC = 70°
Equation of parabola In ∆ACD, ∠CDA = 70°
y = ax2 − 2 …(i) ⇒ ∠CDA = ∠CAD = 70°


5 and y = 4 − bx 2
…(ii) ⇒ CD = CA = CO
O Y In ∆CDO , CD = CO and ∠DCO = 20°
r 180° − 20°
R C (0, 4) ⇒ ∠CDO = = 80°
P
2
7. (a) We have,
B 2a − b, a − 2b and a + b are squares.
O ∴Let 2a − b = x2 ...(i)
Area of region = π (R 2 − r 2 ) X′ X
(x2, 0) D B (x1, 0)
π a − 2b = y2
= πR 2  1 − cos2 
...(ii)
 5 and a + b = z2 ...(iii)
π π where x, y, z ∈ N
= π cosec2  1 − cos2  A (0, –2)
5  5 Y′ From Eqs. (ii) and (iii), we get
π
Q sin = 1  2a − b = y2 + z 2
On solving Eqs. (i) and (ii), we get
 5 R  ∴ x2 = y2 + z 2 ...(iv)
6
 2 π 2 π x intercept are ±
= π  cosec − cot  = π a+ b
From Eqs. (i) and (iii), we get
 5 5 3a = x2 + z 2
Similarly in heptagon, i.e. coordinate of points
x + z is multiple of 3 ⇒ x and z is also
2 2
 6  multiple of 3.
A (0, − 2), B  , 0,
 C (0, 4)
 a + b  From Eqs. (ii) and (iii), we get
6 3b = z 2 − y2 ...(v)
π/7 and D = −
a+ b z − y is a multiple of 3 ⇒ y and z is also
2 2

R1 1 multiple of 3.
r1 Area of kite ABCD = AC × BD
2 Let x = 3x1 , y = 3 y1 , z = 3z1 ⇒ x12 = y12 + z12
1 6 6 Let us assume every two of x, y, z are
⇒ 12 = 6 × 2 ⇒ =2
2 a+ b a+ b coprime.
π  1 − cos2 π 
Area of region = π cosec 2
  6 ⇒ x1 , y1 , z1 are pythagorean triplet.
7  7 ⇒ =4
a+ b ⇒ Out of y1 and z1 , one even ≥ 4 and other
 π π
= π  cosec2 − cos2  6 3 odd ≥ 3.
 7 7 ⇒ a+ b= =
4 2 From Eq. (v), we get
= π
6. (c) Given, ∠ABC = 30° b = 3(z12 − y12 ) = 3(z1 + y1 ) (z1 − y1 )
∴ Both have same area.
⇒ min b = 3(4 + 3) (4 − 3) = 21
∴ A=B ∴ ∠AOC = 2 ∠ABC = 60°
https://iit-jeeacademy.blogspot.com

KVPY Practice Set 5 Stream : SA 227

8. (d) Let the wealth of A, B and C are each even be with another even in each 14. (c) We have, MISSISSIPPI
x, y and z respectively. row or column. There are 9 ways to this. Other than S, seven letters, M, I, I, I,P, P,
Given, A=B+C They are 5! ways to permute odd and 4! I can be arranged in
ways to permute even number. 7!
∴ x= y+ z = 7× 5× 3
5! × 4! × 9 1
A distributes half of his wealth to B and C ∴Required probability = = 4! 2!
9! 14
in the ratio 2 : 1. Now, 4 S can be placed in 8 spaces,
11. (d) Given,
Wealth of B = y +   and C = z +   i.e. 8 C4 .
2 x 1 x
3  2 3  2 AB = BC = CD = 200 ∴Total number of arrangement
Now, B=A+C OA = OB = OC = OD = 200 2 = 7 ⋅ 5 ⋅ 3 ⋅8 C4
x x x x ∠AOB = ∠BOC = ∠COD = 7 × 15 × 8C4
∴ y+ = + z+ ⇒ y−z=
3 2 6 3 200 = 7 × 6C4 × 8C4
B C
Let ‘a’ be the fraction that A should
15. (c) Let the length of train is x m.

20
20

0
distribute and the ratio of distribution is
1 : 2.
E F Length of train and platform = (x + 88) m
A D
1 2 Time taken by train to cross the platform
< √2
200
3 3 O = 21s
A = (1 − a ) x x + 88
Now, ∴ Speed = ...(i)
ax 21
B= y+
3 Time taken by train to cross the man = 9 s
2ax x
C=z+ In ∆OAB and ∆ABE, ∴ Speed = ...(ii)
3 9
∠BAE = ∠AOB
2ax ax From Eqs. (i) and (ii), we get
∴ z+ = y+ + (1 − a ) x ∠ABO = ∠ABE
3 3 x + 88 x
∴ ∆OAB ~ ∆ABE = ⇒12x = 88 × 9
⇒ 3z + ax = 3 y + 3x − 3ax 21 9
OA AB OB
⇒ 4ax = 3( y − z ) + 3x = = 88 × 9
AB BE AE ⇒ x= = 66 m
12
4ax = 3   + 3x Q y − z = 
x x
⇒ AB = AE [QOA = OB ]
 3  3  16. (d) Point P1 is called triple point,
Similarly in ∆OCD and ∆DFE, where fusion curve vaporisation curve
⇒ 4ax = 4x ⇒ a = 1
CD = DF and sublimation curve meets.
9. (d) We can expand the fraction OB
∴ OE = 100 2 = 17. (a) Process AB is isothermal
0. 23 K as follows 2 expansion, so
0. 23 K = 2 ⋅ K −1 + 3 ⋅ K −2 + 2 ⋅ K −3 and EF =
BC
= 100 V
+ 3 ⋅ K −4 + K 2 B
= 2(K + K + K + ...) + 3(K −2 + K −4
−1 −3 −5 ∴ AD = AE + EF + FD
+ K −6 + K) = 200 + 100 + 200 = 500 A
2  1 1  12. (b) T = {4, 5, 6} (mod 15) T
=  1 + 2 + 4 + K
K  K K 
or T = {19, 20, 21}, {34, 35, 36}, {49, 50, 51}, Process BC is isobaric, so
+ 2  1 + 2 + 4 + K
3 1 1 {64, 65, 66} V
K  K K 
{79, 80, 81}, {94, 95, 96}, {109, 110, 111}, B
  {124, 125, 126}
 2 3   1  2K + 3
=  + 2  = {139, 140, 141}, {154, 155, 156}, C
K K   1 − 1  K 2 − 1 T
 {169, 170, 171}, {184, 185, 186} (mod15)
K 
2

7 Now, by direct checking we get smallest 18. (d) Number of neutrons,


Given, 0. 23K = N =A−Z
51 T = 184
2K + 3 7 ∴ Required sum = 1 + 8 + 4 = 81
2 2 2 For 6 Cl13 , N = 13 − 6 = 7
∴ =
K − 1 51
2 and 7 N14 , N = 14 − 7 = 7
13. (d) We have,
⇒ 7K 2 − 102K − 160 = 0 So, they contains same number of
x2 − bx + c = 0
neutrons.
⇒ 7K 2 − 112K + 10K − 160 = 0 Let α , β are the roots of the equations.
⇒ (7K + 10) (K − 16) = 0
19. (c) Part of heat given to A is used up
∴ α + β = b, αβ = c in doing work (against) gravitational force.
−10
⇒ K = 16, K ≠ Given, α −β =1 So, temperature of B will be slightly higher.
7
∴ (α + β )2 − (α − β )2 = 4αβ
10. (b) Sum odd only be formed 20. (c) Kinetic energy of stone decreases
b2 − 1 = 4c then increases. It is never zero in entire
(even, even, odd) or (odd, odd, odd). So can ⇒ b − 4c = 1
2
flight of stone.
focus on placing evens, we need to have
https://iit-jeeacademy.blogspot.com

228 KVPY Practice Set 5 Stream : SA

21. (a) In absence of charge, time period If surface over which points are located is 35. (c) CO2 and CH 4 have zero dipole
m a plane, then Geodesic is a straight line. moment as these are symmetrical in
T1 = 2 π On the surface of a planet, Geodesic is a nature. Between NH3 and NF3 , NH3 has
2k
great circle joining two points. greater dipole moment though in both
In presence of charge, time period NH3 and NF3 , N possesses one lone pair
m 30. (b) Image of first lens is formed at
T2 = 2 π of electrons.
focal point of second lens.
2k This is because in case of NH3 , the net
0.25 m
So, ratio of time period is equal to one. N—H bond dipole is in the same direction
as the direction of dipole of lone pair but
22. (c) Total output energy of battery
in case of NF3 , the direction of net bond
= V (It ) = 12 V × 150 Ah dipole moment of three N—F is opposite
= 12 × 150 × 3600 J to that of the dipole moment of the lone
So, emerging rays are parallel.
= 6.48 × 106 J pair which cancel out the resultant dipole.
31. (c) 100 g of haemoglobin contains
Energy consumed by car lights in time t H
= 0.33 g Fe
= 95t
∴ 67200 g of haemoglobin contains C
Equating both values, we get
0.33 × 67200 O==C==O H H
6.48 × 106 = g Fe
µnet=0
t= ≈ 18. 9 h 100 H
95 = 221.76 g Fe µnet=0
23. (d) When string C is cut, spring ∴ Number of Fe atom present in one
snaps back to regain its unstretched 221.76
molecule of haemoglobin = ≈4 N N
length. So, spring force on m remains 56 F F
H H
same. Hence, accelerations of m and 2m
32. (c) (a) Angular quantum number, l H F
are 2g and g, respectively. signifies the shape of the orbital. e.g. µ net µ net
24. (a) Streamlines may be straight or When l = 0, the shape of orbital is = 4.90 × 10−30 cm = 0.80 × 10−30 cm
curved. spherical.
36. (c) A molecule is set to be chiral, if
In fluids, with low viscosity streamlined When l = 1, the shape of orbital is
all the four groups attached to central
flow occurs only at low flow speed. dumb-bell.
carbon atom are different.
25. (c) [A 4 B −3 C 2 ] Thus, statement (a) is correct.
Br
= [ML2 T −2K −1 ]4 ⋅ [ML2 T −1 ]−3 ⋅ [LT −1 ]−2 (b) According to Bohr’s theory 3 2 1
(a) H3C—CH2—C—Cl
= [M4 − 3 L8 − 6 − 2 T −8 + 3 + 2
K −4 ] For hydrogen like atom
Z2
= [ML0 T −3 K −4 ] En = − 13.6  2  eV Br
= Stefan’s constant. n  1, 1-dibromo-1-chloropropane
26. (c) Time for downward journey is Thus, statement (b) is correct. It is an achiral molecule.
higher as ball can be thrown with any (c) Total number of nodes in orbital
Br
velocity but its downward velocity is = n − l−1
3 2 1
always less than or equal to terminal For 3 s-orbital, n = 3, l = 0 (b) Cl—CH2—CH2—CH
velocity. Number of nodes = 3 − 0 − 1= 2
27. (a) Let car accelerates for time t1 Thus, statement (c) is incorrect. Br
and then it deaccelerates for time t2. (d) According to Bohr’s radius 1, 1-dibromo-3 chloropropane
Then, αt1 − βt2 = 0 and t = t1 + t2 a n2 It is an achiral carbon.
r= 0
 αβ  Z Br
Maximum speed, v = αt1 =   t 3 2 1
α + β a0 (1)2 a (1)2
rHe + = , rH o *
BrCH2—CH2—CH
2 1
28. (b) Taking moments about point of
Thus, statement (d) is correct. Cl
contact O.
33. (d) Let the solubility of CaF2 be S. 1, 3-dibromo-1-chloropropane
For the reaction, It is a chiral molecule with chiral carbon
1m position at 1(*).
0.6 O CaF2 - Ca2+ (aq) + 2F − (aq)
0.8
S 2S Cl Br
Ksp = [Ca2+ ] [F − ]2 = (S) (2S)2 = 4S 3 3 2 1
(d) H2C—C—CH2
F × 0.6 = 100 × 0.8 ∴ Ksp = 4 (2 × 10−4 )3 = 32 × 10−12 M3
100 × 0.8 34. (a) Let, the oxidation state of Fe in
⇒ F= = 133.3 N ~
− 134 N Br Cl
0.6 [Fe(H 2 O)5 (NO)] SO4 be x . 1, 3-dibromo-2-chloropropane
29. (d) Shortest distance between two ∴ x + 5 (0) + 1 = + 2 It is an achiral molecule.
points is called Geodesic. ⇒ x=+1
https://iit-jeeacademy.blogspot.com

KVPY Practice Set 5 Stream : SA 229

37. (b) When an electron withdrawing As it contains  C CH3 group. Due to the presence of chiral centre (*), it
group, like NO2, Cl is attached to the  shows the optical activity and its mirror
phenol ring, it stabilises the negative O image are non-superimposable.
Thus, it will show positive iodoform test.
charge on the oxygen of the phenoxide Hence, it shows one enantiomeric pair.
ion. Due to this reason, acidic character (d) CH 3 CH 2 CH 2 CH 2 CHO
(Pentanal) 45. (b) H2 ( g ) + Cl 2 ( g ) —→ 2HCl( g )
of phenol increases. Between compound
III and IV, III is more acidic as NO2 is It gives negative iodoform test. ∆H = [(BE)H − H + (BE)Cl-Cl ] − 2 [(BE)HCl ]
more stronger EWG than Cl. 41. (c) The structures of given species = [435 + 243] − 2 [431] = − 184 kJ mol −1
But when an electron donating group, are shown below: The moles of HCl( g ) are formed from its
like CH3 is attached to the phenol ring, it O F element, hence
destabilises the ring and hence acidic F F − 184
Xe Xe ∆ f H °( HCl ) = = − 92 kJ mol −1 .
character of phenol decreases. Thus, the F F 2
correct order of acidic character is
F 46. (d) Glycocalyx is the outermost
OH OH OH OH XeOF4
XeF2 mucilage layer of the cell envelope. It gives
Square pyramidal
L inear
CH3 (sp3d2) sticky character to the bacterial cell.
(sp3d2)
> > > 47. (c) Ectocarpus and Fucus
F F respectively show haplodiplontic and
Xe Xe diplontic life cycle. In Ectocarpus, sporic
NO2 Cl O O F F meiosis occurs and haploid biflagellate
III IV I II O meiozoospores are formed. They
XeF4
XeO3 Square planar germinate to produce gametophytic
38. (d) Anion is formed by the gain of
Pyramidal (sp3d2) thalli. The gametophytes liberate
electron to the neutral atom and cation is
(sp3) gametes which fuse to form diploid zygote
formed after the loss of electron from the which gives rise to a diploid plant.
neutral atom. Hence, cation has smaller 42. (d) Sodium peroxide which is a
yellow solid, reacts with moisture and In Fucus, there is a single somatic
size due to increased nuclear charge
CO2 of air (when exposed to air) and phase. It is diploid and produces haploid
whereas anion has bigger size than its
becomes white due to the formation of gametes. They fuse during fertilisation to
neutral atom. give rise to diploid individual.
NaOH and Na2 CO3 .
Thus, the correct order of size is I− > I > I+ .
2Na2 O2 + H 2 O → 4NaOH + O2 48. (d) Among the given options, except
39. (d) According to Gay Lussac’s law, at 2NaOH + CO2 → Na2 CO3 + H 2 O lipids all are polymers. These are formed
constant volume, the pressure of given by the polymerisation of monomers. The
mass of the gas is directly proportional to 43. (c) As CH 3 group is ortho-para basic unit of lipid are fatty acids and
its absolute temperature, i.e. directing, so the major products will be glycerol molecules that do not form
p ∝ T or p = kT formed at o and p-position only. repetitive chains. Instead they form
CH3 triglycerides from 3 fatty acids and one
Thus, the correct representation is given CH3 glycerol molecules. Protein monomers are
in option (d). amino acids and they bond together in
Br2/FeBr3 Br
Electrophilic repetitive chains just as carbohydrate
CH3 substitution monomers are monosaccharides.
p
CH3 49. (c) The body wall of a common
+
Br2/FeBr3 sponge consists of three layers i.e.
T CH3 pinacoderm, choanoderm and mesophyll
40. (c) Positive iodoform test are given layers. Choanoderm is inner cellular
by those carbonyl compounds which CH3 layer which consists of highly specialised
contain CH 3  C group. Br
flagellated cells called choanocytes. The
 CH3 beating of their flagella creates water
O Br current.
(a) CH 3 CH 2 CH 2 CH 2 CH 2 OH CH3 50. (d) A decrease in blood
(Pentanol) Not possible due pressure/volume stimulates the
to steric Hindrance
It gives negative iodoform test. hypothalamus to release ADH
Thus, the correct option is (c). (Antidiuretic Hormone) as well as JGA
(b) H 3 C  CH 2  C  CH 2  CH 3
 44. (d) CH 3  CH  CH  CH 3 + Cl 2 cells to release renin. Renin by renin
O   angiotensin mechanism activates the
(Pentan-3-one) CH3 CH3 adrenal cortex to release aldosterone.
2, 3-dimethyl butane ANF (Atrial Natriuretic factor) is
It does not show iodoform test.
CH3 produced by atria of heart during
(c) H 3 C  CH 2  CH 2  C  CH 3 increased blood pressure. It can cause

 → CH 3  CH  C H  CH 2 Cl vasodilation and thereby decrease the
O  * blood pressure. Therefore, option (d) is
(Pentan-2-one) CH3 correct.
1− chloro-2, 3 dimethyl butane
https://iit-jeeacademy.blogspot.com

230 KVPY Practice Set 5 Stream : SA

51. (c) Vascular cambium located pair of sister chromatids at this stage When expressed in lowest term, the
between xylem and phloem in the stems each chromosome and hence each end denominator of this fraction will always
and roots of vascular plants. It produces would have two copies of the allele. be a divisor of 99 = 3 ⋅ 3 ⋅ 11
secondary xylem towards the pith and 61. (b) Let us partition in to n parts and This gives us the possibilities
secondary phloem towards the bark.
each part has sum = K, then {1, 3, 9, 11, 33, 99}. As a and b both are not
52. (d) Pancreas consists of exocrine and nK = 1 + 2 + 3 + K + 20 both 9 and not both zero the denominator
endocrine parts. Exocrine part secretes 1 cannot be possible.
⇒ nK = 210
alkaline pancreatic juice. This juice ∴ Possible denominators are
contains trypsinogen, chymotrypsinogen, ∴K divides 210.
{3, 9, 11, 33, 99}.
procarboxypeptidase, lipase, amylase, Also, K must be ≥ 20 .
elastase. 64. (c) In ∆ABC,
Now, 210 = 2 × 3 × 5 × 7
∠C = 90°, AB = 10, ∠B = 60°
53. (a) Archaebacteria are the most So, proper divisor of 210 are {1, 2, 3, 5, 6,
primitive form of bacteria. These live in 7, 10, 14, 15, 21, 30, 35, 42, 70, 105, 210} B
diverse habitat, e.g. extreme hot ⇒ K can be 21, 30, 35, 42, 70,105
temperature, saline condition, variable 10
For K = 21, we have (1, 20) (2, 19) ... P
pH, etc. Saline bacteria are called
halophiles (e.g. Halobacterium, (10, 11)
Halococcus). ⇒ 21 is a good number.
54. (c) If anaphase promoting complex is For K = 42, join two-two pairs C D A
defective in a human cell, the For K = 105, join five-five pairs In ∆ABC,
chromosome will not segregate ⇒ 42 and 105 are also good numbers BC = AB cos B = 10 × cos 60° = 5
during anaphase of mitosis. APC triggers For K = 30, we have AC = AB sin B = 10 × sin 60° = 5 3
the transition from metaphase to Choose a P ′ and get a corresponding D′
anaphase by tagging specific proteins for {20, 10}, {19, 11}, {18, 12}, {17, 13}, {16, 14},
{15, 9, 6}, {1, 2, 3, 4, 5, 7, 8} such that BD′ = 5 2
degradation.
⇒ K = 30 is also good number BD > 5 2 > BC 2 + CD 2 > 25 + CD 2
55. (d) Zygotic meiosis is represented in
the haplontic life cycle of many algae Similarly, 35 and 70 also good numbers. ⇒ CD > 5
including Chlamydomonas. In such a ∴There are total 6 good numbers. Thus, the point P may only lie in the
life cycle, all cells are haploid except 62. (c) We have, ∆ABD′
zygote. This is because meiosis occurs in
C1 : (x − 10)2 + y2 = 36 Area of ∆ABD ′
Required probability =
the zygote itself resulting into four Area of ∆ABC
haploid cells that give rise to haploid C2 : (x + 15)2 + y2 = 81
plants. Centre of C1 = (10, 0) and radius = 6 The ratio of AD′ to AC because the
Centre of C2 = (−15, 0) and radius = 9 triangle have identical altitudes.
56. (c) Gibberellins and ABA are AD ′ AC − CD ′
antagonistic to each other. Gibberellins So, ratio = =
Q AC AC
promote seed germination whereas ABA
CD ′ 5 3−1
promotes seed dormancy. 9 = 1− = 1− =
9 6 AC 5 3 3
57. (b) Ornithine cycle removes both B
ammonia and urea from the blood. It A(–15,0) (10,0) ∴Required probability
6
converts ammonia into urea (in liver) P ( 3 − 1) × 3 3 − 3
= =
and transports it to kidneys by the blood. 3× 3 3
Hence, it plays a key role in detoxification The length of smallest line segment PQ is
of our blood. This cycle occurs in the liver. 65. (a) We have,
the indirect common tangent of circle
P (x) = (x − 1) (x − 2) (x − 3)
58. (b) In the dicot stem, the cortical ∴ PQ = AB 2 − (r1 + r2 )2
cells get differentiated to give rise to and P[Q (x)] = P (x) ⋅ R (x)
another meristematic tissue, which is ⇒ PQ = (25)2 − (9 + 6)2 ∴ P[Q (x)] = [Q (x) − 1] [Q (x) − 2] [Q (x) − 3]
called cork cambium or phellogen. On the = (x − 1) (x − 2) (x − 3) R (x)
[Q AB = (10 + 15)2 − 02 , AB = 25]
other side, it forms phellem (cork) and in Since, degree of P (x) = 3
the inner region it forms secondary ⇒ PQ = 625 − 225 and degree of R (x) = 3
cortical cells (phelloderm). ⇒ PQ = 400 = 20 ∴Degree of [P (x) ⋅ R (x)] = 6
59. (d) Sucrose shows the most substrate 63. (d) The repeating decimal 0.ab is Thus, degree of P (Q (x)) = 6, so degree
specificity since it hydrolyses only the equal to Q (x ) = 2
disaccharide, sucrose.
x = 0. abababab ...(i) P (Q (1)) = (Q (1) − 1) (Q (1) − 2) (Q (1) − 3) = 0
60. (c) Since the allele is recessive, both 100x = ab ⋅ abababab ...(ii)
homologous chromosomes in a somatic P (Q (2)) = (Q (2) − 1) (Q (2) − 2) (Q (2) − 3) = 0
On subtracting Eq. (i) from Eq. (ii), we get P (Q (3)) = (Q (3) − 1) (Q (3) − 2) (Q (3) − 3) = 0
cell of an albino person would have the
allele. After meiosis-I, each end would 99x = ab Hence, we conclude Q (1), Q (2) and Q(3)
ab 10a + b must each be 1, 2, 3. Since, a quadratic is
have a homologous chromosome with the ⇒ x= =
allele. As the chromosome is existing as a 99 99 uniquely determined by the three points.
https://iit-jeeacademy.blogspot.com

KVPY Practice Set 5 Stream : SA 231

There can be 3 × 3 × 3 = 27 different series and current remains same in both, Thus, terminal alkyne (A) with molecular
quadratic. so IRAB < IRBC . formula C4 H 6 is CH 3 CH 2 C ≡≡ CH. Thus,
However, we have included Q (x) which Now, for a parallel combination, option (a) is correct.
are not quadratic. They are line. V2 1
P= or P ∝ . CH2— CH—CH— CH2
Then, Q (1) = Q (2) = Q (3) = 1 R R
⇒ Q (x ) = 1 B
Hence, a larger potential drop occurs
(Isomer of A)
Q (1) = Q (2) = Q (3) = 2 across 4Ω bulb.
⇒ Q (x ) = 2 Hence, 4Ω bulb glows brightest in given + –
Q (1) = Q (2) = Q (3) = 3 circuit. CH2 — CH — CH— CH2
⇒ Q (x ) = 3 69. (b) At extreme positions, total Br2
Q (1) = 1, Q (2) = 2, Q (3) = 3 energy is potential energy.
CH2 — CH — CH— CH2
kx2 0.5x2


⇒ Q (x ) = x So, U (x ) = ⇒1 =
Q (1) = 3, Q (2) = 2, Q (3) = 1 2 2 Br Br
⇒ Q (x ) = 4 − x ⇒ x2 = 4 ⇒ x = ± 2m 1, 4 dibromobut-2-ene

So, these linear function are not included So, particle turns back after reading 2 m 73. (b) Number of moles of the gas
mark. 2.0 × 1021
∴ Total number of polynomials = mol
∴Amplitude of oscillation of particle is
= 27 − 5 = 22 6.023 × 1023
2 m.
66. (d) For first stone, = 3.32 × 10−3 mol
1 2
70. (a) Let m = mass of ice initially
x1 = ut + at present. From ideal gas equation
2 pV = nRT
Heat absorbed from t = 0 to t = 50 min is
= 15t − 5t 2 [Q a = g = − 10 ms−2] pV 7.57 × 103 × 10−3
Q1 = miceL T = =
Now, x1 = − 200 m nR 3.32 × 10−3 × 8.314
= m(kg) 333 (J/g)
⇒ 15t − 5t 2 = − 200
= m (3.33 × 105 ) J = 274.25 K
⇒ 5t − 15 t − 200 = 0
2
and heat absorbed from t = 50 min to Root mean square speed,
⇒ 5t 2 − 40t + 25t − 200 = 0
t = 60 min, 3RT
⇒ 5t (t − 8) + 25 (t − 8) = 0 vrms =
Q2 = ms∆T M
⇒ t = 8 s or t = − 5 s
= (10 + m) (4186
. × 103 ) (10 − 0) 3 × 8.314 × 274.25
(Q t = −5 s not acceptable) =
= (10 + m) (4186
. × 103 ) (10) 28 × 10−3
∴Time for which first stone remains in
air = 8 s.
Given that, rate of heat supply is = 494.26 ms−1
constant.
So, graph is x2 − x1 = 15t Q1 Q
74. (d) (a) Electronic configuration of
So, = 2 Br2 = σ1s2 σ * 1s2 σ 2s2 σ * 2s2
(For t = 0 to t = 8 s) ∆t1 ∆ t2
4− 4
and x2 − x1 = 200 + 30t − 5t 2 m (3.33 × 105 ) (10 + m) (4186
. × 103 ) (10) B.O = =0
= 2
(For t > 8 s to t = 10 s) 50 10
m(33.33) Thus, it does not exist. Hence, statement
67. (d) When object is placed at 2f ⇒ = (10 + m)(4186
. ) (a) is correct.
distance, image formed is also at 2f . 5
2− 2
⇒ 33.33 m = 209.3 + 20.93 m (b) He 2 = σ1s2 σ * 1s2 B.O = =0
So, from graph, 2
2f = − 30 cm ⇒ 12.4 m = 209.3 Hence, it will not exist.
2−1
⇒ f = − 15 cm ⇒ m = 16.87 kg He +2 = σ1s2σ * 1s2, B.O = = 0.5
2
68. (a) Circuit can be redrawn as 71. (d) Amount of heat liberated by Since, the bond order is not zero, this
0.05 g of C molecule is expected to exist.
2Ω 4Ω
= mC∆T (c) N 2 = σ1s2σ * 1s2 σ 2s2 σ * 2s2
A 3Ω B C = 2000 × 1 (26 − 24) π 2 px2 ≈ π 2 py2 σ 2 pz2
10 − 4
= 4000 cal = 4 kcal B.O = =3
6Ω 5Ω 4 kcal × 1 2
Calorific value = = 8 kcal per g
0.5 Thus, it has the maximum bond strength
among the other diatomic molecule
∴ Heat of combustion = − 8 × 12
belonging to the second period.
+ – = − 96 kcal per mol
20 V (d) The correct electronic configuration of
72. (a) As the organic compound forms N 2 is σ1s2 σ * 1s2σ 2s2 σ * 2s2
As RAB and RAC are nearly 1 Ω and 2Ω white precipitate with AgNO3 /NH4 OH π 2 px2 ≈ π 2 py2 σ 2 pz2
respectively, a larger potential drop and red precipitate with Cu 2Cl 2/NH4 OH,
Thus, statement (d) is incorrect.
occurs across BC. As AB and BC are in it must be a terminal alkyne.
https://iit-jeeacademy.blogspot.com

232 KVPY Practice Set 5 Stream : SA

75. (a) Equivalent mass of M quaternary structure. Tertiary structure 79. (a) There are 12 chromosomes
mass of metal of protein is a single polypeptide chain present. In a mitotic division, the number
= ×8
mass of oxygen folded and twisted. Primary structure of of chromosomes remains the same, i.e.
10 protein is amino acids joined end to end 2n → 2n. In a meiotic division, the
= × 8 = 100 with each other. Secondary structures number is halved, i.e., 2n → n so, there
0.8
are folded into α-helix and β-pleated should only be 6 chromosomes in the
Approximate atomic mass sheets. nucleus.
6.4 6.4
= = = 193.3 g 77. (d) The amount of light given is 80. (b) Phosphate group, ribose or
specific heat 0.033
saturated, not limited. Therefore the deoxyribose group and cytosine group
193.93
Valency of M = = 1.98 ≈ 2 answer is (d). The concentration of carbon form the nucleotide.
100 dioxide is in short supply, hence limiting Phosphate (H 3 PO4 )
So, accurate atomic mass the rate of photosynthesis.
Ribose (C 5 H10O5 )
= equivalent mass × valency 78. (c) The drugs will only prevent the Deoxyribose (C 5 H10O4 )
= 100 × 2 = 200 g response of the muscle to an electrical
Cytosine has 4 carbon atoms, as it is a
molar mass 199.87 stimulus if it prevents the release of
Atomicity = = =1 acetylcholine and so inhibits the increase
pyrimidine.
atomic mass 200
in membrane permeability to sodium Therefore, the nucleotide should have
76. (d) The folding of two or more ions, and if the drug competes with 5C + 4C = 9C
polypeptide chains constitutes the
acetylcholine at the receptor sites.

You might also like